General Naplex Study Guide

Lakukan tugas rumah & ujian kamu dengan baik sekarang menggunakan Quizwiz!

Which of the following medications is formulated so it can be dosed once weekly? A.) Exenatide B.) Pioglitazone C.) Sitagliptin D.) Metformin E.) Liraglutide

A.) Exenatide Exenatide can be dosed once weekly with the extended release formulation (Bydureon). The other agents are dosed once or twice daily.

Which of the following best outlines the central dogma of molecular biology? A.) mRNA→DNA→Protein B.) DNA→mRNA→Protein C.) Protein→RNA→DNA D.) DNA→Protein→mRNA E.) Protein→DNA→mRNA

B.) DNA→mRNA→Protein DNA is transcribed into mRNA, which is translated ultimately to protein.

Which of the following drugs is indicated for bedwetting in children? A.) Vasopressin B.) Desmopressin C.) Cosyntropin D.) Oxymetholone E.) Methimazole

B.) Desmopressin Desmopressin, an antidiuretic hormone analog, may be used to control nocturnal enuresis.

Based on pharmacokinetic changes associated with aging, which of the following medications would be most likely to accumulate with repeated doses? A.) Alprazolam B.) Diazepam C.) Donepezil D.) Penicillin E.) Selegiline

B.) Diazepam Explanation: Diazepam is a lipid-soluble drug and will accumulate in older adults secondary to increased storage in body fat. The potential for accumulation is much less with the other medications listed.

A 64-year-old woman who weighs 143 lbs presents to an emergency department that is not percutaneous coronary intervention capable. She has an ST-elevation myocardial infarction (MI). The team would like to administer alteplase for the treatment of the MI. The dose for a ST-elevation MI for a patient weighting < 67 kg is 15 mg IV bolus followed by an infusion of 0.75 mg/kg (not to exceed 50 mg) over 30 minutes and then 0.5 mg/kg (not to exceed 35 mg) over 1 hour. What is the total calculated dose in mg to be administered for this patient? (Answer must be numeric; round the final answer to the nearest HUNDREDTH).

96.25

Of the agents listed, which chemotherapy agent has the highest emetic risk? A) Cyclophosphamide B) Etoposide C) Paclitaxel D) Vincristine

A) Cyclophosphamide Cyclophosphamide (A) has high emetogenic potential. More than 90% of patients experience nausea and vomiting if dosed ? 1,500mg/m 2 , or if given with doxorubicin or epirubicin if appropriate antiemetic prophylaxis is not given. The routine use of antiemetic is necessary when giving cisplatin. Etoposide (B) and paclitaxel (C) have low emetogenic potentials. Vincristine (D) has minimal emetogenic potential.

The administration of which of the following agents would be indicated in the setting of tricyclic antidepressant overdose associated with seizures or QRS interval >115 ms? A) Sodium bicarbonate B) Flumazenil C) Physostigmine D) Procainamide E) Both a and c

A) Sodium bicarbonate Serum alkalinization with a goal of establishing a systemic pH between 7.45 and 7.55 is indicated in the setting of tricyclic antidepressant (TCAD) toxicity with evidence of widening QRS interval and/or seizure activity. Mechanisms by which sodium bicarbonate administration may provide benefit are as follows: 1. Increased systemic sodium levels may overcome blocked sodium channels and/or increased systemic pH results in increased nonionized TCAD, decreasing ligand receptor interaction between ionized TCAD and the sodium channel. 2. Wide complex dysrhythmia and hypotension can be reversed by administration of sodium bicarbonate in sufficient dose. Seizures will not respond to this sodium bicarbonate therapy, and should be treated with standard measures, but if seizures occur in the setting of TCAD overdose sodium bicarbonate therapy is warranted because they indicate significant toxicity. 3. Other potential therapies could include administration of hypertonic 3% saline to increase systemic sodium levels or hyperventilation to increase systemic pH; however, these have not been found to provide the same level benefit as treatment with sodium bicarbonate.

BB is a 5'5", 200 pound, 58 year old female with stage II breast cancer who is s/p lumpectomy with negative surgical margins and negative sentinel lymph node. Her oncologist recommends adjuvant chemotherapy with doxorubicin 60 mg/m2 IV day 1 and cyclophosphamide 600 mg/m2 IV day 1, both given every 3 weeks for four cycles. What is the weight that will be used to calculate the body surface area that will be used to calculate AA's chemotherapy doses? A.) Actual body weight. B.) Ideal body weight. C.) Adjusted body weight. D.) Lean body weight.

A.) Actual body weight. While it may seem that a 5'5" 200 pound female should have her weight adjusted downward for body surface area calculation, this practice is not supported by research. When the goal of chemotherapy is cure, adjuvant effect (as in this case), or survival extension, research has shown that the patient's actual body weight produces best treatment effect and no excess of toxicity.

Which of the following is the drug of choice for the treatment of acute febrile seizures? A.) Diazepam B.) Carbamazepine C.) Felbamate D.) Fosphenytoin E.) Lamotrigine

A.) Diazepam EXPLANATION: Benzodiazepines, like rectal diazepam and lorazepam, are the drugs of choice for prolonged febrile seizures. Carbamazepine and phenytoin are not effective in managing febrile seizures; severe side effects of felbamate and the need to titrate lamotrigine preclude their use for this type of seizure.

The pharmacy exemption from the stringent labeling requirements imposed on pharmaceutical manufacturers does not apply to which of the following? A.) Drugs that are dispensed pursuant to a diagnosis by mail B.) Mail order pharmacies C.) Prescribers who dispense drugs D.) The exemption applies to all dispensed medications, regardless of the circumstances.

A.) Drugs that are dispensed pursuant to a diagnosis by mail This provision is designed to protect consumers against fraud and quackery in situations where there is no physician-patient relationship because they are diagnosed and prescribed for by mail, or by means of the Internet. This provision does not apply to mail order pharmacies. If the prescription is a result of a diagnosis by mail or a diagnosis over the Internet, the dispenser must follow the labeling rules that apply to manufacturers. The FDA has ruled that the labeling requirements imposed on pharmacists do not apply to prescribers who dispense medication. See, § 503(b)(2) of the FDCA; 21 U.S.C. § 353(b)(2)

When one compounds an o/w emulsion containing a flavoring agent, the flavor should be in which emulsion phase? A.) External B.) Internal C.) Dispersed D.) Discontinuous

A.) External For the flavoring agent to be tasted, it must be in the external or continuous phase. In an o/w emulsion, the oil is the internal, dispersed, or discontinuous phase.

Which of the following is not a possible side effect of minoxidil? A.) Gynecomastia B.) Hypertrichosis C.) Scalp dermatitis D.) Dizziness E.) Chest pain

A.) Gynecomastia Gynecomastia is not a side effect of minoxidil.

Which of the following agents that Ms. Tiny is taking can be used to treat breast and prostate cancer? I. Zoladex II. Tamoxifen III. Celebrex A.) I only B.) III only C.) I and II only D.) II and III only E.) I, II, and III

A.) I only Zoladex (goserelin) is an LHRH agonist that can be used to treat both breast and prostate cancer. LHRH agonists are FDA approved for premenopausal women, as they inhibit estrogen production from the ovaries.

Which of the following is required for adequate vitamin B12 absorption? A.) Intrinsic factor B.) Ferritin C.) Hemoglobin D.) Acidic environment E.) Transferrin

A.) Intrinsic factor Explanation: Intrinsic factor is critical for the absorption of vitamin B12.

A patient with heart failure develops hyperkalemia while treated enalapril. Which alternative medication should be used in this patient to improve survival? A.) Isosorbide dinitrate and hydralazine B.) Lisinopril C.) Valsartan D.) Digoxin E.) Dofetilide

A.) Isosorbide dinitrate and hydralazine The combination of isosorbide dinitrate and hydralazine improves survival in patients with heart failure and these medications are not associated with hyperkalemia. Valsartan and lisinopril are as likely to cause hyperkalemia as enalapril. Neither digoxin nor dofetilide improve heart failure survival.

For which of the following agents is hypersensitivity upon reexposure a clinical concern? A.) Lepirudin B.) Fondaparinux C.) Bivalirudin D.) Argatroban E.) Warfarin

A.) Lepirudin

Which of the following could potentially increase LDL? A.) Lovaza B.) Lipitor C.) Tricor D.) Questran E.) Welchol

A.) Lovaza Fish oils can potentially increase LDL levels by about 20-30%.

Which of the following are not true concerning memantine? A.) Memantine is FDA approved for mild to moderate Alzheimer's disease dementia. B.) Side effects of memantine include drowsiness, dizziness, headache, and motor restlessness. C.) Dose should be reduced to 5 mg twice daily in patients with a creatinine clearance of less than 30 mL/min. D.) Dextromethorphan should be used with caution when taken with memantine. E.) Dosage is increased to 10 mg twice daily in Alzheimer's disease patients

A.) Memantine is FDA approved for mild to moderate Alzheimer's disease dementia. Memantine is FDA approved only for moderate to severe Alzheimer's dementia. All other statements are correct.

Which of the following medications may cause seizures in an adult patient with renal failure? A.) Meperidine B.) Phenobarbital C.) Carbamazepine D.) Lamotrigine E.) Theophylline

A.) Meperidine Normeperidine, a metabolite of meperidine, can accumulate in patients with renal failure who receive normal doses and cause seizures. The other agents listed are not eliminated renally in adults.

Which electrolyte abnormality typically occurs in patients with severe kidney dysfunction (i.e., creatinine clearance < 15 mL/min)?n)? A.) Metabolic acidosis B.) Metabolic alkalosis C.) Hypophosphatemia D.) Hypernatremia

A.) Metabolic acidosis Metabolic acidosis is a common secondary complication of AKI. Other electrolyte abnormalities include hyperkalemia and hyperphosphatemia. Sodium disorders are usually caused by other concomitant disorders, but not by AKI alone.

AJ is an 84 year-old female with no significant medical history. She presents to her primary care provider with complaints of frequent urination and dysuria that began 4 days ago. What antibiotic regimen is most appropriate for AJ? A.) Nitrofurantoin x 5 days B.) Ampicillin x 5 days C.) Moxifloxacin x 3 days D.) Ceftriaxone x 7 days E.) Gentamicin x 3 days

A.) Nitrofurantoin x 5 days Nitrofurantoin x 5 days is the most appropriate regimen for a female with an uncomplicated UTI. Response B is only indicated for an enterococcal UTI and the other choices are not appropriate treatment for uncomplicated UTIs.

Long-term use of which medication may lead to iron depletion? A.) Omeprazole B.) Trimethoprim-Sulfamethoxazole C.) Hydrochlorothiazide D.) Ascorbic acid

A.) Omeprazole Omeprazole. Proton-pump inhibitors reduce the acidic environment in the stomach which reduces iron absorption as the ferrous form (Fe+2) is absorbed the best. Vitamin C (ascorbic acid) may slightly improve absorption.

Which of the following disease states or clinical conditions would usually require the administration of parenteral nutrition? A.) Postoperative ileus B.) Motor vehicle crash resulting in femur fracture and head injury C.) 20% body surface area burn from a house fire D.) Laparoscopic cholecystectomy E.) Acute exacerbation of hepatic encephalopathy

A.) Postoperative ileus Feeding a patient with postoperative ileus enterally or orally is inappropriate. The other clinical conditions, such as trauma and burns, would occur in patients in whom the gastrointestinal tract could and should be used for nutrition support. A patient receiving laparoscopic cholecystectomy would not need nutrition support. Most patients with hepatic encephalopathy can be fed enterally if they require nutrition support.

Which of the following antiarrhythmic agents' mechanism of action is primarily the result of sodium ion transport blockade? A.) Propafenone B.) Ibutilide C.) Sotalol D.) Verapamil E.) Diltiazem

A.) Propafenone Explanation: Propafenone blocks sodium entry into the cardiac cell, slowing depolarization. Ibutilide and sotalol primarily act by blocking potassium transport, whereas verapamil and diltiazem inhibit the calcium channel.

Which one of the following is contraindicated with proton pump inhibitors? A.) Rilpivirine B.) Etravirine C.) Efavirenz D.) Raltegravir E.) Atazanavir/ritonavir

A.) Rilpivirine Explanation: Rilpivirine is contraindicated with proton pump inhibitors. Proton pump inhibitors can be used with boosted atazanavir in PI-naïve patients. The PPI dose should not exceed a dose equivalent to omeprazole 20 mg daily and the patient should be PI naïve. PPIs are contraindicated with unboosted atazanavir.

All of the following are true about the use of furosemide in heart failure EXCEPT: A.) The drug reduces mortality and slows heart failure progression B.) Hypokalemia is a common adverse effect C.) Response can be evaluated by monitoring patient weight D.) Oral absorption is slowed in patients with acute decompensated heart failure E.) Furosemide's bioavailability is reduced by food

A.) The drug reduces mortality and slows heart failure progression Although furosemide plays an important role in patients with heart failure by interfering with sodium and water retention, it only provides symptomatic benefit. Neither furosemide nor other diuretics improve survival or affect heart failure progression.

What innate cell represents the majority of granulocytes and serves as the primary defense against bacterial infections? A) Lymphocytes B) Neutrophils C) Monocytes D) Eosinophils E) Basophils

B) Neutrophils Neutrophils (B) represent the majority of granulocytes (80%-90%) and leukocytes (40%-70%) and serve as the primary defense against bacterial infections. Neutrophils, also termed as segs or polymorphonuclear cells, migrate from the bloodstream into infected or inflamed tissue. In this migration process, known as chemotaxis, neutrophils reach the desired site and recognize, adhere to, and phagocytose pathogens. During phagocytosis, the pathogen is internalized within the phagocyte. The neutrophil releases its granular contents, which lead to destruction of the engulfed pathogen.

Select the innate cell that represents the majority of granulocytes and serves as the primary defense against bacterial infections. A) Lymphocytes B) Neutrophils C) Monocytes D) Eosinophils E) Basophils

B) Neutrophils Neutrophils represent the majority of granulocytes (80%-90%) and leukocytes (40%-70%) and serve as the primary defense against bacterial infections. Neutrophils, also termed as segs or polymorphonuclear cells, migrate from the bloodstream into infected or inflamed tissue. In this migration process known as chemotaxis, neutrophils reach the desired site and recognize, adhere to, and phagocytose pathogens. During phagocytosis, the pathogen is internalized within the phagocyte. The neutrophil releases its granular contents which lead to destruction of the engulfed pathogen.

A significant drug interaction with aprepitant may occur with which of the following? A) Gabapentin B) Oral contraceptives C) Enoxaparin D) Nystatin

B) Oral contraceptives Reduced efficacy of oral contraceptive pills (B) has been shown when taken with aprepitant. Women should use backup contraceptives for 1 month after discontinuing aprepitant. No interaction with aprepitant exists with gabapentin (A), enoxaparin (C), or nystatin (D).

How much A1C reduction would you expect with liraglutide (Victoza)? A.) 2-2.5% B.) 0.5-1% C.) 1-1.5% D.) 0.5-1.2% E.) 0.6-0.9%

B.) 0.5-1% EXPLANATION: You would expect a 0.5-1% reduction in A1c when liraglutide is used to treat type 2 diabetes. Choice E would be expected with sitagliptin. Choice A is more likely seen in insulin-treated patients.

What is the IBW of a female patient whose height is 5 ft 8 in? A.) 68 kg B.) 64 kg C.) 150 lb D.) 121 lb E.) 53 kg

B.) 64 kg

Which equation is used to predict the stability of a drug product at room temperature from experiments at increased temperatures? A.) Stokes's equation B.) Arrhenius equation C.) Michaelis-Menten equation D.) Fick's equation E.) Noyes-Whitney equation

B.) Arrhenius equation Stability at room temperature can be predicted from accelerated testing data by the Arrhenius equation: log (k 2/k 1) = E a(T 2 T 1)/(2.303 RT 2 T 1), where k 2 and k 1 are the rate constants at the absolute temperatures T 2 and T 1, respectively; R is the gas constant; and E a is the energy of activation. Stokes's equation is used to determine the sedimentation rate of a suspension, whereas the Noyes-Whitney equation is used to determine the dissolution rate.

What is the recommended treatment for uncomplicated gonorrhea? A.) Ceftriaxone 125 mg IM once B.) Ceftriaxone 250 mg IM once plus azithromycin 1 g po once C.) Gemifloxacin 320 mg po once D.) Ceftriaxone 1g IM or IV every 24 hours until improvement, then cefixime 400 mg po twice daily to complete a total of at least 7 days of treatment E.) Treatment is not necessary in patients with uncomplicated gonorrhea.

B.) Ceftriaxone 250 mg IM once plus azithromycin 1 g po once Response A is not the appropriate dose of ceftriaxone in an adult. Response C is incorrect owing to increased resistance to fluoroquinolones. Response D is the treatment for disseminated gonococcal infections. Response E is incorrect because all cases of gonorrhea require treatment.

Which one of the following immunosuppressants interact with mycophenolate mofetil ? A.) Azathioprine B.) Cyclosporine C.) Everolimus D.) Tacrolimus E.) None of the above

B.) Cyclosporine Cyclosporine coadministration inhibits mycophenolic acid glucuronide excretion, thus interfering with mycophenolic acid enterohepatic recycling and reducing mycophenolic acid levels.

Alteplase (Activase) is an rDNA protein of which of the following types? A.) Hormone B.) Enzyme C.) Clotting factor D.) Chemokine E.) Cytokine

B.) Enzyme Alteplase (Activase) is an rDNA protein of the enzyme type.

Regarding stability and compatibility of parenteral nutrition formulations, which of the following statements is false? A.) Generally, phosphate should be added to the formulation first. B.) Iron dextran can be added to total nutrient admixtures (TNA), but it should not be added to two-in-one formulations. C.) Preparation of TNAs using dual-chambered bags can enhance the shelf-life of a parenteral nutrition formulation. D.) Calcium chloride should not be used in parenteral nutrition because of its high reactivity with phosphate.

B.) Iron dextran can be added to total nutrient admixtures (TNA), but it should not be added to two-in-one formulations. EXPLANATION: Iron dextran can be added to two-in-one parenteral nutrition formulations but should be avoided in TNAs.

Case-control studies are the same as randomized clinical trials except for which study design characteristic: A.)Evaluate causal relationship between study variables. B.) Prospective design. C.) Exposure to a test treatment or condition. D.) Inferred that results can be extrapolated to the general population of interest.

B.) Prospective design. Case-control studies are always retrospective. Case control studies do share commonality with randomized clinical trials in that they evaluate causal relationships between study variables, involve exposure to a test treatment or condition and may be able extrapolate findings to the general population.

All of the following clinical study design characteristics are consistent with a randomized clinical trials except: A.) Parallel design B.) Retrospective C.) Blinded D.) Controlled

B.) Retrospective Parallel designs, blinding and controls are all study design characteristics commonly utilized with randomized clinical trials. A retrospective design is used typically in observational studies and is the exception.

Which of the following would be an important drug interaction with Isentress? A.) Metoprolol B.) Tums C.) Citalopram D.) Sulfamethoxazole/trimethoprim E.) Truvada

B.) Tums Integrase inhibitors chelate with cations and should be separated from medications that contain them.

One of the most commonly reported adverse reactions with epoetin alfa and darbepoetin alfa is A.) nausea. B.) hypertension. C.) constipation. D.) anaphylaxis.

B.) hypertension. Hypertension is the most common adverse effect in patients receiving ESAs.

The two macrocytic anemias are A.) vitamin B12 deficiency and iron deficiency anemias. B.) vitamin B12 deficiency and folic acid deficiency anemias. C.) iron deficiency and folic acid deficiency anemias. D.) sickle cell anemia and anemia of chronic kidney disease. E.) iron deficiency and pernicious anemias.

B.) vitamin B12 deficiency and folic acid deficiency anemias. Both vitamin B12 deficiency anemia and folic acid deficiency anemia are macrocytic (large cell) anemias. Both iron deficiency anemia and sickle cell anemia are microcytic and hypochromic anemias.

Select the medication contraindicated in patients with documented sulfa allergy. A) Ultram B) Toradol C) Celebrex D) Aspirin E) Hyaluronic acid

C) Celebrex An allergy to sulfa is a contraindication to celecoxib.

UGT1A1*28 mutations have been shown to cause what change in pharmacokinetics? A) Decreased fluorouracil clearance B) Increased fluorouracil clearance C) Decreased irinotecan clearance D) Increased irinotecan clearance

C) Decreased irinotecan clearance Patients with homozygous mutations in the UGT1A1*28 allele are recommended by the FDA-approved labeling to reduce starting doses of UGT1A1*28 by one level (C). Patients with heterozygous mutations have reduced clearance, but dose reductions are not empirically recommended. UGT1A1*28 mediated glucuronidation is not important for fluorouracil clearance. In contrast, DPD deficiency can significantly reduce fluorouracil clearance (A, B, and D).

Select the receptor antagonists that should be administered with the alkylating agent dacarbazine. A) Temozolomide B) Interleukin-2 C) Ondansetron D) Cisplatin

C) Ondansetron Ondansetron is a 5-HT3 receptor antagonist used for nausea and vomiting associated with the highly emetogenic dacarbazine.

You need to compound 60 mL of an isotonic solution of boric acid. The solubility of boric acid is 1 g in 18 mL of water. The sodium chloride equivalent of boric acid is 0.5. How much boric acid should you weigh out? A.) 0.506 g B.) 0.900 g C.) 1.080 g D.) 1.640 g E.) 1.800 g

C.) 1.080 g It takes 0.54 g of sodium chloride to make 60 mL isotonic; 0.54 ÷ 0.5 = 1.08.

Which of the following is an osmotic concentration that is compatible with biological fluids? A.) 150 mOsmol/L B.) 450 mOsmol/L C.) 310 mOsmol/L D.) 908 mOsmol/L

C.) 310 mOsmol/L If you convert the isotonic solution of NaCl (0.9%) to osmotic pressure, you obtain the 310 mOsmol/L.

Which of the following statements is true? A.) Bronchitis is not associated with fever. B.) Pneumonia rarely causes productive cough. C.) Chest x-ray findings are absent in bronchitis. D.) Bronchitis rarely causes productive cough. E.) The most common cause of pneumonia is viral.

C.) Chest x-ray findings are absent in bronchitis. EXPLANATION: Bronchitis usually has a clear chest x-ray. Both pneumonia and bronchitis are associated with productive cough and fever. The most common cause of bronchitis is viral infection. The most common cause of pneumonia is bacterial infection.

Which of the following conditions occurs with long-term combination hormone replacement therapy, according to the Women's Health Initiative? A.) Increased risk of colon cancer B.) Decreased risk of cardiovascular events C.) Decreased risk of hip fracture D.) Decreased risk of breast cancer

C.) Decreased risk of hip fracture EXPLANATION: The Women's Health Initiative concluded that combination hormone replacement therapy (progesterone plus estrogen) was associated with an increased risk of cardiovascular events and breast cancer, and a decreased risk of colon cancer and hip fracture. The estrogen-only study had different findings.

Which of the following is a common side effect associated with unfractionated heparin? A.) Hypokalemia B.) Hypoglycemia C.) Ecchymosis D.) Nausea E.) Hyponatremia

C.) Ecchymosis Minor bleeding and bruising are common side effects of heparin therapy. Other common areas for bleeding are the urogenital and gastrointestinal tracts.

Which anti-infective agent can cause congestive heart failure? A.) Linezolid B.) Chloramphenicol C.) Itraconazole D.) Anidulafungin E.) Penicillin

C.) Itraconazole EXPLANATION: Itraconazole has been reported to cause congestive heart failure.

Which of the following medications can cause systemic lupus erythematosus? A.) Cyclophosphamide B.) Prednisone C.) Procainamide D.) Azathioprine

C.) Procainamide Procainamide is a Class 1A antiarrhythmic agent which may cause systemic lupus erythematosus especially in patients that are slow acetylators.

Which of the following is associated with cognitive dysfunction? A.) Lamotrigine B.) Phenytoin C.) Topiramate D.) Gabapentin E.) Felbamate

C.) Topiramate Two anticonvulsants are associated with changes in cognitive function—phenobarbital and topiramate

The major adverse effect with the use of triptans for migraine headache treatment is A.) dependence. B.) muscle weakness. C.) chest pain. D.) ischemia. E.) diarrhea.

C.) chest pain. A well-recognized adverse effect of the triptans is angina-like pain.

First-degree atrioventricular heart block can be categorized as a disorder of A.) automaticity. B.) reentry. C.) conduction. D.) increased ventricular excitation. E.) slowed sinus node firing.

C.) conduction.

Which of the following T-scores is the bone mineral density (BMD) threshold value in the diagnosis of osteoporosis? A.) T-Score < -0.5 B.) T-Score < -1.0 C.)T-Score < -1.5 D.)T-Score < -2.5 E.) T-Score < -3.0

Correct answer: D The T-score shows the standard deviation of an individual's bone mineral density from the mean value of healthy young white women. The lower the score, the lower the bone mineral density.Both the World Health Organization and the American Association of Clinical Endocrinologists use aT-score of -2.5 or below as the definition of osteoporosis. Answers A & E: Both the World Health Organization and American Association of Clinical Endocrinologists use a T-score of -2.5 or below as the definition of osteoporosis.Answers B & C: Both the World Health Organization and American Association of Clinical Endocrinologists use a T-score of -2.5 or below as the definition of osteoporosis. A T-score between -1 to -2.5 is consistent with osteopenia.Bottom Line: A T-score of ≤ -2.5 is consistent with the diagnosis of osteoporosis, while a T-score between -1 to -2.5 is consistent with osteopenia.

Sancuso transdermal patch delivers antiemetic activity for: A) 12 hours B) 24 hours C) 3 days D) 7 days

D) 7 days

Select the supportive care medication that is associated with fluid retention. A) Darbepoetin B) Peg-filgrastim C) Amifostine D) Oprelvekin

D) Oprelvekin Answer d is correct.Oprelvekin may cause fluid retention; therefore, use caution in heart failure and hypertension patients. Other significant cardiovascular events associated with oprelvekin include arrhythmia, pulmonary edema, and cardiac arrest. Answer a is incorrect. Darbepoetin is not associated with causing fluid retention. It has been associated with worsening outcomes in patients with cancer and should only be used in patients who have cancer that is considered to be incurable. Answer b is incorrect. Peg-filgrastim is not associated with causing fluid retention. The most common side effected attributed to CSFs, such as peg-filgrastim, is bone pain. Answer c is incorrect. Amifostine is not associated with causing fluid retention. The most notable side effects from amifostine are hypotension and nausea/vomiting.

SM is a 52-year-old man with a new diagnosis of metastatic adenocarcinoma of the lung. Complete pathologic review revealed the following profile: ALK negative, EGFR negative, BRAF V600E negative. The oncologist has recommended to start chemotherapy with carboplatin/pemetrexed. Which of the following monoclonal antibodies could be added to SM's chemotherapy regimen to help improve efficacy? A) Necitumumab B) Atezolizumab C) Cetuximab D) Pembrolizumab

D) Pembrolizumab Answer d is correct. Pembrolizumab has been shown to improve the efficacy of combination chemotherapy in the treatment of metastatic, non-squamous NSCLC. It has specifically been studied in combination with carboplatin and pemetrexed. Answer a is incorrect.Necitumumab has been shown to improve the efficacy of the cisplatin/gemcitabine in the treatment of squamous cell NSCLC. In clinical trials it was ineffective in the treatment of non-squamous NSCLC and had increased risk of side effects. Answer b is incorrect. Like pembrolizumab, atezolizumab targets the programmed cell death 1 pathway. It has not been shown to be effective as first-line chemotherapy in NSCLC and it has not been studied in combination with chemotherapy. Answer c is incorrect. Like necitumumab, cetuximab is a monoclonal antibody that inhibits EGFR. It has been studied in various lung cancer scenarios and has provided only slight benefit with increased toxicities. It is not recommended for the treatment of NSCLC in patients such as the one provided in the question.

Which antiemetic should be diluted when given intravenously to minimize extravasation potential? A) Droperidol B) Fosaprepitant C) Granisetron D) Promethazine

D) Promethazine Promethazine can cause serious tissue damage when extravasation occurs. Several organizations recommend diluting promethazine prior to injection to minimize the extravasation potential.

Select the brand name for eletriptan. A) Maxalt B) Zomig C) Ergomar D) Relpax

D) Relpax Answer d is correct. Relpax is the brand name for eletriptan. Answer a is incorrect. Maxalt is the brand name for rizatriptan. Answer b is incorrect. Zomig is the brand name for zolmitriptan. Answer c is incorrect. Ergomar is the brand name for ergotamine tartrate.

According to the package labeling for prasugrel, how many days should the drug be held prior to major surgery (i.e., CABG)? A.) 2 days B.) 14 days C.) 5 days D.) 7 days E.) Holding prasugrel is not necessary.

D.) 7 days The package labeling recommends holding prasugrel for 7 days prior to surgery to minimize the risk of bleeding.

How many milliliters of tween 80 solution containing 55% w/v of tween 80 should be mixed with 120mL of another tween 80 solution containing 20% w/v of tween 80 to make a solution containing 35% w/v of tween 80? A.) 1000mL B.) 600mL C.) 300mL D.) 90mL

D.) 90mL Use the allegation alternate method

Which prophylactic migraine treatment or therapy would be most appropriate for an expectant mother? A.) Ergomar B.) Feverfew C.) Diltiazem D.) Biofeedback E.) Treximet

D.) Biofeedback EXPLANATION: Nonpharmacologic therapies are always preferred as first-line therapies in pregnancy and lactation. Biofeedback is advantageous because it is a noninvasive procedure. However, a drawback is that it may take multiple sessions to be effective. Acetaminophen can be safely used in pregnancy.

The modified or updated Schwartz equation offers an advantage over the traditional Schwartz equation for calculating creatinine clearance in: A.) Neonates within the first week of life B.) Infants with normal renal function C.) Infants with mild renal insufficiency D.) Children with moderate-severe kidney disease E.) Dialysis patients with end stage kidney disease

D.) Children with moderate-severe kidney disease The modified or updated Schwartz equation, formally referred to as the Bedside Chronic Kidney Disease in Childhood (CKiD) equation, may be a better estimate of GFR in pediatric patients with moderate-severe kidney disease.

Which is a characteristic of Poloxamer 407? A.) Frequently used as a levigating agent B.) Oily phase in a PLO gel C.) Acts as a preservative D.) Decreases in viscosity when refrigerated E.) When in solution, amber in color

D.) Decreases in viscosity when refrigerated Poloxamer 407 is reverse thermal; it is a liquid when refrigerated and a gel at room temperature.

Which one of the following medications inhibits the activation of mTOR (mammalian target of rapamycin) ? A.) Azathioprine B.) Belatacept C.) Cyclosporine D.) Everolimus E.) Mycophenolate mofetil

D.) Everolimus Everolimus inhibits mTOR.

A 53-year-old female (body weight = 65 kg) with hypertension and hypercholesterolemia is seen in the outpatient nephrology clinic for evaluation of kidney disease progression. Her current blood pressure is 156/82 mmHg, SCr is 2.6 mg/dL (stable for the past 4 months), BUN is 44 mg/dL, and urinary albumin excretion rate is 600 mg/day. Her medications are enalapril 20 mg/day × 1 year and simvastatin 20 mg daily × 2 years. According to this patient's estimated creatinine clearance, she would be classified in which of the following KDIGO categories of chronic kidney disease? A.) G1 B.) G2 C.) G3a D.) G4

D.) G4 This patient's estimated creatinine clearance determined using the Cockcroft-Gault equation is 26 mL/min, classified as category G4 CKD (GFR 15-29 mL/min/1.73 m2), with the result multiplied by 0.85 for a female. Note: The estimated GFR determined using the Modification of Diet in Renal Disease equation is 25 mL/min/1.73 m2.

L. W. is a 48-year-old white female who has been experiencing increased anxiety and excessive worry for the past 2 months. She complains of chronic muscle tension, headaches, insomnia, and GI problems. Which of the following is the most likely cause of her symptoms? A.) Obsessive-compulsive disorder B.) Panic disorder C.) Social anxiety disorder D.) Generalized anxiety disorder E.) Simple phobia

D.) Generalized anxiety disorder Explanation: Although all are anxiety disorders, generalized anxiety disorder commonly manifests itself with associated GI symptoms, fatigue associated with insomnia, and headaches. L. W. does not appear to be consumed with an obsessive thought or compulsive behavior. Panic disorder will present suddenly with more severe symptoms of fear, palpitations, and hyperventilation and will usually resolve within 30 minutes.

The following medications are considered to be DMARDs except A.) Sulfasalazine B.) Enbrel C.) Arava D.) Mobic

D.) Mobic Mobic, also known as meloxicam, is an NSAID, not a DMARD.

R. J. is a 48-year-old black female interested in hormone replacement therapy to control her vasomotor symptoms. She has had a total abdominal hysterectomy and has a history of breast cancer. Which of the following medications is most appropriate for her? A.) Phytoestrogens B.) Premarin C.) Raloxifene D.) None of the above

D.) None of the above EXPLANATION: Both hormone replacement therapies, such as Premarin, and phytoestrogens are contraindicated in patients with a history of breast cancer. Raloxifene may decrease the risk of breast cancer, but it will not treat vasomotor symptoms. Hot flashes are an adverse effect of raloxifene.

Which of the following immunosuppressant drugs has the longest elimination half-life? A.) Azathioprine B.) Corticosteroids C.) Mycophenolate sodium D.) Sirolimus E.) Tacrolimus

D.) Sirolimus Sirolimus has the longest elimination half-life of the immunosuppressants listed.

J.M. is a 72 y.o. male who has to urinate several times during the day as well as getting up frequently during the night. Of the following which would be the best initial drug therapy for his condition? A.) Avodart B.) Darifenacin C.) Tolterodine D.) Terazosin E.) Vesicare

D.) Terazosin The symptoms are consistent with over-flow incontinence and would benefit from an alpha adrenergic receptor blocking agent such as terazosin. Avodart (dutasteride) would be useful in decreasing prostate size which could improve symptoms but requires many weeks for benefits and is usually begun if alpha adrenergic blockade is insufficient. The others listed are all anticholinergic and would decrease bladder contractions and could actually worsen over-flow incontinence.

Which of the following may cause osteoporosis? A.) Oral contraceptives B.) Metformin C.) H2 antagonists D.) Thiazolidinediones

D.) Thiazolidinediones Explanation: EXPLANATION: Epidemiologic trials have suggested that thiazolidinediones may increase fracture risk.

Which of the following oral chemotherapeutic agent has been used in the treatment of unresectable melanoma? A) Capecitabine B) Lapatinib C) Erlotinib D) Procarbazine E) Vemurafenib

E) Vemurafenib Answer e is correct.Vemurafenib (Zelboraf™) is a BRAF kinase inhibitor FDA approved for treatment of metastatic or unresectable melanoma with BRAF V600E mutation (+) disease. Answer a is incorrect. Capecitabine (Xeloda™) is FDA approved for metastatic colorectal and breast cancer, and as adjuvant for Stage III colon cancer. Answer b is incorrect.Lapatinib (Tykerb™) is FDA approved for advanced or metastatic breast cancer in combination with Capecitabine. Answer c is incorrect. Erlotinib (Tarceva™) is FDA approved for advanced or metastatic nonsmall cell lung and pancreatic cancer. Answer d is incorrect.Procarbazine (Matulane™) is FDA approved for Hodgkin disease as part of the MOPP chemotherapy regimen which contains chemotherapy with Mechlorethamine,Oncovin™, Procarbazine, and Prednisone.

A DATA waived practitioner is a practitioner who wishes to prescribe or dispense controlled substances in schedules III, IV, or V for addiction treatment. Which of the following organizations grants this waiver? A.) Department of Justice B.) Drug Enforcement Agency C.) Consumer Product Safety Commission D.) Food and Drug Administration E.) Center for Substance Abuse Treatment within the Substance Abuse and Mental Health Services Administration

E.) Center for Substance Abuse Treatment within the Substance Abuse and Mental Health Services Administration

Which of the following medications is recommended in a patient with warm and wet acute decompensated heart failure (ADHF) that is not responding to IV furosemide? A.) Oral furosemide B.) Amlodipine C.) Metoprolol succinate D.) Eplerenone E.) Milrinone

E.) Milrinone Intravenous inotrope therapy can improve urine output and reduce volume overload in patients not responding to IV loop diuretics. Either milrinone or dobutamine can be effective.

The onset of action of sodium nitroprusside is A.) 20-30 minutes. B.) 10-20 minutes. C.) 5-10 minutes. D.) 1-2 minutes. E.) immediate.

E.) immediate. EXPLANATION: Sodium nitroprusside, which can be used for most types of hypertensive emergencies, has an immediate onset with a 1-2 minute duration of action.

What medication is used to reverse the effects of heparin?

Protamine

A new antibiotic is being studied and is available in PO and IV dosage forms. After administration of a single 250 mg IV dose, pharmacokinetic studies have shown that the area under the concentration curve (AUC) of this dose when given as an IV bolus is 540 mg x h/L. After administration of a single 500 mg PO dose, the AUC was 374 mg x h/L. What is the bioavailability of the PO capsule?

0.35

RT is a 65-year-old woman who presents with PSVT with a regular rhythm. RT is experiencing mild symptoms and was given unilateral carotid sinus massage with no success. If the patient has a narrow QRS interval, what medication(s) is(are) first-line agent(s) to treat this patient? Select ALL that apply. A) Adenosine B) Verapamil C) Procainamide D) Amiodarone

A) Adenosine B) Verapamil Answer a is correct. Adenosine is indicated for the treatment of patients with PSVT with a narrow QRS interval and a regular rhythm. Answer b is correct. Verapamil is indicated for the treatment of patients with PSVT with a narrow QRS interval and a regular rhythm. Answer c is incorrect.Procainamide is used for the treatment of patients with PSVT and an irregular rhythm. Answer d is incorrect. Amiodarone is used for patients with PSVT with a wide QRS interval

Amiodarone requires substantial safety monitoring during long-term therapy due to its numerous side effects. Which of the following is required to be routinely performed in a patient on long-term amiodarone therapy? A) Hepatic function panel B) Renal function panel C) Erythrocyte sedimentation rate D) B-type natriuretic peptide (BNP) levels

A) Hepatic function panel Answer a is correct. Amiodarone can cause elevations in liver function test (LFT) and may require dose adjustment in patients with liver impairment. Hepatic function panels should be monitored routinely. Answer b is incorrect. Amiodarone is not appreciably eliminated by the kidneys. Answer c is incorrect. Amiodarone is not known to affect sedimentation rates. Answer d is incorrect. Amiodarone does not affect BNP levels, which is considered a measurement of HF status and response to ventricular stretch.

Select the treatment goal(s) for PAD in patients with intermittent claudication. Select all that apply. A) Increase maximal walking distance B) Increase duration of walking C) Increase amount of pain-free walking D) Decrease preload E) Decrease afterload

A) Increase maximal walking distance B) Increase duration of walking C) Increase amount of pain-free walking

Which of the following may be used to treat elevated blood pressure in acute stroke patients who have concomitant renal dysfunction? Select all that apply. A) Labetalol B) Nicardipine C) Sodium nitroprusside D) Perindopril E) Indapamide

A) Labetalol B) Nicardipine Answers a and b are correct. Both labetalol and nicardipine may be initiated in patients with renal dysfunction and they both are the preferred medications to use for blood pressure lowering in acute stroke. Answer c is incorrect. Sodium nitroprusside's metabolite accumulates in renal insufficiency and can lead to cyanide toxicity. Answer d is incorrect.Perindopril is not indicated for blood pressure treatment in the acute stroke setting; however, ACE inhibitors may be used for blood pressure control starting after the initial 24 hours. Answer e is incorrect.Indapamide is not indicated for blood pressure treatment in the acute stroke setting; however, thiazide-like diuretics may be used for blood pressure control starting after the initial 24 hours.

Which of the following is referred to as an "inodilator," having both inotropic and vasodilatory properties? A) Milrinone B) Dobutamine C) Nesiritide D.) Nitroprusside

A) Milrinone Answer a is correct. Milrinone decreases the breakdown of cyclic adenosine monophosphate (cAMP) in cardiac tissue, resulting in an increase in cardiac contractility and output. By increasing cAMP in vascular smooth muscle, systemic and pulmonary vascular resistance are reduced, thus milrinone is often referred to as an "inodilator" (inotrope and vasodilator). Answer b is incorrect. Dobutamine is an inotrope. While dobutamine stimulates β-2 receptors in the periphery causing mild vasodilation, it also stimulates alpha receptors, which counteracts with mild vasoconstriction. Therefore, the decrease in SVR that occurs with dobutamine is a reflexive response to the increase in cardiac output. Answers c and d are incorrect.Nesiritide (Answer c) and nitroprusside (Answer d) are potent arterial and venous vasodilators. These agents dilate the arterial vessels (decrease in SVR) and cause a reflex increase in cardiac output.

Select the choice that ranks the agents from shortest to longest onset of action in the treatment of heartburn/gastroesophageal reflux disease (first option = shortest onset, last option = longest onset). A. Antacids → histamine 2 receptor antagonists → proton pump inhibitors B. Histamine 2 receptor antagonists → proton pump inhibitors → antacids C. Histamine 2 receptor antagonists → antacids → proton pump inhibitors D. Antacids → proton pump inhibitors → histamine 2 receptor antagonists E. Proton pump inhibitors → antacids → histamine 2 receptor antagonists

A. Antacids → histamine 2 receptor antagonists → proton pump inhibitors

Which of the following statements is the most accurate description of the cause of bone loss related to aromatase inhibitor use? A. Aromatase inhibitors inhibit aromatase, which is responsible for the peripheral conversion of androgens to estrogens, resulting in bone loss due to estrogen deficiency. B. Aromatase inhibitors inhibit aromatase, which is responsible for the peripheral conversion of estrogens to androgens, resulting in bone loss due to androgen deficiency. C. Aromatase inhibitors inhibits aromatase, which is responsible for the central conversion of androgens to estrogens, resulting in bone loss due to estrogen deficiency. D. Aromatase inhibitors inhibit aromatase, which is responsible for the central conversion of estrogens to androgens, resulting in bone loss due to androgen deficiency. E. Aromatase inhibitors inhibit aromatase, which is responsible for the peripheral conversion of estrogen to progesterone, resulting in bone loss due to estrogen deficiency.

A. Aromatase inhibitors inhibit aromatase, which is responsible for the peripheral conversion of androgens to estrogens, resulting in bone loss due to estrogen deficiency.

Which of the following is an example of bacteriostatic activity of antibiotics? A. Limiting bacterial growth with use of protein synthesis inhibitors such as aminoglycoside antibiotics B. Inhibiting cell wall synthesis with use of beta lactam antibiotics C. Maximizing the time that serum concentrations of beta lactam antibiotics remain above the minimum inhibitory concentration during dosing interval D. Maximizing the peak concentrations of aminoglycoside antibiotics to more than 10 times above the minimum inhibitory concentration E. Monitoring serum concentrations to maximize efficacy and minimize toxicity of aminoglycoside antibiotics

A. Limiting bacterial growth with use of protein synthesis inhibitors such as aminoglycoside antibiotics Antibiotics typically are classified as bacteriostatic or bactericidal in terms of their activity. Bacteriostatic antibiotics prevent the growth of microbes by inhibiting reproduction without necessarily killing them. This can be achieved in numerous ways, such as interfering with protein production, DNA replication, or any aspects of cellular metabolism. Antibiotics that are bacteriostatic require the patient to have an active immune system in order to remove the already-present bacteria, as these antibiotics halt the future progression of the bacterium. One example of bacteriostatic antibiotics are aminoglycosides. Bactericidal antibiotics result in immediate bacterial death. Beta-lactams are an example of bactericidal antibiotics. This type of activity is preferred in life-threatening disease states that require immediate bacterial death. Answer B: Inhibiting cell wall synthesis would result in bacterial death characteristic of bactericidal antibiotics, not bacteriostatic. Answer C: This is a true statement, however, it is not indicative of bacteriostatic or bactericidal activity. This is the definition of a time-dependent antibiotic. Beta lactams are examples of time-dependent antibiotics. Answer D: This is a true statement, however, it is not indicative of bacteriostatic or bactericidal activity. This is the definition of a concentration-dependent antibiotic such as aminoglycosides. Answer E: This is a true statement, however, is not indicative of bacteriostatic or bactericidal activity. This is a characteristic of monitoring parameters for antibiotic dosing in patients. Bottom Line: Bacteriostatic antibiotics prevent the growth of microbes by inhibiting reproduction without necessarily killing them. This can be achieved in numerous ways such as interfering with protein production, DNA replication, or any aspects of cellular metabolism.

Nitroprusside has the potential to cause cyanide toxicity with prolonged use. Which of the following acid-base disorders might suggest cyanide toxicity? A. Metabolic acidosis B. Metabolic alkalosis C. Respiratory acidosis D. Respiratory alkalosis E. Mixed acid-base disorder

A. Metabolic acidosis

Which of the following produces a significant pharmacokinetic interaction when administered with azathioprine? A.) Allopurinol B.) Diflucan C.) Sirolimus D.) Probenecid E.) A and D

A.) Allopurinol Explanation: Xanthine oxidase is responsible for the elimination of the active metabolites of azathioprine. Concomitant use of allopurinol with azathioprine results in significantly increased azathioprine-induced toxicity. Reduce the dose of azathioprine by 65-75%.

Which of the following statements describes the use of In-111 oxine? A.) Autologous white blood cells are used. B.) It is ideal for leukopenic patients with a fever. C.) Radiolabeling occurs in vivo. D.) It provides palliative therapeutic benefit.

A.) Autologous white blood cells are used. Explanation: In-111 oxine is not useful for leukopenic patients, and radiolabeling occurs in vitro. It is a diagnostic agent rather than a therapeutic radiopharmaceutical.

Which of the following has been associated with the use of valproic acid in children under 2 years of age? A.) Fatal hepatotoxicity B.) Permanent vision loss C.) Oligohidrosis D.) Toxic epidermal necrosis E.) None of the above

A.) Fatal hepatotoxicity EXPLANATION: Fatal hepatotoxicity has been reported in patients of all ages but is most likely to occur in patients who are under the age of 2 years who also have severe epilepsy and are receiving numerous anticonvulsants. For this reason, most pediatric neurologists do not use this medication in patients under 2 years of age.

Which of the following statements best characterize Aminoglycoside antimicrobial activity? I. Active against most aerobic gram-negative bacteria. II. Active against most anaerobic gram-negative bacteria. III. Active against most fungal isolates A.) I only B.) II only C.) III only D.) I & II E.) II & III

A.) I only Explanation: Aminoglycosides are active against most aerobic gram-negative and selected aerobic gram-positive bacteria. They have no activity against anaerobic bacteria, or fungi.

A patient asks which of his medications could cause ED. A review of his medication list reveals that his ED symptoms are most likely caused by which of the following? A.) Metoprolol B.) Docusate C.) Amiodarone D.) Atorvastatin E.) Enalapril

A.) Metoprolol Explanation: β-blockers are associated with symptoms of erectile dysfunction.

What is the laboratory monitoring goal for argatroban? A.) PTT 1.5-3.0 times control B.) PTT 1.5-2.5 times control C.) INR 2.0-3.0 D.) Anti-Xa 0.6-1 units/mL E.) Thrombocytopenia monitoring only

A.) PTT 1.5-3.0 times control Explanation: This recommendation is in the manufacturer's prescribing information.

Your pharmacy sent several expired vials of meperidine to a registered reverse distributor for destruction. Which of the following statements is correct concerning this transaction? A.) The reverse distributor issued DEA Form 222 and DEA Form 41. B.) The pharmacy issued DEA Form 222, and the reverse distributor issued DEA Form 41. C.) The pharmacy issued DEA Form 222 and DEA Form 41. D.) The reverse distributor issued DEA Form 222, and the pharmacy issued DEA Form 41. E.) None of the above is correct.

A.) The reverse distributor issued DEA Form 222 and DEA Form 41. Explanation: The reverse distributor must issue to the DEA both an official order form (DEA Form 222) and a DEA Form 41 (Registrants Inventory of Drugs Surrendered). The applicable section of the DEA Pharmacist's Manual is Section IV.

Before maraviroc can be part of an ART regimen, which lab test should be performed to determine if the patients virus would be susceptible to it? A.) Trofile B.) Genotype C.) Viral load D.) CD4 count E.) Complete metabolic panel

A.) Trofile Explanation: A trofile is necessary to determine the patients tropism which is the type of chemokine receptors they have on their CD4 cells. Only receptors that are of the CCR5 type can be blocked by maraviroc allowing for its use as part of an effective ART regimen.

Which of the following medications is an oral option for the treatment of MRSA cellulitis? A.) Zyvox 600 mg bid B.) Vancocin 250 mg qid C.) Cubicin 500 mg daily D.) Augmentin 875 mg bid

A.) Zyvox 600 mg bid Explanation: EXPLANATION: Zyvox (linezolid) 600 mg po q12h is an appropriate treatment for MRSA cellulitis. Cubicin (daptomycin) and Vancocin (vancomycin) must be given IV to treat systemic infections. Augmentin (amoxicillin/clavulanate potassium) is inactive against MRSA.

The American Diabetes Association recommends annual urine assays to screen for the presence and quantity of _____________ in the urine. A.) albumin B.) glucose C.) insulin D.) ketones E.) nitrites

A.) albumin Explanation: Albuminuria screening is recommended on an annual basis for all patients with diabetes mellitus. The preferred method is a spot urine measurement of the albumin-to-creatinine ratio to better quantify the degree of albuminuria without needing a timed urine collection. Glycosuria (glucose in urine) can be measured with test strips for home use. However, they are poorly sensitive to hyperglycemia compared with fingerstick blood glucose measurement. Testing for ketones in urine is primarily recommended for patients with type 1 diabetes prior to exercise. Nitrites are produced by certain bacteria and may be indicative of a urinary tract infection.

When preparing an IV formulation of amphotericin B deoxycholate (desoxycholate), the lyophilized amphotericin B powder must first be reconstituted with sterile water. What type of IV fluid must the reconstituted amphotericin B be placed in for IV administration? A) 0.9% sodium chloride B) 5% dextrose in water C) Lactated Ringer's solution D) 0.45% sodium chloride

B) 5% dextrose in water

Which of the following should be assured prior to administering intravenous inotropes and vasodilators? A) Adequate filling pressures with a PCWP 6 to 12 mm Hg B) Adequate filling pressures with a PCWP >15 mm Hg C) Adequate filling pressure with an SVR >1200 dyne/s/cm5 D) Adequate filling pressure with an SVR >1500 dyne/s/cm5

B) Adequate filling pressures with a PCWP >15 mm Hg Answer b is correct. Adequate filling pressures as reflected by a PCWP 15 to 18 mm Hg (Answer b) should be assured prior to safely administering IV inotropes or vasodilators. Answer a is incorrect. While a PCWP 6 to 12 mm Hg would suggest adequate filling pressures in a patient with normal cardiac function, patients with HFrEF require a higher filling pressure to optimize cardiac output (starling curve). Answers c and d are incorrect. An SVR measurement (Answers c and d) reflects vascular tone or afterload and not filling pressure.

What lipid-lowering medication(s) should be adjusted in a patient with renal impairment? A) Atorvastatin B) Gemfibrozil C) Ezetimibe D) Cholestyramine E) Niacin

B) Gemfibrozil Answer b is correct. Fibrates require renal adjustment in patients with mild to moderate renal impairment. Fibrates are also contraindicated in patients with severe renal impairment, defined by manufacturers as <30 mL/min for fenofibrate and <10 mL/min for gemfibrozil. Answer a is incorrect. Atorvastatin and fluvastatin (≤40 mg) are confirmed to be safe in this population. Atorvastatin has even been shown safe in patients with diabetes on dialysis. Although statins at currently approved doses are not considered nephrotoxic, all other drugs in this class should be given at lower doses to reduce myopathy risk. Answer c is incorrect.Ezetimibe does not require adjustment. Answer d is incorrect. Cholestyramine and other BAS do not require adjustment. Answer e is incorrect. There are no specific renal adjustments for niacin, although niacin should be used with caution in this population.

Which one of the following statements is most correct regarding macrosubstrates found in PN solutions? A) Dextrose and amino acids can be mixed together by the manufacturer, heat sterilized, and then shipped to hospitals. B) Glycerin and amino acids can be mixed together by the manufacturer, heat sterilized, and then shipped to hospitals. C) Premixed PN solutions containing dextrose, amino acid, and fat all in a single chamber are available from various manufacturers. D) Combinations of dextrose, fat and amino acid solution may be used within seven days if stored in a refrigerator.

B) Glycerin and amino acids can be mixed together by the manufacturer, heat sterilized, and then shipped to hospitals. Because glycerin does not contain a carbonyl group, the Maillard reaction does not occur when combinations of glycerin and amino acids are heat sterilized. The commercially available product called ProcalAmine contains these components in a premixed formulation.

KW is a 53-year-old man with HF (NYHA class I) receiving furosemide 40 mg twice daily, lisinopril 10 mg daily, metoprolol succinate 50 mg daily, digoxin 0.125 mg daily, and spironolactone 25 mg daily. During a routine clinic visit today, pertinent findings include: BP 120/80 mm Hg, HR 70 beats/min, RR 14, K+ 5.1 mmol/L, BUN 35 mg/dL, and creatinine 1.2 mg/dL (baseline). Which of the following is the most appropriate change to optimize KW's medical regimen? Select all that apply. A) Increase ACE inhibitor dose B) Increase β-blocker dose C) Add ivabradine D) Increase spironolactone dose

B) Increase β-blocker dose Explanation: Answer b is correct. For HF management, ACE inhibitor and β-blocker therapy should be titrated to target doses associated with improved outcomes in clinical trials. This patient has adequate blood pressure to increase either therapy. In addition, HR is adequate to further titrate β-blocker therapy. Uptitration of β-blocker therapy is the safest medication change for this patient. Answer a is incorrect. ACE inhibitors are associated with dose-related hyperkalemia and this patient already has borderline high serum potassium. Answer c is incorrect. Although the patient's HR is ≥70 beats/min, β-blocker therapy is not yet at target dose and the patient does not have any evidence of intolerance that might preclude dose uptitration. Because β-blockers improve mortality (whereas ivabradine primarily reduces hospitalizations), ivabradine should be reserved for patients who remain symptomatic with a HR ≥70 beats/min despite maximally tolerated β-blocker therapy. Answer d is incorrect. Aldosterone antagonists such as spironolactone are associated with hyperkalemia. Thus, the spironolactone dose should not be increased and may even need to be reduced to 12.5 mg daily if hyperkalemia persists or worsens.

Potential side effect(s) of dronedarone is (are): (select ALL that apply) A) Gingival hyperplasia B) Increased serum creatinine C) QT prolongation D) Hypothyroidism

B) Increased serum creatinine C) QT prolongation

What is the term defined by the following: When an adverse drug event is misinterpreted as a new medical condition, in which the provider prescribes an additional medication to treat the side effect of the original medication? A) Prescribing downfall B) Prescribing cascade C) Downstream effect D) Provider cascade

B) Prescribing cascade

SD is a 54-year-old man with NYHA class III HF due to nonischemic cardiomyopathy. His past medical history is notable for moderate asthma since childhood and HTN. Current medications include salmeterol, one inhalation twice daily; fluticasone 88 mcg, inhaled twice daily; furosemide 80 mg twice daily; enalapril 20 mg twice daily; and spironolactone 25 mg daily. Which of the following medication changes may provide further mortality benefit for SD once stabilized on β-blocker therapy? A) Addition of digoxin 0.125 mg daily. B) Substitution of sacubitril/valsartan 49 mg/51 mg for enalapril 20 mg twice daily. C) Addition of valsartan 160 mg twice daily. D) Addition of amlodipine 5 mg daily.

B) Substitution of sacubitril/valsartan 49 mg/51 mg for enalapril 20 mg twice daily. Answer b is correct. The combination of sacubitril and valsartan demonstrated a significant mortality benefit over ACE inhibitors in patients with symptomatic HF despite receiving optimal HF therapy with β-blockers and ACE inhibitors. Answer a is incorrect. Digoxin improves symptoms and reduces hospitalization, but does not confer a mortality benefit. Answer c is incorrect.Valsartan reduces HF exacerbations in combination with background ACE inhibitor therapy, but not all-cause mortality when added to an ACE inhibitor. Answer d is incorrect. Amlodipine has neutral effects on mortality in HF patients.

Which of the following are accurate counseling points for patients starting anakinra therapy for rheumatoid arthritis (RA)?(Please select ALL that apply) A. Inject 100 mg subcutaneously weekly on the same day each week B. Allow the solution to warm to room temperature prior to use C. Injection sites include outer area of upper arms, abdomen, thighs, or upper outer buttocks D. It can be used without dosing adjustment in a patient with a CrCl of 45 mL/min E. It is contraindicated in moderate to severe hepatic impairment

B. Allow the solution to warm to room temperature prior to use C. Injection sites include outer area of upper arms, abdomen, thighs, or upper outer buttocks D. It can be used without dosing adjustment in a patient with a CrCl of 45 mL/min

Voriconazole and tacrolimus significantly interact with each other. Voriconazole increases tacrolimus concentrations because voriconazole is a: A. CYP2C19 inhibitor and tacrolimus is a CYP2C19 substrate. B. CYP3A4 inhibitor and tacrolimus is a CYP3A4 substrate. C. CYP3A4 inducer and tacrolimus is a CYP3A4 substrate. D. CYP2D6 inhibitor and tacrolimus is a CYP2D6 substrate. E. UGT1A2 inhibitor and tacrolimus is a UGT1A2 substrate.

B. CYP3A4 inhibitor and tacrolimus is a CYP3A4 substrate.

Which of the following provides reliable information about the risk and safety of medications during pregnancy and breastfeeding? A. TOXNET B. Motherisk C. LactMed D. VAERS E. What To Expect

B. Motherisk The Motherisk Program is a program run by The Hospital for Sick Children. It is a teratogen information service that provides up-to-date information about the risk and safety of medications and other exposures during pregnancy and breastfeeding. The program also conducts research and education in this field. In addition to the information online, The Motherisk Program offers counselors available by phone to provide information to women, their partners, and health care providers. Answer A: TOXNET is a database searching resource that provides information on toxicology, hazardous chemicals, environmental health, and toxic releases. Answer C: The LactMed database contains information on drugs and other chemicals in breastfeeding mothers. It includes information on the levels of substances in breast milk and infant blood, as well as the possible adverse effects for nursing infants. This database also provides suggestions for therapeutic alternatives, where appropriate. All data are derived from the scientific literature, fully referenced, and peer-reviewed to assure scientific validity and currency. The database is updated monthly. Answer D: The Vaccine Adverse Event Reporting System (VAERS) is a national vaccine safety surveillance program. It is sponsored by the Food and Drug Administration (FDA) and the Centers for Disease Control and Prevention (CDC). It is unrelated to the risk and safety of medications during pregnancy and breastfeeding. Answer E: What to Expect is a website that contains various information about pregnancy such as getting pregnant, breastfeeding, and vaccinations. This website does not contain a database about the risk and safety of medications during pregnancy and breastfeeding, but rather has articles about various pregnancy-related topics. These articles are not peer-reviewed to assure scientific validity, and may not be up to date. This website states, "The material on this website is provided for educational purposes only and is not to be used for medical advice, diagnosis or treatment, or in place of therapy or medical care."

PB, a 62-year-old man, is picking up a prescription for sildenafil 100 mg/day PO as needed before sexual activity. His other medications include metoprolol tartrate 25 mg twice daily and amlodipine 10 mg/day. Which of the following counseling point(s) should be reviewed with PB?(Select ALL that apply) A. If ineffective, you may repeat the dose once after 1 hour B. This should not be used with nitrates, as the combination may cause a dangerous drop in blood pressure C. Avoid grapefruit juice because it can increase the levels of this medication in your body D. This can lower your blood pressure and you are taking other medications that lower blood pressure; make sure to report any dizziness or falls E. It may rarely cause prolonged erections lasting more than 4 hours; seek immediate medical attention if you experience an erection persisting longer than 4 hours

B. This should not be used with nitrates, as the combination may cause a dangerous drop in blood pressure C. Avoid grapefruit juice because it can increase the levels of this medication in your body D. This can lower your blood pressure and you are taking other medications that lower blood pressure; make sure to report any dizziness or falls E. It may rarely cause prolonged erections lasting more than 4 hours; seek immediate medical attention if you experience an erection persisting longer than 4 hours

Which of the following scenarios is an indication for acute hemodialysis in a patient with end stage kidney disease? A. Elevated serum creatinine B. Uremia C. Hypokalemia D. Normotension E. Tachycardia

B. Uremia Acute indications for hemodialysis often can be remembered using the mnemonic device: AEIOU. This includes acidosis, electrolyte abnormalities, ingestion of toxins, overload of fluid, and uremia. Although serum creatinine is commonly used to estimate glomerular filtration rate (GFR) in patients with residual kidney function, it is not a targeted efficacy goal for patients who receive hemodialysis. Blood urea nitrogen accumulation can lead to dermatologic, cardiotoxic, and neurologic effects that increase mortality in patients with end stage renal disease. Nephrologists often target Kt/V in dialysis recipients, which is a marker of uremic removal efficacy. Other targets include control of hyperkalemia and peripheral edema. Safety monitoring often involves monitoring for hypotension, electrolyte shifts, and health of the dialysis access port. Answer A: Although serum creatinine is commonly used to estimate GFR in patients with residual kidney function, it is not a targeted efficacy goal for patients who receive hemodialysis. In contrast, nephrologists focus on uremia levels as an indication for acute dialysis due to the negative physiologic effects of accumulation. Answer C: One of the indications for acute hemodialysis includes electrolyte disturbances, however, this is primarily hyperkalemia (not hypokalemia). Patients with end stage kidney disease on hemodialysis are unable to clear potassium effectively, often leading to elevated serum potassium. Elevated potassium can lead to cardiac abnormalities in patients and should be treated as a medical emergency. Answer D: One of the main clinical symptoms of patients needing acute dialysis is fluid overload, which leads to hypertension in most patients. A patient who is normotensive does not have an acute indication for hemodialysis. Answer E: Heart rate is not an indication for acute hemodialysis. Patients can be tachycardic for several reasons outside of renal dysfunction. Bottom Line: Indications for acute hemodialysis include: acidosis, electrolyte abnormalities, ingestion of toxins, overload of fluid, and uremia.

Which of the following statements is true regarding the risk of venous thromboembolism with combined estrogen-progestin oral contraceptives? A. Risk of venous thromboembolism is lower with higher doses of ethinyl estradiol (>50 mcg daily) B. Venous thromboembolism risk is lower for contraceptives that contain a second-generation progestin compared to a first-generation progestin C. The risk of venous thromboembolism is lowest during the first year of combined estrogen-progestin oral contraceptive use D. The risk of venous thromboembolism is higher in patients with a lower body mass index E. Combined estrogen-progestin oral contraceptives are not associated with venous thromboembolism, only progestin-only contraceptives

B. Venous thromboembolism risk is lower for contraceptives that contain a second-generation progestin compared to a first-generation progestin Venous thromboembolism (VTE) is the most common vascular complication of combined estrogen-progestin oral contraceptive use. Patients who use these contraceptives are at a 2-fold to 4-fold higher risk of VTE than nonusers, although the absolute risk is low for both populations. There are several factors that can affect the VTE risk of contraceptives: - Risk of VTE is lower with lower doses of ethinyl estradiol (< 50 mcg daily) - The risk of VTE is highest during the first year of combined estrogen-progestin oral contraceptive use - VTE risk is lower for contraceptives that contain a second-generation progestin compared to a first-generation progestin - Obesity, age (>39 years old), and smoking further increase risk of VTE Answer A: Risk of VTE is lower with lower doses of ethinyl estradiol (< 50mcg daily). Answer C: The risk of VTE is highest during the first year of combined estrogen-progestin oral contraceptive use. Answer D: The risk of VTE is higher in obese patients compared to nonobese patients. Answer E: Combined estrogen-progestin oral contraceptives and progestin-only contraceptives are associated with increased risk of VTE. Bottom Line: There are several factors that can affect the venous thromboembolism (VTE) risk of combined estrogen-progestin oral contraceptives. Risk of VTE is higher with higher doses of ethinyl estradiol (>50 mcg daily), during the first year of use, with first-generation progestins compared to second-generation progestins, in obese patients, in older age (>39 years old), and with smoking.

The radionuclidic purity limit of Tc-99m Pertechnetate is: A.) 0.10microcuries of Mo-99/1mCi of Tc-99m at administration B.) 0.15microcuries of Mo-99/1mCi of Tc-99m at administration C.) 0.50microcuries of Mo-99/1mCi of Tc-99m at administration D.) 1microcuries of Mo-99/1mCi of Tc-99m at administration

B.) 0.15microcuries of Mo-99/1mCi of Tc-99m at administration Explanation: Reason: This is the limit per USP and package insert.

How many milliequivalents of magnesium sulfate are represented in 1 g of anhydrous magnesium sulfate (MgSO4) (mw = 120)? A.) 122 mEq B.) 16.67 mEq C.) 12 mEq D.) 10 mEq E.) 19 mEq

B.) 16.67 mEq

Which of the following drugs is used only for severe infections caused by multidrug-resistant Gram-negative pathogens because of a significant risk of nephrotoxicity? A.) Rifaximin B.) Colistin C.) Clindamycin D.) Fidaxomicin

B.) Colistin Explanation: Colistin is a polymyxin antibiotic that can be used for multidrug resistant Gram-negative pathogens such as P. aeruginosa and Acinetobacter spp. Its main limiting side effect is significant nephrotoxicity, and it should be reserved for severe or multidrug-resistant infections.

Which component of the immune system is associated with humoral responses in both innate and adaptive immunities? A.) B-lymphocyte (cell) B.) Complement C.) Cytotoxic T-lymphocyte (killer T-cell, CD8) D.) Helper T-lymphocyte (CD4) E.) Macrophage

B.) Complement Explanation: Complement has a role in the humoral response associated with both adaptive and innate immunity.

D.G. is a 40 year old white male who presents to your pharmacy with a prescription for hydrochlorothiazide (HCTZ) 25 mg daily and Lithium CR 450 mg twice daily. How should you counsel D.G. when using these two drugs together? A.) HCTZ can decrease lithium concentrations B.) HCTZ can increase lithium concentrations C.) Lithium can increase HCTZ concentrations D.) Lithium can decrease HCTZ concentrations E.) Taking both HCTZ and Lithium can decrease the concentrations of both medications

B.) HCTZ can increase lithium concentrations Explanation: Thiazide and loop diuretics have been shown to decrease the renal elimination of lithium, causing increased serum concentrations and could cause lithium toxicity. Patients taking thiazide or loop diuretics with lithium may require a reduced dose of lithium, more frequent serum concentration monitoring, and more frequent monitoring for signs and symptoms of lithium toxicity. NSAIDs, fluoxetine, and angiotensin-converting enzyme inhibitors have also been shown to increase lithium concentrations.

EP is a 43-year-old HIV-positive male who presents to your clinic for follow-up. He has a CD4+ count of 15 cells/mm3 and viral load of 51,508 copies/mL. He is currently being treated for active TB diagnosed 6 weeks ago. Current medications: Isoniazid 300 mg daily Rifabutin 150 mg daily Pyrazinamide 2,000 mg daily Septra DS daily Pyridoxine 50 mg daily Combivir 150/300 mg po BID Kaletra 400/100 mg q 12 hours Which of the following medications is most likely to cause hyperglycemia in this patient? I. Combivir II. Kaletra III. Isoniazid A.) I and II B.) II only C.) I, II, and III D.) I and III E.) III only

B.) II only Explanation: Kaletra is a combination of two protease inhibitors, and a class toxicity of protease inhibitors includes hyperglycemia. Combivir, a combination of zidovudine and lamivudine, is associated with lactic acidosis and severe hepatomegaly with steatosis. Isoniazid is associated with hepatoxicity and neurotoxicity.

Which group of medications below CANNOT be compounded in polyvinyl chloride (PVC) bags? A.) Paclitaxel, carboplatin, and irinotecan. B.) Paclitaxel, docetaxel, and etoposide. C.) Docetaxel, carmustine, and cisplatin. D.) Carmustine, 5-fluorouracil, and paclitaxel.

B.) Paclitaxel, docetaxel, and etoposide. Explanation: Paclitaxel, docetaxel, and etoposide all leach the plasticizer DEHP from PVC bags, so they are inappropriate for compounding in PVC bags. Carmustine adheres to PVC, so it is also inappropriate for compounding in PVC bags.

Which of the following asthma controller drugs is a leukotriene receptor antagonist? A.) Asmanex (mometasone) B.) Singulair (montelukast) C.) Xolair (omalizumab) D.) Uniphyl (Theophylline) E.) Foradil (Formoterol)

B.) Singulair (montelukast) Explanation: Singulair (montelukast) is a leukotriene receptor antagonist.

J. H. is a 32-year-old person admitted to the hospital complaining of productive cough, fever, chills, night sweats, and weight loss. Chest x-ray showed bilateral apical nodular infiltrates. What culture or stain should be ordered to help confirm tuberculosis? A.) Sputum culture B.) Sputum acid-fast stain C.) Gram stain D.) Mycology culture and smear E.) Acid-fast culture

B.) Sputum acid-fast stain EXPLANATION: Sputum acid-fast stain can confirm the presence of tuberculosis. Tuberculosis is a very slow-growing bacteria, and it could take weeks for sputum cultures to return. Gram stain is used to determine whether Gram-negative or Gram-positive bacteria is the causative pathogen but is not used to identify tuberculosis. Mycology culture and smear are used to identify fungal infections.

MJ is currently taking tenofovir/emtricitabine, lopinavir/ritonavir, enfuvirtide, atovoquone, clarithromycin and ethambutol. She is complaining today about pain, erythema and nodules on abdomen and thighs. Could one of her medicatations be responsible for these symptoms? A.) No, this is most likely a dermatological process B.) Yes, enfuvirtide C.) Yes, clarithromycin D.) Yes, ethambutol E.) Yes, atovoquone

B.) Yes, enfuvirtide Explanation: Enfuvirtide is given by subcutaneous injection twice daily in the abdomen, thigh or back of the arm. Almost 100% of patients experience local injection site reactions consisting of pain, erythema, induration, nodules and cysts, pruritus and ecchymosis.

Patients taking zidovudine should be monitored for A.) renal toxicity. B.) anemia. C.) birth defects. D.) cardiovascular disease. E.) psychosis

B.) anemia. Explanation: Zidovudine can cause bone marrow suppression resulting in anemia and neutropenia. Patients often have a macrocytic anemia with elevations in MCV.

Which of the following laboratory values may be helpful in differentiating HF from other disease states that cause similar symptoms? A) Serum sodium B) Serum creatinine C) BNP D) Norepinephrine

C) BNP

JD is a 55-year-old African American woman with newly diagnosed hypertension. Her average BP is 164/91 mm Hg. Which of the following is the best recommendation for JD? A) Begin hydrochlorothiazide and return to clinic in 3 months. B) Begin metoprolol and prescribe monitoring blood pressure at home. C) Begin two medications since most patients with stage 2 hypertension will not reach goal with one agent alone. D) Prescribe lifestyle modifications first, and return to clinic in 1 month to determine if pharmacotherapy is warranted. E) Begin clonidine patch since a once weekly patch increases patient compliance.

C) Begin two medications since most patients with stage 2 hypertension will not reach goal with one agent alone. Explanation: Answer c is correct. Since JD has been classified with Stage 2 hypertension at the time of diagnosis, dual therapy would be recommended for this patient. With JD being African American, it would be recommended to make sure that at least one of the medications initiated is a thiazide and/or a DHP CCB. Answer a is incorrect. Although JD will most likely require two medications as she has stage 2 hypertension, it is equally logical to begin one and add the other at follow-up to determine efficacy of the first agent. However, follow-up should be monthly until the patient is at goal. Answer b is incorrect. At home BP monitoring should be prescribed and implemented for any hypertensive patient who is willing and able. However, metoprolol is not the best initial choice for this patient. Results are more inconsistent with β-blockers than other medication classes for the treatment of hypertension, and JD does not have any concomitant conditions that would warrant starting with a β-blocker (post-MI, heart failure). Answer d is incorrect. Lifestyle modifications are essential to the treatment and management of hypertension. They should always be prescribed. However, JD definitely requires medications with a BP of 167/92 mm Hg as she will most likely not reach goal with lifestyle modifications alone and in the meantime possess significant cardiovascular risk with an elevated BP at this level. Answer e is incorrect. Although transdermal systems are good options for patients with compliance problems, clonidine and other centrally acting agents are reserved as last-line options for patients who do not respond to first-line agents. RAAS agents, CCBs, and BBs should be tried before centrally acting medications.

BG is a 72-year-old woman complaining of being "extremely tired all the time." Her exercise tolerance is significantly less than it was 2 months ago; she now has to rest during daily activities. This has come on gradually. She has a history of hypertensive cardiomyopathy (LVEF 25%-30% by ECHO 1 year ago). She is strictly adherent with both diet restrictions and medications. Vital signs include BP 92/63 mm Hg, HR 105 beats/min with symptomatic orthostasis upon standing, and RR 14. BG does complain of recent dizziness; however, she denies palpitations and her electrocardiogram (ECG) is normal. On physical examination, her lungs are clear and she has no jugular venous distention, ascites or lower extremity edema. Laboratory analysis reveals sodium 129 mmol/L, potassium 4.2 mmol/L, BUN 65 mg/dL, and SCr 2.1 mg/dL (baseline BUN/SCr 32/0.9). BG has been stable on the following oral regimen for several months: valsartan 80 mg twice daily, metoprolol XL 50 mg/d, furosemide 40 mg twice daily, amiodarone 200 mg/d, and digoxin 0.125 mg/d. Which one of the following clinical categories best describes BG? A) Warm and dry B) Warm and wet C) Cold and dry D) Cold and wet

C) Cold and dry

LE is a 33-year-old woman currently attempting to become pregnant. Her physician decides that benefits of dyslipidemia treatment outweigh fetal risks. Her LDL-C is 240 mg/dL, HDL-C 64 mg/dL, and TG 132 mg/dL. Her PMH includes recent cholelithiasis. What is the most appropriate medication for LE? A) Rosuvastatin B) Niacin C) Colesevelam D) Gemfibrozil E) Omega-3-acid ethyl esters

C) Colesevelam Answer c is correct. Cholesterol and cholesterol derivatives are critical for normal fetal development. Because BAS are not systemically absorbed, they are considered the treatment of choice for women of childbearing age who are lactating or pregnant, or could become pregnant. Colesevelam is pregnancy category B and also not expected to be excreted in breast milk. Clofibrate and cholestyramine are pregnancy category C. Answer a is incorrect. Statins are pregnancy category X and should only be given to women of childbearing age, if they are highly unlikely to conceive. Answer b is incorrect. Niacin is pregnancy category C and insufficiently studied in pregnancy. Answer d is incorrect. Gemfibrozil is pregnancy category C, not indicated since TG are controlled, and contraindicated in patients with gallstones. Answer e is incorrect. Omega-3-acid ethyl esters (Lovaza®) is pregnancy category C, not indicated since TG are controlled, and may actually increase LDL-C further.

GM is a 58-year-old African American man with systolic heart failure presenting with a 10-day history of shortness of breath which limits his normal daily activities and increases lower extremity edema. His weight has recently increased by 12 lb. His physical examination is notable for BP 144/77 mm Hg, HR 85 bpm, RR 22 rpm, rales, and 4+ lower extremity edema. Pertinent laboratory values include: sodium 136 mmol/L, potassium 5.4 mmol/L, BUN 23 mg/dL, creatinine 1.1 mg/dL, and digoxin 1.9 ng/mL. Past medical history is significant for hypertension (HTN), gout, COPD, and atrial fibrillation. Current medications include lisinopril 20 mg daily, diltiazem CD 120 mg daily, digoxin 0.250 mg daily salmeterol/fluticasone 250/50, two puffs bid. GM recently began taking naproxen 220 mg tid for gout pain. Furosemide is initiated at 40 mg twice daily to manage fluid overload. Within the following 24 hours, GM experiences a brisk diuresis with improvement in heart failure signs and symptoms. The naproxen is discontinued to avoid exacerbating fluid retention, and colchine is initiated at 0.6 mg bid to manage acute gout flares. Once optimal fluid status has been achieved, which of the following represents the best option to manage GM's hypertension? A) Discontinue diltiazem and initiate amlodipine 5 mg daily. B) Initiate carvedilol 3.125 mg twice daily. C) Discontinue diltiazem and initiate carvedilol 3.125 mg twice daily. D) Initiate prazosin 2 mg daily

C) Discontinue diltiazem and initiate carvedilol 3.125 mg twice daily. In addition to initiating beta-blocker therapy, diltiazem should be discontinued due to its negative inotropic effect, which would likely cause worsening HF. Also, it would be rare to have a patient (in any disease state) on a beta blocker and non-dihydropyridine calcium channel blocker because of the additive negative inotropic and negative chronotropic pharmacologic properties. Answer (A) is incorrect. Amlodipine has not been shown to reduce mortality and would not be considered until other HF medications shown to reduce mortality have been initiated. Answer (B) is incorrect. This answer is partially correct, but not the best answer. In addition to assisting with managing GA's HTN, carvedilol is one of the three beta blockers shown to reduce mortality in HF patients. The long term benefits of inhibiting the sympathetic nervous system with beta-receptor blockade outweigh the short-term negative inotropic effects. Randomized controlled trials have demonstrated a significant mortality benefit when beta-blocker therapy is initiated in addition to ACE inhibitors. This benefit is present in patients of all HF severities. Not all beta blockers can be used in systolic heart failure (e.g. heart failure with low ejection fraction). Three beta blockers have been proven effective in heart failure and they are: carvedilol, metoprolol succinate, and bisoprolol. Beta blockers should be started at low doses and up-titrated slowly when patients are euvolemic. Even when started or up-titrated under optimal conditions, a patient's symptoms may transiently worsen. When these adjustments are made until a new equilibrium is established. Answer (D) is incorrect. Prazosin has been shown to not benefit HF patients. Prazosin would be used for patients with benign prostatic hypertrophy (BPH).

Which of the following is an appropriate way to counsel patients on taking extended-release dipyridamole plus aspirin therapy? A) Extended-release dipyridamole 200 mg plus aspirin 25 mg po daily B) Extended-release dipyridamole 25 mg plus aspirin 200 mg po daily C) Extended-release dipyridamole 200 mg plus aspirin 25 mg po bid D) Extended-release dipyridamole 25 mg plus aspirin 200 mg po bid

C) Extended-release dipyridamole 200 mg plus aspirin 25 mg po bid

CB is a 56-year-old woman who presents with palpitations, dyspnea, and presyncope. An ECG is performed and PVCs are found. CB has a past medical history of hypertension, hyperlipidemia, and postmyocardial infarction 2 years ago. What is the treatment of choice for CB? A) Flecainide B) Propafenone C) Metoprolol succinate D) Amiodarone

C) Metoprolol succinate Answer c is correct. β-Blockers, such as metoprolol, are indicated for treatment of symptomatic patients with PVCs. Answer a is incorrect. Class Ic antiarrhythmic agents, such as flecainide, should be avoided in patients with premature ventricular contractions (PVCs) due to the increased risk of mortality postmyocardial infarction. Answer b is incorrect. Class Ic antiarrhythmic agents, such as propafenone, should be avoided in patients with PVCs due to the increased risk of mortality postmyocardial infarction. Answer d is incorrect. Amiodarone is not indicated for the treatment of PVCs

Which of the following patients with small-cell lung cancer should receive prophylactic cranial irradiation? A) All patients with limited-stage SCLC B) All patients with extensive-stage SCLC C) Patients with limited-stage SCLC who achieve a complete response to their chemotherapy regimens D) Patients with extensive-stage SCLC who do not respond to their chemotherapy regimens

C) Patients with limited-stage SCLC who achieve a complete response to their chemotherapy regimens Whether limited or extensive stage, patients with SCLC who achieve a complete response to their chemotherapy regimen (C) should receive prophylactic cranial irradiation (PCI). In this situation, PCI decreases the incidence of brain metastases and improves overall survival. Prophylactic cranial irradiation is only recommended in SCLC, whether limited or extensive stage, if the patient achieves a complete response to their chemotherapy regimen (A, B, and D).

Which of the following medications is contraindicated in patients with hypersensitivity to xanthines? A) Aspirin B) Plavix C) Trental D) Pletal

C) Trental

A 79-year old man with a methicillin-sensitive Staphylococcus aureus prosthetic hip infection has completed 6 weeks of treatment with cefazolin with good response. The pain and redness initially associated with the infection have subsided, and there is no sinus tract or purulent drainage. He underwent a debridement procedure 6 weeks ago with the plan for retention of the prosthesis. The treating surgeon would now like to transition the patient to oral antibiotics. How long should the patient continue on oral antibiotics? A. 2 weeks B. 4 weeks C. 3 months D. 6 months E. No further antibiotic therapy is recommended

C. 3 months For patients with staphylococcal prosthetic joint infections receiving debridement with plan for retention of the prosthesis, Infectious Diseases Society of America guidelines recommend completion of 2-6 weeks of intravenous antibiotic therapy followed by oral antibiotics (rifampin combined with another oral anti-staphylococcal antibiotic) for 3 months in patients with a prosthetic hip infection. Answer A: A duration of 2 weeks is too short for adequate oral antibiotic therapy. The recommendation duration is 3 months.Answer B: A duration of 4 weeks is too short for adequate oral antibiotic therapy. The recommendation duration is 3 months. Answer D: Oral antibiotic therapy is recommended for 6 months in patients with total knee arthroplasty infections (after completion of 2-6 weeks of intravenous antibiotic therapy) for patients with staphylococcal prosthetic joint infection. By contrast, 3 months of oral therapy is recommended for staphylococcal prosthetic hip infections (after completion of 2-6 weeks of intravenous antibiotic therapy). Answer E: Infectious Diseases Society of America guidelines recommend continuing oral antibiotic therapy for 3 months (after completion of 2-6 weeks of intravenous antibiotic therapy) for patients with staphylococcal prosthetic hip infection.

Nathan Fox is a 56-year-old man with episodic cluster headaches for t he past year. His physician recommends starting prophylactic therapy. What is the agent of choice? A. Inderal B. Depakote C. Calan D. Imitrex Nasal Spray E. Oxygen

C. Calan Cluster headache is a rare headache disorder characterized by very intense burning or piercing pain in or around one eye on one side of the head that lasts 30 to 180 minutes. Due to its sudden onset, treatment options with a quick onset are preferred. Acute treatment options include high-flow inhaled oxygen or subcutaneous or intranasal triptans. If the patient continues to have episodes, first-line treatment for prophylaxis is verapamil (Calan), which can be initiated at 120 mg daily and titrated up to 480 mg. Corticosteroids and lithium are also used by specialists, but are discouraged due to long-term adverse effects. Answers A & B: Depakote (valproic acid) and Inderal (propranolol) are prophylactic options for migraines, but not for cluster headache. Answer D: Imitrex nasal spray (intranasal sumatriptan) is an acute treatment option for cluster headaches, but not for prophylactic treatment. Answer E: Oxygen is used for the acute treatment of cluster headaches, but not for prophylactic treatment.

What is the most commonly reported adverse effect of Ofev? A. Hypotension B. Weight gain C. Gastrointestinal upset D. Hyperthyroidism E. Headache

C. Gastrointestinal upset Nintedanib (Ofev) is indicated for the treatment of idiopathic pulmonary fibrosis (IPF). The most commonly reported adverse effects (reported in >10% of clinical trial participants) are related to gastrointestinal upset or discomfort, including diarrhea, nausea, abdominal pain, vomiting, and decreased appetite. Increased liver enzymes were also relatively common. Less common adverse effects include hypertension, headache, weight loss, and hypothyroidism. Answer A: Less common adverse effects of Ofev include hypertension, headache, weight loss, and hypothyroidism. Answers B & D & E: Less common adverse effects include hypertension, headache, weight loss, and hypothyroidism. Bottom Line: Nintedanib (Ofev) is indicated for the treatment of idiopathic pulmonary fibrosis (IPF). The most commonly reported adverse effects are related to gastrointestinal discomfort, including diarrhea, nausea, abdominal pain, vomiting, and decreased appetite. Less common adverse effects include hypertension, headache, weight loss, and hypothyroidism.

Which of the following is simeprevir's mechanism of action? A. CCR5 antagonist B. NS5B nucleoside polymerase inhibitor C. NS3/4A protease inhibitor D. NS5A inhibitor E. NS5B non-nucleoside polymerase inhibitor

C. NS3/4A protease inhibitor

The onset of action for oral clonidine is A.) 1 hour. B.) 20-30 minutes. C.) 5-15 minutes. D.) 1-2 minutes. E.) immediate

C.) 5-15 minutes. Explanation: EXPLANATION: The onset of action for clonidine, which can be used to treat hypertensive emergencies, is 5-15 minutes with a duration of action ranging from 6 to 12 hours.

A patient was treated with radioactive iodine (RAI) for hyperthyroidism, but his condition persists. When would a second dose of RAI be recommended? A.) 1 month B.) 3 months C.) 6 months D.) 12 months E.) 24 months

C.) 6 months Explanation: A second dose of RAI can be repeated in 6 months if the patient remains hyperthyroid.

In addition to patient education and environmental control, which of the following orders is best for long-term management of intermittent asthma? A.) Formoterol as needed B.) Montelukast at bedtime C.) Albuterol MDI as needed and before exercise D.) Theophylline at bedtime and before exercise E.) Budesonide dry-powder inhaler twice daily

C.) Albuterol MDI as needed and before exercise EXPLANATION: For long-term management of intermittent asthma, albuterol MDI as needed and before exercise is best according to the NIH Expert Panel Report 3 guidelines.

Which of the following medications used to treat acute otitis media may cause red discoloration of stools? A.) Amoxicillin-clavulanate B.) Clindamycin C.) Cefdinir D.) Ceftriaxone E.) Cefuroxime

C.) Cefdinir Explanation: Cefdinir (Omnicef), when co-administered with iron-containing products, may cause discoloration of stool as a result of binding in the gastrointestinal tract. Although this reaction has no known side effect, it should be part of patient education when dispensing this medication

Which of the following electrolyte abnormalities may be seen in a patient on thiazide diuretics? A.) Increased potassium B.) Decreased uric acid C.) Decreased sodium D.) Decreased calcium E.) Increased magnesium

C.) Decreased sodium Thiazide diuretics decrease potassium, sodium, and magnesium while increasing uric acid and calcium.

The FDA has warned against use of aliskiren in combination with an angiotensin converting enzyme inhibitor or angiotensin receptor blocker due to the increased risk of renal impairment, hypotension and hyperkalemia in patients with which of the following? A.) Coronary artery disease B.) Hypertension C.) Diabetes D.) HIV E.) Liver disease

C.) Diabetes Explanation: In April of 2012 the FDA warned of possible risks when using aliskiren in combination with ACEIs and ARBs in patients with diabetes or renal impairment. Recommendations being added to drug labels for aliskiren-containing products include a new contraindication against the use of aliskiren with ACEIs or ARBs in patients with diabetes due to the risk of renal impairment, hypotension and hyperkalemia and a warning to avoid use of these combinations in patients with moderate to severe renal impairment (i.e., when GFR is < 60 mL/min).

Which of the following secretagogues (sulfonylurea or meglitinide) should be avoided in elderly patients because of risk of hypoglycemia? A.) Repaglinide B.) Glipizide C.) Glyburide D.) Glimepiride E.) Nateglinide

C.) Glyburide Glyburide is on the American Geriatrics Society's Beers list of medications to avoid in elderly patients owing to significantly increased risk for hypoglycemic events.

Which of the following is not an effect of glucocorticoids? A.) Immunosuppression B.) Decreased prostaglandin synthesis C.) Inhibition of glycogenolysis D.) Decreased neutrophils at sites of infection E.) Inhibition of macrophages

C.) Inhibition of glycogenolysis Explanation: Glucocorticoids have potent effects on glucose and carbohydrate metabolism. They promote glycogen breakdown, rather than inhibit it.

A 12-year-old-male was recently diagnosed with type 1 DM. His current weight is 87 lb. Which of the following is the most appropriate starting dose for his basal insulin? A.) Insulin aspart 15 units daily B.) Insulin glargine 20 units daily C.) Insulin detemir 10 units daily D.) Insulin degludec 5 units daily E.) Insulin lispro 10 units daily

C.) Insulin detemir 10 units daily Explanation: Patients with type 1 DM should be initiated at a dose of 0.4-0.5 unit/kg/day of total daily insulin use. Basal insulin should make up 50% of the total daily dose. The patient's current weight of 87 lb is equivalent to 40 kg. 0.5 unit/kg/day would calculate to a total daily insulin dose of 20 units and 50% (10 units) should be initiated as basal insulin

R. B. is a 64-year-old patient who has been in the hospital for 10 days. Twenty-four hours ago, he vomited and aspirated, and he now has evidence of infection and a new infiltrate on chest x-ray. What is the best empiric antibiotic option for R. B.? A.) Clindamycin B.) Vancomycin C.) Piperacillin-tazobactam D.) Azithromycin E.) Ampicillin

C.) Piperacillin-tazobactam Explanation: Piperacillin-tazobactam provides the best coverage of the Gram-negative enteric bacteria associated with hospital-acquired aspiration pneumonia. Clindamycin covers anaerobic pathogens associated with community-acquired aspiration pneumonia. Vancomycin covers only Gram-positive bacteria. Azithromycin and ampicillin lack the spectrum of activity to cover the majority of bacteria associated with hospital-acquired aspiration pneumonia.

Pramlintide should be avoided in a patient with: A.) type 2 diabetes with an A1C of 8.5% on metformin. B.) type 1 diabetes with an A1C of 8.5% on insulin. C.) type 2 diabetes with an A1C of 9.5% on rosiglitazone (Avandia). D.) type 2 diabetes with an A1C of 8% on pioglitazone (Actos). E.) type 1 diabetes with an A1C of 8% on insulin

C.) type 2 diabetes with an A1C of 9.5% on rosiglitazone (Avandia). EXPLANATION: Pramlintide should be avoided in any diabetic patient with an A1C ≥ 9%. Pramlintide should also be avoided in patients with hypoglycemia unawareness, with severe gastrointestinal disease (including gastroparesis and recurrent severe hypoglycemia requiring assistance in the past 6 months), and with poor adherence to their current insulin regimen or self-monitoring of blood glucose.

Which of the following statin doses may be dispensed to a patient also taking gemfibrozil? Select all that apply. A) Rosuvastatin 20 mg B) Simvastatin 20 mg C) Lovastatin 40 mg D) Fluvastatin 40 mg

D) Fluvastatin 40 mg Answer d is correct. There is no dose limit for fluvastatin in combination with gemfibrozil. Studies suggest that fluvastatin is the only statin that may be safely combined with gemfibrozil, although caution is still advised. Answer a is incorrect. Although rosuvastatin has few drug interactions since 90% is eliminated unchanged, doses should not exceed 5 mg for patients on cyclosporine and 10 mg for those on gemfibrozil, ritonavir, lopinavir, and oral contraceptives. Answer b is incorrect. Simvastatin doses should not exceed 10 mg for patients on gemfibrozil, danazol, and cyclosporine and 20 mg for those on amiodarone and verapamil. Answer c is incorrect. Lovastatin doses should not exceed 20 mg for patients on gemfibrozil, fenofibrate, cyclosporine, and niacin (≥1 g/d) and 40 mg for those on amiodarone and verapamil.

CE is a 74-year-old man with a PMH of CHD, stroke, and hypothyroidism. He currently takes aspirin, levothyroxine, and simvastatin and has now been prescribed cholestyramine. What will you discuss with the patient? A) Take on an empty stomach once daily. B) Mix each dose with at least 12 ounces of juice or soda. C) Sip slowly to reduce side effects. D) Take other medications at least 1 to 2 hours before or 4 to 6 hours after taking cholestyramine.

D) Take other medications at least 1 to 2 hours before or 4 to 6 hours after taking cholestyramine. Answer d is correct. Cholestyramine and colestipol can bind a wide array of medications, including digoxin, warfarin, levothyroxine, phenytoin, niacin, oral contraceptives, ezetimibe, fibrates, statins, and aspirin. Interacting medications should be given 1 to 2 hours before or 4 to 6 hours after these resins. Colesevelam has the lowest likelihood of interactions, but the manufacturer still recommends either closely monitoring or separating medications with a narrow therapeutic index by 4 hours. Answer a is incorrect. BAS should be taken with or just after meals in two to three divided doses daily, with the exception of colesevelam (625 mg, six tablets per day), which may be dosed all at once if tolerated. Answer b is incorrect. Cholestyramine (and colestipol) powder should be mixed with 2 to 6 ounces of water, a noncarbonated beverage such as orange, apple, or grape juice, or a high moisture content pulpy fruit (eg, applesauce). Answer c is incorrect. Powdered resins may cause teeth discoloration, erosion, and decay and must be swallowed quickly.

According to the National Institutes of Health (NIH), which of the following combinations is considered a preferred regimen for the treatment of HIV? A. 1 non-nucleoside reverse transcriptase inhibitor + 1 protease inhibitor + 1 nucleoside reverse transcriptase inhibitor B. 2 non-nucleoside reverse transcriptase inhibitors + 1 nucleoside reverse transcriptase inhibitor C. 2 nucleoside reverse transcriptase inhibitors + 1 protease inhibitor D. 2 nucleoside reverse transcriptase inhibitors + 1 integrase strand transfer inhibitor

D. 2 nucleoside reverse transcriptase inhibitors + 1 integrase strand transfer inhibitor

Which of the following statements regarding the use of Arava during pregnancy is true? A. Arava can be used safely during pregnancy B. Arava should not be used until after the first trimester of pregnancy C. Arava should be used only during the first trimester of pregnancy, and then discontinued D. Arava is contraindicated in pregnant women because of the potential for fetal harm E. The effects of Arava during pregnancy are unknown because they have not been studied

D. Arava is contraindicated in pregnant women because of the potential for fetal harm Arava (leflunomide) is approved for the treatment of adults with active rheumatoid arthritis, but also might be used off-label for prevention of rejection in solid organ transplant recipients and cytomegalovirus (CMV) disease in transplant recipients who are resistant to standard antivirals. Arava has a US Boxed Warning for use in pregnancy. It is contraindicated in pregnant women because of the potential for fetal harm. Adverse events were observed in animal reproduction studies at lower doses than expected for human exposure. Females of reproductive potential should be tested for pregnancy before starting Arava. They also should be counseled to use effective contraception during treatment. If pregnancy occurs during treatment, therapy should be discontinued immediately and an accelerated elimination procedure with cholestyramine should be used. After treatment, pregnancy should be avoided and contraception used until Arava's serum concentrations are undetectable (< 0.02 mg/L on 2 readings at least 14 days apart).

What is the most common adverse effect of Prolia? A. Facial swelling B. Hearing loss C. Pancreatitis D. Arthralgia E. Hyperkalemia

D. Arthralgia

Ms. McGuire, a 55-year-old woman who is prescribed Nardil 15 mg orally 3 times daily, presents with the chief complaint of acute migraines 1-2 times per month. Which of the following medication is contraindicated with Nardil? A. Naprosyn B. Tylenol C. Dilaudid D. Imitrex E. Fiorinal

D. Imitrex Sumatriptan (Imitrex) is a serotonin (5-HT1B and 5-HT1D) receptor agonist primarily metabolized via monoamine oxide (MAO) A enzyme and is contraindicated with MAO A enzyme inhibitors (or within 2 weeks of their use). Phenelzine (Nardil) is an MAO A and B inhibitor. Other MAO inhibitors include tranylcypromine (Parnate), isocarboxazid (Marplan), and selegiline (Emsam). Methylene blue and linezolid also have MAO inhibitor properties. Thus, concomitant use is contraindicated. If used together, it can increase the risk of serotonin toxicity (serotonin syndrome). Sumatriptan is contraindicated with the concomitant use or within 2 weeks of an MAO inhibitor. Answer A: Naproxen (Naprosyn) is a nonsteroidal antiinflammatory drug (NSAID) that can be used together with sumatriptan. Combination therapy of an NSAID and triptan is often used for the treatment of migraine. A meta-analysis found that sumatriptan plus naproxen was more effective than either medication alone. Answer B: Acetaminophen (Tylenol) is not contraindicated with the use of phenelzine (Nardil). Answer C: Hydromorphone (Dilaudid) is not contraindicated with the use of phenelzine (Nardil). Answer E: Aspirin/butalbital/caffeine (Fiorinal) is not contraindicated with the use of phenelzine (Nardil). Bottom Line: Sumatriptan (and other triptans) are contraindicated with concurrent administration or within 2 weeks of discontinuing a monoamine oxide type A inhibitor.

The recommended antibiotic treatment for anthrax may include which of the following? A.) Ciprofloxacin B.) Doxycycline C.) Amoxicillin D.) Any of the above

D.) Any of the above Explanation: Ciprofloxacin, levofloxacin, and doxycycline are approved by the FDA for treatment of anthrax, while amoxicillin can be used when the other drugs are not tolerated or pose patient-specific risks. These agents can be used separately or in combination, depending on the symptoms and the patient's sensitivity to the agents. Antimicrobial resistance to ciprofloxacin has been growing rapidly because of widespread overuse after the anthrax-contaminated mail episodes in 2002.

A drug discovery strategy that utilizes nucleic acids and amino acids in various combinations to synthesize vast libraries of oligonucleotide or peptide compounds for high throughput lead compound screening is which of the following? A.) Whole cell screening B.) Natural product screening C.) Gene therapy D.) Combinatorial chemistry E.) rDNA technology

D.) Combinatorial chemistry Explanation: A drug discovery strategy that utilizes nucleic acids and amino acids in various combinations to synthesize vast libraries of oligonucleotide or peptide compounds for high throughput lead compound screening is called combinatorial chemistry.

The apparent Solubility of drug can be increased by all of the following except: A.) Using salt form the drug B.) Using amorphous or non-crystalline form of the drug C.) Using surfactants or cosolvent D.) Increasing surface area or decreasing particle size of the drug

D.) Increasing surface area or decreasing particle size of the drug Explanation: Dissolution is kinetic process whereas solubility is thermodynamic process. Increase in the surface area or decrease in particle size would enhance the rate of dissolution whereas by using salt form, amorphous form or by using cosolvent would increase solubility of the drug.

A 66-year-old patient presents to the emergency department with symptoms of dysarthria and left-sided weakness. He is diagnosed with an acute ischemic stroke. What is the interval of time from the onset of symptoms that t-PA is most effective? A.) Less than 24 hours B.) Less than 12 hours C.) Less than 6.5 hours D.) Less than 4.5 hours E.) Less than 48 hours

D.) Less than 4.5 hours A patient who is diagnosed with a stroke should be treated with t-PA within 4.5 hours of the onset of symptoms for the best outcome. If t-PA is administered after 4.5 hours of the onset of symptoms, a less favorable outcome is expected.

Which of the following is a cardioselective β-blocker used for treating heart failure? A.) Carvedilol B.) Atenolol C.) Metoprolol tartrate D.) Metoprolol succinate E.) Propranolol

D.) Metoprolol succinate Explanation: Although atenolol and metoprolol tartrate are cardioselective β-blockers, only metoprolol succinate has been shown to improve survival in patients with heart failure.

Which of the following are preferred for long-term treatment of mild persistent asthma? A.) Budesonide and formoterol B.) Fluticasone and salmeterol C.) Beclomethasone and ipratropium D.) Mometasone dry-powder inhaler and prn albuterol E.) Fluticasone MDI and tiotropium

D.) Mometasone dry-powder inhaler and prn albuterol Explanation: Use of an inhaled corticosteroid (Asmanex (mometasone)) and albuterol (Proair, Ventolin, Proventil) as needed is recommended according to the NIH Expert Panel Report 3 guidelines.

Which of the following combinations for migraine prophylaxis is most appropriate? A.) Sumatriptan and naproxen B.) Ergotamine tartrate and amitriptyline C.) Treximet and Motrin IB D.) Motrin IB and Inderal E.) Amitriptyline and Pamelor

D.) Motrin IB and Inderal EXPLANATION: All of the other medications contain triptans and ergotamines, which are not indicated for migraine prophylaxis but are used for acute treatment of migraines. Amitriptyline and Pamelor would not be an appropriate combination because both are in the same pharmacologic class even though both are used for migraine prophylaxis.

A patient reports to an ambulatory clinic complaining of constipation since starting a new antihypertensive medication. Which of the following may be the most likely agent to cause the adverse event? A.) Olmesartan B.) Hydrochlorothiazide C.) Captopril D.) Verapamil E.) Hydralazine

D.) Verapamil Explanation: A common adverse event with verapamil is constipation. Constipation is not a common adverse effect of the other medications.

A 19-year-old presents to your pharmacy with a new prescription for an anticonvulsant. In reviewing the patient profile, you note that the patient has a drug allergy to sulfa medications. Which of the following anticonvulsants would be contraindicated in this patient? A.) Phenobarbital B.) Phenytoin C.) Topiramate D.) Zonisamide E.) Carbamazepine

D.) Zonisamide Explanation: Zonisamide contains a sulfa moiety and has been associated with life-threatening sulfonamide reactions (e.g., Stevens-Johnson syndrome, toxic epidermal necrolysis, aplastic anemia, agranulocytosis, and fulminate hepatic necrosis). Although some studies have reported no cross-reactivity between the zonisamide and sulfonamides, the manufacturer lists history of allergy to sulfonamide as a contraindication to the use of zonisamide.

The anticoagulant effect of dabigatran is mediated by A.) inhibiting synthesis of vitamin K-dependent clotting factors. B.) blocking the platelet P2Y12 receptor. C.) inhibiting clotting factor Xa. D.) direct thrombin inhibition. E.) inhibiting the VKORC1 enzyme.

D.) direct thrombin inhibition. Explanation: Dabigatran is a direct thrombin inhibitor. It does not affect synthesis of the vitamin K-dependent clotting factors or affect platelet activity.

The major troublesome side effect in nicotinic acid therapy is A.) diarrhea. B.) vomiting. C.) hair growth. D.) flushing. E.) dizziness.

D.) flushing. Explanation: The most common side effect is flushing, which may occur in many patients. To decrease flushing intensity, a patient should take aspirin 325 mg 30 minutes before the first dose of nicotinic acid. Itching may also occur with flushing.

Following an intravenous injection of Ga-67 citrate, Ga-67 disassociates from citrate and becomes immediately bound to A.) erythrocytes. B.) leukocytes. C.) platelets. D.) transferrin.

D.) transferrin. Explanation: Ga-67 immediately binds to plasma transferrin, and it is not dependent on white blood cells, erythrocytes, or platelets for localization.

Which of the following medications does not come in a combination pill with metformin? A.) Sitagliptin B.) Linagliptin C.) Saxagliptin D.) Pioglitazone E.) Acarbose

E.) Acarbose Explanation: Acarbose can be used in combination with metformin, however it does not currently have a combination product on the market. All of the other agents are commercially available as a combination drug.

Which of the following has been shown to alter the course of osteoarthritis? A.) Tylenol B.) Tramadol C.) Capsaicin cream D.) Glucosamine sulfate and chondroitin E.) None of the above

E.) None of the above Explanation: Unlike RA, no therapies used in the management of osteoarthritis have been shown to alter its course; they only palliate the symptoms. Tylenol, tramadol, and capsaicin cream are useful in symptom and pain management.

Which of the following vasoactive agents would increase blood pressure but not increase heart rate or cardiac output? A.) Dobutamine B.) Dopamine C.) Norepinephrine D.) Epinephrine E.) Phenylephrine

E.) Phenylephrine EXPLANATION: Phenylephrine has only α-1 adrenergic activity and no β-adrenergic activity. Thus, it will increase blood pressure but not affect heart rate.

Which of the following is most likely to be considered a health-care associated pathogen rather than a community-acquired pathogen? A.) Streptococcus pneumoniae B.) Haemophilus influenzae C.) Mycoplasma pneumoniae D.) Methicillin-resistant Staphylococcus aureus E.) Pseudomonas aeruginosa

E.) Pseudomonas aeruginosa Explanation: Pseudomonas aeruginosa is primarily a hospital-acquired organism. Prior to the 2000s MRSA was primary hospital-acquired; however, community-acquired MRSA has increased dramatically in the past decade such that MRSA is considered both a community-acquired and hospital-acquired organism.

All of the following statements are true regarding seborrhea except A.) it is a chronic inflammatory skin disease seen in areas of greatest sebaceous gland activity. B.) it fluctuates in severity and is worsened by stress and poor health. C.) moderate to severe cases require topical corticosteroids or the topical antifungal ketoconazole for effective treatment. D.) it is called cradle cap when it occurs in infants. E.) it most commonly occurs on the legs and arms.

E.) it most commonly occurs on the legs and arms. Explanation: Seborrhea most commonly occurs on the face, especially eyebrows and eyelashes, but not on the extremities.

Generic Name: Gemfibrozil Brand: ? FDA approved Uses: Adverse events:

Look up

Levemir carries a Pregnancy Category of: A.) A B.) B C.) C D.) D E.) X

B.) B Levemir has recently been given the pregnancy category of B and can be safely used in pregnancy.

Enter the maximum daily dosage (in milligrams per day) of cetirizine for the treatment of upper respiratory allergies.(Answer must be numeric; round the final answer to the nearest WHOLE number).

10 mg

A penicillin V 500-mg tablet equates to 800,000 units of penicillin activity. A patient is taking penicillin V 500 mg tablets 4 times daily for 10 days. How many units of penicillin activity will this patient receive after 5 days? (Answer must be numeric; round the final answer to the nearest WHOLE number.)

16,000,000 units Correct answer: 16000000 Proportions represent the equity between 2 ratios (a/b = c/d). If one knows 3 of the 4 values of a proportion, the fourth value can be calculated. The units in the numerators must match and the units in the denominators must match, so all the units must be converted before starting proportion calculations.1 tablet contains 500 mg of penicillin (800,000 units).After 5 days of taking tablets 4 times daily, the patient will have taken 20 tablets (5 days x 4 tablets/day).Use proportions to calculate the number of units: 800,000 units/1 tablet = X units/ 20 tablets → 16,000,000 units Bottom Line:Proportions represent the equity between 2 ratios (a/b = c/d). If one knows 3 of the 4 values of a proportion, the fourth value can be calculated. The units in the numerators must match and the units in the denominators must match, so all the units must be converted before starting proportion calculations.

A patient was being treated with furosemide 40 mg IV BID for an acute HF exacerbation. She is now ready for discharge and the team wants to send her home on bumetanide. What would be an equivalent oral dose of bumetanide?

4 mg Po bumetanide 80 mg IV furosemide = 160 mg po furosemide 160 mg po furosemide = 4 mg po bumetanide

How much water should be added to this prescription to make it isotonic, if the E value of atropine sulfate is 0.12? Rx: Atropine sulfate 0.4 g Sodium chloride 0.312 g Purified water qs ?

40 Because you know the amount of sodium chloride needed to make this prescription isotonic and the E value for atropine sulfate, you can set up a ratio to determine the equivalents of sodium chloride that can replace the drug. Add the two amounts of sodium chloride (the contribution of the drug and the 0.312 g).

Please fill in the blank to express 0.025% as a ratio strength.For example, if the ratio strength is 1:90, please type "90."(Answer must be numeric; round the final answer to the nearest WHOLE number.)

4000

The neurologist would like to add lamotrigine to his current regimen. What is the recommended starting daily dose (in milligrams) of lamotrigine?(Answer must be numeric. Please round to the nearest whole number).

50 mg Correct answer: 50 For patients on regimens containing carbamazepine, phenytoin, phenobarbital, primidone, rifampin, and lopinavir/ritonavir, and who are not on valproic acid, the recommended initial dosing for lamotrigine is as follows: Weeks 1 and 250 mg once dailyWeeks 3 and 4100 mg daily in 2 divided dosesWeek 5+Increase by 100 mg daily every 1 to 2 weeks The usual maintenance dose is 300 to 500 mg daily in 2 divided doses, but doses up to 700 mg/day have been used.Bottom Line:For patients on regimens containing carbamazepine, the recommended initial dosing for lamotrigine is 50 mg once daily for weeks 1 and 2. The dose is then uptitrated to a typical maintenance dose of 300 to 500 mg daily in 2 divided doses.

A 23-year-old woman plans to insert NuvaRing today. It is day 3 of her menstrual cycle and she still has menstrual bleeding. She has used no previous hormonal contraception. For how many days should she use a spermicide or barrier method of contraception after initial insertion of NuvaRing?(Answer must be numeric; round the final answer to the nearest WHOLE number.)

7 days Correct answer: 7 NuvaRing is a vaginal ring indicated for the prevention of pregnancy. It contains ethinyl estradiol 0.015 mg and etonogestrel 0.12 mg per 24 hours. For initial treatment, in which the patient has had no hormonal contraceptive use in the past month, the patient should insert 1 ring on the first day of the menstrual cycle ("day 1"). Patients also can insert NuvaRing on days 2 to 5 even if bleeding is not complete, however, a spermicide or barrier method of contraception should be used for the following 7 days.Of note, certain female barrier methods, such as the diaphragm or female condom, might interfere with proper ring placement, and therefore, are not recommended for use with NuvaRing. Bottom Line:For initial treatment, in which the patient has had no hormonal contraceptive use in the past month, the patient should insert 1 NuvaRing on the first day of her menstrual cycle ("day 1"). Patients also can insert NuvaRing on days 2 to 5 even if bleeding is not complete. However, a spermicide or barrier method of contraception should be used for the following 7 days.

JR is a 37-year-old woman patient who was previously diagnosed with iron-deficiency anemia. She has been taking oral iron supplementation as directed for 16 weeks without achieving her goal hemoglobin of 12 g/dL. Her most recent hemoglobin level was 9.8 g/dL. The decision has been made to administer IV iron dextran. She is 5'4" tall and weighs 150 lb. What is the correct dose of iron dextran for CR? A) 19.5 mL B) 21.1 mL C) 24.4 mL D) 29.2 mL

A) 19.5 mL Answer a is correct. Dose of iron dextran, in milliliters can be calculated by using this equation: Dose (mL) = 0.0442 (Desired Hb - Observed Hb) × LBW + (0.26 × LBW) her dose calculates at 27 mL. The variable used are: desired hemoglobin 12 g/dL, observed hemoglobin 9.8 g/dL, LBW (lean body weight) 54.7 kg. LBW is calculated for a male as 45.5 + 2.3 (Ht-60) where Ht is measured in inches. Answer b is incorrect. This is the value that is calculated if you use the LBW formula for a male patient. Answer c is incorrect. This is the value if you calculated the dose using actual body weight instead of LBW. Answer d is incorrect. This is the value if you calculated the dose using 16 g/dL as your target hemoglobin.

What treatment duration would you select for chronic HCV genotype 2 treated with peg-interferon and ribavirin? A) 24 weeks B) 48 weeks C) 1 year D) 5 years

A) 24 weeks Genotype 2 (and 3) only requires 24 weeks (A) of peg-interferon and ribavirin. Patients with genotypes 2 or 3 have an 80% chance of responding to 24 weeks therapy. Only genotypes 1 and 4 require 48 weeks of therapy (B) due to the low efficacy rate (50%-60%) of peg-interferon and ribavirin in this patient population. A 1 to 5 year duration of NRTI therapy for HBV is common (C and D). Topic: Hepatitis

Identify the Arabic value of DCXXIV. A) 624 B) 626 C) 1024 D) 1026

A) 624 Answer a is correct. D = 500, C = 100, X = 10, I = −1, V = 5 (500 + 100 + 10 + 10 −1 +5 = 624). Answer b is incorrect. The Roman numeral for 626 would be DCXXVI. Answer c is incorrect. The Roman numeral for 1024 would be MXXIV. Answer d is incorrect. The Roman numeral for 1026 would be MXXVI.

In which of the following groups has dexamethasone demonstrated a mortality benefit? A) A 2 week old with S. agalactiae (group B) meningitis B) A 17 year old with N. meningitidis meningitis C) A 35 year old with S. pneumoniae meningitis D) It has not demonstrated clear benefit for any type of bacterial meningitis

A) A 2 week old with S. agalactiae (group B) meningitis Answer c is correct. One randomized controlled trial and two subsequent meta-analyses have shown a mortality benefit of administering adjunctive corticosteroids in adult patients with S. pneumoniae meningitis. Answer a is incorrect. There is a lack of data demonstrating a mortality benefit of adjunctive corticosteroids in S. agalactiae (group B) meningitis. Answer b is incorrect. There is a lack of data demonstrating a mortality benefit of adjunctive corticosteroids in N.meningitidis meningitis. Answer d is incorrect. A mortality benefit has been shown for adults with S. pneumoniae meningitis.

CV is a 64-year-old man who completed therapy for pyelonephritis and feels better. Two weeks later, he returns to the emergency department (ED) with general malaise, a temperature of 101.9 °F, pelvic pain, dysuria, and increased urination. What is the likely cause of CV's symptoms? A) Acute bacterial prostatitis B) Benign prostatic hyperplasia C) Cystitis D) Epididymitis

A) Acute bacterial prostatitis CV's initial therapy resolved his pyelonephritis. The prostate is a common site of bacteria persistence. Two weeks later he displays symptoms of acute bacterial prostatitis (A). BPH (B) does not present with these symptoms. Cystitis (C) does not typically produce a fever. Epididymitis (D) causes unilateral testicular pain and swelling.

LO is a 28-year-old woman who found out her boyfriend has chronic hepatitis B (HBV). They are sexually active and plan to marry in 6 months. Which of the following is the best course of action? A) Administer HBIG and begin the HBV vaccine series. B) Administer HBIG. C) Begin the HBV vaccine series. D) Begin treating her for HBV.

A) Administer HBIG and begin the HBV vaccine series. The CDC recommends initiating the HBV vaccine series and giving HBIG as postexposure prophylaxis for HBV in both individuals.

The pharmacy department receives a consult from the staff oncologist for management of a patient's drug therapy. The patient is receiving antineoplastic therapy (including platinum analogues). The patient has a serum glucose of 110 mg/dL, serum creatinine of 1.3 mg/dL, and white blood cell count of 11,000 cells/mm3. What agent could be administered with cisplatin to prevent nephrotoxicity? A) Amifostine B) Epoetin alfa C) Mesna D) Dexrazoxane

A) Amifostine Cisplatin and carboplatin may cause nephrotoxicity and can be prevented by the administration of fluids and amifostine. Answer (B) is incorrect. Recombinant human erythropoietic products (epoetin alfa and darbepoetin alfa) are useful in the management of anemia. Answer (C) is incorrect. Mesna is given to prevent the hemorrhagic cystitis of cyclophosphamide. Answer (D) is incorrect. Dexrazoxane prevents the cardiac toxicity caused by anthracyclines.

Which of the following agents is recommended as therapy for invasive aspergillosis? A) Amphotericin B B) Fluconazole C) Nystatin D) Flucytosine

A) Amphotericin B The Correct Answer is: A Amphotericin B (A) is a drug of choice for the Aspergillus species. Fluconazole (B) lacks activity against molds like Aspergillus. Nystatin (C) lacks activity against the Aspergillus species and is not absorbed orally. Flucytosine (D) is used only in combination therapy with amphotericin B against Cryptococcal meningitis, not Aspergillus infections.

JP is a kidney transplant patient whose biopsy showed cellular rejection. Select the statement that most accurately describes mechanism of JP's rejection. A) An orchestrated immune response that involves alloantigen presentation via APCs that then leads to alloreactive T lymphocytes. B) A cytotoxic immune response mediated via preformed antibodies against antigens present on vascular endothelium. C) A slow process of graft fibrosis and arteriopathy, which results in graft dysfunction. D) A process which inhibits the entire process of immune activation, including antigen presentation by APCs, the release of cytokines such as IL-1, IL-2, IL-6, and TNF α, and subsequently lymphocyte proliferation.

A) An orchestrated immune response that involves alloantigen presentation via APCs that then leads to alloreactive T lymphocytes. Answer a is correct. This is a description of ACR. ACR requires the production of alloreactive T cells via T cell binding at the T cell receptors on APCs, with subsequent cytokine release and immune activation. Answer b is incorrect. Antibody-mediated rejection typically occurs hours to days after transplant if donor-specific antibodies are present at the time of transplant. This type of rejection most frequently results from mismatched blood types and positive cross matches, and the incidence has decreased with the advent of screening. Answer c is incorrect. This is a description of chronic rejection. The etiology of chronic rejection is not known and there is no treatment for this condition. However, ACR has been shown to be a primary risk factor for the development of chronic rejection, thus prevention is a key modifier. Answer d is incorrect. This is a description of the ubiquitous immunosuppressive action that steroids have on immune response.

B and T lymphocytes may be distinguished from each other by the presence of lineage specific membrane markers termed: A) CD B) Complement C) CRP D) Chemokines E) CCR5 coreceptor

A) CD Answer a is correct. Morphologic differentiation of lymphocytes is difficult and visual inspection of a blood smear cannot distinguish between T and B cells. Fortunately, lymphocytes can be distinguished by the presence of lineage-specific membrane markers, termed CD. Mature T cells are CD4 or CD8 and B cells are CD20. Identification of the subtype of lymphocyte is not a routine clinical hematology test; lymphocytes are reported as a total lymphocyte count on the CBC. An exception is the reporting/monitoring of CD4 cells for patients with HIV. Answer b is incorrect. The complement system is a mediator of innate immunity. The complement system consists of multiple proteins that play a key role in immune defense. The complement system serves as an adjunct or "complement" to humoral immunity. Answer c is incorrect. CRP is an acute phase reactant produced by the liver during early stages of infection or inflammation. Acute phase reactants or proteins increase in response to inflammatory stimuli such as tissue injury or infection. Recent clinical evidence found that CRP is also released in response to inflammatory markers present within atherosclerotic plaques and leads to cardiovascular disease. Cholesterol medications (HMG-CoA reductase inhibitors/statins) decrease CRP levels and rosuvastatin was found to decrease cardiovascular disease in patients with elevated CRP levels. Answer d is incorrect. Chemokines play an essential role in linking the innate and adaptive immune response by orchestrating traffic. The chemokine system consists of a group of small polypeptides and their receptors. Chemokines possess four cysteines. Based upon the positions of the cysteines, almost all chemokines fall into one of two categories: (1) CC group or (2) CXC group.

RC is a 47-year-old man with stage IV unresectable melanoma who is in the hospital for his treatment with high-dose IL-2. Select the side effect associated with IL-2 that can lead to hypotension and reduced organ perfusion. A) Capillary leak syndrome B) Myelosuppression C) Anemia D) Hepatotoxicity E) Delirium

A) Capillary leak syndrome Answer a is correct. Vascular or capillary leak syndrome is a dose-limiting toxicity commonly reported with IL2. It can be observed immediately after initiation of therapy; clinical presentations may include weight gain, ascites, peripheral edema, arrhythmias and/or tachycardia, hypotension, oliguria and renal insufficiency, pleural effusions, and pulmonary congestion. Answer b is incorrect. Myelosuppression with neutropenia, anemia, and thrombocytopenia have been reported with IL2, but does not usually lead to hypotension and reduced organ perfusion. Monitor patients closely for any infectious process. Answer c is incorrect. Anemia has been reported with IL2, but does not usually lead to hypotension and reduced organ perfusion. Answer d is incorrect. Hepatotoxicity has been reported with IL2, but does not usually lead to hypotension and reduced organ perfusion. Answer e is incorrect. Deliriums have been observed with IL2 and generally resolve when therapy is discontinued, but does not usually lead to hypotension and reduced organ perfusion

Select the antibiotic used in combination treatment of necrotizing fasciitis. This antibiotic is used to decrease bacterial toxin production, thereby limiting tissue damage. A) Cleocin B) Primaxin C) Augmentin D) Zosyn

A) Cleocin Clindamycin (Cleocin) should be added to decrease bacterial toxin production, thereby limiting tissue damage. Answer (B) is incorrect. Imipenem-cilastatin (Primaxin) may be used in the treatment of NF, but it needs to be combined with clindamycin. Answer (C) is incorrect. Amoxicillin-clavulanate (Augmentin) may be used in the treatment of NF, but it needs to be combined with clindamycin. Answer (D) is incorrect. Piperacillin-tazobactam (Zosyn) may be used in the treatment of NF, but it needs to be combined with clindamycin.

B and T lymphocytes may be distinguished from each other by the presence of lineage specific membrane markers termed which of the following? A) Clusters of differentiation (CD) B) Complement C) C-reactive protein (CRP) D) Chemokines E) CCR5 coreceptor

A) Clusters of differentiation (CD) Morphologic differentiation of lymphocytes is difficult and visual inspection of a blood smear cannot distinguish between T and B cells. Fortunately, lymphocytes can be distinguished by the presence of lineage-specific membrane markers, termed clusters of differentiation (CD), (A). Mature T cells are CD4 or CD8 and B cells are CD20. Identification of the subtype of lymphocyte is not a routine clinical hematology test; lymphocytes are reported as a total lymphocyte count on the complete blood count (CBC). An exception is the reporting/monitoring of CD4 cells for patients with human immunodeficiency virus (HIV).

A group of parents approach you because they are concerned some student-athletes may be using human growth hormone in an attempt to increase muscle mass. They inquire how to test for this agent. Of the following, what would be your best response? A) Currently, there is no definitive test for exogenously administered recombinant HGH and surrogate markers must be utilized. B) While there may be some lean mass accretion, there are no data to confirm an actual ergogenic benefit, so the parents should not be concerned. C) Currently, there is no definitive test for exogenously administered recombinant HGH; monitoring for a SE profile is the best way to detect its use. D) Set up a program and threshold levels and begin measuring serum levels of HGH.

A) Currently, there is no definitive test for exogenously administered recombinant HGH and surrogate markers must be utilized. While isomeric measurement testing is still in developmental stages and may become viable, currently, unless the serum is analyzed within 24 hours of exogenous administration, much speculation and uncertainty exists with regard to identifying illicit use of HGH, thus, surrogate markers such as insulin-like growth factor-1 are used. Choice (B) is incorrect. While current evidence suggests little, if any, ergogenic effect, and many harmful effects, illicit drug use for performance-enhancing purposes should be discouraged. Choice (C) is incorrect. Many SE are associated with other drugs as well (eg, fluid retention) and are long term (eg, acromegaly) and so may not be readily apparent. Choice (D) is incorrect. Measuring serum levels for exogenously administered HGH has not been adequately defined.

Which of the following is generally considered as a narrow therapeutic ratio drug? A) Cyclosporine B) Prednisone C) Mycophenolate mofetil D) Mycophenolate sodium

A) Cyclosporine Cyclosporine is generally considered to have a narrow therapeutic ratio as the blood concentration ranges to effectively prevent rejection overlap with that of nephrotoxic potential, and cyclosporine-concentration monitoring is required. The FDA does not formally designate the narrow therapeutic ratio drugs. According to 21 CFR 320.33(c), narrow therapeutic ratio is defined as follows: 1. There is less than a two-fold difference in median lethal dose (LD50) and median effective dose (ED50) values or there is less than a two-fold difference in the minimum toxic concentrations and minimum effective concentrations in the blood, and 2. Safe and effective use of the drug products requires careful titration and patient monitoring. Choices (B), (C), and (D) are incorrect. Prednisone and mycophenolate derivatives do not meet the FDA criteria for narrow therapeutic ratio drugs and serum concentrations are not typically monitored for these medications

Which of the following is a clinical point about DMARD therapy/management? A) DMARDs reduce or prevent joint damage in RA. B) Onset of action is usually 1 to 2 weeks. C) DMARDs are reserved for use in severe long-term RA. D) If a patient fails one DMARD, they will likely fail all DMARDs.

A) DMARDs reduce or prevent joint damage in RA. Disease-modifying antirheumatic drugs (DMARDs) have been shown to slow or prevent disease progression. Answer (B) is incorrect. DMARDs have a typical onset of 1 to 6 months. Answer (C) is incorrect. It is recommended to initiate DMARDs within 3 months of the onset of symptoms. All RA patients are candidates for DMARD therapy except those with very limited disease or who have very severe disease in which little reversibility is expected. Answer (D) is incorrect. If one DMARD does not provide sufficient control of RA, the dose should be increased or additional DMARDs should be added.

Insomnia and anxiety are common adverse events with bupropion, which of the following counseling points should be discussed with VX to minimize the side effects? A) Do not take the second dose after 5 o'clock in the evening. B) If the patient develops insomnia, omit the second dose of the day. C) Insomnia is a temporary adverse event and will resolve approximately 7 days after increasing the dose to twice daily. D) The insomnia and anxiety are most likely due to nicotine withdrawal and will resolve approximately 7 to 10 days after quitting smoking.

A) Do not take the second dose after 5 o'clock in the evening. Answer a is correct. In order for a patient to avoid such adverse events as insomnia and anxiety that interfere with restful sleep, the patient should avoid taking the second dose of the medication after 5 o'clock in the evening or 5 hours before a scheduled bedtime. Answer b is incorrect. While the patient may omit the second dose of the day to avoid evening and night adverse events, it also jeopardizes his or her ability to be able to deal with the withdrawal symptoms or cravings that the bupropion is attempting to alleviate. Answer c is incorrect. The insomnia is not a temporary effect and should be expected to continue throughout the course of therapy. If the patient is told to try and wait out the adverse event, the wakefulness and inability to sleep may lead to restarting smoking. Answer d is incorrect. Insomnia and irritability are signs and symptoms of nicotine withdrawal, but may also be attributed to the bupropion therapy. It is not appropriate to attribute these effects to the lack of nicotine and not appropriately address the adverse drug reaction.

Which of the following medications is associated with torsades de pointes? A.) Erythromycin B.) Ampicillin C.) Atenolol D.) Verapamil E.) Propafenone

A.) Erythromycin Erythromycin can prolong the QTc interval and increase the risk of torsades de pointes.

What antimicrobial has the same oral and IV dose? A) Doxycycline B) Amoxicillin/clavulanate C) Piperacillin/tazobactam D) Ceftriaxone E) Ciprofloxacin

A) Doxycycline Doxycycline (A) is available in oral and parenteral formulations. The pneumonia dose of doxycycline for oral and IV utilization is 100 mg every 12 hours. Amoxicillin/clavulanate (B) is available in an oral formulation only. Amoxicillin/clavulanate is available in immediate and extended release formulations. The immediate release dose of amoxicillin/clavulanate is 500 mg every 8 hours or 875 mg every 12 hours; the dose for the extended release preparation is 2,000 mg every 12 hours. Piperacillin/tazobactam (C) is available in a parenteral formulation only. The pneumonia dose of piperacillin/tazobactam is 3.375 g every 4 to 6 hours or 4.5 g every 6 hours. Ceftriaxone (D) is available in a parenteral formulation only. The pneumonia dose of ceftriaxone is 1 g daily. Ciprofloxacin (E) is available in oral and parenteral formulations; however, the dose is different based upon the formulation utilized. The oral dose for ciprofloxacin is 500 to 750 mg every 12 hours. The IV dose is 400 mg every 8 to 12 hours. Note: Fluoroquinolones are often referred to as medications that have excellent bioavailability. The majority of fluoroquinolones have complete absorption; however, the bioavailability of ciprofloxacin is 50% to 85%, hence the difference in the oral and IV dose of ciprofloxacin. Levofloxacin is the only other fluoroquinolone that is available in oral and IV formulation. The absorption of levofloxacin is rapid and complete. The oral and IV dose of levofloxacin is the same (500-750 mg daily).

What is presently the best course of treatment for Ebola? A) Fluid replacement, ventilation, and additional supportive care as needed B) High dose ribavirin C) Cryotherapy to drop the core temperature to <95°F D) A cocktail of acyclovir, protease inhibitor, and interferon

A) Fluid replacement, ventilation, and additional supportive care as needed Although supportive care with fluid replacement, ventilation, and additional care as needed has a high mortality rate at present, no other course of treatment appears anymore effective. Choice (B) is incorrect. Ribavirin can be used for other types of hemorrhagic fever. It appears ineffective for Ebola. Choice (C) is incorrect. Nothing to support dropping the core temperature at present. Choice (D) is incorrect. No evidence to support this.

Which of the following agents has an adherence concern due to the dosing concern? A) Flutamide B) Bicalutamide C) Nilutamide D) Enzalutamide E) Abiraterone

A) Flutamide Answer a is correct. Flutamide is dose three times daily and therefore adherence is a concern. Answer b is incorrect. Bicalutamide is a once-daily drug and therefore adherence is less of a concern. Answer c is incorrect.Nilutamide is a once-daily drug and therefore adherence is less of a concern. Answer d is incorrect.Enzalutamide is a once-daily drug and therefore adherence is less of a concern. Answer e is incorrect. Abiraterone is a once-daily drug and therefore adherence is less of a concern.

A patient who currently takes oral sumatriptan often experiences headache recurrence, where the headache comes back within 24 hours after a positive response to the medication. Her physician would like a recommendation of a selective 5-HT 1 receptor agonist (triptan) with a longer half-life. Which of the following would you recommend? A) Frovatriptan B) Rizatriptan C) Eletriptan D) Almotriptan

A) Frovatriptan Her current medication, sumatriptan (Imitrex), has an elimination half-life of 2.5 hours. Frovatriptan (Frova), (A), has an elimination half-life of 26 hours. The longer half-life products may benefit a patient who is responsive to triptans but requires a longer-acting medication to last the duration of the headache. Rizatriptan (Maxalt), (B), has a similar elimination half-life of 2 to 3 hours. Eletriptan (Relpax), (C), has a similar elimination half-life of 4 hours. Almotriptan (Axert), (D), has a similar elimination half-life of 3.1 hours.

WF is a 70-year-old man with a recent diagnosis of stage IV colorectal cancer with the primary tumor in the sigmoid colon and multiple metastases found in his liver. The oncologist indicated that cure is not a realistic goal of treatment for WF because of his advanced disease. He is scheduled to begin chemotherapy with the regimen FOLFOXIRI, which contains 5-fluorouracil, leucovorin, oxaliplatin, and irinotecan. In addition, the patient is to receive bevacizumab. Approximately 8 weeks after the beginning of his chemotherapy, WF begins complaining about feeling more tired and weak lately. He also mentions that he becomes short of breath after even moderate physical activity. Blood tests showed a hemoglobin level of 8.6 g/dL. Which of the following is correct regarding WF's anemia? A) He should begin treatment with erythropoietin to treat chemotherapy-induced anemia. B) He should receive a blood transfusion to treat severe anemia from blood loss. C) He should begin treatment with darbepoetin once his hemoglobin level fall below 8 g/dL. D) He is not a candidate for treatment with an erythropoiesis stimulating agent since his cancer is being treated with curative intent.

A) He should begin treatment with erythropoietin to treat chemotherapy-induced anemia. Answer a is correct. Anemia is a common complication from chemotherapy. Typically, treatment with an erythropoiesis stimulating agent (erythropoietin or darbepoetin) will be considered in these patients. Due to concerns over safety, including evidence that shows these drugs can worsen cancer outcomes, the erythropoietin stimulating agents should only be used in patients who are receiving chemotherapy for palliative intent. That is to say, the cancer is considered to be incurable. Answer b is incorrect. The patient is experiencing mild to moderate symptoms of anemia and blood transfusion is not typically used in this situation. Most clinician will consider transfusion when hemoglobin falls below 7 g/dL or if patient is experiencing severe symptoms such as hypotension or shortness of breath at rest. Answer c is incorrect. There is no need to wait for the hemoglobin to fall below 8 g/dL prior to initiation of darbepoetin. Answer d is incorrect. As stated in the case scenario, WF's cancer is considered to be incurable. This indicates that his cancer treatment is being used with palliative intent.

Intravenous dosing of phenytoin cannot be infused faster than 50 mg/min. Select the adverse reaction that is associated with infusions faster than 50 mg/min. A) Hypotension B) Gingival hyperplasia C) Anemia D) Rash

A) Hypotension The correct answer is (A). The dose cannot be infused faster than 50 mg/min due to the potential risks of hypotension and arrhythmias due to the propylene glycol diluent. Answers (B), (C) and (D) are incorrect. Gingival hyperplasia, anemia and rash are idiosyncratic reactions and not related to infusion rates or dose.

On binding of antigen displayed by the antigen-presenting cell to the T-cell receptor complex, what additional step is required for T-helper-cell activation? A.) Binding of the co-stimulatory pathway B.) Activation of the promoter gene NFAT C.) Transcription of the IL-2 gene D.) No additional step required E.) Translation of IL-2

A.) Binding of the co-stimulatory pathway Activation is dependent on antigen-HLA binding to the T-cell receptor complex and the subsequent binding of a second signal or "co-stimulatory pathway."

LK is a 37-year-old HIV-negative new patient at the clinic you work at. He receives a Mantoux skin test that returns positive 2 days later. He was born in the United States and works as a prison guard. He injects heroin on a regular basis. His chest x-ray comes back normal, he has no symptoms of tuberculosis, and his smear culture is negative. What type of drug therapy would be appropriate for this patient? A) Isoniazid 300 mg daily × 9 months B) Rifampin 100 mg daily × 4 months C) Isoniazid 300 mg and rifampin 600 mg × 6 months D) Isoniazid, rifampin, ethambutol, and pyrazinamide

A) Isoniazid 300 mg daily × 9 months The patient does not have any symptoms or indications of active TB disease; so he needs treatment for latent TB infection. This is the correct first-line regimen for treatment of latent TB infection.

Which opioid is associated with both proconvulsant activities in overdose? A) Meperidine B) Methadone C) Hydrocodone D) Heroin

A) Meperidine Answer a is correct. Seizures are associated with accumulation of the meperidine metabolite normeperidine. Answer b is incorrect. Methadone is associated with prodysrhythmic properties even in therapeutic levels as it can prolong QT/QTc interval, potentially precipitating torsades de pointes; however, seizures are not a frequent adverse effect. Answer c is incorrect. Hydrocodone is not commonly associated with intrinsic proconvulsant or prodysrhythmic activity in overdose. Answer d is incorrect. Heroin is not commonly associated with intrinsic proconvulsant or prodysrhythmic activity in overdose.

Which of the following is referred to as an "inodilator," having both inotropic and vasodilatory properties? A) Milrinone B) Dobutamine C) Nesiritide D) Nitroprusside

A) Milrinone Answer a is correct. Milrinone decreases the breakdown of cyclic adenosine monophosphate (cAMP) in cardiac tissue, resulting in an increase in cardiac contractility and output. By increasing cAMP in vascular smooth muscle, systemic and pulmonary vascular resistance are reduced, thus milrinone is often referred to as an "inodilator" (inotrope and vasodilator). Answer b is incorrect. Dobutamine is an inotrope. While dobutamine stimulates β-2 receptors in the periphery causing mild vasodilation, it also stimulates alpha receptors, which counteracts with mild vasoconstriction. Therefore, the decrease in SVR that occurs with dobutamine is a reflexive response to the increase in cardiac output. Answers c and d are incorrect. Nesiritide (Answer c) and nitroprusside (Answer d) are potent arterial and venous vasodilators. These agents dilate the arterial vessels (decrease in SVR) and cause a reflex increase in cardiac output.

You are counseling a patient who is being discharged today. He received a living related renal transplant 5 days ago, and his postoperative course has been uncomplicated except for mild hypertension. When reconciling his home medications, you notice that the medical team has not restarted his home diltiazem. What course of action do you take? A) Notify the patient's medical team and request a discharge prescription for amlodipine. B) Notify the patient's medical team and instruct the patient to resume his home regimen of diltiazem after discharge. C) Notify the patient's medical team and request a discharge prescription for verapamil. D) Notify the patient's medical team and request addition of diltiazem at discharge.

A) Notify the patient's medical team and request a discharge prescription for amlodipine. Many patients will experience higher postoperative blood pressures following solid organ transplant due to high doses of steroids and calcineurin inhibitors. Patients with preexisting hypertension may have a more difficult time maintaining blood pressure control, while patients who receive renal transplants may experience resolution of their hypertension with resolution of their kidney disease. While the nondihydropyridine calcium channel blockers, such as diltiazem and verapamil may be appropriate choices for some patients prior to transplant, they may complicate management of calcineurin inhibitors postoperatively, due to their interaction-mediated via CYP 3A and P-glycoprotein. Dihydropyridine calcium channel blockers, such as nifedipine and amlodipine, have less potential for clinically significant pharmacokinetic interactions with calcineurin inhibitors. Answers (B), (C), and (D) are incorrect. The nondihydropyridine calcium channel blockers (diltiazem and verapamil) inhibit CYP3A and P-glycoprotein, resulting in elevated concentrations and toxicity of calcineurin inhibitors. These agents can be used safely, with close monitoring of trough levels of calcineurin inhibitors, but reinitiation of these agents at discharge is not appropriate.

Select the upper respiratory tract condition that is defined as the presence of fluid in the middle ear without symptoms of acute illness. A) OME B) Sinusitis C) Pharyngitis D) Laryngitis E) Rhinitis

A) OME Answer a is correct. OME is the presence of fluid in the middle ear without symptoms of acute illness. It is important to differentiate between OME and AOM because antimicrobials are only useful for AOM. AOM is a symptomatic middle ear infection that occurs rapidly with effusion. Answer b is incorrect. Sinusitis is an inflammation and/or infection of the paranasal sinus mucosa. Answer c is incorrect. Pharyngitis is an acute throat infection caused by viruses or bacteria. Answer d is incorrect. Laryngitis is a common and acute inflammation of the larynx that is usually caused by acute vocal strain, irritation of the mucosal surface of the larynx, or an URTI. Answer e is incorrect. Rhinitis is the presence of any one of the following: sneezing, nasal congestion, rhinorrhea, or nasal itching.

JJ is a 49-year-old man who experiences headache cycles two times a year, usually in the spring and fall. The headaches occur for about 3 to 4 weeks. Each day during the headache series he may have up to 5 headaches. He describes the headaches as an unbearable type of pain that comes very suddenly, located in his left eye, and goes within 1 to 2 hours. He experiences severe ocular and nasal symptoms, such as nasal stuffiness or rhinorrhea, ocular lacrimation, and ptosis. He tells you that in order to attempt to stop the pain, he sometimes rubs the areas of pain or even beats his head against objects. Which of the following are appropriate abortive treatment options for this patient's headache? A) Oxygen B) Imitrex (sumatriptan) tablets C) Amitriptyline D) Nortriptyline

A) Oxygen Oxygen (A) administered at 5 to 10 L/min by nonrebreather facemask for approximately 15 minutes is a first-line abortive treatment for cluster headache. This patient exhibits primary symptoms of cluster headache as per the International Headache Society Diagnostic Criteria for Migraine: Headache is unilateral, orbital in location, lasting for 15 to 180 minutes. Nasal congestion, rhinorrhea, ocular lacrimation, and ptosis are present. Frequency of headaches lasts from 7 days to 1 year. Due to the duration of cluster headache, sumatriptan tablets (B) will not act quick enough to have an impact before the headache dissipates. Amitriptyline (C) and nortriptyline (D) are prophylactic options for migraine and tension headache. There are no data to support their use as an abortive therapy for cluster headache.

Which of the following patients with prostate cancer can be managed with observation alone? A) Patientwith a Gleason score of 3 and a PSA value of 5 ng/mL (5 µg/L) B) Patientwith a Gleason score of 8 and a PSA value of 40 ng/mL (40 µg/L) C) Patient with a Gleason score of 7 and a PSA value of 15 ng/mL (15 µg/L) D) Patient with a Gleason score of 7 and a PSA value of 20 ng/mL (20 µg/L

A) Patientwith a Gleason score of 3 and a PSA value of 5 ng/mL (5 µg/L) Asymptomatic patients with a low risk of recurrence [a Gleason score of 2 to 6, and a PSA value less than 10 ng/mL (10 µg/L)] may be managed by observation, radiation (external beam or brachytherapy), or radical prostatectomy. Answer (B) is incorrect. The treatment of patients with a high risk of recurrence [a Gleason score of 8 to 10, and a PSA value greater than 20 ng/mL (20 µg/L)] should be treated with androgen ablation for 2 to 3 years, combined with radiation therapy. Answer (C) is incorrect. Individuals with moderate disease [a Gleason score of 7 and a PSA value ranging from 10 ng/mL to 20 ng/mL (10 µg/L to 20 µg/L)] are considered at intermediate risk for prostate cancer recurrence. Answer (D) is incorrect. Individuals with moderate disease o[a Gleason score of 7 and a PSA value ranging from 10 ng/mL to 20 ng/mL (10 µg/L to 20 µg/L)] are considered at intermediate risk for prostate cancer recurrence.

A patient is newly initiated on erlotinib for the treatment of nonsmall cell lung cancer. Which of the following medications may pose a drug/drug interaction with his new medication? A) Phenytoin B) Pregabalin C) Heparin D) Sulfamethoxazole/trimethoprim

A) Phenytoin Answer a is correct. Many kinase inhibitors, including erlotinib have multiple drug interactions through the CYP450 system. Erlotinib is an inhibitor and substrate of 3A4 and is thus sensitive to other substrates of the enzyme as well as inducers and inhibitors of the enzyme, such as amiodarone. Answer b is incorrect. Erlotinib has drug/drug interactions with many anticonvulsants due to activity related to CYP450 3A4. Pregabalin does not affect this enzyme system. Answer c is incorrect. Heparin does not have drug/drug interaction with erlotinib. However, erlotinib can cause myelosuppression, including thrombocytopenia, and should be used with caution in patients receiving medications that can increase risk of bleeding. Answer d is incorrect. There are no known drug interactions between sulfamethoxazole/trimethoprim and erlotinib.

Which AED has an unbound (free) concentration of 1 to 2 &mgr;g/mL? (Unbound refers to "not bound to serum protein/albumin.") A) Phenytoin B) Carbamazepine C) Valproic acid D) Oxcarbazepine

A) Phenytoin Phenytoin (A) is highly protein bound. The usual total (bound and unbound) concentration for phenytoin is 10 to 20 &mgr;g/mL; however, the unbound (free) concentration is 1 to 2 &mgr;g/mL. The unbound concentration is the component that produces seizure control and adverse reactions. Carbamazepine (B) is highly protein bound (75%-90%), but free carbamazepine levels are not monitored. Valproic acid (C) is highly protein bound, but free valproic acid levels are not monitored. Oxcarbazepine (D) does not have established serum concentrations.

Which of the following condoms conducts heat very well and also protects against STIs? A) Polyurethane B) Latex C) Lamb cecum D) Polyethylene glycol

A) Polyurethane Polyurethane condoms conduct heat better than latex condoms. They also protect against STIs, however, they may break easier than other condoms.

A 79-year-old man weighing 80 kg presents to your hospital emergency room with newly discovered AF with rapid ventricular response and is symptomatic with little rate control achieved after initiation of a diltiazem drip. It is decided by the attending physician to proceed with electrical cardioversion. The patient's wife reports that he underwent a cardiac workup the day before for routine knee replacement surgery. The ECG from that workup is retrieved and shows normal sinus rhythm. What is the next appropriate step in this patient's care? A) Proceed with synchronized direct cardioversion after receiving enoxaparin 80 mg. B) Anticoagulate with warfarin for 3 weeks, target INR 2.0 to 3.0, then cardioversion. C) Obtain transesophageal ECHO to rule out thrombus then cardioversion. D) Proceed with synchronized direct current cardioversion without anticoagulation.

A) Proceed with synchronized direct cardioversion after receiving enoxaparin 80 mg. The first goal of managing a patient presenting with AF is rate control. This has been attempted with the diltiazem drip. However, the patient is still very symptomatic. Because of the symptoms, immediate cardioversion should be considered. Answer a is correct. Because the patient has been in AF for less than 48 hours, it is appropriate to proceed with immediate cardioversion as long as the patient is anticoagulated with full VTE treatment dose heparin, full treatment dose enoxaparin, a therapeutic INR on warfarin, or currently on maintenance therapy with one of the new oral anticoagulants (NOAC). Answer b is incorrect. Because the patient is still symptomatic despite rate control, an attempt should be made to cardiovert now. Answer c is incorrect. Because we know that the patient's AF has been occurring for less than 48 hours, it is appropriate to proceed with cardioversion without the TEE. Answer d is incorrect. Patients receiving DCC should be cardioverted only after receiving full dose anticoagulation.

Which of the following agents is most likely to cause hypercalcemia? A.) Calcitriol B.) Doxercalciferol C.) Ergocalciferol D.) Paricalcitol E.) Sevelamer

A.) Calcitriol Calcitriol is the active form of vitamin D (1,25-dihydroxyvitamin D3) and as a D3 agent is more likely to cause hypercalcemia. Doxercalciferol and paricalcitol are D2 agents and are associated with less risk of hypercalcemia and hyperphosphatemia. Ergocalciferol is the precursor form of active vitamin D and is also a D2. Sevelamer is a phosphate binder that does not contain calcium and does not cause hypercalcemia.

Which of the following UA findings would be indicative of acute GN? A) Protein B) Muddy brown casts C) pH 8.0 D) Eosinophils

A) Protein Answer a is correct. Proteins are large molecule substances that should not be able to cross the barriers in the glomerulus to be excreted in the urine. The presence of proteinuria is most indicative of glomerular nephritis. Answer b is incorrect. Muddy brown casts are the cellular components of necrotic tubular epithelial cells. This is the hallmark finding of ATN. Answer c is incorrect. The pH of the urine may have little to do with the pathophysiology of AKI. A high pH may predispose crystal formation of alkaline substances. This information would be more useful in the evaluation of postobstructive nephropathy. Answer d is incorrect. The presence of eosinophils in the urine is unusual and would be more indicative of a hypersensitivity reaction (AIN).

MN is a 23-year-old obese female diagnosed with PTSD after a car accident several years ago. Although MN has been treated with an SSRI and other agents, she still has recurrent, disturbing dreams of the event with minor daytime hallucinations. Her psychiatrist wants to prescribe a second generation antipsychotic for augmentation therapy and would like to avoid a medication with weight gain. Which agent do you most appropriately recommend as a pharmacist? A) Risperidone B) Olanzapine C) Quetiapine D) Haloperidol

A) Risperidone Answer a is correct. Among the antipsychotics, risperidone carries the least amount of weight gain and has data to support its use in PTSD treatment. Answer b is incorrect. Olanzapine is the antipsychotic with the highest propensity to cause weight gain/metabolic changes Answer c is incorrect. Quetiapine also carries significant weight gain. Answer d is incorrect. Second generation (atypical) neuroleptics are preferred for augmentation therapy.

GC is a 58-year-old man with a recent diagnosis of stage IV colon cancer. His prior medical history is significant for hypertension (on lisinopril and hydrochlorothiazide [HCTZ]), deep vein thrombosis (on warfarin), and atrial fibrillation (on amiodarone). The oncologist informs the patient the plan is for him to receive neoadjuvant chemotherapy followed by surgery. The oncologist informs GC that he will receive the chemotherapy regimen XELOX (capecitabine and oxaliplatin). XELOX: Capecitabine 1000 mg/m2 orally twice daily for 14 days Oxaliplatin 130 mg/m2 IV on day 1 Repeat cycle every 3 weeks Which of the following points should the pharmacist council GC on regarding his capecitabine? A) Take tablets with food. B) If he has trouble swallowing the tablets, crush them and mix in applesauce. C) Avoid eating grapefruit or drinking grapefruit juice while taking capecitabine. D) Avoid drinking cold liquids while taking capecitabine.

A) Take tablets with food Answer a is correct. Capecitabine should be taken with food. Answer b is incorrect. Capecitabine is a chemotherapeutic agent, as such it should never be crushed or otherwise adulterated except by professionals trained in safe handling of cytotoxic drugs using appropriate equipment and safety procedures. Answer c is incorrect. There is no drug-food interaction with grapefruit products. Answer d is incorrect. This is a counseling point related to cold-intolerance related to oxaliplatin, not capecitabine.

Which of the following represents an appropriate counseling point for patients prescribed a phosphate binder? A) Take with meals to reduce phosphate absorption B) Take with meals to increase phosphate absorption C) Take with meals to reduce GI side effects D) Take between meals to reduce food-drug interactions

A) Take with meals to reduce phosphate absorption Answer a is correct. Phosphate binders are to be given with meals to minimize the absorption of dietary phosphate. Answers b, c, and d are incorrect. Phosphate binders are taken with meals to reduce not increase phosphate absorption.

MG is a 62-year-old woman with a prior medical history of CKD, hypertension, and stage II colon cancer. Her social history is significant for a long standing history of alcoholism. She underwent surgical resection of her primary cancer 6 months ago and is currently undergoing chemotherapy with the intent to cure her cancer. Routine laboratory monitoring revealed the patient has a hemoglobin level of 8.8 g/dL. Due to her social history, MG should be evaluated for his anemia being complicated by deficiency in: A) Vitamin B12 B) Iron C) Vitamin D D) Thiamine

A) Vitamin B12 Answer a is correct. MG has a long standing history of chronic alcoholism. Vitamin B12 deficiency is a frequent complication that is seen in this patient population. It is important to assess his vitamin B12 status because deficiency could lead to neurological complications in addition to anemia. Answer b is incorrect. Iron deficiency is not correlated to alcoholism. Although if this patient were to start ESA therapy, an assessment of iron status would be warranted. Answer c is incorrect. Vitamin D deficiency is not associated with anemia. Answer d is incorrect. Thiamine deficiency commonly occurs in patients with chronic alcoholism; however, it is not associated with anemia.

MG is a 62-year-old woman with a prior medical history of CKD, hypertension, and stage II colon cancer. Her social history is significant for a long standing history of alcoholism. She underwent surgical resection of her primary cancer 6 months ago and is currently undergoing chemotherapy with the intent to cure her cancer. Routine laboratory monitoring revealed the patient has a hemoglobin level of 8.8 g/dL. Due to her social history, MG should be evaluated for his anemia being complicated by deficiency in: A) Vitamin B12 B) Iron C) Vitamin D D) Thiamine

A) Vitamin B12 Answer a is correct. MG has a long standing history of chronic alcoholism. Vitamin B12 deficiency is a frequent complication that is seen in this patient population. It is important to assess his vitamin B12 status because deficiency could lead to neurological complications in addition to anemia. Answer b is incorrect. Iron deficiency is not correlated to alcoholism. Although if this patient were to start ESA therapy, an assessment of iron status would be warranted. Answer c is incorrect. Vitamin D deficiency is not associated with anemia. Answer d is incorrect.Thiamine deficiency commonly occurs in patients with chronic alcoholism; however, it is not associated with anemia.

BS is a 24-year-old man who comes to your pharmacy and states, "What can my girlfriend and I use to make sure she doesn't get pregnant and protect ourselves from, you know...diseases?" Select the best regimen to recommend from the following choices below. A) Water-based lubricant + female condom B) Oil-based lubricant + male latex condom C) Female condom + male latex condom D) Female condom + male lamb cecum condom

A) Water-based lubricant + female condom Water-based lubricant is safe to use with latex condoms and do not compromise its ability to protect against STIs

Which of the following agents is not a cardioselective beta-blocker? A.) Carvedilol B.) Atenolol C.) Metoprolol D.) Nebivolol E.) Bisoprolol

A.) Carvedilol Carvedilol has combined alpha and beta blocking activity. The other agents are cardioselective and have affinity for beta 1 receptors.

A 49-year-old woman with previous peptic ulcer disease was admitted with persistent vomiting. She looked dehydrated, with dry mucus membranes and skin tenting. Her blood results were Na 141 mEq/L, K 2.6 mEq/L, Cl 87 mEq/L, pH 7.51, Pa CO 2 50 mm Hg, and HCO 3 40 mEq/L. Has the patient been appropriately compensated for the primary acid-base disorder? A) Yes, the PaCO2 is elevated, indicating appropriate compensation. B) Yes, the PaCO2 is low, indicating appropriate compensation. C) Yes, the HCO3 is normal, indicating appropriate compensation. D) Yes, the HCO3 is low, indicating appropriate compensation

A) Yes, the PaCO2 is elevated, indicating appropriate compensation. Because the pH is elevated secondary to metabolic alkalosis, the most appropriate respiratory compensation would be for the lungs to hold on to acid (Pa CO 2 ), resulting in a respiratory acidosis, which is the case here with a Pa CO 2 of 50 mm Hg (A).As indicated, the Pa CO 2 is not low (B), and we know there is a primary metabolic disorder (C and D).

The live herpes zoster vaccine (ZVL) should be stored at what temperature? A) ≤5°F B) 6°F to 35°F C) 36°F to 46°F D) 47°F to 77°F

A) ≤5°F Answer a is correct. The herpes zoster vaccine needs to be kept frozen. Frozen vaccines should be stored in a freezer with a temperature less than 5°F. Answer b is incorrect. Storing the herpes zoster vaccine at temperatures above 5°F can lead to loss of potency. Answer c is incorrect. Vaccines that require refrigeration should be stored at temperatures between 36°F and 46°F. The herpes zoster vaccine should be kept frozen. Answer d is incorrect. Temperatures between 47°F and 77°F are considered room temperature. The herpes zoster vaccine should be kept frozen.

Which of the following beta-blockers has alpha-blocking properties in addition to its beta-blocking properties? A. Carvedilol B. Metoprolol C. Propranolol D. Pindolol E. Nebivolol

A. Carvedilol

Streptococcus pneumoniae manifests its resistance to antibacterial agents through what mechanism? A.) Alteration of penicillin-binding sites B.) Acquisition of the encoding enzyme β-lactamase C.) Extruding the antibacterial agent from the cell through efflux pump D.) Intracellular mutation causing down-regulation of porin genes E.) Acquisition of the encoding enzyme carbenicillinase

A.) Alteration of penicillin-binding sites Streptococcus pneumoniae is responsible for 40-50% of bacterial causes of otitis media. S. pneumoniae promotes its bacterial resistance through alteration of penicillin-binding proteins on the bacterial cell wall, resulting in reduced penicillin affinity. Higher concentrations of medications, like amoxicillin, are required to overcome this resistance. H. influenzae and M. catarrhalis promote bacterial resistance through production of β-lactamase.

Which of the following β-lactam antibiotics displays the best activity against enterococci? A.) Ampicillin B.) Cefepime C.) Meropenem D.) Aztreonam

A.) Ampicillin Ampicillin is a drug of choice for enterococci that are susceptible. No cephalosporins display reliable activity against enterococci.

JB is a 12 year old boy (23kg) recently diagnosed with seizures. JB has a PMH of seasonal allergies. He was recently initiated on oxcarbazepine to be titrated up to a maintenance dose of 300mg BID. In case of breakthrough seizures at home while titrating up his dose, his physician wrote for a prescription of Diastat 5mg. Select all of the following that are true related to the dispensing of Diastat. A. The pharmacist should remove Diastat from the casing and set and lock the dosage to 5mg before dispensing to the patient B. Diastat must be stored in the refrigerator until used C. Diastat should be used for seizures that are different from the patient's ordinary seizure activity D. All remaining medication in the syringe should be discarded after administration E. JB should be placed flat on his stomach while the medication is administered

A. The pharmacist should remove Diastat from the casing and set and lock the dosage to 5mg before dispensing to the patient C. Diastat should be used for seizures that are different from the patient's ordinary seizure activity D. All remaining medication in the syringe should be discarded after administration Diastat (diazepam) is a commonly dispensed benzodiazepine used in the management of seizures. Answer A is correct as pharmacists MUST lock the syringe with the correct dose prior to dispensing the medication. Diastat comes in 10mg and 20mg syringes which can deliver minimum dosages of 5mg and 12.5mg respectively in increments of 2.5mg. The pharmacist must lock the dose even if the prescription was written for the max dose available in the syringe. Answer B is incorrect because Diastat should be stored at room temperature between 59 and 86 degrees Fahrenheit. Answer C is correct as Diastat is not recommended for chronic, daily use because of the potential of tolerance. Chronic daily use can increase the frequency and/or severity of tonic clonic seizures. Patients and prescribers must be properly trained to determine when it is appropriate to administer Diastat. Answer D is correct. While there will likely be medication remaining in the syringe, the remaining medication should be properly disposed of after single use in a sink or toilet. The actual plunger can then be placed in the trashcan away from children. Answer E is incorrect because patients receiving Diastat should be placed on their side with their upper leg forward to expose the rectum rather than on their back.

Tobramycin (formula weight = 468) has a dissociation factor of 1.0. What is its sodium chloride equivalent? A.) 0.069 B.) 0.0092 C.) 0.117 D.) 0.286 E.) 0.782

A.) 0.069 sodium chloride equivalent = (58.5)(1.0)/(468)(1.8) = 0.069

In a patient with severe sepsis or septic shock, appropriate antimicrobial therapy should be started within how many hours after the onset of symptoms? A.) 1 B.) 3 C.) 6 D.) 12 E.) 24

A.) 1 EXPLANATION: The Surviving Sepsis Campaign guidelines indicate that antimicrobials should be started within 1 hour of the onset of symptoms.

Which of the following administration rates is the most appropriate for IV magnesium sulfate when treating hypomagnesemia in the ICU? A.) 1 g/hr B.) 10 mEq/hr C.) IV push over 10 minutes D.) 7.5 mmol/hr E.) 60 mg/min

A.) 1 g/hr EXPLANATION: One gram per hour is the maximum administration rate for magnesium sulfate when it is given for repletion.

What is the most appropriate calcium intake (mg/d) for adolescents and young adults? A.) 1,200-1,500 mg/d B.) 1,000 mg/d C.) 500-1,000 mg/d D.) 1,700-2,200 mg/d

A.) 1,200-1,500 mg/d EXPLANATION: The calcium requirement is 1,200-1,500 mg/d for adolescents and young adults.

C.P. is a 31 year old African American male admitted for his fifth psychiatric admission thought to be attributed to non-adherence to his medication regimen. After two weeks C.P. is stabilized on fluphenazine 5mg, 1 tablet po BID. The psychiatrist wants to transition him to the long acting fluphenazine decanoate injection. What dose and frequency of fluphenazine decanoate would be appropriate for C.P.? A.) 12.5 mg IM every 2 weeks B.) 25 mg IM every 4 weeks C.) 100 mg IM every 4 weeks D.) 37.5 mg IM every 3 weeks E.) 12.5 mg IM every 4 weeks

A.) 12.5 mg IM every 2 weeks Fluphenazine decanoate conversion from PO to long-acting IM is 1.25 times the total oral daily dose and is given every two weeks. Haloperidol decanoate conversion from PO to long-acting IM is 10 times the total oral daily dose and is given every three to four weeks.

What is the recommended minimum leukocyte count prior to a patient receiving strontium-89 chloride? A.) 2,400 B.) 6,200 C.) 8,800 D.) 10,000

A.) 2,400 Any level over 2,400 is above the minimum level of leukocytes recommended before a patient receives strontium-89 chloride.

The term very low sodium may be used in the labeling of over-the-counter drug products intended for oral ingestion if the amount of sodium in the labeled maximum daily dose is ______ or less. A.) 35 mg B.) 70 mg C.) 140 mg D.) 280 mg E.) 560 mg

A.) 35 mg The applicable DEA regulation is 21 CFR 201.64.

What is the therapeutic range for theophylline based on the NIH guidelines for asthma management? A.) 5-15 mcg/mL. B.) 8-12 mcg/mL. C.) 10-20 mcg/mL. D.) 15-25 mcg/mL. E.) 10-15 mcg/mL.

A.) 5-15 mcg/mL. The currently (2017) accepted range for asthma is 5-15 mcg/mL (not the old range of 10-20 mcg/mL). There is no benefit in exceeding 15 mcg/mL, and many patients receive benefit at lower doses.

What is the most appropriate daily protein intake for a healthy adult male weighing 70 kg? A.) 56 g B.) 70 g C.) 84 g D.) 126 g

A.) 56 g The adult RDA for protein in the United States is 0.8 g/kg/day. For an adult who weighs 70 kg, the total daily protein intake would be 70 kg × 0.8 g/kg/day = 56 g/day.

Which of the following agents has the potential to decrease oral contraceptive effectiveness? A.) Actos B.) Metformin C.) Saxagliptin D.) Acarbose E.) Insulin

A.) Actos EXPLANATION: Actos (pioglitazone) can decrease oral contraceptive effectiveness and may cause resumption of ovulation in anovulatory women.

patient calls your pharmacy complaining of symptoms of swelling of the lips, mouth and face. The patient is having some difficulty talking and some issues breathing. EMS has been called to the patient's home and you begin to discuss with the family about the patient's current medications. Which of the following agents might you also avoid if asked to recommend an alternative agent? A.) Aliskiren B.) Amlodipine C.) Metolazone D.) Spironolactone E.) Carvedilol

A.) Aliskiren Direct renin inhibitors can cause angioedema and should probably be avoided as an alternative unless the cause of the angioedema is not drug related. If so it would be used cautiously and the benefit would have to outweigh the risk for the patients.

Hepatobiliary imaging agents follow the same pathway as: A.) Bilirubin B.) Saline C.) Liver enzymes D.) Factor 9

A.) Bilirubin

Which of the following beta-blockers has alpha-blocking properties in addition to its beta-blocking properties A.) Carvedilol B.) Metoprolol C.) Propranolol D.) Pindolol E.) Nebivolol

A.) Carvedilol Correct answer: A Carvedilol(Coreg, Coreg CR) is an example of a beta-blocker with additional alpha-blocking properties. Labetalol (Trandate) is another beta-blockers (BB) with mixed beta/alpha activity.Patients starting these medications should be titrated to the appropriate dose due to the orthostatic hypotension that can occur. Nebivolol (Bystolic) also requires dose titration due to this adverse effect, but its mechanism is related to an increase in nitric oxide release instead of additional alpha-blocking activity. Answer B:Metoprolol is a β1selective BB, which is more cardio-selective and better tolerated in patients with asthma/COPD. It does not have any alpha blocking activity. Answer C:Propranolol is the BB indicated for use in various conditions such as migraine prophylaxis, tremors, and pheochromocytoma. It does not have any alpha-blocking activity. Answer D: Pindolol is an example of a BB with intrinsic sympathomimetic activity, meaning it has partial effects on beta-receptors and less bradycardia compared to other BBs. Other BBs with intrinsic sympathomimetic activity include acebutolol and penbutalol. Pindolol does not have any alpha-blocking activity. Answer E:Nebivolol is the only BB that has additional nitric oxide-releasing activity. It does not have any alpha-blocking activity. Bottom Line:Beta-blockers have the same general mechanism of action, but can be differentiated based on their selectivity, additional unique mechanisms, and availability of intravenous formulations. Beta-blockers with additional alpha activity include carvedilol (Coreg, Coreg CR) and labetalol (Trandate). Beta-blockers with ISA activity include pindolol, penbutolol, acebutolol, and carteolol (CAPP)

Which medication has a US Boxed Warning for the potential to cause an increased risk of serious adverse cardiovascular thrombotic events, including myocardial infarction and stroke? A.) Celebrex B.) Empagliflozin C.) Colchicine D.) Wellbutrin E.) Duloxetine

A.) Celebrex Correct answer: A All nonsteroidal anti-inflammatory drugs (NSAIDs), includingcelecoxib (Celebrex)have US Boxed Warnings for an increased risk of serious adverse cardiovascular thrombotic events, including myocardial infarction (MI) and stroke. This risk may occur early during treatment, but increases with duration of use.The relative risk of cardiovascular thrombotic events appears to be similar in those with and without known cardiovascular disease or risk factors for cardiovascular disease. However, the absolute incidence of cardiovascular events has been shown to be higher in patients with known disease or risk factors.New onset hypertension or exacerbation of hypertension may occur with NSAID use. NSAIDs also can cause sodium and fluid retention, so they should be used with caution in patients with edema. In general, they should be avoided in patients with heart failure or recent MI. Answer B:Empagliflozin does not have a US Boxed Warning for the potential to cause an increased risk of serious adverse cardiovascular thrombotic events. In fact, post-hoc analysis of the EMPA-REG OUTCOME trial suggests use of empagliflozin in diabetic patients with cardiovascular disease and renal impairment may be associated with decreased cardiovascular mortality. Answer C:Colchicine is an anti-inflammatory often used to treat gout. Colchicine does not have a US Boxed Warning for the potential to cause an increased risk of serious adverse cardiovascular thrombotic events. Answer D:Wellbutrin is an anti-depressant used to treat major depressive disorder and seasonal affective disorder. Wellbutrin does not have a US Boxed Warning for the potential to cause an increased risk of serious adverse cardiovascular thrombotic events. Answer E:Duloxetine is a selective serotonin and norepinephrine reuptake inhibitor antidepressant (SSNRI) used to treat depression. Duloxetine does not have a US Boxed Warning for the potential to cause an increased risk of serious adverse cardiovascular thrombotic events. Bottom Line:All nonsteroidal anti-inflammatory drugs (NSAIDs), including celecoxib (Celebrex) have US Boxed Warnings for an increased risk of serious adverse cardiovascular thrombotic events, including myocardial infarction and stroke.

Which sexually transmitted disease is caused by an atypical bacterial pathogen? A.) Chlamydia B.) Gonorrhea C.) Syphilis D.) Herpes E.) Trichomoniasis

A.) Chlamydia EXPLANATION: Chlamydia is caused by an atypical bacterium and requires treatment with either a macrolide or tetracycline. Gonorrhea and syphilis are both bacterial sexually transmitted diseases but are not caused by atypical pathogens. Herpes and trichomoniasis are not bacterial sexually transmitted diseases

Which of the following drugs taken with alcohol is most likely to cause a disulfiram-like reaction? A.) Chlorpropamide B.) Acarbose C.) NPH insulin D.) Glucagon E.) Pioglitazone

A.) Chlorpropamide Chlorpropamide has had the greatest reporting of this drug interaction relative to the first-generation sulfonylureas. The remaining drugs listed have not had reports of this adverse drug event.

Which of the following is the preferred treatment for clostridial myonecrosis (gas gangrene)? A.) Clindamycin plus penicillin B.) Trimethoprim-sulfamethoxazole plus penicillin C.) Ampicillin-sulbactam D.) Vancomycin E.) Metronidazole

A.) Clindamycin plus penicillin Clindamcyin plus penicillin is the preferred treatment for clostridial myonecrosis caused by Clostridium species.

Morphine can be used to help evaluate: A.) Cystic duct obstruction B.) Renal artery stenosis C.) Biliary atresia D.) Stenotic brain vessels

A.) Cystic duct obstruction Reason: By constricting the sphincter of Oddi, the radiopharmaceutical is pushed up the cystic duct by pressure in the event there is no obstruction.

What is the diagnosis for a patient with an A1C of 6.5% on two separate occasions? A.) Diabetes B.) Impaired fasting glucose C.) Impaired glucose tolerance D.) Prediabetes E.) A1C cannot be used to diagnose diabetes or any related classification

A.) Diabetes This patient meets the criteria for a diagnosis of diabetes. The 4 diagnostic criteria for diabetes are as follows: 1) A1C ⩾ 6.5%; 2) fasting plasma glucose ⩾ 126 mg/dL; 3) 2-hour plasma glucose of ⩾ 200 mg/dL during an oral glucose tolerance test and 4) a random plasma glucose concentration ⩾ 200 mg/dL with classic symptoms of hyperglycemia.

A 54-year-old male taking methyltestosterone for AIDS-related wasting disease. Which of the following should be used to monitor him for adverse effects from this drug? A.) Digital rectal exam B.) Hemoglobin A1c C.) Serum creatinine D.) Urinalysis E.) Serum calcium

A.) Digital rectal exam Androgens may cause prostate enlargement as an adverse effect.

Which of the following medications would be preferred for control of ventricular response in patients with atrial fibrillation and heart failure with reduced ejection fraction? A.) Digoxin B.) Verapamil C.) Diltiazem D.) Amlodipine E.) Dofetilide

A.) Digoxin Digoxin would be the drug of first choice because it will help slow the ventricular rate and it does not have negative inotropic effects. Both verapamil and diltiazem are negative inotropes and should not be used in patients with heart failure and low left ventricular ejection fraction. Although it is a calcium channel blocker, amlodipine does not affect AV nodal conduction. Dofetilide is used for conversion to and maintenance of sinus rhythm, not rate control.

Which of the following dosage forms may use surface-active agents in their formulations? (Mark all that apply.) A.) Emulsions B.) Suspensions C.) Colloidal dosage forms D.) Creams

A.) Emulsions B.) Suspensions C.) Colloidal dosage forms Surface-active agents facilitate emulsion formation by lowering the interfacial tension between the oil and water phases. Adsorption of surfactants on insoluble particles enables these particles to be dispersed in the form of a suspension.

Which of the following opthalmic adverse drug reactions of Alphagan is the most commonly reported? A.) Eye Pruritis B.) Anterior uveitis C.) Miosis D.) Eye Discahrge E.) Photophobia

A.) Eye Pruritis Correct answer: A Alphagan (brimonidine) is an opthalmic medication used to reduce elevated intraocular pressure (IOP) in patients with open-angle glaucoma or ocular hypertension. It is also available as an over-the-counter formulation to treat eye redness due to minor irritation. Alphagan is a selective alpha-2 adrenergic agonist, which causes a reduction in the formation of aqueous humor and increased outflow of aqueous humor

Which of the following symptoms are consistent with a diagnosis of benign prostatic hyperplasia?(Select ALL that apply) A.) Feeling of incomplete emptying B.) Polydipsia C.) Nocturia D.) Urinary urgency E.) Weak urine stream

A.) Feeling of incomplete emptying C.) Nocturia D.) Urinary urgency E.) Weak urine stream Correct answer(s): A, C, D, E The American Urological Association Symptoms Index (AUA-SI) characterizes benign prostatic hyperplasia (BPH) as: incomplete bladder emptying frequent urination intermittent urine stream urinary urgency weak urine stream straining nocturia, as assessed by the AUA-SI. The enlarged prostate gland can cause obstruction of the urethra, leading to decreased urinary flow, weak urine stream, and incomplete bladder emptying. The inability to complete bladder emptying can lead to frequent nocturia in patients with BPH.

The hematopoietic medications that generally should not be given on the same day as cytotoxic chemotherapy are: A.) Filgrastim and peg-filgrastim. B.) Erythropoietin and darbepoietin. C.) Darbepoietin and oprelvekin. D.) Oprelvekin and filgrastim.

A.) Filgrastim and peg-filgrastim. Research has shown that the co-administration of filgrastim and peg-filgrastim with cancer chemotherapy can actually increase the incidence of febrile neutropenia and the incidence and severity of neutropenia. Erythropoietin, darbepoietin, and oprelvekin can be given concurrently with cancer chemotherapy.

What is one of the most common adverse drug events caused by acarbose? A.) Flatulence B.) Hypoglycemia C.) Renal failure D.) Hyperglycemia E.) Weight gain

A.) Flatulence The most common adverse drug events for acarbose are flatulence, abdominal pain, and diarrhea. These adverse events may be decreased by titrating the dose gradually and taking the drug with the first bite of each meal.

A 40 year old patient with a long history of Crohn's disease presents with a macrocytic anemia and complains of extreme fatigue. He denies any neurologic symptoms. His methylmalonic acid concentration is normal but his homocysteine concentration is elevated. What is the most likely etiology for his anemia? A.) Folic acid deficiency B.) Vitamin B12 deficiency C.) Combined folate and B12 deficiency D.) Iron deficiency

A.) Folic acid deficiency Folic acid deficiency. A megaloblastic anemia with elevated HCY and normal MMA is due to folate depletion.

Which of the following is correct? A.) For work done in a horizontal laminar flow workbench, arrange items in the hood so that your hand is never between the HEPA filter and an object. B.) For work done in a horizontal laminar flow workbench, vials that are not being used should be stacked up along the side of the hood to increase workspace in the hood. C.) Before each shift, 70% isopropyl alcohol is used to sterilize the laminar flow workbench. D.) An object placed in the horizontal laminar flow workbench disturbs the airflow downstream of the object equal to two times the diameter of the object. E.) Syringes and IV bags are placed in the hood in their protective overwrap.

A.) For work done in a horizontal laminar flow workbench, arrange items in the hood so that your hand is never between the HEPA filter and an object. In an HLFW, never put your hand behind an object, and in a VLFW, never put your hand above an object. In an HLFW, a vial disturbs the laminar airflow equal to three times the diameter of the object. If the vial is next to the side wall, the airflow is disturbed equal to six times the diameter of the object. Syringes and IV bags should be taken from their overwrap at the edge of the hood.

Which of the following is not recommended in the treatment of hepatitis? A.) Ganciclovir + interferon alfa-2b B.) Lamivudine + interferon alfa-2b C.) Interferon alfa-2b + ribavirin D.) Interferon alfa-2b

A.) Ganciclovir + interferon alfa-2b EXPLANATION: Ganciclovir + interferon alfa-2b is not a recommended treatment for hepatitis. Lamivudine + interferon alfa-2b is recommended for the treatment of chronic hepatitis B. Interferon alfa-2b + ribavirin is recommended for the treatment of chronic hepatitis C. Interferon alfa-2b is recommended for the treatment of acute hepatitis C.

Which of the following disease states or clinical conditions would require PN? A.) High-output enteric fistula B.) Bilateral neck dissection for head and neck cancer C.) Colonoscopy D.) Coronary artery bypass graft

A.) High-output enteric fistula

Which of the following anticonvulsants is given by continuous infusion? I. Midazolam II. Fosphenytoin III. Lamotrigine A.) I B.) I and II C.) II D.) I and III E.) II and III

A.) I Midazolam has an extremely short half-life. For the treatment of status epilepticus, a patient normally receives a loading dose that is immediately followed by a continuous infusion.

Which statements best describe the carbapenem antibiotics? I. Exhibits activity against most gram-positive and gram-negative aerobes and anaerobes. II. Meropenem induces seizures more commonly than imipenem. III. Cilastatin exhibits activity against most gram-positive aerobes. A.) I only B.) II only C.) III only D.) I & II E.) II & III

A.) I only Carbapenems are active against most aerobic and anaerobic gram positive and gram negative bacteria. Imipenem is more likely to induce seizures, and cilastatin inhibits the metabolism of imipenem but has no anti-bacterial activity.

What is the correct answer concerning donepezil? I. Inhibits acetylcholinesterase but not butyrylcholinesterase II. It should be taken with meals in divided doses III. Side effects include tachycardia and blood pressure alterations A.) I only B.) III only C.) I and II only D.) II and III only E.) I, II, and III

A.) I only Donepezil is selective for acetylcholinesterase and does not inhibit butyrylcholinesterase. Donepezil does not have to be taken with meals and is given once daily. Donepezil does not increase heart rate and this class of medications that increase acetylcholine should be used with caution in patients with bradycardia.

Which of the following statements best describe appropriate vancomycin monitoring? I. Trough serum concentrations should be routinely monitored in patients with pre-existing renal dysfunction. II. Peak serum concentrations should be routinely monitored in patients with pre-existing renal dysfunction. III. Serum concentration monitoring is of no benefit in vancomycin monitoring. A.) I only B.) II only C.) III only D.) I & II E.) II & III

A.) I only Vancomycin trough concentrations should be assessed in patients with pre-existing renal dysfunction or worsening renal function. Additionally, serum concentration monitoring may be necessary in those with normal renal function to ensure trough concentrations are at goal (10-20 mcg/mL depending on the infection type/location).

Which of the following anticonvulsants are available in a liquid, chewable tablet, and intravenous formulation? _I. Carbamazepine _II. Phenytoin _III. Valproic acid A.) II only B.) III only C.) I only D.) I and II only E.) I, II, and III

A.) II only Only phenytoin is available as a liquid (125 mg/5 mL), chewable tablet (50 mg), and in an intravenous dosage form. Carbamazepine is not available in an IV dosage form and valproic acid is not available as a chewable tablet.

M. W. is a 25-year-old pregnant female who is admitted to the medical ICU following several days of severe nausea and vomiting. She is hypotensive, tachycardic, and confused, and her urine output is very low. Her serum sodium is 128 mEq/L. Which of the following should be given to treat her fluid and sodium abnormality? A.) IV NS or LR solution B.) IV 5% dextrose in water C.) po water D.) IV furosemide E.) Desmopressin

A.) IV NS or LR solution M. W. is hyponatremic, and her clinical signs and symptoms indicate severe dehydration from GI losses of water and sodium. She requires rapid fluid resuscitation with a fluid that has an approximately physiologic amount of sodium (either NS 154 mEq/L or LR 130 mEq/L). This amount of sodium will increase her serum sodium into the normal range over time, and the osmotic effect will hold water in the extracellular compartment (the vasculature and interstitium) to help restore organ perfusion.

Which of the following delivery agents combines a monoclonal antibody with a toxin, allowing targeted drug delivery with minimal toxicity? A.) Immunotoxins B.) Liposomes C.) PEGylation D.) Combinatorial chemistry E.) Biotherapy

A.) Immunotoxins Immunotoxins are delivery agents that combine a monoclonal antibody with a toxin. Liposomes are microscopic spherical lipid droplets that aid in drug delivery. PEGylation involves the addition of polyethylene glycol to therapeutic proteins to minimize the deleterious immune response to an individual protein. Combinatorial chemistry is a drug development strategy and not a drug delivery system. Biotherapy is any treatment option that involves the use of biologic material.

An example of a radiolabeled peptide that is used in localizing primary and metastatic neuroendocrine tumors is A.) In-111 pentetreotide. B.) I-123 iobenguane. C.) In-111 capromab pendetide. D.) I-123 ioflupane.

A.) In-111 pentetreotide. In-123 iobenguane is used in the detection of pheochromocytomas and neuroblastomas, but it is not a peptide. In-111 capromab pendetide is a monoclonal antibody used in patients with prostate cancer. I-123 ioflupane is used in the evaluation of patients with Parkinsonian syndromes.

Which of the following statements best describes the mechanism of action of etoposide? A.) Inhibits Topoisomerase II and is cell-cycle specific B.) Inhibits Topoisomerase I and is cell-cycle specific C.) Inhibits Topoisomerase I and is cell-cycle nonspecific D.) Potentiates Topoisomerase II and is cell-cycle nonspecific E.) Potentiates Topoisomerase I and is cell-cycle specific

A.) Inhibits Topoisomerase II and is cell-cycle specific Correct answer: A Etoposide inhibits DNA synthesis by forming a complex with topoisomerase II and DNA. This complex induces breaks in double-stranded DNA and prevents repair bytopoisomerase II binding. Accumulated breaks in DNA prevent entry into the mitotic phase of cell division and lead to cell death. Etoposide iscell-cycle specific, acting primarily in the G2 and S phases of the cell cycle. Answer B:Irinotecan, not etoposide, inhibits topoisomerase I and is cell-cycle specific.Answer C:Etoposide inhibits topoisomerase II, not I, and is cell-cycle specific. Answer D:Etoposide does not potentiate, but actually inhibits topoisomerase II and is cell-cycle specific.Answer E:While etoposide is cell-cycle specific, it does not potentiate, but actually inhibits topoisomerase II. Bottom Line:Etoposide inhibits DNA synthesis by forming a complex with topoisomerase II and DNA and acts primarily in the G2 and S phases of the cell cycle.

How is otitis externa, or swimmer's ear, best treated? A.) Instill an antimicrobial and steroid solution into the ear canal. B.) Apply antimicrobial ointment into the ear canal with a cotton swab. C.) Instill an antihistamine solution into the ear canal. D.) Increase pH of the ear canal with administration of Burow's solution. E.) Decrease pH of ear canal with administration of dilute HCl solution.

A.) Instill an antimicrobial and steroid solution into the ear canal. The treatment of otitis externa includes the instillation of an antimicrobial and steroid otic solution into the ear canal. Cotton swabs should be avoided to prevent otitis externa. Antihistamine solutions are not indicated in the treatment of otitis externa. Otic solutions containing acetic acid may also be of benefit in otitis externa by decreasing (not increasing) the pH of the ear canal and lowering its bacteria-harboring potential. Hydrochloric acid in any form should not be used in the ear canal.

Which of the following medications is a combination of sitagliptin and simvastatin? A.) Juvisync B.) Jentadueto C.) Janumet D.) Jalyn E.) Jevtana

A.) Juvisync Juvisync is a combination of Sitagliptin (Januvia) and Simvastatin (Zocor) used for the treatment of Type 2 Diabetes and Hyperlipidemia

A 63-year-old female with end-stage kidney disease is receiving peritoneal dialysis. Her most recent laboratory analysis reveals the following: BUN 58 mg/dL, SCr 5.2 mg/dL, phosphorus 7.4 mg/dL, calcium 9 mg/dL, albumin 2.5 g/dL, and iPTH 542 pg/mL. In addition to dietary restriction of phosphorus, which agent is best for initial management of this patient's hyperphosphatemia? A.) Lanthanum carbonate B.) Calcium acetate C.) Aluminum hydroxide D.) Calcium carbonate

A.) Lanthanum carbonate Lanthanum carbonate or sevelamer carbonate (Renvela) is preferred over a calcium-containing binder for initial management because this patient has a corrected calcium of 10.2 mg/dL [corrected calcium = measured serum calcium + 0.8 × (normal serum albumin − measured serum albumin)]. The elevated calcium and phosphorus increase the risk of metastatic calcifications. She requires a phosphorus-binding agent without calcium to minimize calcium absorbed in the GI tract. Aluminum is not preferred because of the risk of accumulation and adverse effects.

Which of the following is true regarding the mechanism of action of leucovorin? A.) Leucovorin stabilizes the binding of 5-dUMP and thymidylate synthetase, enhancing the activity of fluorouracil B.) Leucovorin inhibits active folate stores that are required for DNA/RNA synthesis C.) Leucovorin should not be given with fluorouracil due to increased risk of toxicity D.) Leucovorin binds directly to methotrexate, thereby preventing methotrexate from binding to binding sites E.) Leucovorin should not be administered with pyrimethamine due to increaed risk of hematologic toxicity

A.) Leucovorin stabilizes the binding of 5-dUMP and thymidylate synthetase, enhancing the activity of fluorouracil Correct answer: A Leucovorin is indicated for the palliative treatment of advanced colorectal cancer to prolong survival, in combination with 5-fluorouracil. It also is used as a rescue agent after high-dose methotrexate, and to decrease the toxicity of impaired methotrexate elimination and overdosage of folic acid antagonists. Lastly, it is indicated for the treatment of megaloblastic anemias due to folic acid deficiency when oral therapy is not possible. Leucovorin is a reduced form of folic acid that supplies the necessary cofactor blocked by methotrexate. It actively competes with methotrexate for transport sites, thereby displacing methotrexate from intracellular binding sites. It also restores active folate stores required for DNA/RNA synthesis. It is used in combination with 5-fluorouracil because it stabilizes the binding of 5-dUMP and thymidylate synthetase, thereby enhancing the activity of fluorouracil. When administered with pyrimethamine for the treatment of opportunistic infections, leucovorin decreases the risk for hematologic toxicity. Answer B: Leucovorin is a reduced form of folic acid, thereby supplying the necessary cofactor blocked by methotrexate. It restores active folate stores required for DNA/RNA synthesis. Answer C: Leucovorin is indicated for the palliative treatment of advanced colorectal cancer to prolong survival, in combination with 5-fluorouracil. It is used in combination with 5-fluorouracil because it stabilizes the binding of 5-dUMP and thymidylate synthetase, thereby enhancing the activity of fluorouracil. Answer D:Leucovorin actively competes with methotrexate for transport sites and displaces methotrexate from intracellular binding sites. Answer E:Leucovorin reduces the risk of hematologic toxicity when administered with pyrimethamine for the treatment of opportunistic infections. Bottom Line:Leucovorin is indicated for the palliative treatment of advanced colorectal cancer to prolong survival, in combination with 5-fluorouracil. It is used in combination with 5-fluorouracil because it stabilizes the binding of 5-dUMP and thymidylate synthetase, thereby enhancing the activity of fluorouracil.

Which of the following statements is correct regarding the treatment of bipolar disorder? A.) Lithium and divalproex sodium are considered first-line therapy options for mood stabilization. B.) When treating a patient with lithium, one must monitor the WBC and ANC because of lithium's propensity to cause agranulocytosis. C.) Patients diagnosed with bipolar I disorder exhibit episodes of mania at all times. D.) Patients with bipolar I disorder can be successfully treated with antidepressants alone. E.) Symptoms associated with mild lithium toxicity are likely to occur when serum drug concentration is greater than 3 mEq/L.

A.) Lithium and divalproex sodium are considered first-line therapy options for mood stabilization. Explanation: Lithium and divalproex are first-line therapy for bipolar disorder. Agranulocytosis is an adverse effect that is monitored with clozapine therapy. Bipolar I patients do not spend all of their time in mania. They also experience depression and normal mood at times. Antidepressants can increase the risks of experiencing a manic episode in patients with bipolar disorder, especially when used as monotherapy.

Which of the following medications is an intrauterine contraceptive containing levonorgestrel? A.) Mirena B.) NuvaRing C.) Implanon D.) Depo-Provera CI

A.) Mirena EXPLANATION: Mirena is an intrauterine contraceptive containing levonorgestrel.

In a patient with mildly symptomatic PSVT, verapamil should be used for which of the following rhythms? A.) Narrow QRS complex, regular interval B.) Wide QRS complex, regular interval C.) Wide QRS complex, irregular interval D.) All wide QRS complex rhythms E.) Narrow QRS complex and hypotension

A.) Narrow QRS complex, regular interval A wide QRS complex signifies conduction via an accessory pathway other than the AV node. Because calcium channel blockers prolong conduction in the AV node and not in the accessory pathways, administration of these agents will block the AV node and force impulses to be conducted via the accessory pathways, which have shorter refractory periods. Consequently, the ventricular response will increase significantly. In the presence of hypotension, direct-current cardioversion should be used; verapamil will worsen hypotension.

Which of the following disease states is most likely to cause a post-renal acute kidney injury? A.) Nephrolithiasis B.) Urinary incontinence C.) Hypotension D.) Heart Failure E.) Allergic nephritis

A.) Nephrolithiasis Correct answer: A Acute kidney injury (AKI) can present in multiple forms: pre-renal, intra-renal as acute interstitial nephritis or acute tubular necrosis, and post-renal.Post-renal AKI is more likely to occur in patients with obstruction in the ureter ("after the kidney"). The most common causes of post-renal AKI are kidney stones (ie,nephrolithiasis) and benign prostatic hyperplasia. Other causes include blood clots, cancer tissue, or neurogenic bladder disorders. Treatment of post-renal AKIs usually consists of relieving the obstruction or resolving the neurogenic cause of the bladder issues. Answer B:Urinary incontinence can cause AKI but is less likely to cause post-renal AKI than nephrolithiasis. Post-renal AKI results from an obstruction in the ureter, not typically from urinary incontinence. Answer C:Hypotension can cause AKI, but it is most often associated with pre-renal AKI. This is due to impaired blood flow to the kidneys, resulting in decreased perfusion and AKI. Answer D:Similar to hypotension, heart failure can also cause AKI, but it is most often associated with pre-renal AKI. This is due to impaired blood flow to the kidneys, resulting in decreased perfusion and AKI. Answer E:Allergic nephritis can cause AKI, but it typically causes an intrinsic renal failure, such as acute interstitial nephritis. Bottom Line:Post-renal acute kidney injury usually occurs as a result of impaired bladder emptying "after the kidney." This is often the result of obstruction from a kidney stone, also known as nephrolithiasis.

Which of the following options would you advise a patient to drink with meals to optimize iron absorption from meals? A.) Orange juice B.) Coffee C.) Tea D.) Milk E.) Wine

A.) Orange juice Tea and milk can decrease the absorption of iron from a meal by more than 50%. Orange juice, however, can double the absorption of iron from food.

A 12-year-old girl with a history of heart transplantation 5 years ago has an immunosuppression regimen which includes prednisone, which she has been on since her transplant. Which of the following adverse events is a risk due to her chronic prednisone use?(Select ALL that apply) A.) Osteoporosis B.) Insomnia C.) Chronic Kidney Disease D.) Tremors E.) Weight loss/decreased appetite

A.) Osteoporosis B.) Insomnia Correct answer(s): A, B Prednisone is an immunosuppressant that is commonly used in patients post-transplantation to help prevent graft rejection. In addition to its anti-inflammatory properties, prednisone suppresses the immune system by reducing lymphatic system activity.Chronic prednisone use is associated with several adverse events; close monitoring is important while patients are on this medication. These adverse events include: hypertension peptic ulcer disease osteoporosis myopathy mood changes (euphoria, mania, or depression) insomnia diabetes increased risk of infections Answer C:Prednisone is not associated with causing renal impairment. However, it should be used with caution in patients with renal impairment as fluid retention may occur. Answer D:Prednisone is not associated with tremors. Answer E:Prednisone is not associated with weight loss. In fact, it is associated with weight gain. Bottom Line:Chronic prednisone use is associated with several adverse events, including osteoporosis and insomnia. Close monitoring is important while on this medication.

Pharmacist needs to order a number of different medications, including controlled substances listed in Schedules II, III, IV, and V, as well as noncontrolled substances. Please choose the most accurate statement that reflects Pharmacist's options. A.) Pharmacist may order all needed medications using the electronic controlled substance ordering system. B.) Pharmacist may order all needed medications using DEA Form 222. C.) Pharmacist may not use any single method to order all of the needed medications. D.) Pharmacist may order all needed medications using the traditional invoice and receipt method.

A.) Pharmacist may order all needed medications using the electronic controlled substance ordering system.

Which of the following antitubercular agents inhibits renal excretion of uric acid and may worsen gout? A.) Pyrazinamide B.) Aminosalicylic acid C.) Ethambutol D.) Rifampin E.) Cycloserine

A.) Pyrazinamide Pyrazinamide inhibits renal excretion of uric acid and may worsen gout.

Which of the following adverse events is associated with propofol infusion syndrome? A.) Rhabdomyolysis B.) Severe skin reactions C.) Prolonged weakness D.) Thrombocytopenia E.) Constipation

A.) Rhabdomyolysis Per the manufacturer's information, rhabdomyolysis is commonly associated with propofol infusion syndrome.

A patient recently underwent a total knee replacement. Which of the following is the most appropriate DVT prophylaxis therapy? A.) Rivaroxaban 10 mg daily B.) Rivaroxaban 10 mg bid C.) Rivaroxaban 20 mg daily D.) Rivaroxaban 20 mg bid E.) Rivaroxaban 30 mg daily

A.) Rivaroxaban 10 mg daily Explanation: Rivaroxaban 10 mg daily is the correct dose for DVT prophylaxis after a total knee replacement. Rivaroxaban 20 mg daily is the dose for stroke prevention in patients with atrial fibrillation. The other doses are incorrect.

A police officer presents to the emergency room with a rash. He fears that he was exposed to a biological weapon several days before the rash appeared. You notice the rash is forming pustules and is most prominent on the face and extremities. The patient says the rash developed all at once. He has possibly contracted which of the following? A.) Smallpox B.) Chicken pox C.) Anthrax D.) Tularemia

A.) Smallpox Smallpox is the most likely agent. The agent causes formation of a pustular rash that is typically most prominent on the face and extremities. Lesions form at the same time. Chicken pox rash is most prominent on the trunk and develops in successive groups of lesions over several days. Anthrax forms painless necrotic lesions. Tularemia causes a nonspecific febrile illness that rapidly develops into pneumonia.

Gynecomastia is an adverse effect of which of the following medications? A.) Spironolactone B.) Eplerenone C.) Lisinopril D.) Bumetanide E.) Hydralazine

A.) Spironolactone Gynecomastia can occur with spironolactone therapy. This adverse effect is not reported with eplerenone.

A radiopharmaceutical that utilizes a nebulizer for its administration in ventilation studies is: A.) Tc-99m DTPA B.) Tc-99m MAA C.) Xe-133 D.) Tc-99m Pertechnetate

A.) Tc-99m DTPA Reason: The other drugs are administered via IV (Tc-99m MAA and Tc-99m Pertechnetate) and Xe-133 is a gas and does not utilize a nebulizer for administration.

Which of the following is correct concerning the bacterial endotoxin test? A.) The BET can dependably detect pyrogens originating from only Gram-negative bacteria. B.) Test results from a BET are less sensitive than those obtained from a pyrogen test. C.) All pyrogens are also bacterial endotoxins. D.) In the performance of the gel-clot method, the rate of gel absorbance is related to the amount of endotoxin present in the sample. E.) The LPS portion of the inner cell of Gram-positive bacteria causes the pyrogenic response

A.) The BET can dependably detect pyrogens originating from only Gram-negative bacteria. The LPS portion of the outer cell wall of Gram-negative bacteria causes the pyrogenic reaction. The BET is 5 to 50 times more sensitive than the pyrogen test and detects only pyrogens from Gram-negative bacteria. The rabbit test is also the pyrogen test and detects the fever-producing potential of a CSP. All endotoxins are pyrogens, but all pyrogens are not endotoxins. With the gel-clot test, the rate of gel formation is related to the amount of endotoxin present in a sample. The bacterial endotoxin test (BET) is also called the LAL and detects the amount of bacterial endotoxin originating from Gram-negative bacteria in a CSP. The preferred test for bacterial endotoxins is the BET because the rabbit test detects all pyrogens. The BET is the test referenced in the USP monographs. Items that cannot withstand temperatures of 200-250ºC may be rinsed with sterile water for injection to remove endotoxins from the surface (for example, rubber closures). Filter sterilizing does not remove endotoxins. Autoclaving does not destroy endotoxins. The bacteria do not have to be living for the reaction to occur.

Which of the following is margin of error? A.) The value obtained by multiplying the reliability coefficient and the standard error B.) Confidence interval C.) Point estimate D.) Difference in population means E.) Reliability coefficient

A.) The value obtained by multiplying the reliability coefficient and the standard error EXPLANATION: The value obtained by multiplying the reliability coefficient and the standard error is referred to as the precision of the estimate. It is also called the margin of error.

Which of the following products is available without a prescription? A.) Tinactin (tolnaftate) B.) Diflucan (fluconazole) C.) Grifulvin-V (griseofulvin) D.) Naftin (naftifine)

A.) Tinactin (tolnaftate) Tolnaftate (Tinactin) is available without a prescription. Fluconazole (Diflucan) and griseofulvin (Grifulvin-V) are prescription systemic antifungal agents, and naftifine (Naftin) is a prescription topical antifungal.

Which long-term COPD regimen is preferred in severe disease? A.) Tiotropium, formoterol, inhaled corticosteroid B.) Tiotropium, salmeterol, theophylline C.) Ipratropium, albuterol, montelukast D.) Omalizumab, theophylline, albuterol E.) Budesonide, ipratropium, short-acting β-agonist

A.) Tiotropium, formoterol, inhaled corticosteroid EXPLANATION: Studies have shown long-acting bronchodilators are preferred and inhaled corticosteroids reduce exacerbations and improve quality of life.

JU is a 30 year-old Caucasian female on chronic hemodialysis. Within the first hour of her Wednesday hemodialysis session, she develops a fever (38.4°C), chills, and tachycardia (HR =131). Blood cultures from her dialysis catheter and a percutaneous site are drawn. A laboratory analysis is also performed. Laboratory results: WBC: 17.6 Neutrophils: 82% Bands: 14% Based on history, laboratory data, and clinical presentation the appropriate empiric antibiotic regimen for JU is ________. A.) Vancomycin B.) Imipenem/cilastatin C.) Linezolid D.) Azithromycin E.) Liposomal Amphotericin B

A.) Vancomycin Vancomycin is a Gram-positive antibiotic and is the only therapy indicated for the treatment of intravascular catheter related infections. Response C is not recommended for endovascular infections. Responses B, D, and E do not provide adequate empiric coverage of coagulase-negative staphylococci and S. aureus.

A 79-year-old man presents to the hospital with complaints of shortness of breath and a feeling that his heart is "fluttering." He is diagnosed with atrial fibrillation. He has a history of type 2 diabetes, chronic kidney disease, hypertension, and gout. What is the most appropriate therapy for stroke prevention in this patient? A.) Warfarin (goal of INR 2.0-3.0) B.) Aspirin 325 mg daily C.) Aspirin 81 mg daily D.) Clopidogrel 75 mg daily E.) Warfarin (goal of INR 2.5-3.5)

A.) Warfarin (goal of INR 2.0-3.0) In a patient with a CHADS2 score of 3, guidelines recommend oral anticoagulation rather than aspirin or clopidogrel. Aspirin or clopidogrel is not as effective as warfarin in this high-risk patient.

Which of the following is a protectant? A.) Zinc oxide B.) Lanolin C.) Urea D.) Colloidal oatmeal E.) Camphor

A.) Zinc oxide Lanolin is an emollient. Urea is a kertain softening agent. Colloidal oatmeal and camphor are antipruritics.

Which of the following antipsychotics is associated with the least sedation and weight gain? A.) Ziprasidone B.) Quetiapine C.) Olanzapine D.) Chlorpromazine E.) Clozapine

A.) Ziprasidone iprasidone (Geodon®) is associated with the least amount of sedation and weight gain of the antipsychotics listed. Quetiapine (Seroquel®) is highly sedating and often used inappropriately for insomnia. Olanzapine (Zyprexa®), clozapine (Clozaril®), and chlorpromazine (Thorazine®) are associated with significant sedation and weight gain.

A quality control manager at Widget Pharmaceuticals has just discovered that the manufacturing process has failed to follow Good Manufacturing Practices. A batch sample of a Widget Pharmaceuticals' intravenous solution has tested pure. If, in fact, the intravenous solution is pure, the solution would be A.) adulterated. B.) misbranded. C.) both misbranded and adulterated. D.) neither misbranded nor adulterated. E.) nonsterile.

A.) adulterated. The applicable section of the federal FDCA is 21 USC 351.

A 7-month-old formula-fed female is brought to your pharmacy by her mother, who describes the infant as having new onset of fever (102.5°F) and increased irritability in the past 24 hours. The mother states that she stayed home with the infant today instead of sending her to day care. Family history is significant for an older sibling with a recent upper respiratory tract infection. Examination of the infant's ear canal using a pneumatic otoscope reveals a bulging, red tympanic membrane with no mobility on negative or positive pressure. Computer records reveal that she was treated for acute otitis media at 3 months of age. The drug of choice for this patient's current episode of acute otitis media is A.) amoxicillin. B.) amoxicillin-clavulanate. C.) IM ceftriaxone. D.) cefixime. E.) trimethoprim-sulfamethoxazole.

A.) amoxicillin. Despite the emergence of drug-resistant Streptococcus pneumoniae, high-dose amoxicillin (90 mg/kg/day)—because of its excellent pharmacodynamic profile, side effect profile, and cost—remains the drug of choice in uncomplicated acute otitis media. Amoxicillin-clavulanate is considered first-line therapy in patients who have received amoxicillin in the past 30 days, those with conjunctivitis, those with recurrent otitis media unresponsive to amoxicillin, or those in whom β-lactamase coverage for H. influenzae or M. catarrhalis is indicated. IM ceftriaxone is an acceptable alternative or second-line agent, whereas cefixime and trimethoprim-sulfamethoxazole are not included in the AAP/AAFP guidelines as recommended therapies for acute otitis media.

For a transfer of product into the controlled area, A.) bottles, bags, and syringes must be removed from brown cardboard boxes before being brought into the buffer area. B.) vials stored in laminated cardboard may not be brought into the controlled area. C.) stainless steel carts may be used to transfer items into the controlled area directly from the storage area. D.) large-volume parenteral bags of IV solution must be removed from their protective overwrap before being brought into the controlled area. E.) the refrigerator should be placed next to the laminar flow hood for easy access.

A.) bottles, bags, and syringes must be removed from brown cardboard boxes before being brought into the buffer area. Cardboard must be kept out of the buffer area. Vials stored in laminated cardboard may be stored in the buffer area. No items should be brought into the buffer area without being sanitized. Large-volume parenteral bags should be removed from their overwrap just before being used. The refrigerator should not be in the buffer room because it is a source of contamination.

A patient presents to her primary care office and is diagnosed with hypothyroidism. Which of the following would be characteristics of hypothyroidism?(Select ALL that apply) A.) cold intolerance B.) Heat intolerance C.) Weight gain D.) Weight loss E.) Thyroid Stimulating hormone 15 uU/mL

A.) cold intolerance C.) Weight gain E.) Thyroid Stimulating hormone 15 uU/mL Correct answer(s): A, C, E The most common form of hypothyroidism is from autoimmune etiology. Hypothyroidism is characterized by a lower metabolic state, so the signs and symptoms are typically associated with lower metabolism. Since thyroid-stimulating hormone (TSH) will be produced to stimulate the thyroid hormone to produce more T4, patients with hypothyroidism will have elevated TSH to help stimulate more T4 even though they cannot produce more T4. Based on the Clinical Practice Guidelines for Hypothyroidism in Adults, the signs and symptoms of hypothyroidism include: dry skin cold intolerance fatigue muscle cramps weight gain voice changes constipation bradycardia In women, menometrorrhagia can occur. Laboratory blood work will show elevated thyroid-stimulating hormone (TSH) with low T4. Since the development of free T4 assays, its measurement has largely replaced obtaining total T4. Answers B & D: Heat intolerance and weight loss can be a sign/ symptom of hyperthyroidism. Bottom Line: Hypothyroidism is characterized by lower metabolic state, so the signs and symptoms are typically associated with lower metabolism. Since thyroid-stimulating hormone (TSH) will be produced to stimulate the thyroid hormone to produce more T4, patients with hypothyroidism will have elevated TSH to help stimulate more T4 even though they cannot produce more T4.

Coalescence can be reduced by A.) decreasing the difference between the density of the dispersed phase and the density of the medium. B.) adding an agent that reduces the viscosity of the medium. C.) increasing the droplet size of the dispersed phase. D.) all of the above.

A.) decreasing the difference between the density of the dispersed phase and the density of the medium. According to Stokes's law of sedimentation, coalescence can be reduced by decreasing the difference between the density of the dispersed phase and the density of the medium. Decreasing the viscosity or increasing particle size will increase the rate of sedimentation, which may provide the opportunity for particles to coalescence.

According to the Noyes-Whitney equation, the drug concentration immediately at the solid surface within the diffusion layer (stagnant layer) is recognized as the A.) drug solubility. B.) dissolution rate. C.) thickness layer. D.) diffusion coefficient.

A.) drug solubility. According to the Noyes-Whitney equation, the drug concentration in the stagnant layer is equal to the drug solubility.

Cilastatin is combined with imipenem to A.) inhibit metabolism in the kidney. B.) inhibit resistance mechanisms in Gram-negative bacteria. C.) inhibit hepatic metabolism. D.) improve activity against Gram-positive bacteria. E.) decrease the incidence of seizures.

A.) inhibit metabolism in the kidney. Cilastatin prevents renal metabolism of imipenem by dehydropeptidase—an enzyme present on the brush border of the proximal renal tubule—thereby increasing the serum AUC (area under the curve) and the urinary concentrations of the active drug. Cilastatin has no antibacterial activity.

If you were to examine a peripheral blood smear of a patient with iron deficiency anemia, you would find cells that are A.) microcytic and hypochromic. B.) macrocytic and hypochromic. C.) macrocytic and normochromic. D.) microcytic and normochromic.

A.) microcytic and hypochromic. Iron deficiency produces a hypochromic (low-Hgb) anemia, given that iron is a component of the Hgb molecule. The cells also are microcytic (meaning "small cell"), because they spend a longer time in the marrow awaiting proper Hgb synthesis and therefore divide more

The preferred route of opioid administration is A.) oral. B.) intravenous. C.) subcutaneous. D.) rectal.

A.) oral. Oral medications should be used whenever possible because of convenience, flexibility, and steady serum levels.

JA is a HIV+ patient who presents with new onset seizure disorder and headaches. He is found to have ring enhancing lesions on a CT exam of his brain. A diagnosis of Toxoplasmosis gondii encephalitis is made. Which of the following would be a reasonable choice for acute treatment of this infection? A.) sulfadiazine, pyrimethamine and leucovorin B.) amphotericin B C.) Ethambutol, clarithromycin and rifabutin D.) fluconazole E.) dexathmethasone

A.) sulfadiazine, pyrimethamine and leucovorin Toxoplasmosis preferred therapy for treatment is sulfadiazine, pyrimethamine and leucovorin for at least 6 weeks. Alternative treatment plans include: pyrimethamine, leucovorin and clindamycin; SMZ-TMP DS; Atovoquone +/- pyrimethamine and leucovorin; or azithromycin, pyrimethamine and leucovorin.

Under the "Five Percent Rule," a pharmacy registered to dispense controlled substances to another registrant may do so without registering as a distributor as long as the amount of controlled substances distributed does not exceed 5% of the A.) total number of dosage units of all controlled substances dispensed by the pharmacy during a calendar year. B.) dosage units on hand at the time of the dispensation to the other registrant. C.) total dollars spent on controlled substances in one month. D.) individual controlled substances ordered in that month. E.) total weight (mg) of controlled substances on hand at the time of the sale.

A.) total number of dosage units of all controlled substances dispensed by the pharmacy during a calendar year.

JK is a 48-year-old woman scheduled to receive her first 175 mg/m2 dose of paclitaxel for metastatic breast cancer. She is 5'3" tall and weighs 127 lb. Using the Mosteller formula (BSA (m2) = √Ht [cm] × Wt [kg]/3600) what dose of paclitaxel would you prepare? A) 2780 mg B) 278 mg C) 412 mg D) 4120 mg

B) 278 mg Answer b is correct. The patient's BSA is 1.6 m2 - (175 mg/m2 × 1.6 m2 = 278 mg) Answer a is incorrect. A dose of 2780 mg represents a 10-fold over dose that could result by misplacing a decimal point during calculations. Answer c is incorrect. This dose would have resulted by failing to convert the patient's weight to kg when calculating the BSA. Answer d is incorrect. This dose would have resulted by failing to convert the patient's weight to kg when calculating the BSA and misplacing a decimal point when performing calculations.

BG is a 50-year-old African American woman who is 63 in tall and weighs 130 lb. Her current SCr is 1.6 mg/dL. What is BG's GFR as estimated by the MDRD equation? A) 25 mL/min B) 35 mL/min C) 45 mL/min D) 55 mL/min

B) 35 mL/min Answer b is correct. GFR = 186 × 1.6−1.154 × 50−0.203 × 0.742 = ∼36 mL/min

An 87-year-old man presents to the anticoagulation clinic for follow-up after recent initiation of warfarin for treatment of a PE. You ask the patient what dose he is currently taking but the patient cannot remember. You then ask him what color tablets the patient has and how many he takes per day. The patient responds that he takes 1 blue tablet every day. Based upon this information, what is the patient's current warfarin dose? A) 2.5 mg daily B) 4 mg daily C) 5 mg daily D) 7.5 mg daily

B) 4 mg daily Answer b is correct. All manufacturers of warfarin produce each tablet strength in the same color to provide an easier way to determine a patient's dose if they forget the numerical strength. The table below describes the available warfarin tablet strengths and corresponding tablet color. Answers a, c and d are incorrect. These tablet strengths are green, peach, and yellow in color, respectively.

A premenopausal woman with ER negative, node positive breast cancer is starting doxorubicin and cyclophosphamide adjuvant treatment. What would you recommend to determine the severity of the most common toxicity associated with this treatment regimen? A) An electrocardiogram 1 week after chemotherapy. B) A complete blood count including platelets 1 week after administration of the chemotherapy. C) Serum bilirubin and aspartate transaminase 1 week after chemotherapy. D) Urinalysis 1 week after chemotherapy.

B) A complete blood count including platelets 1 week after administration of the chemotherapy. Myelosuppression (neutropenia, thrombocytopenia) is the most common treatment related adverse effect associated with this adjuvant treatment regimen. Nearly 100% of patients receiving this treatment regimen will experience myelosuppression. Choice (A) is incorrect. Although doxorubicin has been associated with cardiomyopathy that increases in incidence with cumulative doses exceeding 400 mg/m2. The incidence of cardiomyopathy in patients administered cumulative doses exceeding 400 mg/m2 is in the range of 5%. It is not the most common toxicity associated with this treatment regimen. Choice (C) is incorrect. Although these drugs may cause elevations of serum bilirubin and aspartate transaminase, it occurs less frequently than myelosuppression. Choice (D) is incorrect. Although urinalysis could be useful in detecting hematuria caused by cyclophosphamide, this adverse effect (hemorrhagic cystitis) occurs infrequently with this adjuvant chemotherapy regimen.

CJ is a 24-year-old female patient who received kidney transplantation 3 months ago. She fills her prescription for tacrolimus, mycophenolate sodium, and prednisone at your pharmacy. Which of the following represents two adverse effects specific to tacrolimus that CJ may experience? A) Diarrhea and leukopenia B) Alopecia and hyperglycemia C) Hypertriglyceridemia and nephrotoxicity D) Hirsutism and gingival hyperplasia

B) Alopecia and hyperglycemia

CJ is a 24-year-old female patient who received kidney transplantation 3 months ago. She fills her prescription for tacrolimus, mycophenolate sodium, and prednisone at your pharmacy. Which of the following represents two adverse effects specific to tacrolimus that CJ may experience? A) Diarrhea and leukopenia B) Alopecia and hyperglycemia C) Hypertriglyceridemia and nephrotoxicity D) Hirsutism and gingival hyperplasia

B) Alopecia and hyperglycemia Answer b is correct. Alopecia and hyperglycemia are adverse effects specific to tacrolimus. Hyperglycemia is further exacerbated by the concomitant use of corticosteroids and can lead to post-transplant diabetes mellitus. Answer a is incorrect. Diarrhea and leukopenia are adverse effects specific to mycophenolate products. Diarrhea can be associated with tacrolimus, but leukopenia is not common with tacrolimus. Answer c is incorrect. Both cyclosporine and tacrolimus can cause nephrotoxicity, but hypertriglyceridemia is more frequently associated with mTOR inhibitors such as sirolimus and everolimus. Answer d is incorrect. Hirsutism and gingival hyperplasia are adverse effects specific to cyclosporine. It is important to counsel patients who are initiating cyclosporine on the importance of good oral hygiene.

Which of the following functions is performed by neutrophils? A) Antigen presentation to T lymphocytes B) Engulfing pathogens C) Lysing virally infected cells D) Secreting antibody

B) Engulfing pathogens he major role of neutrophils is to engulf and destroy the infectious pathogens (B). Only APCs (e.g., macrophages and dendritic cells) present antigen to T-lymphocytes (specifically helper T lymphocytes), (A). Only CD8(+) cytotoxic T lymphocytes have the machinery to lyse cells that are infected with viruses (C). B lymphocytes become activated to plasma cells, which secrete antibody or immunoglobulin (D).

In a patient with risk factors for hyperkalemia and history of hyperkalemia, which of the following agents would be acceptable treatment to avoid hyperkalemia risk? A) Amiloride B) Amlodipine C) Enalapril D)Spironolactone E) Valsartan

B) Amlodipine Answer b is correct. CCBs (such as amlodipine) are not associated with hyperkalemia. Answer a is incorrect. Amiloride is a potassium-sparing diuretic (similar to triamterene). Answer c is incorrect. ACEI (such as enalapril) are associated with hyperkalemia because of their effects on aldosterone. Answer d is incorrect. Spironolactone is a potassium-sparing diuretic (aldosterone antagonist). Answer e is incorrect. ARBs (such as valsartan) are associated with hyperkalemia because of their effects on aldosterone.

What antineoplastic agents form free radicals, which combine with oxygen to form superoxide, and which may form hydrogen peroxide? A) Antimetabolites B) Anthracyclines C) Alkylating agents D) Antibiotics

B) Anthracyclines Anthracyclines (B) may form free radicals and the free radicals combine with oxygen to form superoxide, which can make hydrogen peroxide. Oxygen free radical formation is a cause of cardiac damage. The cardiac damage is dose dependent; therefore, each anthracycline has a lifetime maximum dose that can be administered. Antimetabolites (A), alkylating agents (C), and antibiotics (D) are classes of antineoplastic agents that are not associated with the formation of free radicals.

BV is a 50-year-old African American male patient with a new diagnosis of heart failure. His past medical history includes hypertension, diabetes mellitus, bilateral renal artery stenosis, and previous myocardial infarction. Which medication should be initiated to decrease pre-load and afterload for appropriate management of his heart failure? A) Prinivil B) BiDil C) Diovan D) Inspra

B) BiDil Hydralazine and isosorbide dinitrate (B) consist of a venous and arterial vasodilator, and may be used in addition to the standard heart failure therapy of ACE inhibitors and beta blockers in African Americans or any patient who cannot tolerate an ACE inhibitor or ARB. Lisinopril (Prinivil), (A), is an ace inhibitor and should be initiated in heart failure patients. However, BV has an absolute contraindication to ACE inhibitor therapy, bilateral renal artery stenosis. Other absolute contraindications for ACE inhibitors are pregnancy and angioedema. Valsartan (Diovan), (C), is an ARB and may be initiated in heart failure patients as an alternative to ACE inhibitors. BV has an absolute contraindication to ARB therapy, bilateral renal artery stenosis. Eprelenone (Inspra), (D), is an aldosterone antagonist used in the management of systolic heart failure after the patient is on standard therapy with an ACE inhibitor and beta blocker.

Which of the following is the generic name for Rexulti®? A) Iloperidone B) Brexpiprazole C) Lurasidone D) Asenapine

B) Brexpiprazole Answer b is correct. Brexpiprazole is the generic name for Rexulti. Answer a is incorrect.Iloperidone is the generic name for Fanapt. Answer c is incorrect.Lurasidone is the generic name for Latuda. Answer d is incorrect. Asenapine is the generic name for Saphris.

It is recommended that the human leukocyte antigen type B, HLA-B*1502, allele is genotyped in Asian patients prior to taking which of the following medications? Select all that apply. A) Lithium B) CBZ C) Lorazepam D) Haloperidol E) Quetiapine

B) CBZ Answer b is correct. CBZ is associated with causing both SJS and TEN, two potentially fatal dermatologic reactions. The allelic variation of the HLA-B gene, HLA-B*1502, is associated with these serious dermatologic reactions, and it is recommended by the manufacturer that patient who are of ancestry in genetically at-risk populations be screened for the presence of this genetic marker. All medications can potentially cause a rash; however, there is a target genetic maker that has been linked specifically to CBZ. When a rash is seen with the use of lamotrigine, the medication should be discontinued. Answer a is incorrect. There is no genetic marker linked with the rash induced by lithium. Answer c is incorrect. There is no genetic marker linked with the rash induced by lorazepam. Answer d is incorrect. There is no genetic marker linked with the rash induced by haloperidol. Answer e is incorrect. There is no genetic marker linked with the rash induced by quetiapine.

AQ is a 44-year-old female patient with a past medical history of hypertension (HTN) and dyslipidemia. Medications include lisinopril and simvastatin. AQ has developed pneumonia and would like to take an oral agent that will not interact with her medications. Which of the following antibiotics used in the treatment of CAP is a strong inhibitor of the CYP-450 3A4 hepatic enzyme and would have a drug interaction with her medications? A) Azithromycin B) Clarithromycin C) Amoxicillin D) Cefpodoxime

B) Clarithromycin Answer b is correct. Clarithromycin is utilized for treatment of CAP, inhibits protein synthesis, and is a strong inhibitor of the CYP-450 3A4 hepatic enzyme. Therefore, drug interactions with the macrolide antimicrobial may be common. Medications that are substrates of the CYP-450 3A4 hepatic enzyme will have their metabolism/clearance decreased. Common examples of substrates of the CYP-450 3A4 hepatic enzyme are the azole antifungals, calcium channel blockers (verapamil and diltiazem), and hydroxymethylglutaryl-coenzyme A (HMG-CoA) reductase inhibitors, and many others. Answer a is incorrect. The azalide azithromycin is utilized for CAP and inhibits protein synthesis; however, it is a weak inhibitor of the CYP-450 3A4 hepatic enzyme. Drug interactions are a major difference between the macrolides (erythromycin and clarithromycin) and the azalide azithromycin. Answer c is incorrect. Amoxicillin may be used as part of a treatment regimen for CAP; however, amoxicillin does not inhibit the CYP-450 system. Answer d is incorrect. Cefpodoxime may be used as part of a treatment regimen for CAP; however, cefpodoxime does not inhibit the CYP-450 system.

BK is receiving FOLFOX chemotherapy. Which of the following should BK avoid most in the short term for the purpose of avoiding acute neurotoxicity exacerbations caused by oxaliplatin? A) Direct sunlight B) Cold C) Heat D) Tight fitting clothing E) OTC acne treatments

B) Cold Oxaliplatin commonly causes an acute sensory neuropathy that is precipitated or exacerbated by cold temperatures.

What is the most common mode of transmission for the hepatitis A virus (HAV)? A) Blood B) Fecal-oral route C) Perinatal exposure D) Semen

B) Fecal-oral route The fecal-oral route (B) is the most common mode of transmission of HAV and often occurs through contact with contaminated food, poor hygienic practices, and in poorly developed countries. Intravenous drug users can acquire HAV through exposure to HAV infected blood (A), but this is not a common mode of transmission. Perinatal transmission (C) is not associated with HAV, but is infrequently associated with HBV and very rarely for HCV. Transmission of viral hepatitis from semen (D) is not associated with HAV, but is rarely associated with other viral hepatitis types.

Aldosterone antagonists should be avoided in which HF patients? A) Serum potassium <3.5 mmol/L B) Creatinine clearance <30 mL/min C) Concomitant ARB therapy D) NYHA class III to IV despite standard HF therapy

B) Creatinine clearance <30 mL/min When evaluating a heart failure patient for aldosterone antagonists (e.g., spironolactone) treatment, creatinine should be < 2.5 mg/dl in men and < 2.0 mg/dl in women, or creatinine clearance < 30 ml/min (B). Renal dysfunction increases the risk of hyperkalemia and thus is a contraindication. Aldosterone antagonists (ARA) can cause hyperkalemia and should be avoided in patients with a serum potassium >5 mmol/L; however, they can be safely prescribed in a patient with a serum potassium <3.5 mmol/L (A). Therapy with aldosterone antagonists may be initiated with a concomitant ARB (C) or ACE inhibitor, as long as hyperkalemia is not present. Combination ACE inhibitor plus ARB plus ARA is not recommended due to an increased risk of hyperkalemia. Aldosterone antagonist therapy is recommended in patients who are NYHA class III to IV despite standard HF therapy (D).

DC is a 59-year-old Caucasian man who reports to his primary care physician complaining of 2-week history of fatigue and fever. A CBC with differential reveals an elevated WBC (25,000 U/L) and profound thrombocytopenia (platelets 30,000 U/L). His peripheral blood has 20% blasts. A bone marrow biopsy was performed and DC was diagnosed with acute myeloid leukemia (AML-M4). Molecular testing revealed—FLT3 negative, NPML1 negative, C-KIT negative. Following induction therapy, DC achieves a complete remission. Next month he arrives at your institution to receive high-dose cytarabine (HDAC) for consolidation therapy. You receive the following order— Cytarabine 3000 mg/m2 IV Q12 hours days 1, 3, and 5 Patient characteristics: Height 6'0", Weight 165 lb Which of the following represents a correct dosing strategy for patient DC? A) Cytarabine 5850 mg IV days 1, 3, and 5 B) Cytarabine 5850 mg IV Q12 hours days 1, 3, and 5 C) Cytarabine 5550 gm IV days 1, 3, and 5 D) Cytarabine 5550 mg IV Q12 hours days 1, 3, and 5

B) Cytarabine 5850 mg IV Q12 hours days 1, 3, and 5 Answer b is correct. DC's calculated BSA is 1.95 m2 since height is 182.88 cm and weight is 75 kg. This answer represents correct dosing and strategy. Answer a is incorrect. DC should receive cytarabine twice daily given every 12 hours. Answer c is incorrect. This dose would represent a major dosing error. Note units of gm not mg. Answer d is incorrect. DC's calculated BSA is 1.95 m2 since height is 182.88 cm and weight is 75 kg. This answer represents a dosing strategy for a BSA of 1.85 m2.

The effects of which anticoagulant(s) can be reversed by idarucizumab? Select all that apply. A) Rivaroxaban B) Dabigatran etexilate mesylate C) Warfarin D) Apixaban E) Heparin

B) Dabigatran etexilate mesylate Answer b is correct. The effects of dabigatran can be reversed by idarucizumab (Praxbind). Answers a and d are incorrect.Apixaban and rivaroxaban can be reversed by andexanet alpha (Andexxa). Answer c is incorrect.Warfarin can be reversed by utilizing vitamin K, fresh frozen plasma (FFP), or prothrombin complex concentrate (PCC). Answer e is incorrect. Heparin can be reversed by utilizing protamine.

YM is a 62-year-old woman with a recent diagnosis of stage III diffuse large B-cell lymphoma. Her disease is characterized by multiple areas of involvement including the spleen, pelvic lymph nodes, and mediastinal lymph nodes. She is scheduled to begin chemotherapy with the regimen EPOCH-R, which includes etoposide, prednisone, vincristine, cyclophosphamide, doxorubicin, and rituximab. On her second cycle, during the infusion of the doxorubicin YM begins experiencing extreme pain at the injection site. The nurse observes new onset redness and swelling around the infusion site and is concerned that extravasation has occurred. Which of the following agents is the most appropriate to treat this extravasation injury? A)Hyaluronidase B) Dexrazoxane C) Sodium thiosulfate D) Silver sulfadiazine

B) Dexrazoxane Answer b is correct. Dexrazoxane is indicated for the treatment of extravasation injury from anthracyclines. It works by inhibiting the production of free radicals. It should be administered as soon as extravasation is detected and repeated for 3 total days of therapy. Answer a is incorrect.Hyaluronidase is incorrect. Hyaluronidase breaks down hyaluronic acid, allowing an extravasated drug to dissipate, decrease the concentration of the drug at the site of extravasation. Before dexrazoxane was known to be beneficial for anthracycline extravasation, hyaluronidase was commonly used. Answer c is incorrect.Sodium thiosulfate is used in the treatment of extravasation of alkylating agents. It binds to the active sites on the alkylating agents, thus neutralizing them. Answer d is incorrect.Silver sulfadiazine is a topical preparation that is used in the treatment of skin burns. It does not have a role in the treatment of chemotherapy extravasation.

YM is a 62-year-old woman with a recent diagnosis of stage III diffuse large B-cell lymphoma. Her disease is characterized by multiple areas of involvement including the spleen, pelvic lymph nodes, and mediastinal lymph nodes. She is scheduled to begin chemotherapy with the regimen EPOCH-R, which includes etoposide, prednisone, vincristine, cyclophosphamide, doxorubicin, and rituximab. On her second cycle, during the infusion of the doxorubicin YM begins experiencing extreme pain at the injection site. The nurse observes new onset redness and swelling around the infusion site and is concerned that extravasation has occurred. Which of the following agents is the most appropriate to treat this extravasation injury? A) Hyaluronidase B) Dexrazoxane C) Sodium thiosulfate D) Silver sulfadiazine

B) Dexrazoxane Dexrazoxane is indicated for the treatment of extravasation injury from anthracyclines. It works by inhibiting the production of free radicals. It should be administered as soon as extravasation is detected and repeated for 3 total days of therapy. Answer a is incorrect.Hyaluronidase is incorrect. Hyaluronidase breaks down hyaluronic acid, allowing an extravasated drug to dissipate, decrease the concentration of the drug at the site of extravasation. Before dexrazoxane was known to be beneficial for anthracycline extravasation, hyaluronidase was commonly used. Answer c is incorrect.Sodium thiosulfate is used in the treatment of extravasation of alkylating agents. It binds to the active sites on the alkylating agents, thus neutralizing them. Answer d is incorrect.Silver sulfadiazine is a topical preparation that is used in the treatment of skin burns. It does not have a role in the treatment of chemotherapy extravasation.

Which antiemetic should be avoided in patients with the potential for a prolonged QT-interval? A) Aprepitant B) Droperidol C) Metoclopramide D) Dexamethasone

B) Droperidol The Correct Answer is: B QT-interval prolongation may occur with recommended doses of droperidol (B). Droperidol now has a black box warning alerting practitioners of the serious potential for QT-interval prolongation. QT-interval prolongation is not a concern with aprepitant (A), metoclopramide (C), or dexamethasone.

What lipid-lowering medication(s) should be adjusted in a patient with renal impairment? A) Atorvastatin B) Gemfibrozil C) Ezetimibe D) Cholestyramine E) Niacin

B) Gemfibrozil

What lipid-lowering medication(s) should be adjusted in a patient with renal impairment? A) Atorvastatin B) Gemfibrozil C) Ezetimibe D) Cholestyramine E) Niacin

B) Gemfibrozil Fibrates require renal adjustment in patients with mild to moderate renal impairment. Fibrates are also contraindicated in patients with severe renal impairment, defined by manufacturers as <30 mL/min for fenofibrate and <10 mL/min for gemfibrozil. Answer a is incorrect. Atorvastatin and fluvastatin (≤40 mg) are confirmed to be safe in this population. Atorvastatin has even been shown safe in patients with diabetes on dialysis. Although statins at currently approved doses are not considered nephrotoxic, all other drugs in this class should be given at lower doses to reduce myopathy risk. Answer c is incorrect. Ezetimibe does not require adjustment. Answer d is incorrect. Cholestyramine and other BAS do not require adjustment. Answer e is incorrect. There are no specific renal adjustments for niacin, although niacin should be used with caution in this population.

Which serotonin (5-HT3) receptor antagonist is available as a transdermal patch? A) Dolasetron B) Granisetron C) Ondansetron D) Scopolamine

B) Granisetron Sancuso is the trade name for the granisetron transdermal patch (B). It received FDA approval in September 2008. Dolasetron (A) is not available in a transdermal patch. Ondansetron (C) is not available in a transdermal patch. Scopolamine (D) is available in a transdermal patch but is not a 5-HT3 antagonist.

A 51-year-old patient has hepatic encephalopathy that is refractory to standard medical therapy including lactulose. He is intolerant of enteral feedings and is being considered for PN. When compared to a standard amino acid formulation, which of the following amino acid profiles best describes a specialty formulation designed for patients such as this? A) Higher in branched chain amino acids, same level of aromatic amino acids B) Higher in branched chain amino acids, lower in aromatic amino acids C) Higher in essential amino acids, lower in nonessential amino acids D) Fortified with dipeptides containing glutamine

B) Higher in branched chain amino acids, lower in aromatic amino acids Answer b is correct. Aromatic amino acids tend to accumulate in hepatic failure, whereas branched chain amino acids in the blood are decreased. According to the false neurotransmitter theory, aromatic amino acids compete for transport across the blood-brain barrier with branched chain amino acids and are converted to false neurotransmitters such as octopamine, leading to hepatic encephalopathy. Feeding of a parenteral amino acid formula fortified with branched chain amino acids and with lesser amounts of aromatic amino acid formulas can help normalize amino acid levels in the blood. Such formulas have been shown to help patients "wake up" from hepatic encephalopathy, but data demonstrating improved outcomes in terms of mortality with use of these products are limited. They should be reserved for use in hepatic encephalopathy patients who have failed standard medical therapy. Answer a is incorrect. Products containing more branched chain amino acids but the same amount of aromatic amino acids as in standard products have been marketed as Aminosyn-HBC or FreAmine HBC for use in highly stressed patients. Branched chain amino acids are preferentially broken down during stress and are thus fortified in these products. These products are not frequently utilized, largely because of expense and stability concerns limiting their concentration and therefore the amount that can be delivered to a patient in a reasonable fluid load. Answer c is incorrect. Specialty products for use in renal failure contain higher amounts of essential amino acids and lower amounts of nonessential amino acids compared to standard amino acid formulations. This allows for recycling of some endogenous urea nitrogen for synthesis of nonessential amino acids. Although some data support lowering of blood urea nitrogen in patients with renal failure receiving these products, data on improved clinical outcomes are limited. Answer d is incorrect. Amino acid products available in the United States do not contain glutamine due to stability issues. Some products available in other parts of the world do provide dipeptides containing glutamine; the dipeptides overcome the stability issues with glutamine. Glutamine is often depleted during stress and is the preferred fuel source for small intestinal cells.

MN is a 28-year-old pregnant patient. She is currently receiving amoxicillin for a urinary tract infection caused by Escherichia coli. She comes to your pharmacy wanting an influenza vaccination. She hates shots and prefers not to receive any injection. During last year's influenza season she received treatment with oseltamivir. What is the appropriate agent for influenza vaccination for MN? A) LAIV B) IIV3/IIV4 C) Oseltamivir D) Rimantadine

B) IIV3/IIV4 IIV3/IIV4 (B) may be administered to pregnant patients. Vaccination is recommended regardless of the stage of pregnancy. LAIV (A) should not be used in pregnant patients at this time. LAIV is indicated for healthy, nonpregnant patients 2 years of age to 49 years of age. Oseltamivir (C) should not replace IIV3/IIV4 or LAIV as prevention of influenza unless the patient has a contraindication to receiving either IIV3/IIV4 or LAIV (i.e., anaphylaxis to a previous dose). Also, there are insufficient human data to determine the risk to a pregnant woman or developing fetus. Rimantadine (D) should not replace IIV3/IIV4 or LAIV as prevention of influenza. Recent evaluations have demonstrated that the adamantanes (rimantadine and amantadine) have become resistant to seasonal influenza strains and this has limited their use. Also, rimantadine has been found to be embryotoxic in high-dose rat studies.

Question 65 of 100 TC is a 62-year-old man recently diagnosed with Stage 3 CLL. TC has been recently complaining of painful lymphadenopathies as well as easy bruising. His physician has chosen to begin rituximab for 6 cycles. Patient TC relapses following rituximab therapy and now their oncologist is contemplating an oral therapy option such as ibrutinib, idelalisib, or ventoclax. TC has been experiencing diarrhea episodes from irritable bowel syndrome. Which of the following novel oral therapies should not be recommended for patient TC? A) Ibrutinib B) Idelalisib C) Ventoclax D) None of the above

B) Idelalisib Answer b is correct.Idelalisib has FDA Black Box warnings for diarrhea or colitis as well as fatal/serious intestinal perforations. With these warning, idelalisib should be avoided in patient TC. Answer a is incorrect.Ibrutinib is an effective option for CLL and does not have any contraindications for TC. Answer c is incorrect. Ventoclax is an effective option for relapsed CLL and does not have any contraindications for TC. Answer d is incorrect. See above for rationale for answer b.

A 50 kg patient with no history of epilepsy presents in status epilepticus. The patient is given an adequate dose of lorazepam and is about to be given a loading dose of IV phenytoin. Assuming a phenytoin Vd of 0.6 L/kg, what dose of phenytoin should be given to achieve a serum phenytoin concentration of ~16-18 mg/L? A.) 18 mg/kg B.) 500 mg C.) 30 mg/kg D.) 50 mg E.) 5 g

B.) 500 mg The equation for calculation of a loading dose is as follows: Dose = Cp (serum concentration) desired × Vd (volume of distribution). So Cp desired is ~17 × (0.6 L/kg × 50 kg) ≈ 510 mg.

DS is a 35-year-old man with stage IV unresectable melanoma who is in dermatology clinic for follow-up visit and assessment for his second treatment with ipilimumab. What side effect(s) would you monitor in DS prior to treatment? A) Immune-mediated constipation B) Immune-mediated enterocolitis C) Capillary leak syndrome D) Myelosuppression E) Immune-mediated depression

B) Immune-mediated enterocolitis Enterocolitis (eg, diarrhea) was reported as the most common adverse reaction observed with ipilimumab. Median onset occurs about 6 to 7 weeks after initiation of treatment. Moderate cases of noninfectious enterocolitis (diarrhea ≤ six stools) can be treated with antidiarrheals. For severe cases of diarrhea, high-dose corticosteroids (eg, methylprednisolone 1-2 mg/kg/d) should be given intravenously until symptoms subsided, then continue steroids taper over ≥1 month to avoid exacerbation of symptoms. Treatment should be withheld for moderate and severe enterocolitis.

OU is a 63-year-old woman with heart failure (NYHA class I) receiving furosemide 20 mg twice daily, lisinopril 10 mg daily, metoprolol succinate 50 mg daily, digoxin 0.125 mg daily, and spironolactone 25 mg daily. During her routine clinic visit today, pertinent findings include: BP 125/77 mm Hg, HR 70 bpm, RR 13 rpm, K+ 5.1 mmol/L, BUN 32 mg/dL, creatinine 1.1 mg/dL (baseline), and serum digoxin concentration 0.8 ng/mL. Which of the following is the most appropriate change to optimize OU's medical regimen? A) Increase ACE inhibitor dose B) Increase beta-blocker dose C) Increase digoxin dose D) Increase spironolactone dose

B) Increase beta-blocker dose For heart failure management, beta-blocker therapy should be titrated to target doses demonstrated in clinical trials to improve major outcomes such as morbidity and mortality. This patient has adequate blood pressure to increase therapy. In addition, the heart rate is adequate to further titrate her beta-blocker dose. Up-titration of beta-blocker therapy is the safest medication change for this patient. Answer (A) is incorrect. For heart failure management, ACE inhibitor therapy should be titrated to target doses demonstrated in clinical trials to improve outcomes such as morbidity and mortality. This patient has adequate blood pressure to increase therapy. Unfortunately, ACE inhibitors are associated with dose-related hyperkalemia and this patient already has borderline high serum potassium. Answer (C) is incorrect. The desired serum digoxin concentration is less than 1 ng/mL, and thus, this patient's digoxin dose should not be changed. Answer (D) is incorrect. Aldosterone antagonists such as spironolactone are also associated with hyperkalemia. Thus, the spironolactone dose should not be increased and may even need to be reduced to 12.5 mg daily if hyperkalemia persists or worsens.

DE is a 49-year-old woman with a diagnosis of breast cancer that is currently being treated with chemotherapy. Her prior medical history is significant for allergic rhinitis and chronic heartburn. Her home medications include cetirizine, pantoprazole, and transdermal norelgestromin/ethinyl estradiol. The patient was recently diagnosed with chemotherapy-induced anemia with a hemoglobin level of 8.8 g/dL. She was prescribed darbepoetin to treat her anemia. DE may require supplementation with which of the following agents to optimize the effectiveness of darbepoetin? A) Folic acid B) Iron C) Vitamin B12 D) Thiamine

B) Iron Answer b is correct. Patients receiving ESA therapy may develop a functional iron deficiency and require iron supplementation. Clinical trials have also shown that empiric iron supplementation without assessment of iron stores increases the hemoglobin response from erythropoietin. Answer a is incorrect. Typical dietary intake of folic acid is sufficient to compensate for increased use in the setting of increased RBC in response to anemia. Answer c is incorrect. Typical dietary intake of vitamin B12 is sufficient to compensate for increased use in the setting of increased RBC in response to anemia. Answer d is incorrect.Thiamine is not essential in the production of RBCs.

Which of the following is true regarding gonococcal urethritis and/or cervicitis? A) Women older than 30 years are at the highest risk of infection. B) Men are typically symptomatic and have significant discomfort with infection. C) Decreased transmission of HIV infection is associated with gonococcal infection. D) Antibiotic susceptibility data can be obtained using nonculture diagnostic tests for gonorrhea.

B) Men are typically symptomatic and have significant discomfort with infection. Women with gonorrhea are usually asymptomatic or have only minor symptoms. However in men, symptoms of gonorrhea include dysuria and purulent urethral discharge. Because of the early presentation and discomfort associated with symptoms in men, treatment is often sought early enough to prevent complications (B). Women younger than 25 years are at the highest risk of gonococcal cervicitis (A). In both men and women, gonorrhea can cause increased susceptibility to and transmission of human immunodeficiency virus (HIV) infection (C). Nonculture diagnostic tests cannot provide antibiotic susceptibility results (D), which may be necessary in cases of infection that persists after treatment.

A liver transplant patient presented with elevated liver function tests and she admits to not taking her immunosuppressive regimen for the past week, as she was out of town and forgot her medications. The medical team wants to treat her for acute cellular rejection and wants you to recommend an appropriate agent. Which of the following would treat acute cellular rejection most effectively? A) Basiliximab B) Muromonab-CD3 C) Daclizumab D) Rituximab

B) Muromonab-CD3 Muromonab-CD3 is a murine monoclonal antibody directed against CD3 antigen on the surface of T cells, which leads to T-cell inactivation. Muromonab-CD3 can be used both in induction regimens as well as treatment of acute cellular rejection. Choice (A) is incorrect. While the IL-2 receptor antagonists, such as basiliximab and daclizumab can be used in induction regimens, they are not indicated for the treatment of acute cellular rejection. Choice (C) is incorrect. While the IL-2 receptor antagonists, such as basiliximab and daclizumab can be used in induction regimens, they are not indicated for the treatment of acute cellular rejection. Choice (D) is incorrect. Rituximab is a monoclonal antibody directed against CD20 on B cells. Rituximab currently has no role in the treatment of acute cellular rejection, as this type of rejection is largely mediated by T cells.

For a patient receiving etoposide, you should counsel the patient about which of the following side effects? A) Severe acute nausea and vomiting within hours of receiving etoposide B) Myelosuppression about 1 week after receiving etoposide C) Neurotoxicity starting after two or more cycles of etoposide D) Nephrotoxicity occurring after three or more cycles of etoposide

B) Myelosuppression about 1 week after receiving etoposide The dose-limiting toxicity of etoposide is myelosuppression, which occurs 7 to 10 days after receiving etoposide (B). Etoposide is a low-risk agent for causing acute nausea and vomiting (A). Neurotoxicity is not an anticipated toxicity of etoposide (C). One should closely monitor patients receiving cisplatin and taxanes for neurotoxicity. Nephrotoxicity is not an anticipated toxicity of etoposide (D). One should closely monitor patients receiving cisplatin for nephrotoxicity.

HM is a 54-year-old man with a recent diagnosis of metastatic squamous NSCLC. He is scheduled to receive chemotherapy with cisplatin and gemcitabine. Which monoclonal antibody should be added to HM's chemotherapy to improve the efficacy of his chemotherapy? A) Bevacizumab B) Necitumumab C) Pembrolizumab D) Ramucirumab

B) Necitumumab Answer b is correct. In clinical trials necitumumab added to cisplatin/gemcitabine in the treatment of metastatic squamous NSCLC was shown to improve median overall survival. Answer a is incorrect. Bevacizumab has shown benefit when added to carboplatin/paclitaxel in the treatment of non-squamous NSCLC. Bevacizumab should not be used in the treatment of squamous NSCLC due to increased risk of bleeding. Answer c is incorrect.Pembrolizumab has shown to improve the efficacy of carboplatin/pemetrexed for the treatment of non-squamous NSCLC. It has not been shown to improve the efficacy of chemotherapy for squamous NSCLC. Pembrolizumab is approved as a monotherapy for the treatment of metastatic NSCLC, regardless of histology. Answer d is incorrect.Ramucirumab has been shown to improve the efficacy of docetaxel when used in the treatment of relapsed NSCLC. This patient is receiving his first chemotherapy regimen and ramucirumab-containing therapy would not be appropriate at this time.

JH is a 56-year-old woman who was recently diagnosed with stage IIIa adenocarcinoma of the lung. She has an extensive smoking history, smoking one and a half packs of cigarettes per day for 30 years. Her oncologist indicated that the cancer is too large to operate on and she will receive chemotherapy first to shrink the tumor, followed later by surgery. She is to receive chemotherapy with cisplatin and pemetrexed. Which of the following organ toxicities is most likely to occur as a result of the patient receiving cisplatin? A) Hepatotoxicity B) Nephrotoxicity C) Pulmonary toxicity D) Cardiotoxicity

B) Nephrotoxicity Answer b is correct. The dose limiting toxicity of cisplatin is nephrotoxicity. Cisplatin is eliminated renally and can cause direct damage to the kidneys. This can result in increased serum creatinine and electrolyte disturbance such as hypokalemia. Cisplatin is typically administered with aggressive hydration to promote the elimination of cisplatin and decrease nephrotoxicity. Answer a is incorrect. Cisplatin is not known to cause hepatotoxicity. Answer c is incorrect. Cisplatin is not known to cause pulmonary toxicity. Answer d is incorrect. Cisplatin is not known to cause cardiotoxicity.

Select the innate cell that represents the majority of granulocytes and serves as the primary defense against bacterial infections. A) Lymphocytes B) Neutrophils C) Monocytes D) Eosinophils E) Basophils

B) Neutrophils Answer b is correct. Neutrophils represent the majority of granulocytes (80%-90%) and leukocytes (40%-70%) and serve as the primary defense against bacterial infections. Neutrophils, also termed as segs or polymorphonuclear cells, migrate from the bloodstream into infected or inflamed tissue. In this migration process known as chemotaxis, neutrophils reach the desired site and recognize, adhere to, and phagocytose pathogens. During phagocytosis, the pathogen is internalized within the phagocyte. The neutrophil releases its granular contents which lead to destruction of the engulfed pathogen. Answer a is incorrect. Lymphocytes are not part of the granulocyte cells; they are key cells for the adaptive immune system and consist of B lymphocytes (humoral) and T lymphocytes (cell-mediated). T lymphocytes are tailored to defend against infections that are intracellular (viral infections), whereas B lymphocytes secrete antibodies that neutralize pathogens prior to their entry into host cells. Answer c is incorrect. Monocytes account for 1% to 10% of circulating granulocytes and leukocytes, remove dead/damaged tissues, destroy cancer cells, and regulate against foreign substances. Answer d is incorrect. Eosinophils account for less than 7% of circulating granulocytes and leukocytes and are present in the intestinal mucosa and lungs, two locations where foreign proteins enter the body. Eosinophils can phagocytize, kill, and digest bacteria and yeast. Elevations of eosinophil counts are highly suggestive of parasitic infections, or may be associated with allergies and asthma. Answer e is incorrect. Basophils are the least common granulocyte, accounting for 0.1% to 0.3% of granulocytes. Signs and symptoms of allergic responses are linked to basophil and mast cell products. Basophils may be associated with immediate hypersensitivity and delayed hypersensitivity reactions, and increase in chronic inflammation and leukemia.

The most common analytical test used to assess whether exogenous testosterone or one of its derivatives has been used is which of the following? A) Androstenedione: testosterone ratio B) Testosterone: epitestosterone ratio C) Urine sample of 19-Nandrolone D) Blood specimen of 19-NA

B) Testosterone: epitestosterone ratio The most common test (urine) conducted examines the ratio of testosterone to epitestosterone. Normally, this occurs in a 1:1 ratio. Occasionally, some athletes will have an increased ratio of 2:1, and rarely 3:1. A threshold ratio of 4:1 currently exists with WADA; values above this ratio are consistent with exogenous administration of testosterone and its derivatives, suggesting illicit use. Choice (A) is incorrect. No established ratio for A:T has been suggested to determine exogenous administration. Choice (C) is incorrect. While this is a confirmatory test, it has been associated with false positives. The most common analytical test is T:E ratio. Choice (D) is incorrect. While potentially doable, the feasibility and cost of blood specimens is not warranted at this time unless in-depth confirmatory testing is warranted.

A 62-year-old woman has been hospitalized in the ICU for several weeks. She has had a complicated hospital course with sepsis secondary to pneumonia, requiring prolonged courses of antibiotics. Over the past few days, she began spiking fevers and is having a lot of diarrhea. Her stool was positive for C. difficile by polymerase chain reaction. Laboratory values include: Na+ 140 mEq/L, Cl- 110 mEq/L, HCO3- 17 mEq/L, albumin 4.1 g/dL, pH 7.32, and PaCO2 33 mm Hg. What is the most likely primary acid-base disturbance? A) Increased anion gap metabolic acidosis B) Normal anion gap metabolic acidosis C) Metabolic alkalosis D) Respiratory acidosis

B) Normal anion gap metabolic acidosis Answer b is correct. The patient is experiencing diarrhea from an infection with C. difficile. This leads us to believe there is a metabolic process occurring. Looking at the pH, we note the patient to be acidemic. We should examine the HCO3− next, because we believe this to be a primary metabolic disorder, based upon patient presentation, and discover it to be low (metabolic acidosis). When we discover this, it is recommended to next check the anion gap, and it is (140−110−17) 13. The patient's expected anion gap is 12.3 (3 × albumin of 4.1); therefore, the anion gap is not elevated, nor is there anything in the history to point toward an increased anion gap (KILU). Answers a, c, and d would then be incorrect.

AQ is a patient with a past medical history of myocardial infarction (2 years ago), systolic heart failure, chronic kidney disease, hypertension and diabetes. AQ has been short of breath, weak and pale. After a complete evaluation he was diagnosed with pernicious anemia. Select the treatment for AQ. A) Oral cyanocobalamin B) Parenteral vitamin B12 C) Epoetin D) Sodium ferric gluconate

B) Parenteral vitamin B12 Parenteral vitamin B12 is the treatment of choice for pernicious anemia. The parenteral formulation is desired because pernicious anemia patients have an impaired absorption of vitamin B12. Answer (A) is incorrect. Oral vitamin B12 is not the recommended treatment of pernicious anemia. Pernicious anemia is a vitamin B12 deficiency anemia caused by impaired absorption of vitamin B12; therefore, the nutrient needs to be administered parenterally. Oral replacement for pernicious anemia is rarely utilized, but if it is utilized, high doses will be required. Answer (C) is incorrect. Epoetin-alfa (Procrit or Epogen) or darbepoetin-alfa (Aranesp) is utilized for chemotherapy and chronic kidney disease induced anemia. Answer (D) is incorrect. Sodium ferric gluconate (Ferrlecit) is a parenteral formulation of iron. Parenteral formulations of iron are utilized for iron-deficient patients unable to tolerate the oral formulation, noncompliance, or nonresponders (Iron dextran [InFED] is the only parenteral formulation with that indication). Sodium ferric gluconate (Ferrlecit) and iron sucrose (Venofer) are indicated to treat anemia associated with CKD in patients receiving erythropoietin products.

GW is a 58-year-old man with metastatic colon cancer on irinotecan plus cetuximab therapy. When he arrives to the infusion center he is complaining of new "pimples" appearing all over his chest and face. The oncology nurse asks you to counsel him on managing this new finding. Your counseling points would include all of the following except: A) Recommend him to ask his doctor about isotretinoin, give him the FDA approved med guide, and explain about the iPLEDGE program to reduce birth defects. B) Recommend him to apply moisturizing lotion to the affected areas. C) Warn him to use sunscreen since direct sunlight can exacerbate his condition. D) Recommend him to ask his doctor about initiating a tetracycline like doxycycline or minocycline. E) Reassure him that this side effect actually may be predictive of a positive tumor response to this regimen.

B) Recommend him to apply moisturizing lotion to the affected areas. This is not an appropriate counseling point. Although rash caused by EGFR agents have been described in the literature as acneiform, typical acne treatments like retinoids will have no efficacy and may even exacerbate the rash

GM is a 58-year-old African American man with systolic heart failure presenting with a 10-day history of shortness of breath which limits his normal daily activities and increases lower extremity edema. His weight has recently increased by 12 lb. His physical examination is notable for BP 144/77 mm Hg, HR 85 bpm, RR 22 rpm, rales, and 4+ lower extremity edema. Pertinent laboratory values include: sodium 136 mmol/L, potassium 5.4 mmol/L, BUN 23 mg/dL, creatinine 1.1 mg/dL, and digoxin 1.9 ng/mL. Past medical history is significant for hypertension (HTN), gout, COPD, and atrial fibrillation. Current medications include lisinopril 20 mg daily, diltiazem CD 120 mg daily, digoxin 0.250 mg daily salmeterol/fluticasone 250/50, two puffs bid. GM recently began taking naproxen 220 mg tid for gout pain. Furosemide is initiated at 40 mg twice daily to manage fluid overload. Within the following 24 hours, GM experiences a brisk diuresis with improvement in heart failure signs and symptoms. The naproxen is discontinued to avoid exacerbating fluid retention, and colchine is initiated at 0.6 mg bid to manage acute gout flares. Once optimal fluid status is achieved, diltiazem is discontinued and carvedilol initiated at 3.125 mg twice daily to manage hypertension. What additional medication change should be considered to decrease morbidity outcomes (e.g. hospitalizations) for GM? A) Increase lisinopril to 40 mg daily. B) Reduce digoxin to 0.125 mg daily. C) Initiate spironolactone 25 mg daily. D) Initiate candesartan 4 mg daily.

B) Reduce digoxin to 0.125 mg daily. While digoxin does not reduce mortality in HF, it has been shown to reduce hospitalization and improve symptoms. The goal serum digoxin concentration is < 1 ng/mL and serum digoxin concentrations >1 ng/mL may negatively impact GM's morbidity and even mortality. GM's digoxin dose should be reduced to 0.125 ?g daily in an attempt to lower his serum digoxin concentration <1 ng/mL.

Which of the following acid-base disturbances would you expect to see in an acute asthma exacerbation? A) Respiratory acidosis B) Respiratory alkalosis C) Metabolic acidosis with respiratory compensation D) Metabolic alkalosis with respiratory compensation

B) Respiratory alkalosis In acute asthma exacerbations, the bronchioles have become inflamed, resulting in airflow obstruction. This obstruction causes patients with acute exacerbations to breathe more rapidly in an effort to oxygenate their tissues more. Therefore, an increased respiratory rate causes CO 2 to be "blown off," resulting in low Pa CO 2 levels-respiratory alkalosis (B). The primary acid-base disturbance is respiratory alkalosis, so respiratory acidosis (A), metabolic acidosis with respiratory compensation (C), and metabolic alkalosis with respiratory compensation (D) are incorrect.

What is the parenteral asthma medication contraindicated in patients with a corn allergy? A) Prednisone B) Theophylline C) Budesonide D) Omalizumab

B) Theophylline Theophylline (B) is contraindicated in patients with a corn allergy. The premixed injection may contain corn-derived dextrose.

Which of the following statement is correct about sirolimus? A) Sirolimus is not metabolized via cytochrome P450 enzymes, thus decreasing the propensity for drug interactions. B) Sirolimus is less nephrotoxic than calcineurin inhibitors. C) Sirolimus is available in many different formulations, thereby facilitating ease of dosing. D) Sirolimus does not require therapeutic drug monitoring.

B) Sirolimus is less nephrotoxic than calcineurin inhibitors. Answer b is correct.Sirolimus has decreased risk for nephrotoxicity compared to calcineurin inhibitors, therefore, it is commonly used in calcineurin inhibitor-sparing regimens to protect patients from calcineurin inhibitor-induced renal insufficiency. Answer a is incorrect. CYP 3A is the major metabolic pathway for sirolimus. Like cyclosporine and tacrolimus, sirolimus is affected by inducers and inhibitors of CYP 3A. Answer c is incorrect.Sirolimus is only available orally and there is no IV formulation. Furthermore, the tablet is triangular, disallowing splitting to create unavailable tablet strengths. Answer d is incorrect. Therapeutic drug monitoring was deemed unnecessary during clinical trials, as sirolimus was used primarily as an adjunctive agent, in the place of an antiproliferative agent. However, the risk of drug interactions and variability in pharmacokinetics warrant monitoring of sirolimus concentrations.

FR is a 56-year-old African American woman diagnosed with hypertension and started on an ACE inhibitor for blood pressure. About a week later she noticed a persistent cough. At the next visit, FR wants to change the medication, as she cannot tolerate the cough. Unfortunately, she missed her follow-up and returns to you in 6 months. In between appointments she was admitted to the hospital and diagnosed with type 2 diabetes. Which of the following recommendations is best for FR? A) Switch to Lopressor B) Switch to Atacand C) Switch to Altace D) Switch to Cardizem E) Continue her current medication as this side effect usually resolves in a couple of months, and may take up to a year

B) Switch to Atacand FR is a diabetic with hypertension and should be on an RAAS agent, either an ACEI or an ARB (Atacand = candesartan), (B). RAAS agents are proven to slow the progression of target organ damage in diabetics and should be used unless contraindicated. Since cough was the reason for discontinuing the ACEI, switching to an ARB is the best option to control FR's hypertension. ARBs do not cause the breakdown of bradykinin and therefore do not induce a nagging cough.

Which of the following is a correct statement regarding administration of medications via nasogastric tubes? A) The medication must be compatible with isotonic fluids. B) Tablets must be fully crushed and mixed with 15 to 30 mL of water. C) The tube must be flushed with 250 mL of water before and after medication administration. D) Capsule beads should be crushed and mixed with 15 to 30 mL of water.

B) Tablets must be fully crushed and mixed with 15 to 30 mL of water Tablets should be crushed, ground to a very fine powder, and mixed with water prior to administration (B). The tip of a nasogastric tube terminates in the gastrum of the stomach, which is acidic. There is no requirement for compatibility with isotonic fluids (A). Nasogastric tubes should be flushed with 30 mL of water before and after medication administration (C). Capsule beads should not be crushed (D). For those capsules that can be opened, the beads should be given intact and flushed with 30 mL water before and after administration.

SM is a 52-year-old man with a new diagnosis of metastatic adenocarcinoma of the lung. Complete pathologic review revealed the following profile: ALK negative, EGFR negative, BRAF V600E negative. The oncologist has recommended to start chemotherapy with carboplatin/pemetrexed. The carboplatin for SM should be dosed based on: A) Body surface area B) Target area under the curve C) Ideal body weight D) Actual body weight

B) Target area under the curve Answer b is correct. Carboplatin doses are based on target area under the curve (AUC). The dose is calculated using the Calvert formula: Dose = Target AUC × (GFR + 25), where GFR is glomerular filtration rate. Answer a is incorrect. Most chemotherapy is dosed based on area under the curve however carboplatin is not. Answer c is incorrect. Ideal body weight is seldom used as a dosing parameter for chemotherapy. Answer d is incorrect. Actual body weight is used as a dosing parameter for some chemotherapy agents; however, body surface area is most commonly used. For carboplatin, target AUC is the correct dosing parameter.

Identify the medication that can affect osteoblast activity and function. A) Calcitonin B) Teriparatide C) Risedronate D) Raloxifene

B) Teriparatide Answer b is correct.Teriparatide is the only anabolic agent for the treatment of osteoporosis. It targets osteoblast activity and function to build bone. Answers a, c, and d are incorrect. These medications are antiresorptive agents and focus on suppressing osteoclast activity.

Why do patients require flow cytometry prior to administration of a second course of muromonab-CD3? A) To evaluate for the presence of antibodies which could make the patient more susceptible to infusion reactions B) To evaluate for the presence of antibodies which could result in drug inactivation C) To evaluate for the presence of antibodies which could result in heightened immunosuppressive effect D) To evaluate for the presence of antibodies indicating an episode of humoral rejection

B) To evaluate for the presence of antibodies which could result in drug inactivation Prior to a second course of muromonab-CD3 flow cytometry should be conducted to test for the presence of human anti-mouse antibodies (HAMA). If these antibodies are present and circulating, the drug will be inactivated prior to exerting its effect.

Why do patients require flow cytometry prior to administration of a second course of muromonab-CD3? A) To evaluate for the presence of antibodies which could make the patient more susceptible to infusion reactions B) To evaluate for the presence of antibodies which could result in drug inactivation C) To evaluate for the presence of antibodies which could result in heightened immunosuppressive effect D) To evaluate for the presence of antibodies indicating an episode of humoral rejection

B) To evaluate for the presence of antibodies which could result in drug inactivation Prior to a second course of muromonab-CD3 flow cytometry should be conducted to test for the presence of human anti-mouse antibodies (HAMA). If these antibodies are present and circulating, the drug will be inactivated prior to exerting its effect. Choice (A) is incorrect. Because muromonab-CD3 is nonhumanized murine monoclonal antibody, risk of infusion reactions is high and use of this agent requires vigorous premedication with corticosteroids, antihistamine and antipyretic agent. However, flow cytometry will not predict a patient's risk for an infusion reaction, so all patients should be universally premedicated. Choice (C) is incorrect. Flow cytometry conducted prior to muromonab-CD3 infusion is done to detect antibodies formed against the agent itself. If antibodies are present, they will bind the agent, and therefore decrease its immunosuppressive potential, not increase it. Choice (D) is incorrect. Flow cytometry is conducted to determine the presence of antibody-mediated rejection or humoral rejection. However, this is unrelated to the flow cytometry that is performed prior to a second course of muromonab-CD3, which is done to detect the presence of HAMA, which will inactivate the drug, decreasing its efficacy.

JF is a 57-year-old woman who presents to the hospital with ADHF. Vital signs include BP 105/67 mm Hg, HR 83 beats/min, and RR 21. Physical examination reveals 12 cm JVD elevated, +S3, bilateral rales on auscultation, abdominal ascites, and 4+ bilateral edema extending to her thighs. Chest radiograph reveals pulmonary edema and pleural effusions. Hemodynamic measurements obtained by PAC include PCWP 29 mm Hg, CI 1.7 L/min/m2, and SVR 700. Her laboratory values are all normal, except sodium 132 mmol/L, BUN 49 mg/dL, and SCr 2.1 mg/dL (baseline BUN/SCr 32 and 0.9). Her drugs on admission include enalapril 10 mg twice daily, carvedilol 12.5 mg twice daily, bumetanide 2 mg twice daily, hydralazine 50 mg three times daily, isosorbide dinitrate 40 mg three times daily, and aspirin 81 mg/d. Discharge for JF is planned with outpatient inotropic therapy based on your recommendations. According to the American College of Cardiology/American Heart Association (ACC/AHA) HF performance measures, which one of the following should be completed prior to JF's discharge? A) BNP measurement B) Written documentation of her EF C) Care of her IV access site D) Advance directives

B) Written documentation of her EF Measurement of BNP (Answer a), intravenous access site care (Answer c), and advance directives (Answer d) are all important to address prior to discharging a patient for ADHF. However, written documentation of EF (Answer b) is one of several performance measures recommended by ACC/AHA to be obtained prior to discharge for ADHF. Prescribing of an ACE inhibitor or ARB as well as a β-blocker in HF with reduced ejection fraction (HFrEF) is also a current performance measure. In JF's case, if she is unable to tolerate these guideline-recommended medications (ie, due to hypotension or other adverse effects), intolerance should be documented in the medical record.

DA is a 55 yo female who has PMH including depression, seizures, HTN, dyslipidemia, bipolar disorder, T2DM, and schizophrenia. She presents to the hospital today with complaints of drowsiness, weight gain, altered mental status, and a coarse hand tremor. Her vitals are the following: BP 160/88 mm Hg, HR 110 bpm, RR 20. Her labs include Na 130 mEq/L, K 4.8 mEq/L, SCr 1.2 mg/dL. Her current medication list includes the following: ASA 81 mg PO daily Atorvastatin 40 mg PO daily Bupropion 150 mg SR PO BID Clozapine 12.5 mg PO BID Hydrochlorothiazide 12.5 mg PO daily Lisinopril 40 mg PO daily Lithium 300 mg PO daily Metformin 1000 mg PO BID Which of the following adverse effects-medication matches are correct? Select all that apply. A. Atorvastatin - seizures B. Clozapine - neutropenia C. Lithium - hyperthyroidism D. Lisinopril- hyperkalemia E. Hydrochlorothiazide - hypocalcemia

B. Clozapine - neutropenia D. Lisinopril- hyperkalemia Answer A is incorrect because atorvastatin (statins) is not associated with seizure risk. These agents are more associated with muscle pain, myalgias, and worry for a rare side effect of rhabdomyolysis. NOTE: In this patient the main medication we would be concerned with seizures is bupropion, which is contraindicated in patients with a history of seizures due to lowering of seizure threshold. Answer B is correct because clozapine is associated with neutropenia. Neutropenia causes an impaired immune system putting the patient at risk of increased infections. While the mechanism is not fully known, neutropenia is defined as a low absolute neutrophil count (ANC). As the ANC level drops < 1500/microL, there are different dosing and ANC monitoring recommendations. This is why clozapine is considered a REMS drug requiring the patient to have their ANC monitored while taking this medication. Answer C is incorrect because lithium is associated with hypothyroidism. In this patient you could argue that the patient is having signs of lithium toxicity (altered mental status and coarse hand tremor) and ordering a lithium level would be warranted. Lithium can also cause serotonin syndrome, especially when taking with serotonergic agents (e.g. SSRIs). NOTE: DDI in this case include lisinopril and HCTZ as both of these can cause increased levels of lithium. Additionally, this patient is hyponatremic, which would put the patient at an increased risk of lithium toxicity. A normal therapeutic range for lithium for bipolar disorder is 0.8 to 1.2 mEq/L. Answer D is correct because ACE inhibitors, like lisinopril, are associated with increased potassium levels. Aldosterone is responsible for the excretion of potassium from the kidneys via the renin-angiotensin-aldosterone-system (RAAS). Since ACE inhibitors prevent aldosterone formation, this causes an increase in potassium reabsorption leading to hyperkalemia. Answer E is incorrect because thiazide diuretics are associated with hypercalcemia NOT hypocalcemia. This differs from loop diuretics which do cause hypocalcemia.

If the source of the phenol is Liquefied Phenol, USP containing 10% water and having a specific gravity of 1.06, how many mL would be required? A.) 0.04 B.) 0.06 C.) 0.05 D.) 0.07 E.) 0.03

B.) 0.06 0.5% requires 0.05 g of phenol (10 g x 0.005 = 0.05 g). Liquefied phenol is a w/v solution containing 90% phenol or 90 g in 100 mL; thus 90 g/100 mL = 0.05 g/x, with x = 0.056 mL or 0.06 mL. Knowing the specific gravity is not required.

The particle size range for blocking pulmonary capillaries for perfusion imaging is: A.) 1-10micrometers B.) 10-150micrometers C.) 0.150-1centimeters D.) 1.5-15centimeters

B.) 10-150micrometers Reason: Any smaller would bypass the lung capillaries and become localized in the RES via the liver, spleen, and bone marrow. Larger particles would cause adverse reactions and increase clearance time.

For purposes of classifying a drug as an orphan drug, a rare disease or condition is defined as follows: a disease or condition that affects (a) fewer than ______ persons in the United States or (b) more than _____ persons in the United States provided that there is no reasonable expectation that the cost of developing and making the drug available will be recovered. A.) 100,000 B.) 200,000 C.) 250,000 D.) 500,000 E.) 1,000,000

B.) 200,000 The applicable section of the federal FDCA is 21 USC 360bb.

Cough and cold products are not to be used in children less than _____ years of age. A.) 2 B.) 4 C.) 6 D.) 8 E.) 12

B.) 4 The Consumer Health Care Products Association announced in October 2008 that manufacturers were voluntarily updating all cough and cold products to state "do not use" in children under 4 years of age.

You are the clinical pharmacist in a critical care unit where a full-term 6-month-old has been admitted. She is 62 cm in height and 6 kg in weight. What is her estimated creatinine clearance (using the Schwartz equation) if her serum creatinine is 0.68 mg/dL? A.) 71 mL/min/1.73m2 B.) 41 mL/min/1.73m2 C.) 62 mL/min/1.73m2 D.) 47 mL/min/1.73m2 E.) 54 mL/min/1.73m2

B.) 41 mL/min/1.73m2 EXPLANATION: The Schwartz equation is as follows: CrCL = (k × length in cm) / SCr, where the constant k = 0.45 for a term infant younger than 1 year of age. Thus, CrCL = (0.45 × 62 cm) / 0.68 mg/dL = 41.03 mL/min.

Prasugrel will provide the greatest clinical benefit in which one of the following patients? A.) 42-year-old female with NSTEMI B.) 50-year old-male with DM and STEMI C.) 52-year-old male with STEMI and history of stroke D.) 70-year-old female weighing 55 kg with UA E.) 83-year-old male with NSTEMI and CKD

B.) 50-year old-male with DM and STEMI Subgroups in the TRITION TIMI-38 study showing the greatest benefit included those with STEMI and DM (choice B is the correct choice). Those groups experiencing harm or no net benefit with prasugrel included those with history of a stroke or TIA (absolute contraindication to the use of prasugrel), patients 75 year of age or older, and patients weighing less than 60 kg. Although prasugrel could be used in a 42-year-old patient with NSTEMI (choice A), this response is not the best choice because those with DM or STEMI (choice B) would be expected to achieve greater benefit from prasugrel without additional risk of bleeding. Choices C, D, and E are incorrect because of history of stroke (contraindicated), weight (< 60 kg), and age (> 75 years), respectively.

How many codons encode the 20 different amino acids as dictated by the genetic code? A.) 50 B.) 64 C.) 62 D.) 60 E.) 66

B.) 64 Sixty-four codons encode 20 different amino acids as dictated by the genetic code.

What weight of magnesium chloride (MgCl2, mw = 95.3) is required to prepare 200 mL of a solution that is 5 millimolar? A.) 191 mg B.) 95.3 mg C.) 19.1 mg D.) 0.5 mg E.) 910 mcg

B.) 95.3 mg A 1 millimolar solution will contain 95.3 mg in 1,000 mL. A 5 millimolar solution will contain 95.3 mg × 5 = 477 mg in 1,000 mL. Thus, 200 mL of a 5 millimolar solution will contain 477 mg/5 = 95.3 mg in 200 mL.

A hospitalized patient who has received 1 week of antibiotic therapy with piperacillin/tazobactam (Zosyn) has developed acute kidney injury, and his clinical presentation includes rash, fever, and eosinophils in the urine. Which of the following types of acute kidney injury is most suggested by this presentation? A.) Prerenal acute kidney injury B.) Acute allergic interstitial nephritis C.) Acute tubular necrosis D.) Glomerulonephritis E.) Postrenal acute kidney injury

B.) Acute allergic interstitial nephritis EXPLANATION: Acute allergic interstitial nephritis is associated with a triad of findings that include rash, fever, and eosinophiluria. (Note: these findings do not occur in all cases of allergic interstitial nephritis.) Penicillins, including Zosyn, are known to cause this type of kidney injury.

The ability of an aging individual to live independently is closely tied to which of the following three functions? A.) Ambulation, confidence, financial security B.) Ambulation, bowel/bladder control, cognitive functioning C.) Financial independence, cognitive functioning, visual acuity D.) Visual acuity, bowel/bladder control, ambulation E.) Cognitive functioning, ability management food intake, toileting ability

B.) Ambulation, bowel/bladder control, cognitive functioning The big three in geriatrics is the ability to ambulate independently, being able to successful management bowel and bladder functions and to be cognitively intact. The loss of one or more of these abilities can often result in the loss of independent living.

Empiric therapy for meningitis for patients up to 1 month of age includes: A.) Vancomycin and ampicillin B.) Aminoglycoside and ampicillin C.) Ceftriaxone and vancomycin D.) Vancomycin and aminoglycoside E.) Ampicillin and ceftriaxone

B.) Aminoglycoside and ampicillin This regimen covers the most likely organism(s) for meningitis in this age group: gram-negative enterics, such as E coli, group B streptococci, and Listeria monocytogenes.

After the initial successful cardioversion, the patient in the previous question continues to have recurrent symptomatic atrial fibrillation episodes. Chronic therapy to maintain sinus rhythm is to be initiated. Which of the following antiarrhythmic drugs would be the best choice to maintain sinus rhythm? A.) Flecainide B.) Amiodarone C.) Sotalol D.) Ibutilide E.) Dronedarone

B.) Amiodarone Because the patient has heart failure, the results of the Cardiac Arrhythmia Suppression Trial (CAST) indicate that class Ic agents should be avoided because of increased risk of death. Sotalol may worsen heart failure. Ibutilide is indicated for conversion only, not for maintenance of sinus rhythm. Dronedarone is contraindicated in patients with heart failure because of an increased risk of mortality.

All of the following are acceptable techniques for managing data from those patients who withdraw from a study prior to completing the study protocol except: A.) Analyzing the data using an intention to treat method including patients competing the trial and those who withdrew from the study B.) Analyzing the data using a per protocol analysis including those patients who were compliant with the protocol until the time of their withdrawal. C.) Analyzing the data using a per protocol method with categorization of withdrawal subject characteristics including reason for withdrawal. D.) Analyze the data using both intention to treat and per protocol methods.

B.) Analyzing the data using a per protocol analysis including those patients who were compliant with the protocol until the time of their withdrawal. Explanation: Patients who withdraw from a study prior to completing the study protocol can be managed by analyzing data using an intention to treat method, intention to treat and per protocol methods and by categorizing patients according to reason for withdrawal using a per protocol method. Analyzing only data from compliant patients without regard for patients withdrawing from the study is inappropriate and is the exception.

A patient presents with a black, necrotic, painless skin lesion on her arm. She also complains of fever, malaise, headache, and swelling of her underarm lymph nodes. Which of the following is the possible biological agent responsible for these symptoms? A.) Hemorrhagic fever virus B.) Anthrax C.) Botulism D.) Arsine

B.) Anthrax Anthrax forms a painless, necrotic ulcer. Hemorrhagic fever viruses cause a rash that develops into petechiae, ecchymosis, hemorrhages, and other bleeding symptoms. Botulism causes a symmetric descending paralysis. Tularemia causes a nonspecific febrile illness that rapidly develops into pneumonia. Arsine is a chemical agent that causes nausea, vomiting, hemolysis, and secondary renal failure. Arsine is produced when water comes into contact with metallic arsenide or when acids come into contact with metallic arsenic or arsenical compounds. The mechanism of hemolysis is not specifically known, but the most recent mechanism postulated involves a direct arsine-hemoglobin interaction that forms arsenic metabolites, causing direct alteration of the erythrocyte cell membrane.

A young female enters your pharmacy and informs you that she plans on becoming pregnant and would like you to review her medication profile to see if any of her medications would be potentially harmful. On reviewing her profile, you notice that she is taking Arava for RA. Which of the following is the most appropriate response? A.) Arava is a category C drug and could potentially harm the fetus. She should discuss the risks and benefits of becoming pregnant with her health care provider first. B.) Arava is contraindicated in pregnanacy, and she should undergo the drug elimination procedure with cholestyramine before trying to conceive. C.) Arava is a category X drug with no active metabolites and a short half-life; therefore, she should discontinue the drug and wait 1-2 weeks before trying to become pregnant. D.) Arava is a category B drug, and the risk of toxicity to the fetus is extremely low.

B.) Arava is contraindicated in pregnanacy, and she should undergo the drug elimination procedure with cholestyramine before trying to conceive. Because Arava is a teratogenic agent with an active metabolite with a long half-life, a drug elimination procedure should be performed before becoming pregnant.

What is the generic name for Imuran? A.) Mycophenolate mofetil B.) Azathioprine C.) Cyclosporine D.) Tacrolimus E.) Prednisone

B.) Azathioprine

Which one of the following immunosuppressants is associated with pancreatitis ? A.) Antithymocyte globulin B.) Azathioprine C.) Basiliximab D.) Cyclosporine D.) Tacrolimus

B.) Azathioprine

Of the following medications, which one would be expected to cause the most interference with the beneficial effects of donepezil therapy? A.) Azilect B.) Benztropine C.) Dextromethorphan D.) Fluoxetine (CYP2D6 inhibitor effects) E.) Namenda

B.) Benztropine EXPLANATION: Benztropine is a potent anticholinergic and would counter any benefits from the use of donepezil. Donepezil is an acetylcholinesterase inhibitor and, therefore, would increase acetylcholine activity. Namenda is an NMDA (N-methyl-D-aspartate) receptor antagonist and is used in conjunction with donepezil. Dextromethorphan is also an NMDA receptor antagonist and should not have a negative effect on donepezil efficacy. Fluoxetine is a CYP2D6 enzyme inhibitor. Because donepezil is a CYP2D6 substrate, levels of donepezil would be increased when given currently. Although increasing the donepezil level might increase the risk of adverse effects, it could actually improve the clinical benefits of donepezil. The concurrent use of the monoamine oxidase inhibitor Azilect would not affect the clinical benefits of donepezil.

A 48-year-old woman with Stage III ER+, Her-2 negative breast cancer has completed 2 years of treatment with tamoxifen. At a recent follow-up with her oncologist, the patient expresses concerns about her changes in mood and requests an antidepressant. Which of the following is the safest antidepressant to initiate in this patient? A.) Paroxetine B.) Citalopram C.) Fluoxetine D.) Buproprion E.) Duloxetine

B.) Citalopram Correct answer: B Citalopram is metabolized by CYP3A4, 2C19, and 2D6; citalopram does not inhibit 3A4 or 2D6 and can be used safely with tamoxifen without compromising its efficacy.Tamoxifen is metabolized via CYP2D6 to 4-hydroxytamoxifen and via CYP3A4/5 to N-desmethyl-tamoxifen. Each is then further metabolized into endoxifen (4-hydroxy-tamoxifen via CYP3A4/5 and N-desmethyl-tamoxifen via CYP2D6. Both 4-hydroxy-tamoxifen and endoxifen are 30-fold to 100-fold more potent than tamoxifen. Answer A:Paroxetine is a potent inhibitor of CYP2D6 and would result in decreased plasma concentrations of the potent tamoxifen metabolites and therefore efficacy. Answer C:Fluoxetine is a potent inhibitor of CYP2D6 and would result in decreased plasma concentrations of the potent tamoxifen metabolites and therefore efficacy. Answer D:Hydroxybupropion, an active metabolite of bupropion, is an inhibitor of CYP2D6 and would result in decreased plasma concentrations of the potent tamoxifen metabolites and therefore efficacy. Answer E:Duloxetine is an inhibitor of CYP2D6 and would result in decreased plasma concentrations of the potent tamoxifen metabolites and therefore efficacy. Bottom Line:Antidepressants that inhibit CYP2D6 reduce the efficacy of tamoxifen in ER+ breast cancer patients. Citalopram is metabolized by CYP3A4, 2C19, and 2D6; citalopram does not inhibit CYP3A4 or 2D6 and can be used safely with tamoxifen without compromising its efficacy.

Which calcium supplement would be best for a patient who is taking chronic omeprazole for severe gastroesophageal reflux disease? A.) Caltrate B.) Citracal C.) Oscal D.) Tums

B.) Citracal EXPLANATION: Calcium citrate (Citracal) does not require acid to be absorbed and is preferred over calcium carbonate products for patients taking concurrent proton pump inhibitors or for those with achlorhydria.

Which benzodiazepine is metabolized by nitro reduction? A.) Alprazolam B.) Clonazepam C.) Lorazepam D.) Diazepam E.) Halazepam

B.) Clonazepam Alprazolam, diazepam, and halazepam are metabolized through an oxidative process. Lorazepam is metabolized through conjugation.

Of the following medications to treat Parkinson's disease which would never be considered for monotherapy? A.) Benztropine B.) Comtan C.) Pramipexole D.) Selegiline E.) Sinemet

B.) Comtan Comtan (entacapone) is always used as an adjunctive agent with carbidopa/levodopa. Entacapone inhibits catecholamine O-methyl transferase (COMT) which is a secondary metabolic pathway for levodopa. In combination with carbidopa there is increased levodopa availability.

Which of the following is a black box warning for itraconazole? A.) Tendon Rupture B.) Congestive Heart Failure C.) Dementia-related psychosis D.) Suicidal ideation E.) Stevens-Johnson Syndrome

B.) Congestive Heart Failure Correct answer: B Itraconazole is an azole antifungal used to treat a variety of fungal infections. Its black box warning states, "Negative inotropic effects have been observed following intravenous administration of itraconazole. Discontinue or reassess use if signs or symptoms of heart failure occur during treatment." Answer A:Tendon rupture is not a black box warning for itraconazole, but it is an important black box warning for fluoroquinolones. Answer C:Dementia-related psychosis is not a black box warning for itraconazole, but it is an important black box warning for antipsychotics. Answer D:Suicidal ideation is not a black box warning for itraconazole, but it is an important black box warning for antidepressants. Answer E:Stevens-Johnson syndrome is not a black box warning for itraconazole, but it is an important black box warning for lamotrigine. Bottom Line:Itraconazole is an azole antifungal agent that has a black box warning of congestive heart failure.

Rank the following systemic corticosteroids in order from least to most potent.(ALL options must be used.) A.) Prednisone B.) Cortisone C.) Hydrocortisone D.) Dexamethasone E.) Triamcinolone

B.) Cortisone (weakest) C.) Hydrocortisone A.) Prednisone E.) Triamcinolone D.) Dexamethasone (strongest) The dose equivalence of each corticosteroid is detailed in the chart below. The higher the dose equivalence, the less potent the drug is. Glucocorticoid Equivalent Potency (mg) Cortisone 25mg Hydrocortisone 20 mg Prednisone 5 mg Prednisolone 5mg Triamcinolone 4mg Methylprednisolone 4mg Betamethasone 0.6 mg Dexamethasone 0.75 mg The potency can be determined by looking at the dose equivalence. The more potent an agent, the less amount needed to exert the same effect. As outlined in the chart, dexamethasone is one of the most potent corticosteroids, with cortisone and hydrocortisone being the least potent. Bottom Line: Dexamethasone is the most potent corticosteroid, followed by prednisone, hydrocortisone, and cortisone.

A patient with initially mild symptoms of organophosphate poisoning deteriorates and develops seizures. Which of the following drugs would be indicated? A.) Phenytoin B.) Diazepam C.) Lithium D.) Buprenorphine

B.) Diazepam Recommended treatment for acute-onset seizures attributable to organophosphate agents or drug overdoses is either diazepam or lorazepam. Phenytoin is a seizure medication, but benzodiazepines are preferred over phenytoin for the control of acute seizures. Lithium is not a seizure medicine and, in fact, may cause seizures with elevated blood concentrations. Buprenorphine is a medication-assisted therapy for opioid use disorder.

Which of the following statements regarding the discontinuation of lamotrigine is true? A.) Unlike other antiepileptic medications, lamotrigine can be stopped abruptly due to its long half life B.) Do not abruptly discontinue; the dose should be gradually decreased by about 50% a week over at least 2 weeks C.) Lamotrigine can be abruptly discontinued as long as the patient is already on an alternative antieplileptic medication D.) Lamotrigine withdrawal can cause headache, fatigue and irritability E.) Stopping lamotrigine in a patient also on valproic acid may cause higher levels of valproic acid

B.) Do not abruptly discontinue; the dose should be gradually decreased by about 50% a week over at least 2 weeks Correct answer: B All anticonvulsants, including lamotrigine,should not be discontinued abruptlybecause of the risk of increased seizure frequency. The dose should be gradually decreased by ~50% per week over at least 2 weeks, if possible. Valproic acid may increase serum concentrations of lamotrigine, therefore stopping valproic acid a patient also on lamotrigine may cause lamotrigine levels to decrease. Answers A & C:All anticonvulsants, including lamotrigine, should not be discontinued abruptly because of the risk of increased seizure frequency. Answer D:All anticonvulsants, including lamotrigine, should not be discontinued abruptly because of the risk of increased seizure frequency. Headache, fatigue, and irritability are not associated with lamotrigine withdrawal. Answer E:Valproic acid may increase serum concentrations of lamotrigine, therefore stopping valproic acid a patient also on lamotrigine may cause lamotrigine levels to decrease. Lamotrigine does not affect levels of valproic acid, so stopping lamotrigine will have no impact on valproic acid. Bottom Line:All anticonvulsants, including lamotrigine, should not be discontinued abruptly because of the risk of increased seizure frequency. The dose should be gradually decreased by ~50% per week over at least 2 weeks, if possible.

Which of the following amendments to the federal Food, Drug, and Cosmetic Act created the legal distinction between legend (prescription) and nonlegend (nonprescription) drug products? A.) Kefauver-Harris B.) Durham-Humphrey C.) Waxman-Hatch D.) Dingell-Williams E.) Rogers-Williams

B.) Durham-Humphrey Enacted in 1951, the Durham-Humphrey Amendment to the federal Food, Drug, and Cosmetic Act created the distinction between legend (prescription) and nonlegend (nonprescription) drugs. The amendment created this distinction by exempting certain drugs (legend drugs) from the "adequate directions for use" labeling requirement. After the amendment, legend drugs could be sold only by a pharmacy or other authorized dispenser pursuant to a prescription and could be dispensed in a container with a label that included directions for use. To distinguish legend from nonlegend drugs, the amendment required a commercial container of a legend drug to carry on the label the statement "Caution: Federal law prohibits dispensing without a prescription." This language was known as the "federal legend" and is the reason prescription drugs are also referred to as legend drugs. This statement was replaced with the "Rx Only" symbol by enactment of the Food and Drug Administration Modernization Act in 1997.

Which of the following vasoactive agents should be limited to second line use for treating hypotension in septic shock because it has an increased risk of tachycardia? A.) Vasopressin B.) Epinephrine C.) Norepinephrine D.) Dobutamine E.) Phenylephrine

B.) Epinephrine Per the 2012 SCCM guidelines, epinephrine should be considered a second line agent because of tachycardia (among other adverse events). Some human and animal studies suggest epinephrine has deleterious effects on splanchnic circulation and produces hyperlactatemia, but no clinical evidence shows that epinephrine results in worse outcomes, and it should be the first alternative to norepinephrine. Epinephrine may increase aerobic lactate production via stimulation of skeletal muscles' ?2-adrenergic receptors and thus may prevent the use of lactate clearance to guide resuscitation.

What alcohol is used to compound a nonsterile preparation when its type and percentage are not specified? A.) Ethyl alcohol 100% B.) Ethyl alcohol 95% C.) Ethyl alcohol 70% D.) Isopropyl alcohol 70%

B.) Ethyl alcohol 95% Alcohol, USP (95% ethyl alcohol), is to be used for nonsterile compounded preparations whenthe formula does not specify the type or percentage of alcohol.

Which of the following refers to a plasmid designed to allow for the expression of an inserted gene within a host cell for the production of the specified protein? A.) Cloning vector B.) Expression vector C.) Transcription factor D.) Translation initiation factor E.) Transposable genetic element

B.) Expression vector A cloning vector is used to carry a fragment of DNA into a cell for cloning. A transcription factor is a protein that regulates transcription in eukaryotic cells. A translation initiation factor, as its name implies, is involved in the initiation of translation. A transposable genetic element, or transposon, is a portion of DNA that can move from one part of the genome to another.

If a USP/NF component is unavailable for a compounded preparation, which chemical grade would be the next best choice to use? A.) ACS B.) FCC C.) AR D.) Tech

B.) FCC SP/NF or FCC grade components are preferred for compounding. Always obtain the certificate of analysis (COA) for every component used, and use components manufactured in FDA-registered facilities. Other component grades may be used if the COA is reviewed to determine that they are appropriate for use.

If 10 g of solute is dissolved in enough purified water to produce 100 mL of solution, you have produced a 10% w/w solution. A.) True B.) False

B.) False EXPLANATION: You have produced a 10% w/v solution.

A pharmacy may dispense a prescription that contains the words "for office use" in place of a patient's name and address when the prescription has been issued by a practitioner seeking to purchase controlled substances for administration to patients while in the practitioner's office. A.) True B.) False

B.) False The applicable DEA regulation is 21 CFR 1306.04.

The person granted power of attorney to obtain and execute a DEA Form 222 must be a registrant with the DEA. A.) True B.) False

B.) False The statement is false. Pharmacists can execute a DEA Form 222, but they do not have to be registrants. Moreover, there is no requirement that the person granted power of attorney be a health care practitioner. A pharmacy can grant power of attorney to any individual, whether or not the individual is located at the registered location. The applicable section of the DEA Pharmacist's Manual is Section VIII

Which of the following is major side effect with hydroxychloroquine therapy? A.) Pulmonary Toxicity B.) Retinal Damage C.) Bone marrow suppression D.) Stomatitis

B.) Retinal Damage Therapy with hydroxychloroquine can lead to retinal damage. This occurs as a result of deposition of the drug in the melanin layer of the cones. Many times this is a dose depended reaction and may occur with doses over 400mg daily. The other answer choices are complications of methotrexate therapy.

Which class of antibiotics should be given intravenously, if possible, when treating a severe infection while a patient is on continuous enteral feeds? A.) Macrolides B.) Flouroquinolones C.) Cephalosporins D.) Vancomycin

B.) Flouroquinolones Correct answer: B Several medications can have altered absorption kinetics when coadministered with enteral nutrition (EN). Some medications will require dose adjustments, frequency changes, or holding of the EN feeds before and after medication administration. In general,fluoroquinolones(FQs) should be given intravenously (IV) if possible when a patient is receiving continuous EN. This class of antibiotics has a high propensity towards binding with the cations in EN feeds. If converting to IV isn't possible, FQs should be separated from the EN feeds by at least 1 hour and the EN tubes should be flushed with at least 15 mLs of water before and after the medication is given. Answer A:There are no interactions between macrolides (i.e. azithromycin) and EN. Answer C:Absorption of cephalosporins are generally thought to be decreased by 10% when given with EN, however, no adjustments to therapy is suggested. Answer D:There are no interactions between vancomycin and EN. No adjustments are needed. Bottom Line:Fluoroquinolone antibiotics have a binding interaction with enteral nutrition. Feeds should be separated by 1 hour if the medication is given orally. Intravenous administration is recommended if a patient is receiving continuous enteral feeds.

Which of the following corticosteroid agents has the highest ratio of mineralocorticoid to glucocorticoid activity? A.) Hydrocortisone B.) Fludrocortisone C.) Prednisone D.) Methylprednisolone E.) Dexamethasone

B.) Fludrocortisone Fludrocortisone is almost purely mineralocorticoid and is used for increasing blood pressure rather than for anti-inflammatory effects.

Which oral triptan has the longest duration of action? A.) Amerge B.) Frova C.) Zomig D.) Imitrex E.) Midrin

B.) Frova EXPLANATION: Frova has the longest duration of action of all the triptans. It has a half-life of about 25 hours. Frova has a long half-life but a shorter onset. The other triptans listed have shorter durations of action, but they may be required when a more rapid onset is desired. Midrin is not a triptan.

Which of the following combinations represents optimal pharmacotherapy of patients with stage C HFrEF? A.) Furosemide, clonidine, hydrochlorothiazide, and propranolol B.) Furosemide, lisinopril, spironolactone, and carvedilol C.) Carvedilol, verapamil, amlodipine, and nesiritide D.) Cardizem, hydrochlorothiazide, digoxin, and sotalol E.) Dobutamine, amiodarone, furosemide, and nitroglycerin

B.) Furosemide, lisinopril, spironolactone, and carvedilol Furosemide, lisinopril (an ACEI), spironolactone (an aldosterone antagonist), and carvedilol (a ß-blocker) in combination should be used routinely in patients with HFrEF to reduce mortality, improve symptoms, and slow heart failure progression as described in the 2017 ACC-AHA Guidelines for the Management of Heart Failure.

Dapsone can cause hemolytic anemia in patients with which of the following? A.) Presence of HLA-B5701 gene B.) Glucose-6-Phosphate dehydrogenase deficiency C.) von Willebrand factor deficiency D.) Antihemophilic Factor VIII deficiency E.) Folic acid deficiency

B.) Glucose-6-Phosphate dehydrogenase deficiency Correct answer: B Dapsone is an alternative treatment option for Pneumocystis pneumonia prophylaxis and treatment as well as prophylaxis for toxoplasmosis when a patient cannot tolerate Bactrim. Dapsone may cause hemolytic anemia in patients with glucose-6-phosphate dehydrogenase deficiency. Therefore, it is important to test patients for this deficiency prior to initiating dapsone therapy. Answer A:Presence of HLA-B5701 gene predisposes patients to hypersensitivity to the HIV medication abacavir. Dapsone would not cause hemolytic anemia in these patients. Answer C:Von Willebrand factor is a blood glycoprotein involved in hemostasis and its deficiency may cause thrombotic thrombocytopenic purpura, Hedye's syndrome, and possibly hemolytic-uremic syndrome. Dapsone would not cause hemolytic anemia in these patients.Answer D:Antihemophilic factor VIII deficiency, or hemophilia A, is a genetic deficiency in clotting factor VIII that causes increased bleeding and usually affects men. Dapsone would not cause hemolytic anemia in these patients. Answer E:Folic acid deficiency may be due to medications, alcoholism, and vegetarian diets, which may cause folic acid deficiency anemia. Dapsone would not cause hemolytic anemia in these patients. Bottom Line:Dapsone can cause hemolytic anemia in patients with glucose-6-phosphate dehydrogenase deficiency, so it is important to test for this deficiency prior to initiation of dapsone therapy.

Which of the following is a risk factor for torsades de pointes with sotalol? A.) Hypercalcemia B.) Hypomagnesemia C.) Hypernatremia D.) Hypothyroidism E.) Hyperkalemia

B.) Hypomagnesemia Hypomagnesemia, and hypokalemia, are important risk factors for the development of torsades de pointes.

Which of the following is true? I. Unless a patient is experiencing a life-threatening adverse effect, anticonvulsants should not be abruptly discontinued. II. Patients with juvenile myoclonic epilepsy should receive lifelong therapy. III. Only 10% of patients who remain seizure free for 2 years can have their anticonvulsants successfully withdrawn. A.) I B.) I and II C.) I and III D.) II and III E.) I, II, and III

B.) I and II Abrupt discontinuation of an anticonvulsant may cause a patient to develop status epilepticus. Patients with juvenile myoclonic epilepsy have a genetic abnormality that will require therapy for life.

SA states she has problems swallowing large pills. Her current ART regimen is tenofovir/emtricitabine, etravirine, darunavir and ritonavir capsules. What are some changes that can be made to help her swallow these medications. I. Instruct patient to dissolve etravirine in water and drink II. Change ritonavir capsules to tablets III. Change darunavir tablets to solution A.) I and III B.) I and II C.) II and III D.) III only E.) I, II and III

B.) I and II Explanation: Etravirine can be dissolved in about 5 mL of water, stir until water looks milky then if desired, add to more water, orange juice or milk. The glass should be rinsed several times with the liquid and swallowed to make sure patient gets the entire dose. Grapefruit juice, warm or carbonated beverages should be avoided. Ritonavir tablets are much smaller than the capsules and do not require special storage precautions. Ritonavir is also available as a solution but taste can be a barrier for many patients.

Which of the following statements best describe the penicillins? I. Exhibit concentration-dependent bacterial killing II. Exhibit time-above-MIC-dependent bacterial killing III. Exhibit excellent MRSA activity A.) I only B.) II only C.) III only D.) I & II E.) II & III

B.) II only Penicillins exhibit time-dependent bacterial killing and have no activity against MRSA

Which of the following statements concerning Alzheimer's disease is (are) true? I. Dementia of Alzheimer's disease is slowly progressive, and patients do not develop hallucinations or delusions unless they have concurrent vascular dementia. II. Neurofibrillary tangles are composed of amyloid proteins and disrupt function when they are deposited on neurons. III. The diagnosis of Alzheimer's disease requires the presence of memory loss and at least one or more additional symptoms. A.) I only B.) III only C.) I and II only D.) II and III only E.) I, II, and III

B.) III only EXPLANATION: Hallucinations and delusional thinking can be seen in Alzheimer's disease independent of additional diagnoses. Neurofibrillary tangles exist within neurons, whereas neuritic plaques are composed of amyloid proteins and are deposited on the outside of neurons. The diagnosis of Alzheimer's disease requires the presence of memory loss and at least one other symptom such as aphasia, apraxia, or agnosia.

Which of the following agents is associated with nephrolithiasis? I. Enalapril II. Radiocontrast dye III. Indinavir A.) I only B.) III only C.) I and II only D.) II and III only E.) I, II, and III

B.) III only Indinavir is known to cause crystalluria and nephrolithiasis. Enalapril is most likely to cause prerenal AKI, and radiocontrast dye can lead to prerenal and intrinsic kidney disease.

Which of the following medications can cause or worsen heart failure? I. Amiodarone II. Amlodipine III. Rosiglitazone A.) I only B.) III only C.) I and II only D.) II and III only E.) I, II, and III

B.) III only Rosiglitazone is associated with worsening fluid retention in patients with heart failure.

Which of the following antiarrhythmic agents' mechanism of action is primarily the result of potassium ion transport blockade? A.) Propafenone B.) Ibutilide C.) Lidocaine D.) Verapamil E.) Esmolol

B.) Ibutilide Ibutilide blocks potassium transport during phase 3 of the cardiac action potential.

Which type of immunity involves stimulation of cells and soluble mediators that nonspecifically recognize alloantigens with no altered response on repeat exposure? A.) Autoimmunity B.) Innate immunity C.) Adaptive immunity D.) Acute rejection E.) Hyperacute rejection

B.) Innate immunity Innate immunity is the fundamental type of immunity in which antigens are recognized in a nonspecific manner. This type of immunity is not augmented on repeat exposure.

A 36-year-old man with HIV was recently diagnosed with latent tuberculosis infection. He is taking Triumeq for treatment of his HIV with good adherence. He weighs 75 kg and has good kidney and liver function. What treatment (if any) would you recommend given his new diagnosis with latent tuberculos A.) Isoniazid 300 mg PO daily for 6 months B.) Isoniazid 300 mg PO daily for 9 months C.) Isoniazid 375 mg PO daily for 9 months D.) Isoniazid 900 mg PO daily and rifapentine 900 mg PO once weekly for 3 months E.) No treatment is necessary at this time since the patient is asymptomatic

B.) Isoniazid 300 mg PO daily for 9 months Correct answer: B Although the patient is asymptomatic, treatment of latent tuberculosis infection reduces the patient's risk of developing active tuberculosis. This is particularly important in a patient with HIV. Isoniazid is the preferred drug for treatment of latent tuberculosis infection (5-10 mg/kg, maximum dose of 300 mg daily) and should be continued for 9 months for patients with HIV. Answer A:Patients with HIV should receive isoniazid for 9 months for treatment of latent tuberculosis infection. Answer C:The maximum daily dose of isoniazid for treatment of latent tuberculosis infection with a daily regimen is 300 mg. Answer D:Although Isoniazid 900 mg PO daily and rifapentine 900 mg PO once weekly for 3 months is a viable regimen for some patients, it is not recommended for patients less than 2 years-old, HIV/AIDS patients taking antiretroviral treatment, isoniazid or rifampin resistant strains, pregnant women, or women expecting to become pregnant within the 12-week regimen. Answer E:Since the patient has HIV, he is particularly at risk for developing active tuberculosis disease and should be treated for this latent infection. Bottom Line:Treatment of latent tuberculosis infection is recommended in patients with HIV. The preferred drug of choice is isoniazid (5-10 mg/kg, maximum 300 mg/day) for 9 months.

Urge urinary incontinence secondary to detrusor hyperactivity is most effectively treated with which of the following medications? A.) Avodart B.) Sanctura C.) Tamsulosin D.) Midodrine E.) Proscar

B.) Sanctura Sanctura (trospium) is an anticholinergic that decreases acetylcholine-mediated contractions of the detrusor muscle. This improves bladder control in urge incontinence. Avodart (dutasteride) and Proscar (finasteride) are used to treat overflow incontinence by reducing prostate enlargement. Tamsulosin, an α-adrenergic antagonist, would decrease bladder control in urge incontinence. Midodrine, an α-adrenergic agonist, is used for stress incontinence and is less beneficial for urge incontinence than the use of an anticholinergic

Ampicillin is added to empiric meningitis regimens because of which pathogen? A.) Haemophilus influenzae B.) Neisseria meningitidis C.) Streptococcus pneumoniae D.) Gram-negative enterics (E. coli) E.) L. monocytogenes

E.) L. monocytogenes EXPLANATION: Ampicillin is added to cover Listeria monocytogenes because it is not susceptible to cephalosporins.

Which of the following is true about Sertaline? (Select all that apply) A.) It is contraindicated in patients with a history of seizures B.) It is the preferred antidepressant in people with cardiovascular disease C.) The drug has a boxed warning for increased suicidal risk in young adults D.) It inhibits CYP2D6 E.) This medication does not provide immediate relief and can take at least 4-6 weeks to feel the full effect

B.) It is the preferred antidepressant in people with cardiovascular disease C.) The drug has a boxed warning for increased suicidal risk in young adults E.) This medication does not provide immediate relief and can take at least 4-6 weeks to feel the full effect Sertraline (Zoloft) is a Selective Serotonin Reuptake Inhibitor (SSRI). All SSRIs have a boxed warning for an increased risk for suicidal thinking and behavior in children, adolescents, and young adults. All antidepressants may take 1 to 2 weeks to feel any benefit and 6 to 8 weeks to feel the full effect on mood. Most antidepressants prolong the QT interval and must be used with caution in patients with arrhythmia risk. Cardiovascular adverse events are usually dose-dependent with higher risks at higher doses. Answer A: Sertraline is not contraindicated in seizure history. An SSRI that should be avoided with a history of seizures is Vilazodone (Viibyrd). Answer D: Sertraline does not inhibit CYP2D6. SSRIs that inhibit CYP2D6 include Fluoxetine, Fluvoxamine and Paroxetine. Bottom Line: Antidepressants are best mastered by knowing the properties of an entire class of medications while keeping in mind what is unique to individual medications. Sertraline (Zoloft) is a Selective Serotonin Reuptake Inhibitor (SSRI), and all SSRIs have a boxed warning for an increased risk for suicidal thinking and behavior in children, adolescents, and young adults

Why is blinding of group assignments important for an RCT? A.) It ensures randomization of patients. B.) It minimizes the effect of participant expectations. C.) Blinding is not an important consideration. D.) It limits the follow-up time needed.

B.) It minimizes the effect of participant expectations.

P.R. is a 53 year old white female with bipolar I disorder. When counseling P.R. at the pharmacy window about her mood stabilizer she states that she has noticed a rash starting to develop on her midsection. She remembers her doctor telling her she needed to increase her dose slowly to avoid this rash. Which mood stabilizer is P.R. is most likely taking? A.) Valproic acid B.) Lamotrigine C.) Lithium D.) Oxcarbazepine E.) Quetiapine

B.) Lamotrigine Lamotrigine has a black box warning about severe and potentially life threatening skin rashes, including Stevens-Johnson syndrome and toxic epidermal necrosis. Patients need to follow a specific titration schedule recommended by the manufacturer (See also Lamctial® Starter Pak) to help prevent skin rash. Patients taking lamotrigine with valproic acid will need to reduce total daily dose and titrating dose of lamotrigine by one-half.

Close monitoring of adrenal hormone secretion may be required when administering which of the following? A.) Methyltestosterone B.) Mitotane C.) Desmopressin D.) Iodides E.) Propylthiouracil

B.) Mitotane Mitotane is cytotoxic to adrenal cells and thus reduces cortisol synthesis and release. ACTH increases cortisol release. Close monitoring of cortisol levels is important when mitotane is used.

Which of the following combinations would be recommended for a hypertensive emergency with a myocardial infarction? A.) Phentolamine and labetalol B.) Nitroglycerin and esmolol C.) Labetalol, nicardipine, and nitroprusside D.) Trimethaphan, esmolol, and nitroprusside E.) Nitroprusside, nitroglycerin, and enalaprilat

B.) Nitroglycerin and esmolol EXPLANATION: Nitroglycerin and esmolol are recommended for use in myocardial infarction hypertensive emergencies. Answer A would be used for pheochromocytoma or in a cocaine emergency. Answer C is used to treat encephalopathy. Answer D is recommended for dissecting aortic aneurysm. Answer E treats congestive heart failure.

Which of the following is not an advantage of using LMWH over heparin? A.) Subcutaneous administration B.) No dosage adjustment needed with renal insufficiency C.) Once- or twice-daily dosing D.) Predictable response at lower doses E.) Lower incidence of heparin-induced thrombocytopenia

B.) No dosage adjustment needed with renal insufficiency Because LMWHs are eliminated renally, their doses must be adjusted for patients with renal impairment (creatinine clearance < 30 mL/min). Guidelines recently have been released for enoxaparin dosing in patients with renal impairment. For DVT prophylaxis, enoxaparin should be administered 30 mg subcutaneous q24h rather than q12h. For DVT treatment, enoxaparin should be administered 1 mg/kg subcutaneous q24h rather than q12h.

Which of the following drugs is first-line choice in septic shock? A.) Dopamine B.) Norepinephrine C.) Epinephrine D.) Vasopressin E.) Dobutamine

B.) Norepinephrine

Per protocol analysis includes data from which patients? A.) All randomized patients. B.) Only patients who complete a specified treatment protocol. C.) Patients who were excluded from trial participation. D.) Patients who crossover to an alternative treatment in a parallel design trial.

B.) Only patients who complete a specified treatment protocol. Per protocol analysis includes data only patients who complete a specified treatment protocol. However, all other patients should be accounted for at the end of the study.

Regarding the biologic DMARDs, which of the following statements is correct? A.) Kineret is unique in that it is not immunosuppressive. B.) Patients should have a tuberculin skin test completed before initiation. C.) FluMist is acceptable to use for influenza prevention. D.) Patients receiving therapy are at increased risk for hepatitis.

B.) Patients should have a tuberculin skin test completed before initiation. Do a PPD (purified protein derivative) before initiation of biologic drugs because of their immune-suppressing properties, which could cause the reactivation of a disease such as tuberculosis.

Which of the following β-blockers has ISA activity? A.) Tenormin B.) Sectral C.) Inderal D.) Lopressor E.) Coreg

B.) Sectral β-blockers with ISA activity include Sectral (acebutolol) (Answer B) and Visken (pindolol). Tenormin (atenolol) (Answer A), Inderal (propranolol) (Answer C), Lopressor (metoprolol) (Answer D), and Coreg (carvedilol) (Answer E) do not have ISA activity.

A 66-year-old male with a past medical history of heart failure with reduced ejection fraction and hypertension presents to the emergency room with a 1-week history of intermittent palpitations and dizziness. The ECG reveals atrial fibrillation with a ventricular rate of 130 bpm. The decision is made to attempt to restore normal sinus rhythm. Which of the following represents the best therapeutic approach to cardioverting the patient? A.) Perform TEE; if no thrombus is present, cardiovert; anticoagulation is not necessary. B.) Perform TEE; if no thrombus is present, cardiovert; anticoagulate for at least 4 weeks postcardioversion. C.) Anticoagulate for 4 weeks before cardioversion; discontinue anticoagulation postcardioversion. D.) Anticoagulate for 2 weeks before cardioversion; continue anticoagulation for at least 4 weeks postcardioversion. E.) Direct-current cardiovert immediately.

B.) Perform TEE; if no thrombus is present, cardiovert; anticoagulate for at least 4 weeks postcardioversion. Because the patient appears to have been in atrial fibrillation for 1 week by history, the risk of thromboembolism during conversion to sinus rhythm is significant. Proper treatment would require at least 3-4 weeks of anticoagulation (warfarin INR 2-3 or a DOAC) before cardioversion, followed by at least 4 weeks of anticoagulation postcardioversion. Alternatively, a TEE can be used to rule out an atrial thrombus, allowing immediate cardioversion. Because the atria will require time to recover normal contractile activity, anticoagulation will be required for at least 4 weeks postconversion.

J.S. is a 66-year-old male with a past medical history of congestive heart failure and hypertension for which he is receiving lisinopril 10 mg po qd, digoxin 0.25 mg po qd, carvedilol 25 mg bid, and spironolactone 25 mg po qd at home. J.S. now presents to the emergency room with a 1-week history of intermittent palpitations and dizziness. A stat EKG reveals atrial fibrillation with a ventricular rate of 130 bpm. The decision is made to attempt to restore normal sinus rhythm. Which of the following represents the best therapeutic approach to cardioverting this patient? A.) Perform transesophageal echocardiogram; if no thrombus is present, cardiovert; there is no need for anticoagulation. B.) Perform transesophageal echocardiogram; if no thrombus present, cardiovert after anticoagulation achieved. Anticoagulate for at least 4 weeks post-cardioversion. C.) Anticoagulate for 4 weeks prior to cardioversion. Discontinue anticoagulation post-cardioversion. D.) Anticoagulate for 2 weeks prior to cardioversion. Continue anticoagulation for at least 4 weeks post-cardioversion. E.) Direct current cardiovert immediately.

B.) Perform transesophageal echocardiogram; if no thrombus present, cardiovert after anticoagulation achieved. Anticoagulate for at least 4 weeks post-cardioversion. Since the patient appears to have been in atrial fibrillation for 1 week by history, there is a significant risk of thromboembolism during conversion to sinus rhythm. Proper treatment would require at least 3-4 weeks of anticoagulation (warfarin INR 2-3, dabigatran, rivaroxaban, or apixaban) prior to cardioversion, followed by at least 4 weeks of anticoagulation post-cardioversion. Alternatively, a transesophageal echocardiogram can be used to rule out an atrial thrombus, allowing immediate cardioversion provided anticoagulation is achieved prior to echocardiogram. Since the atria will require time to recover normal contractile activity, anticoagulation will be required for at least 4 weeks post-conversion.

A 53-year-old male has a past medical history of MI and hypertension. He presents to his physician complaining of short (about 10 seconds in duration), intermittent palpitations during the past 2 days. Tests rule out an acute MI, and an echocardiogram shows a left ventricular ejection fraction of 25%. The patient is sent home with a Holter monitor to identify any arrhythmias. The Holter monitor reveals episodes of PVCs and nonsustained ventricular tachycardia. What is the most appropriate intervention for this patient? A.) No therapy is indicated. B.) Place an implantable cardioverter defibrillator. C.) Start propafenone. D.) Start verapamil. E.) Use direct-current cardioversion.

B.) Place an implantable cardioverter defibrillator. Patients with LVEF ≤ 30-40% and previous MI are at increased risk of sudden cardiac death, usually from ventricular fibrillation. Implantation of an ICD in these patients reduces the risk of mortality. Propafenone has negative inotropic effects and may worsen heart failure. The risk of ventricular proarrhythmia is significantly increased if propafenone is used in this patient. Verapamil also would worsen heart failure and not reduce the risk of mortality. Direct-current cardioversion is not indicated because the patient is stable. All standard heart failure medications should be optimized.

Which of the following anticonvulsants is metabolized to phenobarbital? A.) Ethosuximide B.) Primidone C.) Zonisamide D.) Levetiracetam E.) Carbamazepine

B.) Primidone Primidone (Mysoline) is an active anticonvulsant, but it is also metabolized to phenobarbital.

A factor that can decrease radioactive iodine uptake is: A.) Pregnancy B.) Prior radiation to the neck C.) Recovery from subacute thyroiditis D.) Lithium therapy

B.) Prior radiation to the neck Reason: Radiation potentially destroys the mechanism of action for the uptake of Iodine and all other options can increase uptake.

Mr. Steffer presented transferred to the hospital from an outside hospital in septic shock secondary to influenza. He is intubated and sedated on a fentanyl and propofol drip. Nutrition is consulted to determine a tube feeds regimen for this patient. Which of the following medications can alter a patient's caloric needs as it provides calories to the patient? A.) Fentanyl Drip B.) Propofol Drip C.) Norepinephrine Drip D.) Vasopressin drip

B.) Propofol Drip Correct answer: B When enteral nutrition (EN) is being started on a patient, it is important to examine their caloric needs, in addition to any caloric intake they may have. Propofolis a sedative agent that is suspended in a lipophilic solvent. It provides 1.1 kcal/ml to the patient. This is very important to take into consideration when determining the patient's tube feeds regimen. EN provides nutrients to patients via direct access to the stomach. Access to the stomach is obtained via an orogastric, nasogastric, esophagostomy/pharyngostomy, gastrostomy, or a percutaneous endoscopic gastrostomy (PEG) tube. Access to the gastrointestinal tract is obtained via nasoduodenal, nasojejunal, percutaneous endoscopic jejunostomy (PEJ), or a jejunostomy tube. There are several contraindications to EN, some of which include: obstruction of the GI tract absence of GI functionality GI ischemia inability to access the stomach or GI tract due to physical deformity/trauma unresolvable coagulopathies hemodynamic instability ascites high GI fistula output respiratory compromise Answer A:Fentanyl drips provide pain and sedation relief for patients in the ICU. They do not alter the caloric needs of patients. Answer C:Norepinephrine is a vasoactive agent that is used to increase a patient's blood pressure. Although caution should be given to patients on vasopressors, they do not alter the patient's caloric needs. Answer D:Vasopressin is a vasoactive agent that is used to increase a patient's blood pressure. Although caution should be given to patients on vasopressors, they do not alter the patient's caloric needs. Bottom Line:Propofol provides a patient 1.1 kcal/ml, which must be accounted for when calculating caloric needs.

Which of the following would be most important to monitor for a patient taking high doses of meperidine? A.) Weight for edema B.) Respiratory rate for Respiratory depression C.) Blood pressure for hypertension D.) Bowel movements for diarrhea E.) Blood glucose for hypoglycemia

B.) Respiratory rate for Respiratory depression Correct answer: B Meperidine is a narcotic analgesic. As with all narcotics, the main risk of use is respiratory depression. Therefore, it is important to monitor respiratory rate for patients on this medication.The most frequently observed adverse reactions include lightheadedness, dizziness, sedation, nausea, vomiting, and sweating. If patients report these side effects, their doses of meperidine should be lowered.

Which of the following immunosuppressants should not be administered at the same time secondary to an interaction related to timing of doses? A.) Tacrolimus and azathioprine B.) Sirolimus and cyclosporine C.)Cyclosporine and azathioprine D. ) Sirolimus and tacrolimus E.) Sirolimus and mycophenolate mofetil

B.) Sirolimus and cyclosporine Simultaneous administration of sirolimus and cyclosporine increases C max and the area under the curve of sirolimus by 120-500% and 140-230%, respectively. Administration 4 hours apart increases C max and the area under the curve of sirolimus by 30-40% and 35-80%, respectively.

A 7-month-old formula-fed female is brought to your pharmacy by her mother, who describes the infant as having new onset of fever (102.5°F) and increased irritability in the past 24 hours. The mother states that she stayed home with the infant today instead of sending her to day care. Family history is significant for an older sibling with a recent upper respiratory tract infection. Examination of the infant's ear canal using a pneumatic otoscope reveals a bulging, red tympanic membrane with no mobility on negative or positive pressure. Computer records reveal that she was treated for acute otitis media at 3 months of age. Decisions for antimicrobial therapy in this patient should be based on coverage for which of the following pathogens? A.) Staphylococcus epidermidis, Streptococcus pneumoniae, and Pseudomonas aeruginosa B.) Streptococcus pneumoniae, Haemophilus influenzae, and Moraxella catarrhalis C.) Haemophilus influenzae, Streptococcus pyogenes, and Pseudomonas aeruginosa D.) Streptococcus pneumoniae, Staphylococcus aureus, and Moraxella catarrhalis E.) Staphylococcus epidermidis, Pseudomonas aeruginosa, and Burkholderia cepacia

B.) Streptococcus pneumoniae, Haemophilus influenzae, and Moraxella catarrhalis The most common bacterial pathogens in acute otitis media are Streptococcus pneumoniae, Haemophilus influenzae, and Moraxella catarrhalis.

Which of the following agents being taken by the patient requires the premedication regimen of dexamethasone, diphenhydramine, and ranitidine to prevent an anaphylactic reaction? A.) Taxotere B.) Taxol C.) Paraplatin D.) Cisplatin

B.) Taxol Taxol is the brand name of paclitaxel. This agent has been shown to cause hypersensitivity reactions in patients. It is unclear if these reactions are due to the drug itself or the drug's vehicle (Cremophor). All patients receiving paclitaxel should receive a premedication regimen of dexamethasone, diphenhydramine, and ranitidine.

Which of the following statements about assessment of kidney function is correct? A.) The MDRD equation is accurate at GFRs above 60 mL/min/1.73 m2 B.) The Cockcroft Gault equation estimates creatinine clearance; the MDRD equation estimates GFR C.) The MDRD equation may be used in patients with unstable kidney function D.) The Cockcroft Gault equation may be used in patients with unstable kidney function E.) The MDRD equation includes an adjustment factor for African Americans, obesity, and females

B.) The Cockcroft Gault equation estimates creatinine clearance; the MDRD equation estimates GFR The Cockcroft-Gault equation provides an estimate of creatinine clearance while the MDRD equation provides an estimate of GFR since it was developed using a gold standard method to measure glomerular filtration. Both equations are to be used in patients with stable kidney function, not unstable kidney function as observed in a patient with AKI. The MDRD equation is less accurate at GFRs above 60 mL/min/1.73 m2 and an absolute value is not reported for individuals with GFRs above this value. The MDRD equation also has adjustment factors for African-Americans and females, not obese patients.

Choose the correct statement. The plenum in a laminar flow workbench is: A.) Where the air is prefiltered. B.) The area where air is pressurized for distribution over the HEPA filter. C.) The area where compounding takes place. D.) An area that serves no purpose. E.) An area directly above the HEPA filter in a horizontal laminar flow hood.

B.) The area where air is pressurized for distribution over the HEPA filter. The plenum is the area behind the HEPA filter in an HFLW, which allows air to be pressurized for even distribution over the filter.

Which of the following statements about lyophilic colloidal dispersions is true? A.) They tend to be more sensitive to the addition of electrolytes than do lyophobic systems. B.) They tend to be more viscous than lyophobic systems. C.) They can be precipitated by prolonged dialysis. D.) They separate rapidly.

B.) They tend to be more viscous than lyophobic systems. Most lyophilic colloids are organic molecules (including gelatin and acacia); they spontaneously form colloidal solutions and tend to be viscous. Dispersion of lyophilic colloids is stable in the presence of electrolytes.

Which of the following statements about lyophilic colloidal dispersions is true? A.) They tend to be more sensitive to the addition of electrolytes than do lyophobic systems. B.) They tend to be more viscous than lyophobic systems. C.) They can be precipitated by prolonged dialysis. D.) They separate rapidly. E.) All of the above.

B.) They tend to be more viscous than lyophobic systems. Most lyophilic colloids are organic molecules (including gelatin and acacia); they spontaneously form colloidal solutions and tend to be viscous. Dispersion of lyophilic colloids is stable in the presence of electrolytes.

Which beta-blocker is preferred in a patient with intermittent asthma who has heart failure? A.) Coreg B.) Toprol XL C.) Propranolol D.) Atenolol E.) Labetelol

B.) Toprol XL Toprol XL (metoprolol succinate) is cardioselective and approved for use in heart failure. Atenolol (Tenormin) is cardioselective but not approved for use in heart failure.

Which of the following product dosing regimens is correct? A.) Climara Transdermal: Apply to skin once daily. B.) Vagifem: 1 tablet vaginally once daily for 2 weeks, then 1 tablet vaginally twice weekly. C.) Estring: Insert ring intravaginally once daily at bedtime. D.) Premarin tablets: 0.625-2.5 mg tid. E.) Premarin vaginal cream: 20-40 g vaginally once daily.

B.) Vagifem: 1 tablet vaginally once daily for 2 weeks, then 1 tablet vaginally twice weekly. Vagifem dosage is 1 tablet vaginally once daily for 2 weeks; then 1 tablet vaginally twice weekly.

Which of the following is true regarding anticonvulsants and their effect on weight? A.) Valproic acid and phenytoin both decrease weight. B.) Valproic acid increases weight and topiramate decreases weight. C.) Topiramate and phenytoin both increase weight. D.) Topiramate, valproic acid, and phenytoin cause no change in weight. E.) Phenytoin is the only anticonvulsant known to increase weight.

B.) Valproic acid increases weight and topiramate decreases weight. Topiramate can cause significant weight loss, and valproate can cause significant weight gain. Phenytoin does not significantly affect weight.

Which of the following drugs is an oral pro-drug of 5-FU? A.) Fluorouracil B.) Xeloda C.) Fludara D.) Cytoxan

B.) Xeloda Xeloda (capecitabine) is an oral pro-drug of 5-FU. Fluorouracil is another name for 5-FU. Fludarabine (Fludara) is used to treat chronic lymphocytic leukemia and non-Hodgkin lymphoma intravenously. Cyclophosphamide (Cytoxan) is available in IV and po dosage forms.

In the GI tract, calcitriol promotes A.) absorption of calcium and inhibits absorption of phosphorus. B.) absorption of both calcium and phosphorus. C.) decreased binding of calcium and phosphorus. D.) increased elimination of calcium and phosphorus.

B.) absorption of both calcium and phosphorus. Active vitamin D (calcitriol) promotes absorption of both calcium and phosphorus in the GI tract. For this reason, therapy with calcitriol or a vitamin D analog may need to be withheld if the calcium or phosphorus is elevated.

The mechanism of action for carbonic anhydrase inhibitors is A.) increasing aqueous humor outflow. B.) decreasing aqueous humor formation. C.) increasing uveoscleral outflow effect on aqueous humor formation. D.) improving aqueous humor outflow by vasodilation. E.) pupil constriction with resulting increased aqueous humor outflow.

B.) decreasing aqueous humor formation. EXPLANATION: Carbonic anhydrase inhibitors decrease aqueous humor formation with little effect on aqueous humor outflow.

H. F. is a 72-year-old male with a history of hypertension, glaucoma, and hyperlipidemia. He was recently diagnosed with Parkinson's disease and began medication. After 4 weeks of treatment, he continues to have difficulty with walking and writing and has tremors in both hands. He currently takes the following medications: Hydrochlorothiazide 25 mg daily Lisinopril 10 mg daily Aspirin 81 mg daily Simvastatin 20 mg at bedtime Senna tablets, 1 daily for constipation Carbidopa-levodopa 10/100, 1 tablet three times daily The best recommendation to improve his treatment response would be to A.) consider decreasing antihypertensive therapy and to evaluate if symptoms improve. B.) gradually increase the dose of carbidopa-levodopa. C.) switch from carbidopa-levodopa to entacapone. D.) add a monoamine oxidase inhibitor, such as selegiline. E.) add a sustained-release dopamine agent at bedtime.

B.) gradually increase the dose of carbidopa-levodopa. The current dose of carbidopa-levodopa is an initial dosage and should be increased to improve response. The full benefit of carbidopa requires daily divided doses of 100 mg or more. Entacapone is never given as monotherapy, and the addition of other medications is more appropriate after a trial of carbidopa-levodopa at therapeutic doses.

The plenum in a laminar flow workbench is the area A.) where the air is prefiltered. B.) where air is pressurized for distribution over the HEPA filter. C.) where compounding takes place. D.) that serves no purpose. E.) directly above the HEPA filter in a horizontal laminar flow hood.

B.) where air is pressurized for distribution over the HEPA filter. The plenum is the area behind the HEPA filter in an HLFW that allows air to be pressurized for even distribution over the filter.

When the 13-day-old former 35-week gestational age baby is examined at the pediatric hospital, she is also noted to have some lesions. The team has just sent cultures of the lesions as well as HSV PCR of the CSF. Which of the following is an appropriate pharmacologic approach in this patient? A) Wait for the cultures and PCR results to come back, then modify therapy if needed. B) Change antibiotic therapy to ceftriaxone and vancomycin. C) Add IV acyclovir to the current antibiotic regimen. D) Add oral voriconazole therapy to the current antibiotic regimen.

C) Add IV acyclovir to the current antibiotic regimen. Answer c is correct. IV acyclovir should be added as soon as possible as empiric therapy for HSV encephalitis while awaiting the results of diagnostic studies. The earlier the treatment is started, the lower the risk of death or serious sequelae. Answer a is incorrect. Culture and PCR results may take hours to days to return. Such a delay in therapy against HSV would increase the risk of death or serious sequelae in an infected patient. Answer b is incorrect. This change in antibiotics to ceftriaxone and vancomycin would offer no antiviral coverage against HSV. Also, ceftriaxone and vancomycin would be inappropriate empiric therapy for bacterial meningitis in a neonate (see Question 2). Answer d is incorrect. This addition of antifungal therapy with voriconazole would offer no antiviral coverage against HSV. In addition, antimicrobial therapy for CNS infections is generally given the IV route (vs oral) in order to ensure adequate CNS penetration.

Androstenedione (Andro) is a dietary supplement touted to act as an anabolic agent. The purported basic science behind this theory is best described by which of the following? A) Androstenedione blocks the breakdown of endogenous testosterone. B) Androstenedione is a natural anabolic steroidal compound that acts at the androgenic receptors with a higher affinity than testosterone. C) Androstenedione is a precursor in the cascade of steps that produces testosterone. D) Androstenedione has no anabolic potential.

C) Androstenedione is a precursor in the cascade of steps that produces testosterone. Androstenedione is the precursor to testosterone. In theory, providing more substrate precursor should lead to higher concentrations of end product, such as testosterone. This has been shown to be the case with doses of androstenedione >300 mg daily. For this reason, it is a prohibited substance in most athletic competition. Choice (A) is incorrect. Androstenedione is a precursor in the steps forming testosterone and does not inhibit the metabolism of testosterone Choice (B) is incorrect. Androstenedione is not known to act at androgenic receptors. Choice (D) is incorrect. In theory, given a high-enough dose, androstenedione could increase testosterone levels and, indirectly, improve anabolism.

HB is a 54-year-old African American man who presents to the emergency department with symptoms of left-sided paralysis and visual impairment. He has a PMH significant for hypertension, dyslipidemia, and benign prostatic hyperplasia. MRI of the brain confirms the patient has had an ischemic stroke. Which of the following medications would be the most appropriate for secondary stroke prevention in HB? A) Ticlopidine B) Dipyridamole C) Aspirin D) Clopidogrel plus aspirin

C) Aspirin Answer c is correct. Aspirin is the most well-studied antiplatelet agent used in the secondary prevention of stroke. Currently, aspirin, clopidogrel, and extended-release dipyridamole plus aspirin are all appropriate antiplatelet options for initial therapy. Answer a is incorrect. Ticlopidine is not currently recommended by the American Heart Association Stroke Council due to its significant side-effect profile. It can cause severe gastrointestinal side effects and there have been reports of neutropenia, agranulocytosis, aplastic anemia, and thrombotic thrombocytopenic purpura. Answer b is incorrect. Dipyridamole should not be used as monotherapy in the secondary prevention of ischemic stroke. The combination of extended-release dipyridamole plus aspirin is an acceptable antiplatelet option for initial therapy. Answer d is incorrect. The American Heart Association Stroke Council currently does not recommend the combination of clopidogrel with aspirin as there is an increased risk of hemorrhage when used together.

A patient with ATN with anuria has a serum potassium concentration of 6.8 mEq/L with associated electrocardiogram changes of peaked T waves. Which intervention should be initiated first? A) Regular insulin 10 units and 25 g of dextrose 50% IV push over 2 to 5 minutes B) Sodium bicarbonate 8.4% 50 mEq IV push over 2 to 5 minutes C) Calcium gluconate 1 g IV push over 2 to 5 minutes D) Sodium polystyrene sulfonate 15 g po

C) Calcium gluconate 1 g IV push over 2 to 5 minutes Answer c is correct. This patient has signs of cardiotoxicity secondary to hyperkalemia. The most important first step is to antagonize the effect of potassium on the myocardial cells. One gram of calcium (either chloride or gluconate) should be given immediately if any EKG abnormalities are noted. Answer a is incorrect.Insulin stimulates the cellular uptake of potassium, decreasing the extracellular concentration. This is an appropriate step to manage hyperkalemia, though not the first step in this patient. Answer b is incorrect. Administering sodium bicarbonate causes the efflux of H+ from within in the cell in exchange for K+. While this might be an appropriate strategy to manage hyperkalemia (especially if the patient had a metabolic acidosis), it is not the first step in this patient. Answer d is incorrect.Sodium polystyrene sulfonate is an appropriate adjunctive agent to promote elimination of potassium from the body in the feces. It acts to exchange sodium for potassium ions in the gastrointestinal tract. It does not work immediately and would not be the first step.

Select the beta-lactam antimicrobial that does not require dose or interval reductions in patients with renal dysfunction. A) Amoxicillin B) Azithromycin C) Ceftriaxone D) Moxifloxacin E) Cefepime

C) Ceftriaxone The beta-lactam ceftriaxone undergoes dual elimination. Even though ceftriaxone has renal elimination, it is not required to have dose/interval reductions in patients with renal dysfunction. Answer (A) is incorrect. The beta-lactam amoxicillin and other penicillin antimicrobials used for the treatment of pneumonia are required to have dose and/or interval modifications in patients with renal dysfunction. Answer (B) is incorrect. While azithromycin does not require dose or interval reductions in patients with renal dysfunction (azithromycin undergoes hepatic metabolism/elimination), it is not a beta-lactam microbial. Azithromycin is an azalide antimicrobial; however, clinicians often refer to azithromycin as a macrolide. Answer (D) is incorrect. While moxifloxacin does not require dose or interval reductions in patients with renal dysfunction (moxifloxacin undergoes hepatic metabolism and renal and feces elimination), it is not a beta-lactam antimicrobial. Moxifloxacin is a fluoroquinolone antimicrobial. All other fluoroquinolones (levofloxacin, ciprofloxacin, gemifloxacin) require dose or interval modifications in patients with renal dysfunction. Answer (E) is incorrect. The fourth-generation cephalosporin cefepime is a beta-lactam antimicrobial and requires dose or interval reductions in patients with renal dysfunction.

PY is a 62-year-old woman diagnosed with stage IV colon cancer with a primary tumor in the sigmoid colon that has spread to the liver. The patient has undergone surgery 2 weeks ago to remove the solitary tumor in the liver and is ready to begin drug therapy including combination chemotherapy (FOLFOX) and a monoclonal antibody. The patient tested negative for KRAS, NRAS, BRAF, MSI, and MMR. What is the most appropriate monoclonal antibody for PY to receive with FOLFOX? A) Bevacizumab B) Nivolumab C) Cetuximab D) Ramucirumab

C) Cetuximab Answer c is correct. The patient has a tumor on the left side of the colon (sigmoid colon) and no mutations that confer resistance to EGFR antibodies (negative for KRAS/NRAS/BRAF) and is likely to respond to cetuximab or panitumumab. Answer a is incorrect. Bevacizumab is a vascular endothelial growth factor (VEGF) inhibitor and can prolong wound healing times. It is recommended that patients wait at least 4 weeks after a surgical procedure to receive bevacizumab. If there was not another viable option, chemotherapy could be started then bevacizumab added later. However, in this case cetuximab is an acceptable alternative to bevacizumab. Answer b is incorrect. The PD-1 inhibitors, nivolumab and pembrolizumab, are only indicated in MSI-H or dMMR positive tumors that have failed multiple therapies. PY does not meet these criteria. Answer d is incorrect. Like bevacizumab, ramucirumab is a VEGF inhibitor and should not be used soon after surgery. Additionally, it is only indicated as a second-line agent in combination with FOLFIRI. This patient is receiving initial chemotherapy with FOLFOX.

LC is a 76-year-old woman who presents to ED via EMS to a large academic medical center (with a coronary catheterization laboratory) complaining of sudden onset of diaphoresis and nausea. She states, "About 5 hours ago my chest started hurting and I just don't feel well." LC's weight is 65 kg. Past medical history: CAD and arthritis Family history: Father died of acute myocardial infarction at 76 years of age and mother passed away at age 70 from pneumonia Social history: Does not drink alcohol; smokes 1 pack of cigarettes per week Medications: ASA 81 mg orally once daily, atorvastatin 40 mg orally at bedtime, conjugated estrogens 0.625 mg orally daily, and celecoxib 200 mg orally daily Laboratory data: Serum creatinine (SCr) = 1.9 mg/dL, total cholesterol 250 mg/dL, triglycerides 150 mg/dL, high-density lipoprotein (HDL) 40 mg/dL, LDL 130 mg/dL, troponin I = 5.7 ng/mL Electrocardiogram: ST-segment elevation Which one of the following is the preferred approach to reperfuse this patient? A) Chew ASA 81 mg, clopidogrel 75 mg, UFH for 48 hours B) Reteplase 10 units IV for two doses 30 minutes apart and UFH for 48 hours C) Chew ASA 325 mg, administer ticagrelor 180 mg orally once, abciximab 16.25 mcg IV bolus and percutaneous intervention with coronary stent placement D) Streptokinase 1,500,000 units IV over 30 minutes, ASA 81 mg, clopidogrel 300 mg

C) Chew ASA 325 mg, administer ticagrelor 180 mg orally once, abciximab 16.25 mcg IV bolus and percutaneous intervention with coronary stent placement Answer c is correct. LC is experiencing a STEMI that requires emergent reperfusion and has presented to a hospital with cardiac catheterization capabilities. The patient should be loaded (higher doses) with antiplatelets, and started on abciximab since he will be going to catheterization emergently. Answer a is incorrect. LC is experiencing a STEMI and the goal for these patients is always primarily reperfusion. ASA, clopidogrel, and UFH will prevent further platelet aggregation and thrombus formation but they do not dissolve current clots. Answer b is incorrect. A regimen including reteplase is a possibility for LC's treatment since reperfusion can occur either with fibrinolytic or PCI. However, if the facility has PCI capabilities, the patient should receive PCI as it is more effective than thrombolysis. Answer d is incorrect. This is a possibility for treatment since this patient is a STEMI and will need reperfusion either with fibrinolytic or PTCA. However, streptokinase is not a common or highly recommended fibrinolytic due to its lack of specificity and capability of allergic reactions. Newer second generation fibrinolytics are preferred. Also, if the facility has PCI capabilities, the patient should receive PCI as it is more effective than thrombolysis.

NK is a 62-year-old man presenting to urgent care today with dysuria, increased urinating frequency, and flank pain. His past medical history includes hyperlipidemia and migraines. He is allergic to penicillin and sulfa drugs. The patient has a high fever and severe nausea and vomiting (N&V). What is the most appropriate therapy for NK? A) Amoxicillin 500 mg po tid B) Ciprofloxacin 500 mg po bid C) Ciprofloxacin 400 mg IV twice daily D) TMP-SMX 1 double strength (DS) tablet po bid

C) Ciprofloxacin 400 mg IV twice daily Answer c is correct.Ciprofloxacin is the appropriate therapy because NK is allergic to penicillin and sulfa drugs. Answer a is incorrect. NK is allergic to penicillin so amoxicillin would be an inappropriate choice. Additionally, amoxicillin would not be utilized as empiric therapy. Answer b is incorrect. Oral therapy would most likely not be utilized at this time because of the severe N&V. Once the N&V have resided, a transition to oral therapy will be utilized. Answer d is incorrect. NK is allergic to sulfa drugs, and TMP-SMX includes a sulfa component. If the patient did not have an allergy, sulfa antibiotics can be utilized in uncomplicated pyelonephritis. An uncomplicated pyelonephritis would be a mild case where a patient could be treated as an outpatient with oral antibiotics.

Which of the following medications would be the best agent for migraine prophylaxis for a professional football player? A.) Propranolol B.) Topiramate C.) Divalproex D.) Amitriptyline E.) Paroxetine

C.) Divalproex EXPLANATION: Divalproex is the best agent for this patient. Propranolol is associated with fatigue and exercise intolerance. Topiramate is associated with increased risk of heat stroke.

MK is a 62-year-old woman with a recent diagnosis of stage II squamous NSCLC. Complete pathological evaluation showed ALK positive, EGFR negative, BRAF V600E negative. She has a prior medical history of hypertension, type 2 diabetes, stage 3b chronic kidney disease (last estimated glomerular filtration rate 35 mL/min), and gastroesophageal reflux disease. She underwent surgery and is ready to receive chemotherapy. Four years after completing chemotherapy, MK presents to her primary care physician with new onset coughing with hemoptysis. Complete evaluation revealed the patient's lung cancer has relapsed, now with multiple tumors in the original lung as well as several suspicious tumors in the liver. The pathologic evaluation reveals the same genetic features as her original tumor. Which of the following is the most appropriate treatment for MK at this time? A) Docetaxel and ramucirumab B) Atezolizumab C) Crizotinib D) Erlotinib

C) Crizotinib The patient is positive for ALK. Since her initial regimen was used in the adjuvant setting for the treatment of early stage lung cancer chemotherapy was initially used. Now, in the relapsed setting, targeted therapy is preferred when possible. Since the patient is positive for ALK, therapy targeting this kinase can be used. Crizotinib is the preferred initial anti-ALK drug. Answer a is incorrect. Since her tumor is ALK positive she should receive targeted therapy prior to receiving any further chemotherapy. Typically patients who are ALK positive will receive multiple ALK targeted drugs before receiving chemotherapy for relapsed disease. Answer b is incorrect. Since her tumor is ALK positive she should receive therapy targeted at this mutation prior to other targeted therapies like the PD-1/PDL-1 inhibitors. Typically patients who are ALK positive will receive multiple ALK targeted drugs before attempting other therapies for relapsed disease. Answer d is incorrect. This patient has ALK positive, EGFR negative lung cancer, targeted EGFR kinase inhibitors such as erlotinib would not be recommended until the patient has failed ALK-targeted medications.

YM is a 62-year-old woman with a recent diagnosis of stage III diffuse large B-cell lymphoma. Her disease is characterized by multiple areas of involvement including the spleen, pelvic lymph nodes, and mediastinal lymph nodes. She is scheduled to begin chemotherapy with the regimen EPOCH-R, which includes etoposide, prednisone, vincristine, cyclophosphamide, doxorubicin, and rituximab. Approximately 4 hours after the infusion of the first cycle of chemotherapy, YM begins experience severe nausea and has episodes of vomiting. Which of the medications in her treatment regimen is the most likely to be causing her nausea and vomiting? A) Etoposide B) Prednisone C) Cyclophosphamide D) Vincristine

C) Cyclophosphamide Answer c is correct. Cyclophosphamide is an alkylating agent and is considered to be a noncell cycle specific chemotherapy. Like all alkylating agents, CINV is a concern. At doses typically used in the treatment of cancer, cyclophosphamide is moderately emetogenic (risk of nausea/vomiting is 30%-90%). However, when you use high doses (greater than 1000 mg/m2) or in combination with an anthracycline such as doxorubicin, it is highly emetogenic (≥90% risk of nausea/vomiting) Answer a is incorrect. Etoposide is a topoisomerase II inhibitor and is considered to be cell cycle specific chemotherapy. It has a low emetogenic potential. Answer b is incorrect. Prednisone is a corticosteroid. The oral use of prednisone could be associated with some nausea and vomiting and taking it with food should be considered. Corticosteroids like prednisone, and especially dexamethasone, are frequently used to prevent chemotherapy-induced nausea/vomiting. Answer d is incorrect.Vincristine is a vinca alkaloid and is considered to be cell cycle specific. It is considered to be minimally emetogenic.

GM is a 58-year-old African American man with systolic heart failure presenting with a 10-day history of shortness of breath which limits his normal daily activities and increases lower extremity edema. His weight has recently increased by 12 lb. His physical examination is notable for BP 144/77 mm Hg, HR 85 bpm, RR 22 rpm, rales, and 4+ lower extremity edema. Pertinent laboratory values include: sodium 136 mmol/L, potassium 5.4 mmol/L, BUN 23 mg/dL, creatinine 1.1 mg/dL, and digoxin 1.9 ng/mL. Past medical history is significant for hypertension (HTN), gout, COPD, and atrial fibrillation. Current medications include lisinopril 20 mg daily, diltiazem CD 120 mg daily, digoxin 0.250 mg daily salmeterol/fluticasone 250/50, two puffs bid. GM recently began taking naproxen 220 mg tid for gout pain. Furosemide is initiated at 40 mg twice daily to manage fluid overload. Within the following 24 hours, GM experiences a brisk diuresis with improvement in heart failure signs and symptoms. The naproxen is discontinued to avoid exacerbating fluid retention, and colchine is initiated at 0.6 mg bid to manage acute gout flares. Once optimal fluid status has been achieved, which of the following represents the best option to manage GM's hypertension? A) Discontinue diltiazem and initiate amlodipine 5 mg daily. B) Initiate carvedilol 3.125 mg twice daily. C) Discontinue diltiazem and initiate carvedilol 3.125 mg twice daily. D) Initiate prazosin 2 mg daily

C) Discontinue diltiazem and initiate carvedilol 3.125 mg twice daily. In addition to initiating beta-blocker therapy, diltiazem should be discontinued due to its negative inotropic effect, which would likely cause worsening HF. Also, it would be rare to have a patient (in any disease state) on a beta blocker and non-dihydropyridine calcium channel blocker because of the additive negative inotropic and negative chronotropic pharmacologic properties. Answer (A) is incorrect. Amlodipine has not been shown to reduce mortality and would not be considered until other HF medications shown to reduce mortality have been initiated. Answer (B) is incorrect. This answer is partially correct, but not the best answer. In addition to assisting with managing GA's HTN, carvedilol is one of the three beta blockers shown to reduce mortality in HF patients. The long term benefits of inhibiting the sympathetic nervous system with beta-receptor blockade outweigh the short-term negative inotropic effects. Randomized controlled trials have demonstrated a significant mortality benefit when beta-blocker therapy is initiated in addition to ACE inhibitors. This benefit is present in patients of all HF severities. Not all beta blockers can be used in systolic heart failure (e.g. heart failure with low ejection fraction). Three beta blockers have been proven effective in heart failure and they are: carvedilol, metoprolol succinate, and bisoprolol. Beta blockers should be started at low doses and up-titrated slowly when patients are euvolemic. Even when started or up-titrated under optimal conditions, a patient's symptoms may transiently worsen. When these adjustments are made until a new equilibrium is established. Answer (D) is incorrect. Prazosin has been shown to not benefit HF patients. Prazosin would be used for patients with benign prostatic hypertrophy (BPH).

A 65-year-old patient presents with AF and HF. Which of the following is the best choice for pharmacologic cardioversion of AF in this patient? A) Flecainide B) Sotalol C) Dofetilide D) Dronedarone

C) Dofetilide Answer c is correct. Dofetilide, a Class III antiarrhythmic is a safe drug to use in patients with structural heart disease and LV dysfunction. Amiodarone is the other antiarrhythmic that is safe in this patient population. Answers a, b, and d are all incorrect. Flecainide, sotalol, and dronedarone are not safe for use in patients with structural heart disease.

Which of the following agents is available as an SQ injection? A) Abatacept B) Cytoxan C) Etanercept D) Remicade E) Rituxan

C) Etanercept Etanercept (C) is given as an SQ injection and is dosed weekly or semiweekly. Abatacept (A) is given as an IV infusion at weeks 0, 2, and 4 and then every 4 weeks thereafter. Cytoxan (B) is available orally and by IV injection. Remicade (D) is available as an IV infusion given at weeks 0, 2, and 6 and then every 8 weeks thereafter. Rituxan (E) is available as an injectable drug and is given by IV infusion. Topic: Rheumatoid Arthritis

Which of the following is an appropriate way to counsel patients on taking extended-release dipyridamole plus aspirin therapy? A) Extended-release dipyridamole 200 mg plus aspirin 25 mg po daily B) Extended-release dipyridamole 25 mg plus aspirin 200 mg po daily C) Extended-release dipyridamole 200 mg plus aspirin 25 mg po bid D) Extended-release dipyridamole 25 mg plus aspirin 200 mg po bid

C) Extended-release dipyridamole 200 mg plus aspirin 25 mg po bid Aggrenox is supplied as a capsule containing extended-release dipyridamole 200 mg plus aspirin 25 mg. This capsule is given twice daily.

What side effect is associated with Twinrix (hepatitis A/B vaccine)? A) Stevens-Johnson syndrome B) Neuroleptic syndrome C) Guillain-BarrÉ syndrome D) Red man syndrome

C) Guillain-BarrÉ syndrome Guillain-BarrÉ syndrome (C) is a rare side effect associated with Twinrix. Stevens-Johnson syndrome (A) is not a reported side effect of Twinrix. Neuroleptic syndrome (B) is not a reported side effect of Twinrix. Red man syndrome (D) is not a reported side effect of Twinrix.

Select the answer that is defined as a superficial skin infection caused by ?-hemolytic Streptococcus and S. aureus. Treatment consists of soaking the lesions with soap and water, use of skin emollients to dry areas and antimicrobials. Topical mupirocin may be used alone when there are few lesions. A) Erysipelas B) Lymphangitis C) Impetigo D) Necrotizing fasciitis

C) Impetigo Impetigo is a superficial skin infection caused by ?-hemolytic Streptococcus and S. aureus. The microorganisms colonize the skin and invade through abrasions, insect bites or other small traumas. Impetigo lesions are numerous, well-localized and erythematous. Antimicrobials with activity against group A Streptococcus (GAS) and the beta-lactamase producing (penicillinase) S. aureus are preferred and include first-generation cephalosporins and penicillinase-stable penicillins. Topical mupirocin may be used alone when there are few lesions. Answer (A) is incorrect. Erysipelas is an infection of the superficial layers of the skin and cutaneous lymphatics. The intense red color and burning pain led to the common name of St. Anthony's fire. The infection is most often caused by GAS. Mild to moderate cases of erysipelas are treated with intramuscular procaine penicillin G or oral penicillin VK. Answer (B) is incorrect. Lymphangitis is an inflammation of the lymphatic channel that ensues when a local skin infection is not contained, most often caused by GAS. Lymphangitis is characterized by red linear streaks that extend from the infection site toward regional lymph nodes. Because these infections are potentially serious and rapidly progressive, initial treatment should be with IV penicillin G 1 to 2 million units every 4 to 6 hours. Answer (D) is incorrect. Necrotizing fasciitis (NF) is an uncommon, rapidly progressive, life- threatening infection of the subcutaneous tissue and fascia. After resuscitation and hemodynamic stabilization, prompt surgical intervention is key in the treatment of NF. Broad-spectrum IV antimicrobial therapy should be initiated immediately in patients with NF.

DE is a 49-year-old woman with a diagnosis of breast cancer that is currently being treated with chemotherapy. Her prior medical history is significant for allergic rhinitis and chronic heartburn. Her home medications include cetirizine, pantoprazole, and transdermal norelgestromin/ethinyl estradiol. The patient was recently diagnosed with chemotherapy-induced anemia with a hemoglobin level of 8.8 g/dL. She was prescribed darbepoetin to treat her anemia. Six weeks after initiating darbepoetin, DEs hemoglobin level is 9.6 g/dL. Which of the following represents the most appropriate course of action for DEs darbepoetin? A) Continue darbepoetin at the same dose B) Decrease the dose of darbepoetin by 25% C) Increase the dose of darbepoetin by 50% D) Discontinue the darbepoetin

C) Increase the dose of darbepoetin by 50% Answer c is correct. This patient has had an inadequate increase in hemoglobin (<1 g/dL in 4-6 weeks) and a dose increase is warranted. The recommended dose increase in ESA nonresponders is 25%. Answer a is incorrect. The patient has not had an adequate response to his ESA therapy so a dose increase is indicated. Answer b is incorrect. The dose of ESAs is decreased in patients whose hemoglobin exceeds the goal level or who have too rapid of a rise in hemoglobin level. Answer d is incorrect. The patient has not had an adequate response in hemoglobin since initiating ESA therapy. Six weeks of therapy is not long enough

EQ is a 64-year-old male patient presenting to urgent care for pain in his great toe. The pain is severe and began suddenly. EQ is diagnosed with an acute gout attack. Past medical history includes hypertension, dyslipidemia, diabetes and HIV. Medications include lisinopril, pravastatin, metformin and highly active anti-retroviral therapy (emtricitabine, tenofovir, ritonavir, atazanavir). Labs are within normal limits. Family and social history are positive for a myocardial infarction in his father at age 49 who is also a social beer drinker. Select the medication(s) to treat EQ's acute gout attack. A) Allopurinol B) Colchicine C) Naproxen D) Febuxostat

C) Naproxen Naproxen is used in the management of acute gout and would be the agent of choice for EQ. Answer (A) is incorrect. Urate-lowering therapies are of no benefit for acute gout and should generally not be initiated during an acute attack. However, in patients already receiving these agents (e.g., allopurinol, febuxostat), the urate-lowering medication should be continued without interruption. Answer (B) is incorrect. Colchicine may be used for acute gout attacks; however, colchicine should be avoided with strong CPY 450 3A4 inhibitors (e.g. ritonavir and atazanavir) if possible and are contraindicated in the setting of renal impairment. This may lead to increased colchicine levels and toxicity. There are numerous things that providers and patients need to know about colchicine (e.g. side effects, administration and drug interactions); therefore, the FDA requires a medication guide to be dispensed. That guide can be found at http://www.fda.gov/ucm/groups/fdagov-public/@fdagov-drugs-gen/documents/document/ucm176363.pdf. Answer (D) is incorrect. Urate-lowering therapies are of no benefit for acute gout and should generally not be initiated during an acute attack. However, in patients already receiving these agents (e.g., allopurinol, febuxostat), the urate-lowering medication should be continued without interruption.

TB was recently started on fluphenazine for the treatment of schizophrenia. He has been taking the fluphenazine for 6 days and is not feeling well. TB is experiencing muscle rigidity, hyperthermia and hypertension and presents with an altered level of consciousness. Select the type of reaction that TB is experiencing. A) Tardive dyskinesia B) Dystonia C) Neuroleptic malignant syndrome D) Serotonin syndrome E) Hypertensive crisis

C) Neuroleptic malignant syndrome Neuroleptic malignant syndrome (NMS) may occur within the first 24 to 72 hours after antipsychotic treatment. NMS may occur with FGA and SGA treatment. Common signs and symptoms associated with this syndrome include hyperthermia, hypertension, altered level of consciousness, rigidity and increased creatine kinase. Answer (A) is incorrect. Tardive dyskinesia is an abnormal involuntary movement which develops after chronic administration of an antipsychotic. Hyperthermia, hypertension and an altered level of consciousness do not occur with tardive dyskinesia. Answer (B) is incorrect. Dystonia is muscle spasms, a type of EPS. These muscle spasms/contractions occur in the neck, head and trunk areas. Dystonic reactions may occur within a few days of antipsychotic treatment; however, hyperthermia, hypertension and an altered level of consciousness do not occur with dystonia. Answer (D) is incorrect. Serotonin syndrome has a similar presentation as NMS; however, serotonin syndrome occurs when two or more serotonergic drugs are combined. Answer (E) is incorrect. A hypertensive crisis occurs when a monoamine oxidase inhibitor (MAO-I) is combined with tyramine-containing foods.

Which of the following antiandrogens is associated with interstitial pneumonitis? A) Flutamide B) Bicalutamide C) Nilutamide D) Enzalutamide E) Abiraterone

C) Nilutamide Answer c is correct.Nilutamide has been associated with interstitial pneumonitis. Answer a is incorrect. Flutamide is not associated with interstitial pneumonitis. Answer b is incorrect. Bicalutamide is not associated with interstitial pneumonitis. Answer d is incorrect.Enzalutamide is not associated with interstitial pneumonitis. Answer e is incorrect. Abiraterone is not an antiandrogen and is not associated with interstitial pneumonitis.

RF is a 62-year-old man with nonischemic cardiomyopathy (LVEF 30%-35%) presenting to the emergency department (ED) with an acute HF exacerbation. His vital signs include BP 155/90 mm Hg, heart rate (HR) 85 beats/min, RR 20, and O2 sat 94% on 4 L/min of oxygen by nasal cannula (NC). Physical examination reveals jugular venous distension (JVD), regular rate and rhythm (RRR), crackles bilaterally at bases, and 2+ bilateral lower extremity edema. He admits to a 12-lb weight gain in the past 2 weeks since his carvedilol dose was increased and reports strict adherence to both dietary restrictions and medications. In the ED, he has already received furosemide 80 mg IV ×1 dose with minimal response in urine output. Pertinent laboratory results include potassium 3.9 mmol/L, BNP 1550 pg/mL, BUN 37 mg/dL, and SCr 1.3 mg/dL (baseline). RF's home medications include lisinopril 10 mg daily, carvedilol 25 mg twice daily, digoxin 0.125 mg/d, and furosemide 80 mg orally twice daily. Based on the BNP value, RF is experiencing which one of the following? A) Active myocardial ischemia B) Shortness of breath due to a noncardiac etiology C) Significant volume overload and ventricular wall stretch D) Renal insufficiency

C) Significant volume overload and ventricular wall stretch Answer c is correct. Brain natriuretic peptide (BNP) is released and elevated in the setting of significant volume overload causing stretch of the ventricular wall. Answer a is incorrect. Common laboratory tests for assessing active myocardial ischemia include creatinine kinase, creatinine kinase-myocardial fraction, and troponin. Answer b is incorrect. BNP may be used to rule out other etiologies of shortness of breath due to a noncardiac etiology, in which cases the BNP level will be normal. RF's shortness of breath (SOB) is due to a heart failure exacerbation with pulmonary edema from cardiac failure, which would be a cardiac etiology. Answer d is incorrect. While BNP may be altered in the setting of renal insufficiency, it is not to the same degree as the level of elevations which occurs in the setting of fluid overload.

What is the most common gram-positive cause of a UTI? A) Staphylococcus aureus B) Staphylococcus epidermidis C) Staphylococcus saprophyticus D) Streptococcus pneumoniae

C) Staphylococcus saprophyticus Staphylococcus saprophyticus (C) is the most common gram-positive bacteria causing UTIs (5%-10%). Staphylococcus aureus (A) may cause UTI; however, it is not a common cause. S. aurues (including MRSA) may cause catheter-associated UTIs. Staphylococcus epidermis (B) is an infrequent cause of UTIs. Please note that S. epidermis is often reported as coagulase-negative Staphylococci. Staphylococcus saprophyticus is also a coagulase-negative staph. Streptococcus pneumoniae (pneumococcus), (D), is an infrequent cause of UTIs. Pneumococcus is a common cause of respiratory tract infections.

TK is a 32-year-old HIV-positive patient. He does not want influenza to develop and would like to be vaccinated since he is immunocompromised. Select the appropriate vaccination for TK. A) LAIV B) Oseltamivir C) TIV D) Amantadine E) Immunocompromised patients should not be vaccinated

C) TIV TIV may be used in immunocompromised patients, including patients with HIV. Answer (A) is incorrect. LAIV is indicated for healthy, nonpregnant patients aged 2 to 49 years. Since TK has HIV, he is not a candidate for LAIV. Answer (B) is incorrect. Oseltamavir cannot be used as vaccine, nor should prevention or postexposure use of oseltamavir replace influenza vaccine TIV or LAIV. Answer (D) is incorrect. Amantadine is not an influenza vaccine and, because of increased resistance, it is seldom used as a preventive or treatment option. Answer (E) is incorrect. Immunocompromised patients should be vaccinated with TIV, unless there is a contraindication. 71% of users answered correctly.

Question 16 of 25 A 3-year-old child and her 65-year-old grandmother were riding in car that was involved in a three-vehicle accident at 3 am. They have only minor injuries, but one of the other vehicles was carrying I131 for use as an imaging agent. The I131 container was not properly stored or sealed and it entered the side window of the vehicle with the child and grandmother. Many of the compounded I131 capsules were ruptured and dispersed throughout the car. They are 20 minutes post accident and no one is answering the phone at the nuclear pharmacy, there is no paperwork with the I131 product and the delivery driver is unconscious. Which is the best advice for the grandmother and child? A) Treat both with potassium iodide. B) Treat the grandmother with potassium iodide and leave the child untreated. C) Treat the child with potassium iodide and leave the grandmother untreated. D) Treat both with Prussian blue. E) Treat neither.

C) Treat the child with potassium iodide and leave the grandmother untreated. There are too many unknowns in this case. An argument could be made to withhold potassium iodide if the dose was known. However the dose is not known and the product was inadequately stored, the child was definitely exposed and given children's sensitivity to I131 treating is the appropriate course for the child. Grandmother is older than 40 years of age. Her risk of thyroid disease secondary to the exposure is virtually nil.

A 55-year-old man was admitted to the hospital with a 3-day history of persistent vomiting. The following laboratory values are taken: pH 7.40, PaCO2 40 mm Hg, HCO3 24 mEq/L, Na 149 mEq/L, Cl 100 mEq/L, BUN 110 mg/dL, and Cr 8.7 mg/dL. What would you expect the pH, PaCO2, and HCO3 to be in a patient who has persistent vomiting (↑, ↓, or N)? A) pH ↑; PaCO2 ↓; HCO3 ↑ B) pH ↓; PaCO2 ↓; HCO3 ↓ C) pH ↑; PaCO2 N; HCO3 ↑ D) pH ↓; PaCO2 N; HCO3 ↓

C) pH ↑; PaCO2 N; HCO3 ↑ Answer c is correct. In a patient with persistent vomiting, one would expect to see metabolic alkalosis, with or without respiratory compensation. Therefore, the pH should be elevated, the HCO3 should be elevated (because of a loss of hydrogen ions from the upper GI tract), and the PaCO2 should be normal or elevated (depending on the duration of metabolic alkalosis). Answer a is incorrect. PaCO2 should be normal or elevated (depending on the duration of metabolic alkalosis). Answers b and d are incorrect. The patient should be alkalemic not acidemic.

Which of the following medications can be used to maintain sinus rhythm in a patient with atrial fibrillation and heart failure? A.) Digoxin B.) Metoprolol succinate C.) Amiodarone D.) Propafenone E.) Adenosine

C.) Amiodarone EXPLANATION: Only amiodarone or dofetilide is recommended to maintain sinus rhythm in patients with atrial fibrillation and heart failure. Digoxin, metoprolol succinate, and adenosine are not effective for maintaining sinus rhythm. Propafenone is contraindicated in patients with heart failure because of its negative inotropic effects and its increased risk of proarrhythmia in this population.

KF is a 72" tall, 173lb, 30 year-old male with normal renal function that has been on a ventilator in the trauma ICU after a motor vehicle accident. Five days into his hospital stay KF spikes a temperature of 39.4°C, labs reveal WBC count of 17, and there is an infiltrate noted on chest x-ray. The attending surgeon requests assistance with antibiotic coverage, and would like an agent to cover MRSA and two agents to double-cover for P. aeruginosa until cultures and sensitivities are known. Select the appropriate initial dose of Amikacin for KF utilizing extended-interval dosing. A.) 500mg IV Q12H B.) 500mg PO Q8H C.) 1500mg IV Q24H D.) 550mg IV Q24H E.) 1500mg IV Q8H

C.) 1500mg IV Q24H The recommended dose of amikacin in HCAP or VAP is 20mg/kg based on ideal bodyweight dosed once daily in patients with normal renal function. This patient's IBW is 77.6kg, thus 1500mg would be approximately 19.3mg/kg/day.

The maximum daily dose of galantamine in patients with renal impairment is A.) 8 mg/day. B.) 12 mg/day. C.) 16 mg/day. D.) 24 mg/day. E.) 32 mg/day.

C.) 16 mg/day. With renal or hepatic dysfunction, galantamine doses should not exceed 16 mg/day. With severe renal or hepatic dysfunction, galantamine should not be used.

Calcium chloride (CaCl2) is a three-ion electrolyte that dissociates at 70% of the relevant concentration. What is its dissociation factor? A.) 1.8 B.) 2.2 C.) 2.4 D.) 2.6

C.) 2.4 Because this salt dissociates into three ions (Ca++, Cl-, and Cl - ) and the leftover of undissociated CaCl 2, , the total amounts of ions are 70, 70, 70, and 30. Therefore, the total amount of four species in solution is 240. The dissociation factor becomes 240/100.

Calculate the dose for a child 4 years of age, 39 inches in height, and 32 lb in weight for a drug with an adult dose of 100 mg. Use the BSA equation. A.) 45.3 mg B.) 25.4 mg C.) 36.4 mg D.) 40.1 mg

C.) 36.4 mg EXPLANATION: Find the body surface area (BSA) from the equation, and then set up a ratio between dose (100 mg) per 1.73 m2 and the surface area of the child.

Calculate the amount of water (in grams) in 100 mL of 65% (w/w) syrup that has a density of 1.313. A.) 30 g B.) 75 g C.) 46 g D.) 65 g E.) 35 g

C.) 46 g 65% w/w means 65 g in 100 g syrup. The total weight of the 100 mL syrup is 100 mL × 1.313 (density) = 131.3 g

One of your regular patients comes into your pharmacy and reports that he developed a dry, hacking cough 2 days ago, and although he has been taking a nonprescription liquid cough suppressant, it has been ineffective in controlling the cough. He says that not too long ago, another pharmacy sold him a nonprescription cough suppressant containing codeine that was very effective. He asks if you have such a product, and you reply that you do and agree to sell him the product. Assuming that state law permits selling such a product without a prescription, you may sell not more than 240 cc (8 oz) of a controlled substance containing opium, nor more than 120 cc (4 oz) of any other controlled substance, nor more than 48 dosage units of a controlled substance containing opium, nor more than 24 dosage units of any other controlled substance in any given period of how many hours? A.) 24 B.) 36 C.) 48 D.) 60 E.) 72

C.) 48 The applicable DEA regulation is 21 CFR 1306.26.

Which of the following is not true about the chi-square test? A.) Observed frequencies are the number of observed individuals or other entities that fall into the various categories of the variable of interest. B.) The chi-square statistic measures the agreement between the observed frequencies and the expected frequencies. C.) A very small test statistic leads to the rejection of the null hypothesis. D.) The chi-square test is commonly used when the variables are categorical. E.) The chi-square test is commonly used when an investigator has frequency or count data.

C.) A very small test statistic leads to the rejection of the null hypothesis. EXPLANATION: A very large test statistic leads to the rejection of the null hypothesis for the chi-square test

A progestin-only oral contraceptive would be preferable over a combination oral contraceptive in which of the following cases? A.) A 24-year-old college student who is sexually active B.) A 48-year-old perimenopausal female with irregular menstrual cycles C.) A woman who is breast-feeding her infant D.) A 30-year-old obese patient with type 2 diabetes mellitus E.) A 52-year-old postmenopausal woman

C.) A woman who is breast-feeding her infant Progestin-only oral contraceptives are preferred in breast-feeding women because they do not negatively affect milk supply. Perimenopausal females may take combined oral contraceptives to regulate their menstrual cycle.

A 62-year-old man with newly diagnosed ED was given samples of Stendra by his physician. He complains that the medication is not working. The most appropriate action is to verify that the patient was instructed on proper medication use. Which of the following counseling points for Stendra is correct? A.) Food may impair absorption. B.) Place the tablet under the tongue. C.) Administer 15 minutes before sexual intercourse. D.) Priapism has not been reported.: E - Floppy iris syndrome can occur.

C.) Administer 15 minutes before sexual intercourse. Stendra, a PDE-5 inhibitor, should be taken orally 15 minutes before sexual activity regardless of meals. Priapism is possible with all PDE-5 inhibitors. Floppy iris syndrome is associated with alpha blockers.

Which of the following medications has the following warning: "For patients switching from immediate-release niacin, therapy with this drug should be initiated with a low dose and then titrated to the desired therapeutic response"? A.) Pravigard B.) Vytorin C.) Advicor D.) Atorvastatin E.) Ezetimibe

C.) Advicor Advicor (Niaspan + lovastatin) contains Niaspan, which is not dose-equivalent to immediate-release niacin or modified-release (sustained-release or time-release) niacin preparations.

A. J. is started on abacavir/lamivudine, atazanavir, and ritonavir. Four weeks later, she appears jaundiced and has icterus. Her total bilirubin is 5.2. Which of her medications could be causing this? A.) Abacavir B.) Lamivudine C.) Atazanavir D.) Ritonavir E.) None. Her symptoms are not medication related; check for acute hepatitis.

C.) Atazanavir Atazanavir can cause a reversible increase in indirect bilirubin due to UGT 1A1 inhibition in 22% to 47% of patients. This increase is medically inconsequential but may cause jaundice or icterus. If patients develop jaundice or icterus, consideration should be given to discontinuing atazanavir. Also, no safety data are available for patients with continued bilirubins greater than five times the upper limit of normal, and discontinuation may be warranted.

A health care provider inquires about the recommended monitoring parameters for patients started on Ridaura. Which of the following represents the most appropriate response? A.) Baseline ophthalmologic exam, CBC, and serum creatinine, followed by a yearly CBC, serum creatinine, and ophthalmologic exam B.) Baseline LFTs, CBC, and albumin, followed by monthly LFTs C.) Baseline CBC, serum creatinine, and urinalysis for protein, followed by a CBC and urinalysis for protein every 1-2 months D.) No recommended monitoring parameters at this time

C.) Baseline CBC, serum creatinine, and urinalysis for protein, followed by a CBC and urinalysis for protein every 1-2 months Gold therapy is associated with glomerulonephritis, thrombocytopenia, and leukopenia; therefore, a baseline renal evaluation and periodic testing should occur during the entire course of therapy.

Question 12 of 25Next Question|Previous Question DM is the leading cause of which of the following microvascular complications? A.) Pancreatitis B.) Fatty liver C.) Blindness D.) Stroke E.) Deafness

C.) Blindness Diabetes is the leading cause of new cases of blindness among adults 20-74 years of age in the United States. There is also an increased incidence of stroke, but diabetes is not the leading cause of this problem. The other items are incorrect.

Toprol-XL is an agent that: A.) Is contraindicated in heart failure B.) Blocks beta-1, beta-2, and alpha-1 receptors C.) Blocks only beta-1 receptors D.) Should not be used in combination with Zestril E.) Increases the serum digoxin concentration

C.) Blocks only beta-1 receptors

What laboratory value should be monitored weekly in patients taking daptomycin? A.) Liver enzymes B.) Creatinine C.) CPK D.) PT/INR E.) CRP

C.) CPK EXPLANATION: CPK (creatine phosphokinase) should be monitored weekly in patients receiving daptomycin because of concerns of myotoxicity. Although nephrotoxicity and hepatotoxicity have been reported with use of this drug, weekly monitoring is not recommended. Daptomycin can cause a false elevation in INR, which can make evaluation of warfarin difficult but does not necessitate routine monitoring.

Which of the following agents may be used in the enteric coating of tablets? (Mark all that apply.) A.) Hydroxypropyl methylcellulose B.) Carboxymethylcellulose C.) Cellulose acetate phthalate D.) Acacia

C.) Cellulose acetate phthalate An enteric-coated tablet has a coating that remains intact in the stomach but dissolves in the intestine when the pH exceeds 6. Enteric-coating materials include cellulose acetate phthalate, polyvinyl acetate phthalate, and hydroxypropyl methylcellulose phthalate.

AM, an ovarian cancer patient, originally presented with creatinine 1.8 mg/dL and after surgery her creatinine is stable at 2 mg/dL. Which of the agents normally used in adjuvant chemotherapy of ovarian cancer will AM's oncologist most likely avoid? A.) Topotecan and liposomal doxorubicin. B.) Paclitaxel and docetaxel. C.) Cisplatin. D.) Carboplatin.

C.) Cisplatin. Cancer patients with stable compromised renal function should not receive cancer medications known to be renally excreted and known to cause renal damage. Topotecan and carboplatin are excreted renally but are not known as renal toxins. Paclitaxel and docetaxel are not excreted renally. Cisplatin is excreted renally and is a well-known direct renal tubular toxin.

Patient is requesting a formulation of lamotrigine that can be dissolved in water and then swallowed as a liquid because he has difficulty swallowing pills. Which of the following is an important counseling point for this patient? A.) Take the lamotrigine solution with food B.) Take the lamotrigine solution without food C.) Consume the solution immediately after the tablet is dispersed D.) Unused solution could be stored in the refrigerator E.) Lamotrigine cannot be dissolved in water, it must be swallowed whole

C.) Consume the solution immediately after the tablet is dispersed Correct answer: C Lamictal chewable tablets may be chewed, dispersed in water or diluted fruit juice, or swallowed whole. To make a lamotrigine solution, add the chewable tablet to just enough liquid to cover tablet. Let the tablet sit in the liquid until dispersed (~1 minute), then swirl solution and consume immediately. Partial doses should not be consumed, and full or partial dose should not be stored for later use. Answers A & B: Lamotrigine can be taken with or without food. Answer D:Partial doses should not be consumed, and full or partial dose should not be stored for later use. Answer E:Lamictal chewable tablets may be chewed, dispersed in water or diluted fruit juice, or swallowed whole. Bottom Line:Lamictal chewable tablets may be chewed, dispersed in liquid, or swallowed whole. To make a lamotrigine solution, add the chewable tablet to just enough liquid to cover tablet. Let the tablet sit in the liquid until dispersed (~1 minute), then swirl solution and consume immediately. Partial doses should not be consumed, and full or partial dose should not be stored for later use.

Which of the following is not used to determine a sample size for statistical analysis? A.) The desired statistical power. B.) The estimated effect size. C.) Disease prevalence D.) Variability and experimental error

C.) Disease prevalence The desired statistical power, estimated effect size, and variability and experimental error are all used to calculate a sample size. The disease prevalence is not is not used in calculating sample size and is the exception.

A 76-year-old female has a history of rheumatoid arthritis. Her medications for this condition include etanercept 50 mg subcutaneously once weekly and prednisone 5 mg po daily. She is admitted to the general medical ward of her local hospital for a deep venous thrombosis, and her physician asks for recommendations regarding her home medications. Which of the following is the best recommendation for her regarding her prednisone therapy? A.) Discontinue prednisone therapy while patient is in the hospital. B.) Change the dose to prednisone 2.5 mg po daily while patient is in the hospital. C.) Continue the prednisone at the current dose of 5 mg po daily while patient is in the hospital. D.) Increase the dose of prednisone to 20 mg po daily while patient is in the hospital. E.) Convert prednisone po to prednisone 10 mg IV daily while patient is in the hospital.

C.) Continue the prednisone at the current dose of 5 mg po daily while patient is in the hospital. Discontinuation or decrease of the prednisone the patient takes at home may precipitate withdrawal symptoms caused by HPA axis suppression. Increasing the dose to four times the home dose or converting to an IV formulation is not necessary according to the clinical situation of the patient.

Which of the following conditions could Synthroid exacerbate? A.) Hypercholesterolemia B.) Anemia C.) Coronary artery disease D.) Hypertension E.) Constipation

C.) Coronary artery disease Thyroid hormones enhance oxygen consumption and increase oxygen demand. They should be used with caution in patients with coronary artery disease.

What is the mechanism of action of dalteparin? A.) Dalteparin inactivates factor Xa only B.) Dalteparin inactivates factor IIa only C.) Dalteparin inactivates factor Xa and factor IIa but has higher affinity for factor Xa D.) Dalteparin inactivates factor Xa and factor IIa but has higher affinity for factor IIa E.) Dalteparin has no effect on factor Xa or IIa

C.) Dalteparin inactivates factor Xa and factor IIa but has higher affinity for factor Xa Correct answer: C Dalteparin is a low-molecular-weight heparin, and its anticoagulation effect is byinactivating factor Xa and factor IIa, but it has greater binding affinity for factor Xa. The ratio of anti-factor Xa to anti-factor IIa has been thought to be as high as 4:1. Answer A:Dalteparin inactivates both factor Xa and factor IIa, not just factor Xa. An example of an anti-factor Xa inhibitor is fondaparinux. Answer B:Dalteparin inactivates both factor Xa and factor IIa, not just factor IIa. Answer D:Although dalteparin inactivates both factor Xa and factor IIa, it has a greater affinity for factor Xa, not factor IIa. Answer E:Dalteparin works by binding to and inactivating factor Xa and factor IIa. Bottom Line:Dalteparin is a low-molecular-weight heparin that works by inhibiting factor Xa and factor IIa, but it has greater binding affinity for factor Xa.

Which of the following statements is TRUE regarding the pathophysiology of secondary hyperparathyroidism? A.) Decrease in functional mass of the kidney leads to elevated serum calcium levels B.) Decrease in functional mass of the kidney leads to hyperactivation of vitamin D3 C.) Decrease in functional mass of the kidney leads to increased serum phosphorus D.) Secondary hyperparathyroidism commonly occurs in patients with GFR < 90 mL/min E.) Elevated parathyroid hormone levels commonly lead to overt clinical symtpoms

C.) Decrease in functional mass of the kidney leads to increased serum phosphorus Correct answer: C Hyperparathyroidism and the subsequent disorders in phosphorus, calcium, and vitamin D are common clinical manifestations seen in patients with chronic kidney disease (CKD). Complications due to secondary hyperparathyroidism most commonly manifest in stages 3, 4, or 5 CKD (glomerular filtration rate [GFR] ≤ 60 mL/min/1.73 m2).As kidney function declines, impairment in serum phosphorus excretion leads to hyperphosphatemia.Subsequent chelation of calcium leads to hypocalemia. Uremic overtone decreases vitamin D activation. In response to these changes, parathyroid release is increased. This stimulates osteoclasts to lyse bone to release calcium in the serum. The end product is severe electrolyte derangements, bone loss, and increase parathyroid tone

A 78 yo patient has atrial fibrillation and newly diagnosed hypothyroidism. The patient currently stable on warfarin 3 mg daily. How and why will the addition of levothyroxine and subsequent correction of hypothyroidism affect the INR? A.) Decrease, due to induction of CYP3A4 B.) Increase, due to inhibition of CYP3A4 C.) Decrease, due to increased clotting factor metabolism D.) Increase, due to decreased clotting factor metabolism E.) None of the above

C.) Decrease, due to increased clotting factor metabolism During the hypothyroid state patients may be resistant to warfarin due to decreased vitamin K-dependent clotting factor metabolism. As the patient becomes euthyroid the INR may decrease requiring a higher warfarin dose.

Which of the following sedative agents is least likely to cause respiratory depression? A.) Lorazepam B.) Propofol C.) Dexmedetomidine D.) Midazolam E.) Fentanyl

C.) Dexmedetomidine Dexmedetomidine does not cause respiratory depression. All the other agents can do so.

Which of the following conditions is a common side effect of aliskiren? A.) Tachycardia B.) Bradycardia C.) Diarrhea D.) Vomiting E.) Flushing

C.) Diarrhea EXPLANATION: Diarrhea is the most common side effect of aliskiren with a reported incidence of 2.3%. Other associated side effects include headache, nasopharyngitis, dizziness, and fatigue. More rare adverse events include elevated uric acid, gout, angioedema, and rashes.

C. H. is an 82-year-old male with a history of falls related to low blood pressure and now reports urinary incontinence. Which of the following would have the greatest potential for worsening his orthostatic hypotension? A.) Alfuzosin B.) Darifenacin C.) Doxazosin D.) Dutasteride E.) Tamsulosin

C.) Doxazosin Doxazosin, an α-adrenergic antagonist, could improve urine flow but has the potential to induce orthostatic hypotension and falls with geriatric patients being at greater risk. Alfuzosin and tamsulosin have a similar mechanism of action but may produce less orthostatic hypotension because of greater selectivity to bladder function.

By altering the design of the dosage form of the drug, one can modify the following property A.) Intrinsic Solubility of the drug B.) Pharmacodynamics of the drug C.) Drug release of the drug D.) All of the above

C.) Drug release of the drug Dosage form determines the drug release at the site of absorption. By altering the dosage form once can modify the amount of drug released in the small intestine which is available for absorption. Extended release product modify the rate of drug release thereby provide sustained plasma levels of the drug.

Hypokalemia is a possible adverse effect of all of the following medications except A.) Hydrochlorothiazide B.) Chlorthalidone C.) Eplerenone D.) Torsemide E.) Metolazone

C.) Eplerenone Eplerenone causes increased potassium. All the other medications listed cause a decrease in serum potassium.

Which one of the following products would be most appropriate to use in a patient with a history of hypertriglyceridemia? A.) Premarin (conjugated estrogens) B.) Estratest (esterified estrogens/methyltestosterone) C.) Estrogel (estradiol micronized topical gel) D.) Prempro (estrogen/medroxyprogesterone) E.) Activella (estradiol/norethindrone)

C.) Estrogel (estradiol micronized topical gel) Estrogen may exacerbate certain disease states including hypertriglyceridemia. The exacerbation of hypertriglyceridemia can be avoided or minimized by using a transdermal product rather than oral.

What is the drug of choice for absence seizures in a child < 2 years of age? A.) Phenytoin B.) Phenobarbital C.) Ethosuximide D.) Valproic acid E.) Primidone

C.) Ethosuximide Although valproic acid is extremely effective and is frequently used as monotherapy for absence seizures, it should not be given to a patient < 2 years of age.

Which of the following agents can be used as systemic therapy to treat fungal dermatological conditions? A.) Miconazole B.) Clotrimazole C.) Fluconazole D.) Tolnaftate E.) Undecylenic acid

C.) Fluconazole Fluconazole is a systemic antifungal agent. All others are used topically for fungal conditions.

Insomnia, GI upset, and headache are common side effects of which of the following antidepressants? A.) Phenelzine B.) Amitriptyline C.) Fluoxetine D.) Trazodone E.) Both B and C

C.) Fluoxetine SSRIs commonly cause insomnia, GI upset, anxiety, headache, and sexual dysfunction. Phenelzine, amitriptyline, and trazodone cause sedative effects.

Which of the following biological agents is indicated for treatment of ovulatory failure? A.) Ganirelix (Antagon) B.) Glucagon (GlucaGen) C.) Follitropin alfa (Gonal-F) D.) Eptifibatide (Integrilin) E.) Thyrotropin (Thyrogen)

C.) Follitropin alfa (Gonal-F) Ganirelix (Antagon) is a recombinant hormone indicated for the treatment of luteinizing hormone surge during fertility therapy. Glucagon (GlucaGen) is a recombinant hormone indicated for treatment of hypoglycemia. Eptifibatide (Integrilin) is a recombinant enzyme indicated for the treatment of acute coronary syndromes. Thyrotropin (Thyrogen) is a recombinant hormone indicated for the treatment of thyroid cancer.

A 69 year old male with stage 4 non-small cell lung cancer presents to your outpatient chemotherapy service for cycle 2 day 1 of gemcitabine chemotherapy (he previously failed to respond to three cycles of paclitaxel and carboplatin). His labs are drawn today and are: WBC 2900 cells/mcL, absolute neutrophil count (ANC) 990 cells/mcL, platelets 101,000 cells/mcL, Hgb 10.3 mg/dL, Hct 31%, SCr 0.9, AST 29, ALT 28, total bilirubin 0.4, conjugated bilirubin 0.2. The chemotherapy nurse is concerned about these lab values and calls the oncologist. The oncologist's most rational decision is to: A.) Treat the patient with the chemotherapy as ordered and consider adding Neupogen, Leukine, or Neulasta to the regimen. B.) Treat the patient with a 20% dose reduction and consider adding Neupogen, Leukine, or Neulasta to the regimen. C.) Hold the patient's chemotherapy for one week, then consider a dose reduction. D.) Stop chemotherapy permanently in this patient.

C.) Hold the patient's chemotherapy for one week, then consider a dose reduction. For stage 4 non-small cell lung cancer, second line chemotherapy is definitely palliative. There is rarely a reason to be aggressive with dose intensity with palliative care chemotherapy. There is also little pharmacoeconomic data to support the use of colony stimulating factors.

Which of the following are true regarding Accutane (isotretinoin)? I. Pharmacies, prescribers, patients, and wholesalers must register with the iPledge system. II. Women must have documentation of a negative pregnancy test before the dispensing of the medication each month. III. Liver function tests (LFTs), hemoglobin A1c (HgA1c), and complete blood count (CBC) should be monitored. A.) I only B.) III only C.) I and II D.) I and III E.) I, II, and III

C.) I and II LFTs, lipid panel—not HgA1c—and CBC should be monitored when an individual is taking Accutane.

Which of the following should initial treatment of cryptococcal meningitis include? I. Amphotericin B II. Flucytosine III. Fluconazole A.) I only B.) II only C.) I and II only D.) II and III only E.) I, II, and III

C.) I and II only Amphotericin B plus flucytosine is the recommended treatment for at least the first 2 weeks for cryptococcal meningitis. Fluconazole is the drug of choice for non-CNS cryptococcal infections and for the 10-week consolidation phase of cryptococcal meningitis treatment.

Which of the following medications may have its action potentiated (i.e., increased) by corticosteroids, aminoglycosides, or calcium channel blockers? A.) Fentanyl B.) Lorazepam C.) Vecuronium D.) Dexmedetomidine E.) Propofol

C.) Vecuronium EXPLANATION: Because of some similarities in chemical structures, corticosteroids, aminoglycosides, or calcium channel blockers can enhance the effect of neuromuscular blockers such as vecuronium.

Which of the following statements concerning Parkinson's disease is (are) true? I. Medications are the primary but not the only treatment option. II. Rigidity of limbs may be experienced in patients who have tremors. III. Symptoms include micrographia and frequent diarrhea. A.) I only B.) III only C.) I and II only D.) II and III only E.) I, II, and III

C.) I and II only EXPLANATION: In addition to medications, a variety of nondrug therapies are part of the treatment of Parkinson's disease. Physical therapy to improve movement and endurance and occupational therapy to improve functionality in the home as well as dietary consultations are all examples of appropriate nondrug therapies. Although micrographia is a symptom of Parkinson's disease, most patients will experience constipation secondary to decreased mobility and fluid intake.

Moxifloxacin exhibits antibacterial activity against which of the following organisms? I. Atypical bacteria II. Streptococcus pneumoniae III. Pseudomonas aeruginosa A.) I only B.) II only C.) I and II only D.) II and III only E.) I, II, and III

C.) I and II only Moxifloxacin is active against atypical pathogens and Streptococcus pneumoniae, making it an ideal drug for the treatment of community-acquired pneumonia. Moxifloxacin is not as active against Pseudomonas aeruginosa as ciprofloxacin or levofloxacin and should not be used for treatment when Pseudomonas is known or suspected to be the causative pathogen.

Which of the following are not associated with any drug-drug interactions? I. Gabapentin II. Carbamazepine III. Levetiracetam A.) I only B.) II only C.) I and III only D.) II and III only E.) I and II only F.) III only

C.) I and III only At this time, neither gabapentin nor levetiracetam are associated with significant drug-drug interactions. The absorption of gabapentin may be reduced by concurrent administration of aluminum- and/or magnesium-containing antacids; hence antacids should be given 2 hours before or after a dose of gabapentin. Carbamazepine is an inducer that is associated with numerous drug-drug interactions.

Which of the following opioids are active at the kappa receptor? I. Buprenorphine II. Fentanyl III. Pentazocine A.) I only B.) III only C.) I and III only D.) II and III only E.) I, II, and III

C.) I and III only Explanation: Both buprenorphine and pentazocine are active at the kappa receptor. Fentanyl is a pure µ-agonist.

Which of these fungal infections may be treated effectively with the use of topical antifungal agents? I. Tinea capitis (ringworm of the scalp) II. Tinea unguium (fungal infection of the nails) III. Tinea corporis (ringworm of the skin) A.) I only B.) II only C.) III only D.) I and II E.) I, II, and III

C.) III only Topical antifungal agents are the first line of therapy against tinea corporis (ringworm of the skin). However, systemic antifungal therapy is usually required for treatment of tinea capitis (ringworm of the scalp) and tinea unguium (fungal infection of the nails).

A 24-year-old female presents to the emergency department with complaints of severe shortness of breath, dyspnea, and chest pain. She is also experiencing tachycardia and tachypnea. Two days earlier she noticed pain and swelling in her left lower extremity. Her medical history is negative for thrombosis. Her current medications include Tri-Levlen daily and ibuprofen 600 mg q6h prn for pain. A duplex ultrasound of the left lower extremity revealed a DVT. A CT scan of her chest reveals a pulmonary embolism. Her vital signs are T, 98.4°F; P, 124/min; R, 36/min; BP, 162/100 mm Hg; Wt, 220 lb (100 kg); Ht, 5' 4". This patient is started on heparin therapy. Which dosage regimen is most appropriate? A.) IV heparin 20,000 IU bolus, then 5,000 IU/h B.) IV heparin 8,000 IU bolus, then 1,800 IU/h C.) IV heparin 5,000 IU bolus, then 500 IU/h D.) IV heparin 5,000 IU q12h E.) Subcutaneous heparin 5,000 IU q12h

C.) IV heparin 5,000 IU bolus, then 500 IU/h Several studies have indicated that weight-based dosing of heparin is more effective in obtaining therapeutic aPTT than standard heparin titration. A weight-based protocol with an 80 IU/kg IV bolus followed by an infusion of 18 IU/kg/h should produce aPTTs close to the therapeutic range. The other doses are not appropriate.

Which of the following administration rates is most appropriate for IV calcium gluconate when treating hypocalcemia in the ICU? A.) 1 g/hr B.) 10 mEq/hr C.) IV push over 10 minutes D.) 7.5 mmol/hr E.) 60 mg/min

C.) IV push over 10 minutes EXPLANATION: According to the manufacturer's information, a calcium gluconate dose can be administered over 10 minutes.

A patient with normal kidney function (serum creatinine [SCr] 1.12 mg/dL, blood urea nitrogen [BUN] 21 mg/dL) is admitted to the hospital with severe sepsis. Labs repeated the next day show an SCr of 2.52 mg/dL and BUN of 46 mg/dL. On the basis of this information, how would that patient be classified according to the following RIFLE classifications for acute kidney injury? A.) Risk because he has severe sepsis B.) Failure because his SCr and BUN have more than doubled C.) Injury because his SCr has more than doubled D.) Injury because his BUN has doubled E.) Risk because his SCr has more than doubled

C.) Injury because his SCr has more than doubled EXPLANATION: According to this information, the patient would fall into the injury category because his serum creatinine has more than doubled in a short time. By definition, injury is a twofold increase in SCr or a decrease in glomerular filtration rate by 50% or urine output less than 0.5 mL/kg/h for 12 hours.

Which of the following statements about digoxin immune Fab is true? A.) Digoxin bound to digoxin immune Fab retains digoxin-like pharmacologic activity. B.) It enhances digoxin elimination in the bile and stools. C.) It is derived from sheep serum that has antibodies to digoxin. D.) It demonstrates virtually no cross-reactivity to digoxin-like substances. E.) It can be administered only if the ingested dose of digoxin is known.

C.) It is derived from sheep serum that has antibodies to digoxin. The digoxin antibody is derived from sheep serum and should be used cautiously in people with sheep allergy. The bound digoxin is inactive and is eliminated in the urine. There is some cross-reactivity. The dose is determined by steady-state serum concentrations, by dose ingested, and by empirical method.

Which is a characteristic of a preparation in the moderate compounding category according to USP Chapter <795>? A.) It requires manipulation of a commercial product by adding a component. B.) It has a USP compounding monograph. C.) It requires special calculations to determine component quantities per preparation of individualized units. D.) It has appeared in a peer-reviewed journal.

C.) It requires special calculations to determine component quantities per preparation of individualized units. The compounding of troches is an example of a preparation in the moderate category. The cavities of the mold used to make them require calibration.

S. A. has been diagnosed with histoplasmosis. His current medications include tenofovir/emtricitabine, darunavir, ritonavir, dapsone, azithromycin, and lansoprazole. Which of the following choices would be the most appropriate for his treatment of histoplasmosis? A.) Itraconazole tablets B.) Fluconazole C.) Itraconazole solution D.) Voriconazole E.) Caspofungin

C.) Itraconazole solution Itraconazole is the azole of choice for treatment of histoplasmosis. The tablets require an acidic environment for absorption; the solution does not. Therefore, if patients require antacids, H 2 receptor antagonists, or PPIs, itraconazole solution is preferred.

Which of the following is not a counseling point for a patient recently initiated on Chlorthalidone? A.) Take early in the day to avoid nocturia B.) May be taken with food or milk C.) May decrease blood glucose in diabetics D.) Report problems with muscle cramps E.) May increase sensitivity to sunlight

C.) May decrease blood glucose in diabetics All of the answers listed are patient counseling points to discuss with a patient except for answer C. Thiazide diuretics may increase blood glucose in diabetics. Patients with diabetes should monitor blood glucose and report any changes to their healthcare provider.

A 32-year-old Caucasian male presents with bloody diarrhea (fewer than four stools per day) for 2 days. Complete blood counts and erythrocyte sedimentation rate are normal. Physical exam is normal. Colonoscopy reveals distal colitis. The patient has been using a mesalamine enema nightly. Which of the following is the best choice for initial therapy for him? A.) Prednisone 40 mg po daily B.) Sulfasalazine 4-6 g po daily C.) Mesalamine 1-4 g PR nightly D.) Mesalamine 4-6 g po daily E.) Methylprednisolone 16 mg IV q8h

C.) Mesalamine 1-4 g PR nightly Topical aminosalicylates (answer C) are more effective than oral aminosalicylates (answer B) and topical steroids for mild distal ulcerative colitis (although oral aminosalicylates or topical steroids may be used first line if the patient prefers). Oral and IV steroids (answers A and E, respectively) are reserved for more severe cases of ulcerative colitis or cases that do not respond to oral and topical aminosalicylates. The oral dose of mesalamine in answer D is incorrect.

Which of the following drugs is used to treat Cushing's disease? A.) Clotrimazole B.) Propylthiouracil C.) Mitotane D.) Vasopressin E.) Prednisone

C.) Mitotane Mitotane is used to treat Cushing's disease. Propylthiouracil is used to treat hyperthyroidism. Clotrimazole is a topical antifungal, and vasopressin is used to treat diabetes insipidus.

Which of the following medications are the generic names for Dulera, the new inhaled corticosteroid and long-acting inhaled β2-agonist combination product? A.) Beclomethasone and salmeterol B.) Budesonide and formoterol C.) Mometasone and formoterol D.) Mometasone and salmeterol E.) Fluticasone and salmeterol

C.) Mometasone and formoterol

Which of the following effects is mediated by antagonism of peripheral nicotinic acetylcholine receptors? A.) Bradycardia B.) Bronchoconstriction C.) Muscle paralysis D.) Sialorrhea E.) Tachycardia

C.) Muscle paralysis Correct answer: C Acetylcholine acts on both muscarinic and nicotinic receptors. These 2 types of acetylcholine receptors are differentiated by the effects they mediate. Muscarinic receptors were named as such because these are the acetylcholine receptors that muscarine specifically stimulates, and nicotinic receptors were named as such because these are the acetylcholine receptors that nicotine specifically stimulates.Peripheral nicotinic acetylcholine receptors mediate skeletal muscle contraction. Antagonism of these receptors will result in muscle paralysis. Succinylcholine, vecuronium, and cisatracurium are examples of medications that act as antagonists at peripheral nicotinic acetylcholine receptors. In contrast, peripheral muscarinic acetylcholine receptors mediate parasympathetic effects such as bronchoconstriction, secretory functions, gastrointestinal motility, bladder contractility, and negative chronotropic effects. Answer A:Bradycardia is not mediated by nicotinic acetylcholine receptors. Stimulation of peripheral muscarinic acetylcholine receptors causes increased vagal tone and may lead to bradycardia. Answer B:Bronchoconstriction is mediated by stimulation of peripheral muscarinic acetylcholine receptors in smooth muscle. Smooth muscle tone is not modulated by nicotinic acetylcholine receptors. Answer D:Sialorrhea is not mediated by nicotinic acetylcholine receptors. Stimulation of peripheral muscarinic acetylcholine receptors causes increased secretory functions and may result in sialorrhea. Answer E:Tachycardia is not mediated by nicotinic acetylcholine receptors. Antagonism of peripheral muscarinic acetylcholine receptors decreases vagal tone and may result in tachycardia. Bottom Line:Peripheral nicotinic acetylcholine receptors mediate skeletal muscle contraction. Peripheral muscarinic acetylcholine receptors mediate parasympathetic effects.

Which of the following monitoring goals is the most appropriate for argatroban? A.) Anti-Xa level 0.6-1 unit/mL B.) PTT 1.5-2.5 times control C.) PTT 1.5-3.0 times control D.) INR 2.0-3.0 E.) Monitor only in pregnancy or renal impairment.

C.) PTT 1.5-3.0 times control EXPLANATION: According to the manufacturer's information, the therapeutic range for argatroban is PTT 1.5-3.0 times control.

Which arthritis drugs below are safe in pregnancy? A.) Arava B.) Methotrexate C.) Plaquenil D.) Leflunomide

C.) Plaquenil Because Arava (leflunomide) is a teratogenic agent (pregnancy category X) with an active metabolite with a long half-life, a drug elimination procedure should be performed before becoming pregnant. Methotrexate is also category X, but hydroxychloroquine is often used safely in pregnancy.

Intramuscular injection of iron dextran is not generally recommended due to: A.) Increased delayed hypersensitivity B.) Pain at the injection site C.) Potential permanent staining of the skin D.) Delayed transfer to the reticuloendothelial system

C.) Potential permanent staining of the skin Potential permanent staining of the skin. Intramuscular injection of iron dextran must be administered by z-track injection. Z-track injection is a method by which the medication is administered into a large muscle which prevents the leakage of medication back into the subcutaneous tissue. It is called Z-track because the skin is pushed away from the injection site prior to the injection so that the injection is implemented by creating a zigzag injection path sealing the drug within the muscle.

Which of the following is not a measure of central tendency? A.) Median B.) Mean C.) Range D.) Mode

C.) Range Range is a measure of dispersion

Which of the following references would be the best source to identify drug information regarding whether a medication is lactose free? A.) AHFS Drug Information B.) FDA Orange Book C.) Red Book D.) Martindale: The Complete Drug Reference

C.) Red Book

Use the following case study to answer question. A 50-year-old man comes to your pharmacy for cholesterol and medication monitoring. His medical history is notable for hypertension, recent-onset type 2 diabetes, and hypercholesterolemia. Family history is noncontributory. Social history indicates that he neither smokes nor uses alcohol. He has no known allergies. His medication history reveals that he occasionally takes acetaminophen for headaches and no other OTC medications or herbal products. Current medications include hydrochlorothiazide 25 mg/day (for 4 years) and a new prescription today for atorvastatin 10 mg/day. Your physical assessment reveals the following: BP 144/90 mm Hg; pulse, 70 and regular; weight, 185 lb; height, 5′9ʺ. Other pertinent labs include an A1c of 7.3%. A FLP today reveals the following: total cholesterol = 250 mg/dL, HDL = 40 mg/dL, and TG = 145 mg/dL. Which of the following baseline labs would be optimal to assess prior to initiation of this patient's medication for dyslipidemia? A.) CBC, SCr, LFTs B.) SCr, LFTs C.) SCr, LFTs, CPK, TSH D.) CBC, LFTs E.) SCr, LFTs, CPK

C.) SCr, LFTs, CPK, TSH In addition to baseline SCr (for renally adjusted statins), CPK, and LFTs, measuring TSH is optimal because hypothyroidism is a factor in statin-induced myalgias.

A 69-year-old man has a new prescription for Neurontin 300 mg PO 3 times daily. Which of the following monitoring parameters should be monitored during treatment? A.) Complete Blood Count B.) Serum Sodium C.) Serum creatinine D.) EKG E.) AST and ALT

C.) Serum creatinine Correct answer: C Neurontin (gabapentin) is an anticonvulsant approved by the FDA for seizures and postherpetic neuralgia. It is renally excreted as unchanged drug and, therefore, should be used in caution in patients with decreased renal function.Renal function should be monitored throughout therapy.

A 15-year-old patient (40 kg) with cystic fibrosis is admitted to the hospital secondary to an acute pulmonary exacerbation. Home medications include Creon as directed, TOBI nebulization, ADEK once daily, and dornase alfa (once daily via nebulization). She is started on empiric ceftazidime 2 g IV q8h and tobramycin 130 mg IV q8h. Which of the following should be ordered in this patient? A.) Serum tobramycin peak concentration B.) Serum tobramycin trough concentration C.) Serum tobramycin peak and trough concentrations D.) Sputum ceftazidime concentration E.) Sputum ceftazidime and tobramycin concentrations

C.) Serum tobramycin peak and trough concentrations Therapeutic drug monitoring is a critical part of the overall therapeutic plan in patients with cystic fibrosis. Patients with cystic fibrosis exhibit altered pharmacokinetic parameters of aminoglycosides, primarily increased clearance and greater volumes of distribution. When using conventional (rather than once-daily) dosing, obtain tobramycin peak concentrations to ensure the dose being given is sufficient to reach concentrations of 8-12 mcg/mL, and obtain trough concentrations to ensure adequate renal clearance (cystic fibrosis patients receive higher mg/kg doses).

Which of the following antacids is considered the most potent in its neutralizing capacity? A.) Calcium carbonate B.) Aluminum hydroxide C.) Sodium bicarbonate D.) Magnesium hydroxide E.) Magnesium chloride

C.) Sodium bicarbonate Sodium is more potent than calcium, which is more potent than magnesium, which is more potent than aluminum.

Which of the following tests does not have to be completed on a high-risk CSP that will be administered by intravascular injection and is prepared in a lot size of 30 single-dose vials before release to a patient? A.) Bacterial endotoxin test B.) Visual inspection C.) Sterility test D.) Verification of the sterilizing filter integrity E.) LAL test

C.) Sterility test The bacterial endotoxin test (LAL test), visual inspection test, and bubble point test should all be completed before the CSP is dispensed. Because the sterility test takes 14 days, the preparation may be dispensed before the results are known. However, a system to recall the CSP must be in place if it does not meet the test's requirement.

With respect to over-the-counter drug packaging, a package having one or more indicators or barriers to entry that, if breached or missing, can reasonably be expected to provide visible evidence to consumers that tampering has occurred is which of the following? A.) Tamper-proof packaging B.) Tamper-resistant packaging C.) Tamper-evident packaging D.) Tamper-protected packaging

C.) Tamper-evident packaging The applicable FDA regulation is 21 CFR 211.132.

A 30-year-old male is being treated with tapentadol after bunionectomy surgery. Which of the following statements is correct? A.) Tapentadol is metabolized to two active metabolites. B.) Tapentadol is used for the management of mild to moderate chronic pain. C.) Tapentadol is mainly metabolized by glucuronidation. D.) Tapentadol causes QT prolongation and torsades de pointes. E.) Tapentadol may be administered as a patch, tablet, or injection.

C.) Tapentadol is mainly metabolized by glucuronidation. EXPLANATION: Tapentadol is an oral opioid indicated for treatment of acute moderate to severe pain. It is metabolized to inactive metabolites mainly through glucuronidation.

Which infectious agent is responsible for smallpox? A.) Bacillis anthracis B.) Clostridium botulinum C.) Variola virus D.) Vibrio cholerae E.) Yersina pestis

C.) Variola virus Smallpox is caused by the Variola virus for which there is a vaccine. Bacillis anthracis causes anthrax, Clostridium botulinum causes botulism, Vibrio cholerae causes cholera, and Yersina pestis causes plague. These latter infections are typically treated with antibiotics and supportive therapy except for botulism which responds to a specific anti-toxin.

M. R. was started on tenofovir/emtricitabine, darunavir, ritonavir, and raltegravir 3 weeks ago. Her serum creatinine before starting medications was 1.1 mg/dL. Today, her serum creatinine is 1.8 mg/dL. Which one of her medications could be causing this? A.) Darunavir B.) Raltegravir C.) Tenofovir D.) Ritonavir E.) Emtricitabine

C.) Tenofovir Tenofovir rarely can cause renal insuffiency, including the Fanconi syndrome, which is characterized by hypophosphatemia, hypouricemia, proteinuria, norm-oglycemic glycosuria, and in some cases, acute renal failure. Patients with underlying renal insuffiency are at higher risk. CrCl should be monitored in patients. Dose adjustments are necessary when CrCl is less than 50 mL/min.

Which of the following drugs is an anabolic agent that is indicated for high-risk patients and contraindicated in patients with a history of bone cancer or radiation to the bone? A.) Ibandronate B.) Denosumab C.) Teriparatide D.) Calcitonin

C.) Teriparatide EXPLANATION: Teriparatide (Forteo) was associated with an increased risk of osteosarcoma in some animal studies. Therefore, it is contraindicated in anyone at risk of bone cancer, including children and adolescents. Its duration of use is indicated for a period of only 2 years.

Which of the following statements is true about dofetilide? A.) The target plasma concentration is 0.5-1.0 ng/mL. B.) Pulmonary fibrosis occurs with prolonged use. C.) The dose is adjusted according to creatinine clearance. D.) Bradycardia and heart block occur with high doses. E.) Blue-gray skin discoloration occurs in areas of the body exposed to the sun.

C.) The dose is adjusted according to creatinine clearance. EXPLANATION: Dofetilide is primarily eliminated by the kidneys; therefore, the dose is adjusted according to creatinine clearance.

A 76-year-old male with a past medical history of hypertension, myocardial infarction one year ago, and normal left ventricular ejection fraction is receiving lisinopril and chlorthalidone daily. He now presents to the emergency room with a 2-week history of palpitations, dizziness, leg swelling, and weight gain. The ECG reveals atrial fibrillation with a ventricular rate of 140 bpm. After controlling the ventricular rate, the medical team wants to cardiovert the patient into normal sinus rhythm. Which of the following is correct regarding cardioverting this patient? A.) The patient should receive aspirin for 1 week before cardioversion and 2 weeks afterward. B.) The patient should receive clopidogrel for 3 weeks before cardioversion and 4 weeks afterward. C.) The patient should receive dabigatran for at least 3 weeks before cardioversion and 4 weeks afterward. D.) The patient should receive aspirin plus clopidogrel for 3 weeks before cardioversion and 4 weeks afterward. E.) Antithrombotic therapy is not indicated in this patient.

C.) The patient should receive dabigatran for at least 3 weeks before cardioversion and 4 weeks afterward. Because the patient appears to have been in atrial fibrillation for at least 2 weeks by history, significant risk of thromboembolism exists during conversion to sinus rhythm. Current guidelines (2014) call for anticoagulation with either dabigatran or warfarin (INR 2-3), dabigatran, rivaroxaban, or apixaban for 3 weeks before cardioversion followed by at least 4 weeks of anticoagulation postcardioversion. Antiplatelet agents are not shown to be effective in this setting.

When requesting a drug sample, a prescriber must provide which of the following items? A.) A list of patients who will receive the samples B.) A list of the diagnoses for which the samples will be used C.) The prescriber's actual signature D.) A statement confirming that the prescriber will not sell these samples E.) All of the above

C.) The prescriber's actual signature The applicable section of the federal FDCA is 21 USC 353.

Which of the following statements is correct? A.) The rabbit test and the LAL test are the same test. B.) LAL reagent will determine the fever-producing potential of the pyrogens. C.) There are two types of techniques for the BET: the gel-clot technique and the photometric technique. D.) The CSP being tested has no effect on the test. E.) All CSPs may be tested using the rabbit test.

C.) There are two types of techniques for the BET: the gel-clot technique and the photometric technique. The pyrogen test, also known as the rabbit test, determines the fever-producing potential of the pyrogens. The BET is also known as the LAL test. The drug product can inhibit or enhance the gel formation in the BET.

Choose the correct statement. A.) The rabbit test and the LAL test are the same test. B.) LAL reagent will determine the fever-producing potential of the pyrogens. C.) There are two types of techniques for the bacterial endotoxin test: the gel clot technique and the photometric technique. D.) The CSP being tested has no effect on the test. E.) All CSPs may be tested by the rabbit test.

C.) There are two types of techniques for the bacterial endotoxin test: the gel clot technique and the photometric technique. The pyrogen test, also known as the rabbit test, determines the fever-producing potential of the pyrogens. The bacterial endotoxin test is also known as the LAL test. The drug product can inhibit or enhance the gel formation in the BET.

C. D. is a 72-year-old female who presents to the emergency room with burning pain in her chest. Her past medical history includes breast cancer s/p surgery and radiation that is in full remission, diabetes, and obesity. She began smoking at age 22 and currently smokes a pack of cigarettes per day. Upon presentation to the hospital, her BP was 136/75 mm Hg, HR 75, temperature 97°F, and respiratory rate 14. Her ECG is consistent with NSTEMI. Pertinent laboratory values include the following: K 3.8 mmol/L; SCr 0.9 mg/dl; troponin × 3: 0.75, 1.35, 2.7; LDL 120 g/dL. She received nitroglycerin SL × three doses and aspirin 81 mg chew tabs × 4 in the ambulance What is the most appropriate initial drug regimen for this patient in which medical management strategy is preferred (ie. PCI is NOT planned)? A.) UFH 60 unit/kg bolus then 12 units/kg/h IV infusion; clopidogrel 300 mg po × 1, then 75 mg po daily; metoprolol 5 mg IV × 3 B.) Enoxaparin 1 mg/kg SQ q12h; clopidogrel 300 mg po × 1, then 75 mg po daily; metoprolol 5 mg IV × 3 C.) UFH 60 unit/kg bolus, then 12 units/kg/h IV infusion; clopidogrel 300 mg po × 1, then 75 mg po daily; metoprolol 25 mg po bid D.) Enoxaparin 40 mg SQ daily; clopidogrel 300 mg po × 1, then 75 mg po daily; metoprolol 25 mg po bid E.) UFH 60 unit/kg bolus, then 12 units/kg/h IV infusion; clopidogrel 300 mg po × 1, then 75 mg po daily; isosorbide mononitrate 30 mg po daily

C.) UFH 60 unit/kg bolus, then 12 units/kg/h IV infusion; clopidogrel 300 mg po × 1, then 75 mg po daily; metoprolol 25 mg po bid According to the 2012 ACCF/AHA Focused Update of the Guideline for the Management of Patients with UA/NSTEMI, the correct initial regimen for the treatment of NSTEMI is an anticoagulant, aspirin, clopidogrel, and oral beta-blocker. Choice C contains all of the appropriate medications. Choices A and B are incorrect because the metoprolol is IV. IV beta-blockers are second-line therapy to beta-blockers by mouth and are indicated only in patients with hypertension who do not have symptoms of cardiogenic shock upon presentation. Choice D is incorrect because the dose of enoxaparin is too low. The correct dose of enoxaparin is 1 mg/kg SQ q12h (reduce to once daily in patients with CrCl < 30 mL/min). Choice E is incorrect because the regimen has no beta-blocker.

Question 7 of 20 Next Question|Previous Question A 70 year old woman presents with extreme fatigue, pallor, and mild glossitis. She is noted to have a significant megaloblastic anemia with a MCV of 110 fL. Her serum homocysteine and methylmalonic acid are both elevated. Her past medical history is significant for hypertension and peptic ulcer disease for which she has intermittently taken histamine-2 antagonists and proton pump inhibitors for the past 10 years. She also takes vitamin D/calcium supplements for mild osteoporosis, a multivitamin, vaginal estrogen cream, and hydrochlorthiazide. Which of the following treatment regimens would be appropriate for this patient? A.) Vitamin B12 100 mcg po daily B.) Vitamin B12 1,000 mcg IM/SC/IV every 3 months C.) Vitamin B12 1,000 mcg IM/SC/IV weekly for 4 to 6 weeks D.) Folate 5 mg po daily

C.) Vitamin B12 1,000 mcg IM/SC/IV weekly for 4 to 6 weeks Vitamin B12 1,000 mcg IM/SC/IV weekly for 4 to 6 weeks. If she had neurologic symptoms, a more aggressive regimen such as 1000 mcg IM/SC/IV daily for 1 week, then 1000 mcg weekly for the next 6 weeks would be indicated. Large oral doses are better reserved for maintenance therapy.

A 42-year-old male (body weight = 70 kg) on hemodialysis three times per week receives epoetin alfa for treatment of anemia. He has been stable on an epoetin dose of 4,000 units IV three times per week with an average hemoglobin of 11 g/dL (hematocrit of 33%). Over the past 3 months, his hemoglobin has dropped to 9 g/dL. Iron indices reveal the following: serum ferritin 78 ng/mL and TSat 12%. The best initial treatment for this patient is to A.) increase the dose of epoetin alfa to maintain a hemoglobin of 11-12 g/dL (hematocrit of 33-36%). B.) administer IV iron (sodium ferric gluconate) at a maintenance dose of 125 mg/wk. C.) administer a 1 g total dose of IV iron in divided doses. D.) begin oral ferrous sulfate 325 mg tid.

C.) administer a 1 g total dose of IV iron in divided doses. This patient is iron deficient, as indicated by his low serum ferritin (< 100 ng/mL) and TSat (< 20%). No change in epoetin alfa should be made until the iron deficiency is corrected (this is the leading cause of resistance to ESAs). This patient will require a full course of iron (1 g administered intravenously in divided doses with each dialysis session) as opposed to a maintenance dose, which should be administered once the patient is iron replete. Sodium ferric gluconate may be administered in doses of 125 mg per dialysis session for eight sessions to give the total 1 g dose (iron sucrose would be administered in 100 mg increments over 10 hemodialysis sessions). Ferumoxytol would be administered as two 510 mg doses given 3-8 days apart. Absorption of oral iron is poor, making IV iron preferred in this hemodialysis patient.

Agents that may be used in the enteric coating of tablets include A.) hydroxypropyl methylcellulose. B.) carboxymethylcellulose. C.) cellulose acetate phthalate. D.) all of the above. E.) none of the above.

C.) cellulose acetate phthalate. An enteric-coated tablet has a coating that remains intact in the stomach but dissolves in the intestine when the pH exceeds 6. Enteric coating materials include cellulose acetate trimellitate, cellulose acetate phthalate, polyvinyl acetate phthalate, and hydroxypropyl methylcellulose phthalate.

A life-supporting or life-sustaining device that requires FDA approval before it can be marketed in interstate commerce in the United States is a A.) class I device. B.) class II device. C.) class III device. D.) class IV device. E.) class V device.

C.) class III device. The federal FDCA provides for three classes of medical devices: class I, class II, and class III. Generally, class I devices are not subject to premarket approval, and class II devices are subject to performance standards. Class III devices are life-supporting or life-sustaining devices and require premarket approval by FDA. The applicable section of the federal FDCA is 21 USC 360c.

A patient who is receiving chronic phenytoin therapy is hospitalized for an elective surgical procedure. Admission labs note that the patient has a phenytoin concentration of 8 mcg/mL (therapeutic range: 10-20 mcg/mL) and an albumin concentration of 3 g/dL. Phenytoin: F = 0.2-0.9, CL-variable, < 1% excreted unchanged in the urine, 88-93% bound to plasma proteins (primarily albumin). Given this information and the therapeutic range of phenytoin, you would recommend that the physician A.) decrease the dose of phenytoin, because high-extraction drugs (e.g., phenytoin) exhibit increased unbound concentrations with increases in fraction unbound in the plasma. B.) increase the dose rate of phenytoin, because low-extraction drugs (e.g., phenytoin) exhibit increased CL with increases in fraction unbound in the plasma. C.) not change the dose rate of phenytoin because low-extraction drugs (e.g., phenytoin) do not exhibit changes in unbound concentrations with increases in fraction unbound in the plasma. D.) not change the dose rate of phenytoin because low-extraction drugs (e.g., phenytoin) exhibit equal and offsetting changes in CL and F with increases in fraction unbound in plasma.

C.) not change the dose rate of phenytoin because low-extraction drugs (e.g., phenytoin) do not exhibit changes in unbound concentrations with increases in fraction unbound in the plasma. Phenytoin must be a low-extraction drug because its bioavailability is as high as 90%. The large range in F is due to variability in the absorption of the drug. You know it is not high extraction because if it were, you could never get an F of 90%. Assume ER < 0.1, f up = 0.07-0.12. Phenytoin is a low-extraction drug and restrictively cleared (assume all low-ER drugs are restrictively cleared for the purposes of this course). Normal albumin range: 3.5-5 g/dL. Thus, the patient probably has increased f up because of decreased albumin. F up (according to the following equation) would be 0.14 (slightly elevated). Because phenytoin is a low-extraction drug, CL is dependent on f up and CL int. Increased f up would lead to increased CL and decreased total plasma concentrations (thus C p of 8, which is below therapeutic range). However, unbound concentrations would be predicted to be normal (therapeutic) even though total concentration is low. You would not recommend an increase in the patient's phenytoin dose because it may result in toxic concentrations. Obtaining free phenytoin plasma concentration, if available from the hospital's lab, may be reasonable to document therapeutic concentrations.

Which of the following statements are true regarding Zyloprim? A.) it works to decrease the formation of uric acid by accelerating the activity of xanthine oxidase. B.) it does not require dosage adjustment in patients with renal insufficiency. C.) skin reactions, including Stevens-Johnson syndrome, have been reported with its use. D.) it should be used for the treatment of an acute gouty arthritis attack.

C.) skin reactions, including Stevens-Johnson syndrome, have been reported with its use. Zylorprim works to decrease the formation of uric acid by inhibiting xanthine oxidase not accelerating its activity. It does require dosage adjustment in patients with renal Skin reactions, including Stevens-Johnson syndrome, have been reported with its use. It should not be used for the treatment of an acute gouty arthritis attack.

TR is a 72-year-old man with diabetes and COPD. He is currently hospitalized and on a ventilator for community-acquired pneumonia. Which of the following regimens would be most appropriate dietary therapy? A) 50% carbohydrate, 30% fat, 20% protein B) 65% carbohydrate, 30% fat, 35% protein C) 35% carbohydrate, 25% fat, 40% protein D) 35% carbohydrate, 50% fat, 15% protein

D) 35% carbohydrate, 50% fat, 15% protein Answer d is correct. The low carbohydrate diet has been supplemented by a higher fat content. This will help decrease the CO2 load while providing adequate calories. Answer a is incorrect. Patients with pulmonary disease should be given a low carbohydrate diet to decrease CO2 production as well as to decrease the risk of hyperglycemia in a diabetic patient. Answer b is incorrect. See answer a above. Answer c is incorrect. The carbohydrate load is appropriate, but the caloric deficit should be replaced with fat.

Sancuso transdermal patch delivers antiemetic activity for: A) 12 hours B) 24 hours C) 3 days D) 7 days

D) 7 days Sancuso provides antiemetic activity over 7 days.

Dofetilide is indicated in which of the following? A) A patient initiated in an outpatient setting B) A patient with creatinine clearance (CrCl) <20 mL/min C) A baseline QTc 510 millisecond D) A patient with LV hypertrophy

D) A patient with LV hypertrophy Answer d is correct. Dofetilide is safer in patients with structural heart disease and ventricular hypertrophy. Answer a is incorrect. Dofetilide must be initiated in an inpatient setting under telemetry. Answer b is incorrect. Dofetilide is contraindicated in patients with a CrCl <20 mL/min. Answer c is incorrect. Dofetilide is contraindicated in patients with a QTc >440 millisecond (or 500 millisecond with a ventricular conduction abnormality).

PR is an 82-year-old woman who is status post drug-eluting stent placement following presentations with a STEMI. She has a past medical history significant for hypertension, dyslipidemia, and hypothyroidism. She has no known drug allergies. Which of the following is the best choice of long-term antiplatelet therapy? A) ASA 325 mg orally daily B) ASA 325 mg plus Ticagrelor 90 mg orally twice daily C) Ticagrelor 180 mg orally twice daily D) ASA 81 mg plus clopidogrel 75 mg orally daily

D) ASA 81 mg plus clopidogrel 75 mg orally daily Answer d is correct. This regimen of low-dose ASA and a thienopyridine represents the best option in this patient. ASA plus clopidogrel should be continued for at least a year following drug-eluting stent placement. Answer a is incorrect. Following treatment with drug-eluting coronary artery stents, DAPT is indicated (ASA plus thienopyridine). Answer b is incorrect. Although use of low-dose ASA is appropriate, Ticagrelor is only approved to be given with 75-100 mg/d of ASA. Answer c is incorrect. Following treatment with coronary artery stents, DAPT is indicated (ASA plus thienopyridine). The dose for ticagrelor is 90 mg orally twice daily.

You are an oncology pharmacist counseling a parent and their child undergoing treatment for pediatric ALL. Patient will be treated with a Children's Oncology Group (COG) protocol. All of the following adverse events should be discussed with them regarding asparaginase except? A) Hyperglycemia B) Risk of allergic reactions C) Potential for bleeding D) Alopecia

D) Alopecia Asparaginase is not associated with alopecia. Choice (A) is incorrect. Hyperglycemia/glucose intolerance has been reported in ~10% of patients which receive asparaginase. Choice (B) is incorrect. Hypersensitivity reactions are a major concern with patients which receive asparaginase. A test dose is recommended with first dose of therapy. Practitioners are instructed to have epinephrine, diphenhydramine, and hydrocortisone available at bedside due to risk. Choice (C) is incorrect. Thrombosis, fatal bleeding, consumption coagulopathy, and intracranial hemorrhage have all been reported.

Which of the following side effects are associated with flutamide? A) Constipation B) Increased libido C) Rash D) Diarrhea

D) Diarrhea Flutamide causes diarrhea (D). In order to minimize diarrhea, it is recommended that you initiate flutamide at 250 mg daily then titrate to 250 mg tid. Flutamide does not cause constipation (A). Flutamide is not associated with increased libido (B). Rash (C) is not a side effect of flutamide therapy.

A 58-year-old male patient with heart failure, erectile dysfunction, and CAD with stable angina underwent testing for PAD after having findings suggestive of PAD on physical examination at his last primary care office visit. Which of the following diagnostic tests is most appropriate to confirm a PAD diagnosis in this patient? A) Fecal occult blood test B) Toe-brachial index C) Cardiac catheterization D) Ankle-brachial index

D) Ankle-brachial index Answer d is correct. Resting ankle-brachial index is the preferred test to confirm diagnosis of PAD in patients who have findings suggestive of PAD upon examination or who are high risk of PAD. Answer a is incorrect. Fecal occult blood test is used to screen for colorectal cancer or to test for blood in stool samples. Answer b is incorrect. Toe-brachial index is only recommended as an alternative to ankle-brachial index as a PAD diagnostic test. Answer c is incorrect. Cardiac catheterization can diagnose atherosclerotic arterial disease in the heart vasculature, not in the lower extremities.

JF is a 57-year-old woman who presents to the hospital with ADHF. Vital signs include BP 105/67 mm Hg, HR 83 beats/min, and RR 21. Physical examination reveals 12 cm JVD elevated, +S3, bilateral rales on auscultation, abdominal ascites, and 4+ bilateral edema extending to her thighs. Chest radiograph reveals pulmonary edema and pleural effusions. Hemodynamic measurements obtained by PAC include PCWP 29 mm Hg, CI 1.7 L/min/m2, and SVR 700. Her laboratory values are all normal, except sodium 132 mmol/L, BUN 49 mg/dL, and SCr 2.1 mg/dL (baseline BUN/SCr 32 and 0.9). Her drugs on admission include enalapril 10 mg twice daily, carvedilol 12.5 mg twice daily, bumetanide 2 mg twice daily, hydralazine 50 mg three times daily, isosorbide dinitrate 40 mg three times daily, and aspirin 81 mg/d. What is the desired PCWP for JF? Select all that apply. A) Less than 2.2 L/min/m2 B) Greater than 2.2 L/min/m2 C) Between 6 and 12 mm Hg D) Between 15 and 18 mm Hg

D) Between 15 and 18 mm Hg

JF is a 57-year-old woman who presents to the hospital with ADHF. Vital signs include BP 105/67 mm Hg, HR 83 beats/min, and RR 21. Physical examination reveals 12 cm JVD elevated, +S3, bilateral rales on auscultation, abdominal ascites, and 4+ bilateral edema extending to her thighs. Chest radiograph reveals pulmonary edema and pleural effusions. Hemodynamic measurements obtained by PAC include PCWP 29 mm Hg, CI 1.7 L/min/m2, and SVR 700. Her laboratory values are all normal, except sodium 132 mmol/L, BUN 49 mg/dL, and SCr 2.1 mg/dL (baseline BUN/SCr 32 and 0.9). Her drugs on admission include enalapril 10 mg twice daily, carvedilol 12.5 mg twice daily, bumetanide 2 mg twice daily, hydralazine 50 mg three times daily, isosorbide dinitrate 40 mg three times daily, and aspirin 81 mg/d. What is the desired PCWP for JF? Select all that apply. A) Less than 2.2 L/min/m2 B) Greater than 2.2 L/min/m2 C) Between 6 and 12 mm Hg D) Between 15 and 18 mm Hg

D) Between 15 and 18 mm Hg Answer d is correct. For patients with left ventricular dysfunction, the optimal PCWP is 15 to 18 mm Hg as maintaining mildly elevated filling pressures is essential to optimizing the Starling curve and assuring optimal cardiac output. Answers a and b are incorrect. These numbers represent cardiac index. A cardiac index of greater than 2.2 L/min/m2 is necessary to maintain adequate tissue perfusion. Answer c is incorrect. For patients with normal cardiac function, the desired PCWP is between 6 and 12 mm Hg. Most patients with heart failure will require higher than normal ventricular filling pressures in order to maximize cardiac output.

PT is a 33-year-old HIV patient. He has a high viral load and low CD4. He has been nonadherent with medication and provider appointments. PT is admitted to the medical center for a change in mental status. He is undergoing a complete work-up to identify the cause. An India Ink stain is reported as positive. Which one of the following organisms is likely to manifest a positive India ink stain on a CSF sample? A) Candida albicans B) Candida glabrata C) Aspergillosis fumigatus D) Candida neoformans

D) Candida neoformans Answer d is correct. The India ink stain adheres to the capsule around C. neoformans. Answers a, b, and c are incorrect.Cryptococcus is evaluated via an India ink stain. The other organisms do not respond to the India ink stain.

DC is a 59-year-old Caucasian man who reports to his primary care physician complaining of 2-week history of fatigue and fever. A CBC with differential reveals an elevated WBC (25,000 U/L) and profound thrombocytopenia (platelets 30,000 U/L). His peripheral blood has 20% blasts. A bone marrow biopsy was performed and DC was diagnosed with acute myeloid leukemia (AML-M4). Molecular testing revealed—FLT3 negative, NPML1 negative, C-KIT negative. What toxicities should DC be counseled on prior to receiving high-dose cytarabine? A) Infusion-related reactions, paralytic ileus, cardiotoxicity B) Cerebellar toxicity, peripheral neuropathy, infusion-related reactions C) Nausea, peripheral neuropathy, ocular toxicity D) Cerebellar toxicity, nausea, ocular toxicity

D) Cerebellar toxicity, nausea, ocular toxicity

DC is a 59-year-old Caucasian man who reports to his primary care physician complaining of 2-week history of fatigue and fever. A CBC with differential reveals an elevated WBC (25,000 U/L) and profound thrombocytopenia (platelets 30,000 U/L). His peripheral blood has 20% blasts. A bone marrow biopsy was performed and DC was diagnosed with acute myeloid leukemia (AML-M4). Molecular testing revealed—FLT3 negative, NPML1 negative, C-KIT negative. What toxicities should DC be counseled on prior to receiving high-dose cytarabine? A) Infusion-related reactions, paralytic ileus, cardiotoxicity B) Cerebellar toxicity, peripheral neuropathy, infusion-related reactions C) Nausea, peripheral neuropathy, ocular toxicity D) Cerebellar toxicity, nausea, ocular toxicity

D) Cerebellar toxicity, nausea, ocular toxicity .Cytarabine is associated with cerebellar, nausea, and ocular toxicity particularly in the high-dose (g/m2) setting. Cerebellar toxicity is characterized by nystagmus, slurred speech, and ataxia. HDAC is listed as a moderate emetogenic risk (30%-90% frequency of emesis) agent by NCCN guidelines. Ocular toxicity manifests itself by conjunctivitis; this can be prevented with the use of prophylactic dexamethasone eye drops. Answer a is incorrect.Cytarabine is not associated with infusion related reactions, paralytic ileus, or cardiotoxicity. Answer b is incorrect.Cytarabine is not associated with peripheral neuropathy or infusion related reactions. Answer c is incorrect.Cytarabine is not associated with peripheral neuropathy.

Which of the following is an antidepressant that has a black box warning for rare cases of liver failure? A) Venlafaxine B) Bupropion C) Imipramine D) Nefazodone

D) Nefazodone The FDA-approved nefazodone (D) labeling includes a black box warning describing rare cases of liver failure. Because of the potential for hepatic injury associated with nefazodone use, treatment should not be initiated in individuals with active liver disease or elevated baseline serum transaminases. Venlafaxine (A), bupropion (B), and imipramine (C) do not have a black box warning for liver failure.

A 61-year-old woman presents to clinic with newly diagnosed squamous cell carcinoma of the lung, which metastasize to the liver and bone. The EGFR status reveals no mutations. All of her laboratory values are within normal limits, and reports that she is extremely active. Based upon this information, which of the following treatments would be rational? A) Best supportive care B) Erlotinib C) Cisplatin + pemetrexed D) Cisplatin + gemcitabine

D) Cisplatin + gemcitabine Patients with squamous cell carcinoma of the lung have a significantly longer survival when treated with cisplatin + gemcitabine as compared to cisplatin + pemetrexed. Answer (A) is incorrect. Because of patient's good performance status of either a 0 or 1, she should receive active therapy. Answer (B) is incorrect. Because the tumor does not exhibit any EGFR mutations, patient is unlikely to benefit from erlotinib. Answer (C) is incorrect. Patients with squamous cell carcinoma of the lung have a significantly shorter survival when treated with cisplatin + pemetrexed as compared to cisplatin + gemcitabine.Patients with squamous cell carcinoma of the lung have a significantly longer survival when treated with cisplatin + gemcitabine as compared to cisplatin + pemetrexed. Answer (A) is incorrect. Because of patient's good performance status of either a 0 or 1, she should receive active therapy. Answer (B) is incorrect. Because the tumor does not exhibit any EGFR mutations, patient is unlikely to benefit from erlotinib. Answer (C) is incorrect. Patients with squamous cell carcinoma of the lung have a significantly shorter survival when treated with cisplatin + pemetrexed as compared to cisplatin + gemcitabine.

What is the best treatment option for a 61-year-old man who is chemotherapy naïve and was recently diagnosed with extensive-stage small-cell lung cancer? A) Carboplatin, pemetrexed, plus pembrolizumab B) Cisplatin plus gemcitabine C) Carboplatin plus paclitaxel D) Cisplatin plus etoposide

D) Cisplatin plus etoposide Answer d is correct. For extensive-stage SCLC to maximize survival, one should utilize cisplatin plus etoposide regimen. Answer a is incorrect. Carboplatin, pemetrexed, and pembrolizumab can be used together for the treatment of metastatic non-squamous NSCLC. Answer b is incorrect. Cisplatin and gemcitabine are the regimens typically used in squamous NSCLC. Answer c is incorrect. Carboplatin and paclitaxel are the regimens utilized in NSCLC not in SCLC.

YQ is a 6-year-old patient with a severe case of acute bacterial sinusitis. She has a type I (immediate) allergic reaction to penicillin. What antibiotic may be utilized to treat the sinus infection? A) Doxycycline B) Moxifloxacin C) Clindamycin D) Clarithromycin

D) Clarithromycin Clarithromycin (D) may be utilized in patients who are allergic to penicillin and is effective in treating sinus infections. While patients allergic to penicillins may take tetracycline antibiotics (A), YQ cannot because she is less than 8 years of age. While patients allergic to penicillin antibiotics may take quinolones (B), YQ cannot because of her age. While patients allergic to penicillin antibiotics may take clindamycin (C), clindamycin does not provide coverage against gram-negative bacteria. Clindamycin would be useful if YQ had a S. pneumoniae infection; however, we do not know the pathogen YQ is infected with and it could be the gram-negative H. influenzae or M. catarrhalis.

Which of the following is an antibiotic with the following mechanism of action: binds to components of the cell membrane and causes rapid depolarization, inhibiting intracellular synthesis of DNA, RNA, and protein? A) Piperacillin-tazobactam B) Imipenem-cilastatin C) Clindamycin D) Daptomycin E) Vancomycin

D) Daptomycin Daptomycin (D) binds to components of the cell membrane and causes rapid depolarization, inhibiting intracellular synthesis of DNA, RNA, and protein. Piperacillin/tazobactam (A) inhibits bacterial cell wall synthesis by binding to one or more of the penicillin-binding proteins. Tazobactam binds and inhibits beta-lactamases that inactivate piperacillin, resulting in piperacillin having an expanded spectrum of activity. Imipenem inhibits bacterial cell wall synthesis by binding to penicillin-binding proteins, which inhibits the final transpeptidation step of peptidoglycan synthesis. Cilastatin prevents renal metabolism of imipenem by inhibition of dehydropeptidase along the brush border of the renal tubules (B). Clindamycin (C) reversibly binds to 50s ribosomal subunits preventing peptide bond formation, thus inhibiting bacterial protein synthesis. Vancomycin (E) inhibits bacterial cell wall synthesis by blocking glycopeptide polymerization through binding of the D-alanyl-D-alanine portion of cell wall precursor.

Which of the following is an antibiotic with the following mechanism of action: binds to components of the cell membrane and causes rapid depolarization, inhibiting intracellular synthesis of DNA, RNA, and protein? A) Piperacillin-tazobactam B) Imipenem-cilastatin C) Clindamycin D) Daptomycin E) Vancomycin

D) Daptomycin Daptomycin (D) binds to components of the cell membrane and causes rapid depolarization, inhibiting intracellular synthesis of DNA, RNA, and protein. Piperacillin/tazobactam (A) inhibits bacterial cell wall synthesis by binding to one or more of the penicillin-binding proteins. Tazobactam binds and inhibits beta-lactamases that inactivate piperacillin, resulting in piperacillin having an expanded spectrum of activity. Imipenem inhibits bacterial cell wall synthesis by binding to penicillin-binding proteins, which inhibits the final transpeptidation step of peptidoglycan synthesis. Cilastatin prevents renal metabolism of imipenem by inhibition of dehydropeptidase along the brush border of the renal tubules (B). Clindamycin (C) reversibly binds to 50s ribosomal subunits preventing peptide bond formation, thus inhibiting bacterial protein synthesis. Vancomycin (E) inhibits bacterial cell wall synthesis by blocking glycopeptide polymerization through binding of the D-alanyl-D-alanine portion of cell wall precursor.

SL is a 69-year-old man who was recently diagnosed with prostate cancer and is initiated on a hormone agent for the first time for androgen deprivation. The pharmacist tells the patient he should not experience tumor flare with this new medication. Which of the following agents was this patient initiated on? A) Leuprolide B) Goserelin C) Triptorelin D) Degarelix E) Enzalutamide

D) Degarelix Answer d is correct.Degarelix is a GnRH antagonist and therefore tumor flare is not expected with this agent due to its mechanism of action. Answer a is incorrect. Leuprolide is an LH-RH agonist and therefore a disease flare-up during the first week may be expected. Answer b is incorrect. Goserelin is an LH-RH agonist and therefore a disease flare-up during the first week may be expected. Answer c is incorrect. Triptorelin is an LH-RH agonist and therefore a disease flare-up during the first week may be expected. Answer e is incorrect.Enzalutamide is not an LH-RH or a GnRH agent, and therefore tumor flare is not an expected side effect. Additionally, this agent is only approved for use in castrate resistant prostate cancer, which this patient does not have.

FG is a 63-year-old man with hormone-refractory metastatic prostate cancer. He has several other comorbid diseases including CHF, diabetes, and hypertension. Which of the following chemotherapy agents is the most appropriate for this patient? A) Estramustine + docetaxel B) Mitoxantrone + prednisone C) Estramustine + docetaxel + mitoxantrone + prednisone D) Docetaxel + prednisone

D) Docetaxel + prednisone he correct answer is (D). Docetaxel + prednisone is the recommended chemotherapy regimen in patients with hormone-refractory metastatic prostate cancer due to its overall survival benefits. Docetaxel should be administered at 75 mg/m2 every 3 weeks, combined with prednisone 5 mg tid. The most common adverse events reported with this regimen are nausea, alopecia, and bone marrow suppression. In addition, fluid retention and peripheral neuropathy, known effects of docetaxel, are observed. Docetaxel is hepatically eliminated; therefore, patients with hepatic impairment may not be eligible for treatment with docetaxel. Answer (A) is incorrect. Although estramustine + docetaxel has survival data, it is not currently recommended first line. Additionally, estramustine would be inappropriate in this patient due to his history of cardiovascular disease. Answer (B) is incorrect. Mitoxantrone + prednisone can be used in hormone-refractory metastatic prostate cancer, although its benefits have yet to be determined. Therefore, it is currently not recommended first line. Additionally, mitoxantrone is associated with cardiotoxicity, and may not be appropriate in a patient with CHF. Answer (C) is incorrect. All four of these agents have not been studied together in combination.

Which of the following cell types plays a critical role in parasitic infections? A) Basophil B) Macrophage C) Plasma cell D) Eosinophil

D) Eosinophil Eosinophils play a major role in parasitic infections.

A patient is admitted to the hospital with seizures that result in a sudden interruption of activities and a blank stare. What is the recommended first-line treatment for this type of seizure disorder? A) Phenytoin B) Felbamate C) Levetiracetam D) Ethosuximide

D) Ethosuximide Answer d is correct. Ethosuximide is the first-line treatment for absence seizures. Answers a, b, and c are incorrect. None of these drugs treat absence seizures.

Select the agent that is administered via intramuscular injection for the treatment of metastatic estrogen receptor positive breast cancer. A) Anastrozole B) Avastin C) Herceptin D) Faslodex E) Femara

D) Faslodex Faslodex is administered by IM injection to women with ER+ metastatic breast cancer. Choice (A) is incorrect. Anastrozole is administered orally to patients with ER+ metastatic breast cancer. Choice (B) is incorrect. Avastin is administered intravenously to patients with breast cancer characterized by an overexpression of HER-2. Choice (C) is incorrect. Herceptin is administered intravenously to patients with breast cancer.

RF is a 62-year-old man with nonischemic cardiomyopathy (LVEF 30%-35%) presenting to the emergency department (ED) with an acute HF exacerbation. His vital signs include BP 155/90 mm Hg, heart rate (HR) 85 beats/min, RR 20, and O2 sat 94% on 4 L/min of oxygen by nasal cannula (NC). Physical examination reveals jugular venous distension (JVD), regular rate and rhythm (RRR), crackles bilaterally at bases, and 2+ bilateral lower extremity edema. He admits to a 12-lb weight gain in the past 2 weeks since his carvedilol dose was increased and reports strict adherence to both dietary restrictions and medications. In the ED, he has already received furosemide 80 mg IV ×1 dose with minimal response in urine output. Pertinent laboratory results include potassium 3.9 mmol/L, BNP 1550 pg/mL, BUN 37 mg/dL, and SCr 1.3 mg/dL (baseline). RF's home medications include lisinopril 10 mg daily, carvedilol 25 mg twice daily, digoxin 0.125 mg/d, and furosemide 80 mg orally twice daily. Which of the following interventions is best for RF on arrival to the intensive care unit (ICU)? A) Dobutamine 2.5 mcg/kg/min infusion B) Milrinone 0.375 mcg/kg/min infusion C) Metolazone 10 mg po now and then daily D) Furosemide 120 mg IV twice daily

D) Furosemide 120 mg IV twice daily Answer d is correct. RF is clearly experiencing fluid overload secondary to acute decompensated heart failure (ADHF). RF has failed an initial dose of IV furosemide but the dose was small. Therefore, it is appropriate to try a higher dose of IV furosemide such as 120 mg IV twice daily. Answers a and b are incorrect. Currently, RF is not exhibiting any signs or symptoms of low cardiac output that would necessitate administration of an inotropic agent such as dobutamine (Answer a) or milrinone (Answer b). Answer c is incorrect. Metolazone is a thiazide-type diuretic, which acts synergistically with loop diuretics for refractory fluid overload. At this time, RF is not refractory to higher furosemide doses.

Which of the following combinations would be most effective to enhance urine production in a patient who has oliguric ARF secondary to ATN? A) Furosemide and ethacrynic acid B) Triamterene and hydrochlorothiazide C) Bumetanide and spironolactone D) Furosemide and metolazone E) Spironolactone and metolazone

D) Furosemide and metolazone The combination of a loop-type (furosemide) and a thiazide diuretic (metolazone) synergistically enhances urine production. By preventing the reabsorption in the loop of Henle, more sodium remains in the tubular lumen and is delivered to the distal tubule. Blocking sodium channels in the distal tubule (thiazides) promotes the excretion of the sodium and water (now from both the loop of Henle and the distal tubule). Answer (A) is incorrect. Both furosemide and ethacrynic acid are loop-type diuretics. They share the same mechanism of action, inhibiting sodium reabsorption in the loop of Henle. The use of two agents with the same mechanism of action would not be expected to produce synergistic effects. Answer (B) is incorrect. Triamterene is a potassium-sparing diuretic and would be relatively contraindicated in oliguria/anuria. Hydrochlorothiazide works in the distal tubule. In oliguria, the delivery of sodium to the distal tubule is decreased and therefore, the efficacy of hydrochlorothiazide would also be decreased. Answer (C) is incorrect. While bumetanide is a loop-type diuretic that would be useful in oliguric ATN, spironolactone would be relatively contraindicated. Spironolactone is an aldosterone antagonist and would therefore increase the elimination of sodium (and water) but enhance the reabsorption of potassium. Answer (E) is incorrect. Spironolactone use may result in hyperkalemia. Metolazone is a thiazide diuretic, and would not be as effective in oliguria.

What is the preferred antiemetic for breakthrough nausea and vomiting? A) Aprepitant B) Dronabinol C) Ondansetron D) Promethazine

D) Promethazine Dopamine antagonists such as promethazine (D) are effective for breakthrough nausea and vomiting for as needed use. Aprepitant (A) should not be prescribed for as needed use. Dronabinol (B) should be reserved for refractory CINV. Ondansetron (C) should not be prescribed for as needed use.

What medication may be used concurrently with high-dose Aldesleukin? A) Dexamethasone B) Indomethacin C) Losartan D) Granisetron

D) Granisetron Granisetron (D) is a 5-HT3 receptor antagonist antiemetic agent and has no reported interaction with high-dose IL-2. Dexamethasone (A) is a corticosteroid. The use of corticosteroids should be avoided in patients receiving treatment with high-dose interleukin-2 (Aldesleukin). Corticosteroids decrease the antitumor effects of IL-2 due to its inhibitory effect on the immune system. Indomethacin (B) is a nonsteroid anti-inflammatory agent. The use of nonsteroid anti-inflammatory agents should be avoided in patients receiving treatment with high-dose interleukin-2 (Aldesleukin). NSAIDs increase the risk of capillary leak syndrome. Losartan (C) is an antihypertensive agent. The use of antihypertensives should be avoided in patients receiving treatment with high-dose interleukin-2 (Aldesleukin). Antihypertensives potentiate the hypotensive effects of IL-2.

JM, a 6-month-old infant, is seeing his pediatrician today in order to receive his 6 month vaccinations. His vaccination records are as follows: Hep B at birth and 2 months; DTaP at 2 and 4 months; Hib (PedvaxHIB) at 2 and 4 months; PCV at 2 and 4 months; IPV at 2 and 4 months. JM does not have any medical conditions or allergies to medications or vaccines. Which vaccines should JM receive today? A) Hep B, RV, DTaP, Hib, PCV, and IPV B) Hep B, RV, DTaP, PCV, and IPV C) Hep B, DTaP, Hib, PCV, and IPV D) Hep B, DTaP, PCV, and IPV

D) Hep B, DTaP, PCV, and IPV At 6 months, JM will need his third dose of Hep B, DTaP, PCV, and IPV.

TN is a 54-year-old man with newly diagnosed stage IV metastatic melanoma. TN has good PS with no comorbid conditions, thought to be perfect candidate for immunotherapy. Which of the following immunotherapy would be treatment of choice for TN? A) Interferon alfa-2b B) Dacarbazine C) Carmustine D) IL-2 E) Paclitaxel

D) IL-2 Answer d is correct. High-dose IL-2 is an immunotherapy that is FDA approved in treatment of metastatic melanoma in selected patients with good PS. Answer a is incorrect. High dose interferon-alfa 2b is an immunotherapy agent that is FDA approved for use in the adjuvant setting for treatment of melanoma in patients who are free of disease (nonmetastatic) but at high risk for systemic recurrence within 56 days of surgery. Answer b is incorrect. Dacarbazine is a chemotherapy agent with FDA approval as single agent for treatment of metastatic melanoma. Answer c is incorrect. Carmustine is a chemotherapy agent used in combination with other chemotherapeutic agents in the Dartmouth regimen for the treatment of Stage IV metastatic melanoma. Answer e is incorrect.Paclitaxel is a chemotherapy agent that has been used in treatment of metastatic melanoma; however, response rate are relatively lower (6%-18%) compared to single agent Dacarbazine.

Select the toxicity that has been associated with the administration of both trastuzumab and bevacizumab. A) Myelosuppression B) Gastrointestinal (GI) perforation C) Alopecia D) Infusion reactions

D) Infusion reactions Answer d is correct. Trastuzumab and bevacizumab are monoclonal antibodies administered by intravenous infusion. Infusion reactions (the onset of chills, fever, changes in blood pressure within 1 hour of administration) have been reported to occur with both agents. Answer a is incorrect. Trastuzumab and bevacizumab are monoclonal antibodies that are rarely associated with myelosuppression. Answer b is incorrect. Although GI perforation is a proven bevacizumab toxicity, it is not associated with trastuzumab. Answer c is incorrect. Monoclonal antibodies are not associated with hair loss.

What type of incontinence can be described as having urinary urgency, frequency, and nocturia along with leakage during exercise? A) Stress urinary incontinence B) Urge urinary incontinence C) Overflow incontinence D) Mixed incontinence E) None of the above

D) Mixed incontinence The above symptoms describe both UUI (urinary urgency, frequency, and nocturia) and SUI (leakage during exercise) and therefore are defined as mixed incontinence because it is more than one type of UI. Choice (A) is incorrect. The above symptoms are not SUI alone. Choice (B) is incorrect. The above symptoms are not UUI alone. Choice (C) is incorrect. The above symptoms are not OI.

A 62-year-old man with a recent diagnosis of prostate cancer presents to oncologist. His oncologist tells him that his prostate cancer has a Gleason score of 3 + 3 or 6. A prostate cancer with a Gleason score of 6 is considered: A) Not differentiated B) Poorly differentiated C) Differentiated D) Moderately differentiated E) Well differentiated

D) Moderately differentiated Answer d is correct. Prostate cancer can be graded systematically according to the histologic appearance of the malignant cell and then grouped into well, moderately, or poorly differentiated grades. Gland architecture is examined and then rated on a scale of 1 (well differentiated) to 5 (poorly differentiated). Two different specimens are examined, and the score for each specimen is added. Poorly differentiated tumors grow rapidly (poor prognosis), while well-differentiated tumors grow slowly (better prognosis). A Gleason score of 5 to 6 is considered moderately differentiated. Answer a is incorrect. The term "not differentiated" is not used in the Gleason scoring system. Answer b is incorrect. A Gleason score of 7 to 10 is considered poorly differentiated. Answer c is incorrect. The term "differentiated" is not used alone in the Gleason scoring system. Answer e is incorrect. Well differentiated is equal to a Gleason score of 2 to 4.

A patient presents to the hospital with symptoms concerning for a bowel obstruction. His past medical history is significant for a liver transplant 3 months ago and he is receiving Myfortic 720 mg po bid. The physician wants to convert the patient from oral Myfortic to IV due to po intolerance resulting from his bowel obstruction. Which of the following would result in comparable plasma concentrations of mycophenolic acid? A) Mycophenolate sodium 720 mg IV bid B) Mycophenolate sodium 1000 mg IV bid C) Mycophenolate mofetil 720 mg IV bid D) Mycophenolate mofetil 1000 mg IV bid

D) Mycophenolate mofetil 1000 mg IV bid CellCept, or mycophenolate mofetil, is available in an oral and IV formulation. When converting a patient from po Myfortic (mycophenolate sodium) to IV therapy, IV mycophenolate mofetil (CellCept) can be used at a therapeutically equivalent dose. Myfortic 720 mg is equivalent to 1000 mg of CellCept. Answers a and b are incorrect. Myfortic, or mycophenolate sodium, is an enteric-coated delayed-release oral formulation and it is not available intravenously. Choice (C) is incorrect. The dosing conversion between mycophenolate sodium and mycophenolate mofetil (CellCept) is not 1:1, but rather 720 mg of mycophenolate sodium is equivalent to 1000 mg of mycophenolate mofetil.

Lupus nephritis is one of the most serious complications of SLE. What other items are included in the diagnostic criteria of SLE? A.) Maculopapular rash B.) GI bleeds C.) Pulmonary hypertension D.) Anemia

D.) Anemia Anemia is commonly seen in SLE and is one of the criteria for diagnosis. Four of the eleven criteria listed in the chapter must be present for diagnosis.

Which of the following is true regarding the stages of syphilis infection? A) The characteristic lesion of primary syphilis is a diffuse rash, usually affecting the palms and soles. B) Manifestations of latent syphilis include regional lymphadenopathy and meningitis. C) Tertiary syphilis is highly transmissible. D) Neurosyphilis can present at any stage of syphilis.

D) Neurosyphilis can present at any stage of syphilis. Answer d is correct. CNS involvement can present at any stage of syphilis. Early neurosyphilis occurs within first few years of infection and usually coexists with primary or secondary syphilis. Late neurosyphilis occurs years to decades after the initial infection and represents a tertiary manifestation of syphilis. Answer a is incorrect. The characteristic lesion of primary syphilis is the chancre (ulcer). The chancre is usually painless and appears at the site of T. pallidum entrance into the body approximately 3 weeks after transmission. Answer b is incorrect. Latent syphilis refers to patients with a positive serologic diagnosis for syphilis, but no clinical symptoms. This stage occurs after secondary syphilis symptoms have subsided and there are two possible outcomes: progression to tertiary syphilis or clinical cure. Answer c is incorrect. Tertiary syphilis encompasses the long-term complications of syphilitic disease such as granulomatous disease (also called gummatous syphilis) and cardiovascular syphilis. Tertiary syphilis is now uncommon due to antibiotic treatment and is not transmissible.

AW is a 37-year-old woman who admits to smoking 1 pack of cigarettes per day. She is getting married and would like to start hormonal contraception. Which of the following products is most appropriate for CS? A) NuvaRing B) Ortho Evra C) Ortho-Novum 7/7/7 D) Nor-QD

D) Nor-QD Nor-QD (D) is a progestin-only oral contraceptive and is not contraindicated in women older than 35 years who smoke. NuvaRing (A) contains ethinyl estradiol and etonogestrel. Ethinyl estradiol is contraindicated for use in women older than 35 years and who smoke more than 15 cigarettes per day. The risk of stroke and clots rise in this population. Ortho Evra (B) contains ethinyl estradiol and norgestrelomin. Ethinyl estradiol is contraindicated for use in women older than 35 years and who smoke more than 15 cigarettes per day. The risk of stroke and clots rise in this population. Ortho-Novum 7/7/7 (C) contains ethinyl estradiol and norethindrone. Ethinyl estradiol is contraindicated for use in women older than 35 years and who smoke more than 15 cigarettes per day. The risk of stroke and clots rise in this population.

MV is a patient with a non-myeloid malignancy that is receiving treatment. MV has received treatment previously and it affected platelet count. What agent may a patient with non-myeloid malignancy receive to decrease the need for platelet transfusions? A) Filgrastim B) Epoetin alfa C) Dexrazoxane D) Oprelvekin

D) Oprelvekin Patients with non-myeloid malignancies who experienced thrombocytopenia with a prior chemotherapy cycle may receive oprelvekin (IL-11). Oprelvekin decreases the need for platelet transfusions and the numbers of platelets required for transfusions. Answer (A) is incorrect. Filgrastim stimulates the production of neutrophils and promotes proliferation of granulocytes. Answer (B) is incorrect. Recombinant human erythropoietic products (epoetin alfa and darbepoetin alfa) are useful in the management of anemia. Answer (C) is incorrect. Dexrazoxane prevents the cardiac toxicity caused by anthracyclines.

Select the medication that is used for trigeminal neuralgia and neuropathies and is associated with significant CYP-450 drug interactions. A) Valproate B) Gabapentin C) Topiramate D) Oxcarbamazepine

D) Oxcarbamazepine Oxcarbamazepine is a substrate and strong inducer of the CYP-450 system; therefore, it is associated with significant drug interactions.

A 63-year-old man presents to your clinic complaining of excruciating pain in his left big toe. After being diagnosed with an acute gout flare, his physician wants to start him on therapy. His medical history is positive for hypertension, hyperlipidemia, peptic ulcer disease, and glaucoma. Which of the following is appropriate therapy for this patient? A) Ibuprofen B) Indomethacin C) Allopurinol D) Prednisone

D) Prednisone Answer d is correct. This would be an option for patients who have contraindications to other acute gout therapy, such as NSAIDs with peptic ulcer disease. Answers a and b are incorrect. They are both NSAIDs and should be avoided in patients with a history of peptic ulcer disease. Answer c is incorrect. Allopurinol should not be initiated as treatment for an acute gout flare as this medication may worsen the flare by rapidly decreasing uric acid and causing mobilization of uric acid stores.

Who should be screened for asymptomatic bacteriuria? A) College students B) Men C) Patients with indwelling catheters D) Pregnant women

D) Pregnant women The IDSA recommends that pregnant women (D) be screened for bacteriuria by urine culture at least once during early pregnancy (12-16 weeks gestation) or at their first prenatal visit. All positive urine cultures, including asymptomatic bacteriuria, should be treated in pregnant women. College students (A), men (B), and patients with indwelling catheters (C) do not need to be screened for asymptomatic bacteriuria. Patients with chronic catheters universally have asymptomatic bacteriuria.

Which of the following agents would be most appropriate to prevent a gout flare in a transplant patient who is currently receiving tacrolimus, azathioprine, and prednisone? A) Indomethacin B) Allopurinol C) Diclofenac D) Probenecid

D) Probenecid Probenecid is indicated for the prevention of gouty attacks, and is not expected to interact with any medication in this patient's immunosuppressive regimen as it is excreted renally. However, if this patient had renal insufficiency with a CrCl < 50 mL/min, this agent should be avoided. Choice (A) is incorrect. Indomethacin is a nonsteroidal anti-inflammatory drug (NSAID). NSAIDs are not recommended in combination with calcineurin inhibitors like tacrolimus due to increased potential for nephrotoxicity via decrease in renal prostaglandin production. Choice (B) is incorrect. Allopurinol inhibits xanthine oxidase and is typically the drug of choice to prevent gouty attacks. The metabolite of azathioprine (6-mercaptopurine) is inactivated via xanthine oxidase. If allopurinol is administered concomitantly with azathioprine, the other metabolic pathway that produces 6-thioguanine nucleotides can be enhanced, leading to severe pancytopenia. Allopurinol should be avoided in patients who are currently receiving azathioprine. Choice (C) is incorrect. Diclofenac is a nonsteroidal anti-inflammatory drug (NSAID). NSAIDs are not recommended in combination with calcineurin inhibitors like tacrolimus due to increased potential for nephrotoxicity via decrease in renal prostaglandins.

Your patient has just been diagnosed with cirrhosis and undergoes an endoscopy. Several large esophageal varices are noted and his hepatologist decides that he should be started on drug therapy to prevent variceal bleeding. Which of the following describes appropriate therapy for primary prevention of variceal bleeding in this patient? A) No primary prevention therapy needed; only patients who have experienced an episode of variceal bleeding in the past should receive prophylaxis therapy. B) Norfloxacin 400 mg po bid. C) Sucralfate 1 g po four times daily. D) Propranolol 20 mg po bid.

D) Propranolol 20 mg po bid. Answer d is correct. Propranolol 20 mg po bid is an appropriate drug and dosing schedule for primary prevention of variceal bleeding in portal hypertension. Non-selective β-blockers reduce portal pressure by diminishing portal venous inflow via two mechanisms: a decrease in cardiac output through β1 receptor antagonism and a decrease in splanchnic blood flow through β2 receptor antagonism. Propranolol should be titrated to the maximum tolerated dose. Possible side effects of non-selective β-blocker therapy include fatigue, shortness of breath, inability to recognize hypoglycemia in patients with diabetes, depression, and erectile dysfunction. Answer a is incorrect. Patients found to have varices should receive primary prophylaxis therapy in an effort to prevent variceal hemorrhage. Answer b is incorrect. Norfloxacin is an antibiotic that is indicated in the treatment of variceal bleeding. Prophylactic antibiotic therapy is indicated for variceal bleeding. Answer c is incorrect. Sucralfate, an aluminum complex, has no indication for the primary prophylaxis of variceal bleeding in portal hypertension, but is used to manage duodenal ulcer.

Central nervous system (CNS) side effects (seizures and mental status changes) are associated with beta lactam and quinolone anti-infectives. What is a risk factor for development of the central nervous system reactions? A) Duration of therapy B) Infusion interval C) Bioavailability D) Renal dysfunction

D) Renal dysfunction Antibiotic-associated central nervous system toxicities may be common effects for penicillin, cephalosporin, carbapenem, and quinolone anti-infectives, especially if the dose or interval is not adjusted for renal dysfunction (D). Duration of therapy (A) would not be a factor in CNS side effects of beta lactams or quinolones, unless the patient was accumulating (not eliminating) the anti-infective. The infusion interval (B) would not impact the central nervous system side effect. Bioavailability (C) would not impact the central nervous system side effect.

Question 12 of 20 YM is a 62-year-old woman with a recent diagnosis of stage III diffuse large B-cell lymphoma. Her disease is characterized by multiple areas of involvement including the spleen, pelvic lymph nodes, and mediastinal lymph nodes. She is scheduled to begin chemotherapy with the regimen EPOCH-R, which includes etoposide, prednisone, vincristine, cyclophosphamide, doxorubicin, and rituximab. The medical team asks the pharmacists if YM needs to receive any medications to prevent allergic reactions from her treatment regimen. Which of the medications in her treatment regimen is most likely to cause an infusion reaction and could benefit from premedication with acetaminophen and diphenhydramine? A) Cyclophosphamide B) Vincristine C) Doxorubicin D) Rituximab

D) Rituximab Answer d is correct. Rituximab is a chimeric MAB ("-xi-") indicating that it has a substantial amount of nonhuman protein. Because of this, patients receiving rituximab are at risk of developing a reaction during the infusion. Patients typically receive acetaminophen and diphenhydramine to lessen the risk of infusion reactions from rituximab. Answer a is incorrect. Cyclophosphamide is not associated with infusion reactions. Answer b is incorrect.Vincristine is not associated with infusion reactions. The primary concern with infusion of vincristine is risk of extravasation since it is a vesicant. Answer c is incorrect.Doxorubicin is not associated with infusion reactions. The primary concern with infusion of doxorubicin is risk of extravasation since it is a vesicant.

MC is a 19-year-old pop singer with a past medical history of allergic rhinitis, hypothyroidism, and hepatitis B. Medications include loratadine, levothyroxine, and tenofovir. MC has cancelled her concert tour because of a persistent cough and chest tightness. What medication would be an absolute contraindication for MC, if given as a single agent therapy for her asthma? A) Mometasone B) Zafirlukast C) Theophylline D) Salmeterol

D) Salmeterol LABAs may increase the risk of asthma-related deaths (black box warning). Monotherapy with an inhaled long-acting beta2-agonist (LABA), such as salmeterol (D) or formoterol, is not recommended. If a LABA is required, it should be used in combination with an ICS, preferably in the same inhaler. The combination inhalers salmeterol/fluticasone (Advair), formoterol/budesonide (Symbicort), and formoterol/mometasone (Dulera) are FDA-approved for use in patients with persistent asthma that is not well-controlled on an ICS alone. The addition of a LABA improves lung function, decreases symptoms and exacerbations, and reduces rescue use of short acting beta2-agonists. Inhaled corticosteroids (A) are considered preferred treatment for persistent asthma either as monotherapy or in combination. Leukotriene modifiers (B) may be considered as alternative monotherapy depending on the patient's asthma severity. Theophylline (C) may be considered as alternative monotherapy depending on the patient's asthma severity.

In which of the following histologies of lung cancer should one avoid bevacizumab, due to its association with causing pulmonary hemorrhage in this histology? A) Adenocarcinoma B) Large cell C) Bronchoalveolar D) Squamous

D) Squamous Bevacizumab should not be used in squamous (D) cell carcinoma of the lung because of its association with causing pulmonary hemorrhage in this subtype of lung cancer. Bevacizumab can be safely used in patients with adenocarcinoma (A) of the lung but not squamous cell carcinoma of the lung. Bevacizumab can be safely used in patients with large-cell (B) carcinoma of the lung but not squamous cell carcinoma of the lung. Bevacizumab can be safely used in patients with bronchoalveolar (C) sub-type of adenocarcinoma of the lung but not squamous cell carcinoma of the lung.

Second generation antihistamines can be used without a prescription in children _____ years of age, provided they have been diagnosed by a physician with allergic rhinitis and approved for nonprescription therapy. A.) There is no age limit B.) > 12 C.) > 6 D.) 2 - 6 E.) < 2

D.) 2 - 6 For children that have been diagnosed by a physician with allergic rhinitis and approved for nonprescription therapy nonprescription second generation antihistamines can be used in children 2 - 6 years of age without a prescription. If a physician has not diagnosed the child with allergic rhinitis, then the child must be ≥ 12 years of age to use nonprescription second generation antihistamines.

According to federal law, what information is required on the label for a controlled substance prescription? A.) The date of filling B.) The name of the patient C.) The phone number of the dispensing pharmacy D.) All of the above E.) A and B only

E.) A and B only The applicable section of the DEA Pharmacist's Manual is Section IX.

CU is a 52-year-old man with cardiomyopathy (left ventricular ejection fraction 31%) following an acute myocardial infarction (MI). After his MI, he developed signs and symptoms of heart failure (HF) including shortness of breath (SOB) at rest. Which of the following characterizes CU's current American College of Cardiology (ACC)/American Heart Association (AHA) HF stage and New York Heart Association (NYHA) class? A) Stage A, NYHA class not applicable B) Stage B, NYHA class I C) Stage C, NYHA class II D) Stage C, NYHA class IV

D) Stage C, NYHA class IV CU meets the criteria for Stage C. Patients such as CU who are Stage C have developed signs and/or symptoms of heart failure. The NYHA classification system categorizes patients as class I if they are relatively asymptomatic and physical activity is not limited by the heart failure disease process. Patients who are NYHA class II and III are symptomatic with moderate and minimal physical activity, respectively. Finally, patients such as CU who have symptoms at rest are classified as NYHA class IV (D). Patients who are Stage A (A) are at risk for developing left ventricular dysfunction and heart failure (e.g., hypertension, coronary artery disease). Patients who are Stage B (B) have developed structural heart disease but have not developed signs and symptoms of heart failure. CU meets the criteria for Stage C. Patients such as CU who are Stage C have developed signs and/or symptoms of heart failure. However, patients who are NYHA class II (C) and III are symptomatic with moderate and minimal physical activity. CU is symptomatic at rest.

Which of the following statements is correct? A) Modular formulas are targeted for patients with hepatic insufficiency. B) Calorically dense formulas provide nutrition targeted to a specific disease state. C) Elemental formulas contain intact proteins and polysaccharides. D) Standard formulas contain intact proteins.

D) Standard formulas contain intact proteins. Standard formulas contain intact proteins, and are meant for patients who can fully digest proteins and do not require hydrolyzed proteins.

Select the cell that is part of cell-mediated immunity. A) B lymphocyte B) Neutrophil C) Macrophage D) T lymphocyte E) Complement

D) T lymphocyte Answer d is correct. T lymphocytes compose the cell-mediated part of the adaptive functional division and are tailored to defend against infections that are intracellular (viral infections). Answer a is incorrect. B lymphocytes compose the humoral part of the adaptive immune system and secrete antibodies that neutralize pathogens prior to their entry into host cells. B cells, once activated by T cells or antigen presenting cells, become a plasma cell that will produce one of five immunoglobulin types: IgA, IgD, IgE, IgG, or IgM. Answer b is incorrect. Neutrophils are key cells of the innate chemical immunity that fight off bacterial infections. Answer c is incorrect. Macrophages are the main scavenger cells of the immune system. In addition to attacking foreign cells, they are involved in the destruction of old erythrocytes, denatured plasma proteins, and plasma lipids. Answer e is incorrect. The complement system is a mediator of innate immunity. The complement system consists of multiple proteins that play a key role in immune defense. The complement system serves as an adjunct or "complement" to humoral immunity.

GB is a 28-year-old woman with a chief complaint of dysuria. Symptoms started 3 days ago. The physician orders a urinalysis and a urine culture. What is appropriate empiric therapy for GB? Patient has normal renal function and no medication allergies. Medications include metoprolol and omeprazole. A) Cefdinir B) Linezolid C) Amoxicillin D) TMP-SMX

D) TMP-SMX TMP-SMX is appropriate empiric therapy for GB. If the local E. coli resistance rate for TMP-SMX is high, ciprofloxacin would be an appropriate choice. Answer a is incorrect. Cefdinir is an oral third-generation cephalosporin commonly used for acute otitis media in pediatrics. It is not indicated for treatment of UTIs. Although cefdinir is not indicated for treatment of UTIs, you may see an oral cephalosporin utilized. Bacteria are becoming increasingly more resistant and management is driven by in vitro results of a culture and susceptibility report. There are often times when an E. coli UTI is resistant to TMP-SMX and quinolones; therefore, alternative therapy may be utilized (even if it is not indicated). Answer b is incorrect. Linezolid is an antibiotic that covers gram-positive bacteria exclusively. It is primarily used to treat methicillin-resistant S. aureus (MRSA) and vancomycin-resistant enterococci (VRE). Both MRSA and VRE can cause UTIs, but GB does not need to be empirically covered with linezolid. Answer c is incorrect.E. coli resistance rate to amoxicillin is high (37% and higher in select regions). Therefore, aminopenicillins (eg, amoxicillin) would not be ideal for empiric therapy. Because E. coli and other fermenting gram-negative rods (eg, Klebsiella) produce β-lactamase, the addition of clavulanate to amoxicillin (Augmentin) may be utilized in the treatment of E. coli UTIs.

A patient reports significant swelling since starting one of his blood pressure medications. Which of the following could be the cause? A.) Indapamide B.) Clonidine C.) Chlorthalidone D.) Amlodipine E.) Spironolactone

D.) Amlodipine Dihydropyridine calcium antagonists can cause peripheral edema due to vasodilation. Answer B does not cause this adverse effect. Answer A, C and E are diuretic agents would help with edema.

MK is a 63-year-old woman with newly diagnosed metastatic breast cancer scheduled to receive her first dose of trastuzumab. She is 5' 6" tall and weighs 175 lb. You receive the following order: trastuzumab 440 mg IV infusion over 1.5 hours. Select the appropriate assessment to discuss with the prescriber. A) Trastuzumab causes significant nausea and vomiting warranting premedication with a serotonin antagonist antiemetic. It would be best to call the prescriber and suggest administration of an antiemetic. B) Trastuzumab can safely be administered as an IV bolus injection. It would be appropriate to call the prescriber and suggest the order be changed to be administered as an IV bolus injection. C) Trastuzumab has been shown effective as an adjuvant treatment but not treatment of metastatic disease. It would be appropriate to call the prescriber and clarify the indication for trastuzumab for this patient. D) The recommended initial dose of trastuzumab is 4 mg/kg (320 mg total for this patient). It would be appropriate to call the prescriber and clarify dosage for this patient. E) Trastuzumab ordered appropriately for this patient. No clarification is indicated.

D) The recommended initial dose of trastuzumab is 4 mg/kg (320 mg total for this patient). It would be appropriate to call the prescriber and clarify dosage for this patient. Answer d is correct. The standard initial dose for this patient would be 4 mg/kg or 320 mg for this patient. It would be appropriate to clarify this patient's dose. Answer a is incorrect.Trastuzumab is not associated with nausea and vomiting that requires pretreatment antiemetic therapy. Answer b is incorrect.Trastuzumab should not be administered as an IV bolus injection. Answer c is incorrect.Trastuzumab has been shown effective as treatment for both micrometastatic and metastatic breast cancer characterized by overexpression of HER-2. Answer e is incorrect. Clarification of the dose ordered for this patient is warranted.

TR is a patient presenting for follow-up to his primary-care provider. His LDL-C is at goal; however, his HDL is 32 mg/dL. The provider would like to start treatment with niacin to increase his HDL-C. TR has a past medical history of hypertension, angina and diabetes. Medications include metoprolol, aspirin and metformin. Social and family history are negative. Basic metabolic panel and complete blood count labs are within normal limits. Select the potential contraindication (drug interaction) if niacin is added to TR's regimen. A) Metoprolol B) Aspirin C) Metformin D) There are no known interactions where it is recommended to avoid concomitant use

D) There are no known interactions where it is recommended to avoid concomitant use There are no known interactions where it is recommended to avoid concomitant use. However, niacin may impact statin levels. Cases of rhabdomyolysis have occurred during concomitant use of niacin and a statin. With concurrent use or if symptoms suggestive of myopathy occur, monitor creatine phosphokinase (CPK). There are patients that will require therapy with niacin and a statin because monotherapy did not achieve the cholesterol goal. Answer (A) is incorrect. Metoprolol does not interact with niacin. Answer (B) is incorrect. Aspirin does not interact with niacin. Answer (C) is incorrect. Metformin does not interact with niacin.

A 59-year-old man with a performance status of 1 returns to clinic with relapsed small-cell lung cancer, and new bone and liver metastases. He completed his previous chemotherapy of carboplatin and etoposide five months ago, and reports no other medical problems. He and his wife request further treatment if it is reasonable. Based upon this information, which of the following treatments would be rational? A) Best supportive care B) Erlotinib C) Cisplatin + paclitaxel + bevacizumab D) Topotecan

D) Topotecan Topotecan demonstrates activity in patients with relapsed SCLC, and has an FDA-approved indication for this patient population. Answer (A) is incorrect. Because the patient's SCLC relapsed more than three months since completing first-line therapy, his performance status is a 1. If he requests therapy, it is rational to give him second-line therapy by offering a clinical trial, topotecan, gemcitabine, or taxanes. Answer (B) is incorrect. Erlotinib is utilized for NSCLC, not SCLC. Answer (C) is incorrect. Cisplatin+ paclitaxel + bevacizumab is a regimen utilized for NSCLC, not SCLC.

BB is a 35-year-old woman who regularly picks up her Arava and Ortho Tri-Cyclen refills at your pharmacy. Today she arrives to pick up her Arava and states she will no longer need her Ortho Tri-Cyclen as she and her husband have decided to start trying to have a baby. Which of the following would be the most appropriate response to this information? A) Continue Arava at a lower dose when she becomes pregnant, as rheumatoid arthritis typically improves during pregnancy. B) Change Arava to methotrexate during pregnancy. C) Discontinue Arava 2 to 3 weeks prior to trying to get pregnant. D) Undergo drug-elimination with cholestyramine prior to trying to get pregnant.

D) Undergo drug-elimination with cholestyramine prior to trying to get pregnant. Arava is pregnancy category X based on data suggesting increased risk for fetal death and teratogenic effects. Due to Arava's active metabolite and long half-life, patients wishing to become pregnant should undergo the drug elimination procedure with cholestyramine (8 g cholestyramine tid for 11 days; plasma levels M1 <0.02 mg/L must be verified on two separate occasions at least 14 days apart). Answer (A) is incorrect. RA symptoms may improve during pregnancy, but Arava is pregnancy category X and must be discontinued prior to conception. Answer (B) is incorrect. Both methotrexate and Arava are pregnancy category X. Answer (C) is incorrect. Arava must be discontinued approximately 3 months prior to conception to allow for a drug elimination and washout period.

Clinical trials that compared the efficacy of tamoxifen and AIs as adjuvant treatments for patients with breast cancer have established which of these outcomes? A) AIs have been better tolerated than tamoxifen. B) Sequential hormonal therapy (tamoxifen followed by an AI) has resulted in significantly longer progression-free survival than tamoxifen monotherapy. C) Tamoxifen without subsequent AI therapy resulted in superior progression-free survival. D) a and b. E) a and c are all outcomes documented in clinical trials.

D) a and b. Both answers a and b are correct. Choice (A) is correct. Aromatase inhibitors are less likely to cause thromboembolic events and uterine cancer. Choice (B) is correct. Several studies have documented improved progression-free survival rates with the sequential administration (tamoxifen followed by an AI). Choice (C) is incorrect. Tamoxifen monotherapy has not resulted in superior outcomes. Choice (E) is incorrect. Choice (A) is correct but Choice (C) is incorrect.

A 64-year-old man is admitted to the hospital with septic shock. He weighs 72 kg and is 72" tall. Assuming a volume of distribution of 0.8 L/kg, what loading dose of IV vancomycin should you give this patient to achieve an initial target concentration of 35 mg/L?(Round the final answer to the nearest 250 mg increment). A.) 1000 mg B.) 1500 mg C.) 1750 mg D.) 2000 mg E.) 2250 mg

D.) 2000 mg Correct answer: D A loading dose may be computed using the following equation where LD is the loading dose, Css is the initial target concentration, V is the volume of distribution, and F is the bioavailability. LD = (Css x V)/F Since the medication in this question will be administered via IV, F = 1. LD = (35 mg/L x 0.8 L/kg x 72 kg)/1LD = 2016 mg (2000 mg when rounding to the nearest 250-mg increment) Answers A & B & C & E: LD = (35 mg/L x 0.8 L/kg x 72 kg)/1 = 2016 mg (2000 mg when rounding to the nearest 250-mg increment)Bottom Line: The loading dose of vancomycin can be calculated using the following equation: LD = (Css x V)/F.

PL is a 67-year-old man with a history of diabetes mellitus (DM) and alcoholism at the clinic for his seventh cycle of oxaliplatin. He is currently taking capecitabine at home. PL's CBC (complete blood count) with differential is all within normal limits. Before preparing the oxaliplatin for infusion, which other parameters would significantly determine if he requires a dose reduction or his dose to be held? i. Renal function estimated by creatinine clearance ii. Liver function estimated by AST, ALT, total bilirubin iii. Assessment of neurotoxicity side effects A) i only B) ii only C) i and ii D) i and iii E) i, ii, and iii

D) i and iii Oxaliplatin is primarily renally excreted like other platinum chemotherapy agents. There are creatinine clearance specific recommendations in the FDA approved labeling, but caution is recommended for severe renal dysfunction. The labeling does have dose modification guidelines based on grade 2 persistent neurosensory events and recommends consideration to discontinue therapy for grade 3 events. Dose modifications are also recommended for severe GI toxicities and myelosuppression.

Diphenhydramine should be used no longer than how many consecutive nights for nonprescription treatment of insomnia? A.) 3 B.) 5 C.) 7 D.) 14 E.) 21

D.) 14 Diphenhydramine should be used no longer than 14 consecutive nights for the short-term nonprescription treatment of insomnia. After taking diphenhydramine for 3 consecutive nights, the patient can be advised to stop for one night to see if insomnia is relieved.

How long at a minimum should a person abstain from consuming alcohol after stopping disulfiram therapy? A.) 1 day B.) 7 days C.) 10 days D.) 14 days

D.) 14 days The package insert states that the alcohol-disulfiram reaction can occur up to 14 days after stopping use. Ideally, the person would abstain from alcohol for the rest of his or her life to prevent relapse to alcohol use disorder.

JP is a 73 year-old female on general medicine service in an academic medical center. She is 5'6" and 197lbs and has past medical history of CKD on hemodialysis. Pharmacy has been consulted to dose vancomycin for a suspected central line infection. What is the goal vancomycin trough if the MIC of the offending pathogen is not known? A.) 4 to 5 mcg/mL B.) 5 to 10mcg/mL C.) 5 to 15mcg/mL D.) 15 to 20 mcg/mL E.) Monitoring of trough concentrations is no longer recommended

D.) 15 to 20 mcg/mL For serious infections, such as bacteremia, the recommended goal trough for vancomycin is 15 to 20mcg/mL unless the MIC of the offending pathogen is known to be less than 1mcg/mL.

Which of the following is the maximum infusion rate for fosphenytoin? A.) 50 mg/min B.) 75 mg/min C.) 100 mg/min D.) 150 mg/min E.) 175 mg/min

D.) 150 mg/min EXPLANATION: Fosphenytoin is a prodrug that is converted to phenytoin within minutes after infusion. Unlike phenytoin, which is solubilized in propylene glycol, fosphenytoin is water soluble. Because phenytoin itself can be cardiotoxic, fosphenytoin should not be infused faster than 150 mg/min in adults

Which of the following patients would be the best candidate for oral combination hormone replacement therapy (estrogen + progesterone)? A.) 59-year-old female with hypertension, diabetes, hyperlipidemia, and history of myocardial infarction B.) 62-year-old female with a history of breast cancer, who has had a total hysterectomy, bilateral salphingoopherectomy, and mastectomy and who is experiencing hot flashes C.) 52-year-old female who is not experiencing vasomotor symptoms, but who is complaining of urinary urgency and pain with sexual intercourse D.) 49-year-old female with hypothyroidism who is experiencing significant vasomotor symptoms including night sweats E.) None of the above

D.) 49-year-old female with hypothyroidism who is experiencing significant vasomotor symptoms including night sweats A newly menopausal female with significant symptoms and no contraindications is the best candidate for hormone replacement therapy. A 59-year-old female with multiple cardiovascular risk factors including an event and no mention of menopausal symptoms would not be a candidate for hormone replacement therapy. Breast cancer is a contraindication to hormone replacement therapy. If a patient's only symptoms are related to vulvovaginal atrophy, intravaginal estrogen cream would be preferred over systemic therapy.

R. C. is a 33-year-old female diagnosed with stage 2 breast cancer. R. C. is 5'4", weighs 200 lb, and is considered obese because of her body mass index of 34.3. Pathology reports show that her breast cancer is estrogen receptor-negative, progesterone receptor-negative, and HER2 negative, known as "triple negative" breast cancer, an especially aggressive tumor type. After a modified radical mastectomy with sentinel lymph node biopsy, R. C. will receive chemotherapy with the dose-dense AC followed by T regimen (doxorubicin 60 mg/m2 intravenous push over 3-10 minutes and cyclophosphamide 600 mg/m2 intravenous infusion over 30 minutes, both given once every 2 weeks for 4 cycles, followed by paclitaxel 175 mg/m2 intravenous infusion over 3 hours once every 2 weeks for 4 cycles, with pegylated filgrastim 6 mg subcutaneous given the day after treatment in each of the 8 cycles to prevent febrile neutropenia). The combination of doxorubicin and cyclophosphamide is highly emetogenic, whereas paclitaxel has low emetogenicity. Before this diagnosis, R. C. had never had cancer or cancer chemotherapy medications. She has smoked 1 pack of cigarettes per day for 20 years and has never consumed alcohol. She is gravida 3, para 3; all children are alive and healthy. Her family history is negative for breast cancer. R.C. is married and lives with her husband and children about 15 minutes from the cancer center. The cyclophosphamide that R. C. receives is an alkylating agent of the nitrogen mustard type. Other examples of nitrogen mustards include which of the following? A.) Ifosfamide B.) Mechlorethamine C.) Fluorouracil D.) A and B only E.) A, B, and C

D.) A and B only Ifosfamide and mechlorethamine are nitrogen mustard alkylating agents, whereas fluorouracil is a pyrimidine analog antimetabolite.

The DEA has extensive recordkeeping requirements. However, which of the following is not required by the DEA? A.) A pharmacy must maintain all unexecuted official order forms (DEA Form 222). B.) A pharmacy must maintain a record of all controlled substances dispensed. C.) A pharmacy must maintain a logbook (or electronic equivalent) for all pseudoephedrine sold without a prescription. D.) A pharmacy must maintain a copy of all pharmacists' licenses. E.) A pharmacy must maintain receipts and invoices for schedule IV controlled substances.

D.) A pharmacy must maintain a copy of all pharmacists' licenses. The applicable section of the DEA Pharmacist's Manual is Section VI.

Which of the following statements is not true about population and samples? A.) A population can be finite or infinite. B.) A simple random sample is a sample selected from a population such that every possible sample of the same sizes has the same chance of being drawn. C.) The process of drawing a simple random sample is called simple random sampling. D.) A population for statistical analysis is a collection of entities that are part of the study population. E.) A population of values includes every member of all the possible values of a random variable that one is interested in at a particular time.

D.) A population for statistical analysis is a collection of entities that are part of the study population. Explanation: EXPLANATION: In statistics, a population includes every member of all entities that one is interested in at a particular time. A sample is a collection of entities that are part of a population.

A 50-year-old women presents to the local physician with a complaint of extreme stiffness in the morning that lasts until noon on most days for the past 2 months. She also states that she is tired all the time and that both of her hands are swollen and painful. On examining the patient, the physician documents the presence of rheumatoid nodules. The patient's laboratory workup is significant for an elevated CRP (C-reactive protein) and ESR (erythrocyte sedimentation rate) and a positive rheumatoid factor. She states that she has been taking over-the-counter naproxen at a dose of 250 mg twice daily without relief. Which of the following represents the best drug therapy option for this patient? A.) Increase the dose of naproxen to 500 mg twice times daily. B.) Increase the dose of naproxen and add methotrexate 25 mg twice daily. C.) Substitute celecoxib 100 mg twice daily. D.) Add Methotrexate 20 mg once weekly with folate 1 mg daily

D.) Add Methotrexate 20 mg once weekly with folate 1 mg daily Although the patient currently has room to increase her dose of the NSAID, she would benefit from the addition of a DMARD. This patient has a disease duration of less than 6 months with moderate disease and poor prognostic factors. Methotrexate represents a viable option; however, the dose of 25 mg twice daily is excessive (it should be dosed once weekly). ACR guidelines have taken out prognostic factors as considerations but the statement is true regarding DMARD. MTX needs to be added as soon as possible and the patient can increase dose of naproxen.

In addition to angiotensin-converting enzyme inhibitors and angiotensin receptor blockers, which of the following agents has shown benefit in reducing proteinuria in patients with diabetic nephropathy and persistent proteinuria despite appropriate therapy? A.) Furosemide B.) Metformin C.) Acetylcysteine D.) Aliskiren E.) NSAIDs

D.) Aliskiren EXPLANATION: The renin inhibitor aliskiren (Tekturna) has been studied in this patient population and has been shown to reduce proteinuria.

A federal government practitioner is exempt from registration with the DEA if his or her _______ is provided on the prescription. A.) Branch of military B.) Service identification number (if military) C.) Social security number (if a public health service employee) D.) All of the above E.) B and C only

D.) All of the above The applicable section of the DEA Pharmacist's Manual is Section IX.

Which of the following side effects may result from vitamin B12 supplementation as a result of an increase in intravascular volume secondary to an increased production of red blood cells? A.) Itching B.) Glossitis C.) Chills D.) Angina E.) Hypertension

D.) Angina Angina and dyspnea may be side effects that can occur after using vitamin B12 supplementation as a result of increased intravascular volume.

Which medication(s) is/are classified as a calcineurin inhibitor? A.) Rapamune B.) Cyclosporine C.) Tacrolimus D.) B and C E.) A and C

D.) B and C

Phenobarbital is used to help differentiate what from other causes of neonatal jaundice? A.) Cystic duct obstruction B.) Sphincter of Oddi relaxation C.) Gall Bladder Spasm D.) Biliary atresia

D.) Biliary atresia Reason: Phenobarbital maximizes sensitivity by activating the liver excretory enzymes, and the lack of any biliary clearance into the bowel within 24hr indicates a true positive test.

Which of the following mechanisms most likely contributes to the benefits of beta-blockers in the treatment of heart failure? A.) Stimulation of beta-2 receptors B.) Increased heart rate and decreased blood pressure C.) Stimulation of beta-1 receptors D.) Blockade of increased sympathetic nervous system activity E.) Blockade of angiotensin II receptors

D.) Blockade of increased sympathetic nervous system activity Explanation: Activation of the sympathetic nervous system plays an important role in the initiation and progression of heart failure. The benefits of beta-blockers are thought to be due to blockade of this increased activity of the sympathetic nervous system.

An adult planning for overseas travel receives two doses of Twinrix 1 month apart. He returns 6 months after the second dose to receive the final dose. However, the health care provider does not have Twinrix stocked. What should be done? A.) Nothing should be done. His series is completed. B.) Because the patient has received two doses of hepatitis A vaccine, he needs to receive only a single dose of hepatitis B vaccine. C.) He should receive only hepatitis A vaccine. D.) Both hepatitis A vaccine and hepatitis B vaccine should be administered. E.) The provider should wait until Twinrix is available

D.) Both hepatitis A vaccine and hepatitis B vaccine should be administered. EXPLANATION: A complete series of hepatitis B vaccine requires three doses, and a complete series of hepatitis A vaccine requires two doses. Twinrix contains only the pediatric dose of hepatitis A; therefore, the hepatitis A series is not completed. Both hepatitis A and hepatitis B vaccines should be given to complete the series.

L. P. is a 26-year-old white female recently diagnosed with major depression. She has no history of drug or alcohol abuse but does take medication for her epilepsy. Which antidepressant should L. P. avoid? A.) Mirtazapine B.) Citalopram C.) Selegiline D.) Bupropion E.) Duloxetine

D.) Bupropion Of the choices, only bupropion has demonstrated that it may decrease the seizure threshold.

Which of the following products is available in both injectable and nasal spray dosage forms? A.) Raloxifene B.) Alendronate C.) Teriparatide D.) Calcitonin E.) Prempro

D.) Calcitonin Injectable and nasal spray dosage forms of calcitonin are available. Teriparatide is available as an injection only. Prempro, raloxifene, and alendronate are available only in oral dosage forms.

Ideal properties for a β-blocker in the treatment of ACS include which of the following? A.) Available as an IV product, cardioselectivity B.) Low lipophilicity, has ISA C.) Has ISA, cardioselectivity D.) Cardioselectivity, low lipophilicity, does not have ISA E.) Noncardioselective, high lipophilicity

D.) Cardioselectivity, low lipophilicity, does not have ISA Ideally, a β-blocker used for the treatment of UA or NSTEMI would have β1-receptor selectivity and no ISA and low lipophilicity. Being available as an IV agent is not an advantage because oral initiation of β-blockers is preferred to avoid adverse effects; Answer A is incorrect. β1-receptor selectivity would reduce the chance for bronchospasm, and low lipophilicity would reduce the neurological side effects; Answer E is incorrect. β-blockers with ISA reduce heart rate to a lesser degree than non-ISA β-blockers, thus producing a smaller decrease in oxygen demand; Answers B and C are incorrect.

The pharmacokinetic parameters for captopril in healthy adults are Clearance: 800 mL/minute f e = 0.5 Vss: 0.81 L/kg Plasma protein binding: 75% When cimetidine (a highly lipid soluble weak base that is highly secreted in the renal proximal tubules) and captopril are co-administered, the renal clearance of captopril is reduced to approximately 125 mL/minute. What is the most likely mechanism to account for this reduction in renal clearance? A.) Cimetidine reduces the filtration clearance of captopril. B.) Cimetidine enhances the reabsorption of captopril. C.) Cimetidine increases the unbound fraction of captopril. D.) Cimetidine blocks the renal secretion of captopril.

D.) Cimetidine blocks the renal secretion of captopril. The most likely mechanism to account for this reduction in renal clearance is that cimetidine blocks the renal secretion of captopril.

A male patient treated with digoxin for heart failure has a serum digoxin concentration of 0.8 ng/mL. His primary care physician now initiates clarithromycin therapy for suspected community acquired pneumonia. Which of the following is correct about the digoxin-clarithromycin interaction? A.) The dose of digoxin should be increased by 50%. B.) The dose of clarithromycin should be increased by 50%. C.) Clarithromycin inhibits the metabolism of digoxin by CYP2C19, leading to increased digoxin plasma concentrations. D.) Clarithromycin inhibits the transport of digoxin by P-glycoprotein, leading to increased digoxin plasma concentrations. E.) Clarithromycin inhibits the metabolism of digoxin by CYP2C9, leading to decreased digoxin plasma concentrations.

D.) Clarithromycin inhibits the transport of digoxin by P-glycoprotein, leading to increased digoxin plasma concentrations. EXPLANATION: Clarithromycin inhibits the drug transporter P-glycoprotein (P-gp), resulting in increased digoxin bioavailability and decreased clearance, thus resulting in increased digoxin plasma concentrations.

MO is a HIV positive patient who is diagnosed with Pneumocystis jirovecii pneumonia. He has a past medical history that includes an allergy to sulfa medications. What would be a reasonable treatment for his Pneumocystis jirovecii pneumonia infection? A.) Ciprofloxacin B.) Clarithromycin C.) Dapsone D.) Clindamycin and primaquine E.) Doxycycline

D.) Clindamycin and primaquine The drug of choice for treatment of Pneumocystis jirovecii pneumonia is SMZ-TMP DS. In patients who are sulfa allergic, alternative treatment includes: clindamycin and primaquine; Dapsone and TMP; or atovoquone.

Which of the following is a drug discovery strategy that uses nucleic acids and amino acids in various combinations to synthesize vast libraries of oligonucleotide or peptide compounds for high-throughput lead compound screening? A.) Whole cell screening B.) Natural product screening C.) Gene therapy D.) Combinatorial chemistry E.) rDNA technology

D.) Combinatorial chemistry

When a pharmacy orders schedule II controlled substances using the DEA official triplicate order form, which copy (copies) of the form is (are) sent by the pharmacy to the supplier? A.) Copy 1 B.) Copy 2 C.) Copy 3 D.) Copies 1 and 2 E.) Copies 2 and 3

D.) Copies 1 and 2 The pharmacy keeps copy 3 and sends copies 1 and 2 to the supplier. The applicable DEA regulation is 21 CFR 1305.12.

Which of the following is the official DEA form that is the actual Certificate of Registration that must be conspicuously displayed at a pharmacy or other location registered with the DEA? A.) DEA Form 111 B.) DEA Form 222 C.) DEA Form 223 D.) DEA Form 224 E.) DEA Form 224a

D.) DEA Form 224 DEA Form 223 is the DEA Certificate of Registration that must be displayed at the registrant's location. DEA Form 111 is not a real DEA form. DEA Form 222 is used to order schedule II controlled substances. DEA Form 224 is the application form for a new DEA Certificate of Registration. DEA Form 224a is the renewal application form that DEA mails to registrants approximately 60 days prior to the expiration of a DEA Certificate of Registration. The applicable DEA regulation is 21 CFR 1301.13.

A 73-year-old female with a past medical history of heart failure (left ventricular ejection fraction 25%) and hypertension is receiving enalapril 10 mg po bid and furosemide 40 mg bid. She now presents to the emergency room with a 2-week history of palpitations, dizziness, leg swelling, and weight gain. The ECG reveals atrial fibrillation with a ventricular rate of 130 bpm. Which of the following should be used to acutely slow the ventricular rate in this patient? A.) Metoprolol B.) Carvedilol C.) Propafenone D.) Digoxin E.) Lidocaine

D.) Digoxin Digoxin would be the initial drug of choice to acutely slow the ventricular rate in this patient with worsening heart failure. Although metoprolol and carvedilol are both very effective rate control agents, in this patient who presents with worsening heart failure (weight gain, leg swelling), they may worsen heart failure even further.

Which of the following may exacerbate GERD symptoms by lowering the lower esophageal sphincter pressure? A.) Quinidine B.) Iron C.) Potassium chloride D.) Diltiazem E.) Tetracycline

D.) Diltiazem Calcium channel blockers decrease LES pressure. Quinidine, iron, potassium chloride, and tetracycline have direct irritant effects on the esophageal mucosa.

A 56-year-old female presents to the emergency department with complaints of flank pain, dysuria, and increased urinary frequency. She is diagnosed with a urinary tract infection. Her past medical history includes type 2 diabetes mellitus, hypertension, and recurrent DVTs. Her medications include metformin 1 g bid, quinapril 40 mg daily, and warfarin 5 mg daily. What would be the most appropriate antibiotic to treat this patient's UTI? A.) Septra DS bid B.) Ciprofloxacin 500 mg bid C.) Rifampin 300 mg qid D.) Doxycycline 100 mg bid E.) Erythromycin 500 mg qid

D.) Doxycycline 100 mg bid Doxycycline 100 mg bid would be the most appropriate therapy for a UTI. Septra DS, erythromycin, and ciprofloxacin will interact with warfarin to elevate the INR.

Which of the following preventive measures is not considered appropriate for prevention of acute otitis externa? A.) Drying ears with hair dryer on lowest heat setting after exposure to water B.) Installation of drops containing isopropyl alcohol C.) Installation of drops containing isopropyl alcohol and acetic acid D.) Drying ears with cotton-tipped swabs after exposure to water E.) Using barrier bathing cap or swimming cap

D.) Drying ears with cotton-tipped swabs after exposure to water EXPLANATION: Any manipulation of the skin of the external auditory canal should be avoided. The use of cotton-tipped swabs has been shown to cause trauma to this area. Other drying techniques, such as the use of hair dryers, have been shown to be effective. Caution must be used to avoid burns. Alcohol drops with or without acetic acid are efficacious to dry the area and reduce the pH of the ear canal.

Which one of the following immunosuppressants requires therapeutic drug monitoring? A.) Azathioprine B.) Basiliximab C.) Belatacept D.) Everolimus E.) Prednisone

D.) Everolimus Everolimus is dosed to achieve levels of 3-8 ng/ml for kidney transplant recipients.

Which of the following tasks should be done to prevent aspiration pneumonia in a patient receiving enteral nutrition? A.) Frequent flushing of the PEG with warm water. B.) Place patient in prone position while receiving enteral nutrition. C.) Periodically assess the patient's abdomen to ensure tolerance. D.) Frequently assess the placement of the feeding tube

D.) Frequently assess the placement of the feeding tube Feeding tube placement should be verified frequently, especially in patients with nasally placed tubes, to decrease the incidence of aspiration. Although flushing of the feeding tube with warm water will decrease incidence of tube clogging, it will not decrease the incidence of aspiration. Patients should have the head of bed at 30°, not placed in the prone position. The patient's abdomen should be assessed frequently, not periodically.

Which of the following radiopharmaceuticals is useful in assisting with the diagnosis of opportunistic infections? A.) Thallous chloride Tl-201 B.) I-123 iobenguane C.) In-111 pentetreotide D.) Ga-67 citrate

D.) Ga-67 citrate Tl-201 chloride is indicated for cardiology, not infectious diseases. I-123 iobenguane and In-111 pentetreotide are both for oncology indications, not infectious diseases. Ga-67 citrate is useful for diagnosis of opportunistic infections, and it has application in oncology as well.

Benzyl alcohol, a preservative found in certain parenteral preparations, may accumulate in neonates and cause toxicity. Which of the following adverse effects is related to the accumulation of benzyl alcohol? A.) Hyperosmolarity B.) Kernicterus C.) Gray baby syndrome D.) Gasping syndrome E.) Arthropathy

D.) Gasping syndrome EXPLANATION: Gasping syndrome occurs because of accumulation of benzyl alcohol in the neonate. Because glycine conjugation pathways are immature, accumulation of benzoic acid occurs. Symptoms of gasping syndrome may include cardiovascular collapse, metabolic acidosis, neurologic deterioration, and respiratory failure.

Each of the following can be a symptom of atrial fibrillation EXCEPT: A.) Dizziness B.) Palpitations C.) Angina D.) Hypertension E.) Sudden-onset slurred speech

D.) Hypertension Due to loss of functional atrial contraction and rapid ventricular rate (producing palpitations), cardiac output may decrease, resulting in decreased perfusion of major organs, particularly the brain (dizziness, confusion, etc) and heart (angina and heart failure exacerbation). Depending on the vascular tone, blood pressure may remain stable or fall as a direct result of decreased cardiac output; however, hypertension would not be expected. Patients with atrial fibrillation are at increased risk of thrombosis, particularly stroke, secondary to pooling of blood in the left atrium and subsequent thrombus formation.

Which of the following statements best describe the macrolides? I. Primarily effective against gram-positive aerobic bacteria and atypicals II. Effective against most Penicillin-resistant streptococci III. Ineffective against Penicillin-resistant streptococci A.) I only B.) II only C.) III only D.) I & II E.) I & III

D.) I & II Erythromycins are primarily gram-positive aerobic antibiotics and atypicals with good activity against most penicillin-resistant Streptococcus isolates.

The currently available vaccine for prevention of smallpox is: I. Dryvax II. A live-virus preparation of the vaccinia virus III. Avoidance of direct contact with infected persons and their body fluids A.) I only B.) I and II only C.) II and III only D.) I, II, and III E.) No vaccine is currently available

D.) I, II, and III

Which of the following statements best describes the use of corticosteroids in the treatment of acne? I. A short course can be used to suppress acne II. Continual use can worsen acne III. Should only be used orally IV. Should be used at doses higher than 50 mg/day A.) I only B.) II only C.) I and II D.) I, II, and III E.) I, II, III, and IV

D.) I, II, and III A short course of 7-10 days of prednisone 20 mg should be used.

Which of the following statements is correct concerning sterility testing of high-risk-level CSPs? A.) Two methods of sterility testing exist: membrane filtration and gel-clot. B.) When it is purchased ready to use from a vendor, medium for the sterility test does not have to undergo growth promotion testing before use. C.) When performing the sterility test in house, one may use only one medium, which is usually TSB. D.) In incubation of the sterility test units, the appropriate temperatures are 20-25°C for soybean casein-digest media and 30-35°C for FTM. E.) In the USP <71> sterility test chapter, another name for the "Validation Test" that must be performed once on each product is the "Inhibition and Enhancement Test."

D.) In incubation of the sterility test units, the appropriate temperatures are 20-25°C for soybean casein-digest media and 30-35°C for FTM. The two methods of sterility testing are membrane filtration and direct inoculation. The two media are TSB and FTM. TSB is incubated at 20-25ºC and FTM is incubated at 30-35ºC, both for a minimum of 14 days. The medium suitability test must be performed on each lot of medium. The validation test, also called the bacteriostasis and fungistasis test, must be performed once on each product. The sterility test does not prove the entire lot is sterile; it just proves the units tested did not show any growth. Therefore, the CSP met the requirements of the test. Two media are used in the sterility test, soybean-casein digest and fluid thioglycollate. Each lot of medium must undergo the medium suitability test, which includes proving it is sterile and is capable of supporting the growth of USP organisms. These tests may be done at the same time the sterility test is performed. The sterility test is done on the CSP, and the media fill is used to verify the compounding skills of the operator.

A patient presents to her primary care physician complaining of increased shortness of breath. It seems to worsen on exertion when climbing stairs and at night when she is lying down. Based on her history and symptoms, the treating physician suspects heart failure and is considering starting a pharmacologic option to manage her fluid balance. What is the mechanism of action of furosemide? A.) Inhibits sodium reabsorption in the distal tubules B.) Competes with aldosterone in the distal renal tubules C.) Inhibits the enzyme carbonic anhydrase D.) Inhibits reabsorption of sodium, potassium, and chloride in the ascending loop of Henle

D.) Inhibits reabsorption of sodium, potassium, and chloride in the ascending loop of Henle Correct answer: D Loop diuretics exert their action by inhibiting the sodium-potassium-chloride cotransport system located within the thick ascending limb of the loop of Henle. This results in excretion of sodium and water, leading to a decreased fluid balance for the patient.Fluid overload and shortness of breath, particularly on exertion, is one of the first signs of heart failure. Fluid overload associated with heart failure is managed by daily weight measurements and pharmacologic measures such as diuretics. Loop diuretics (i.e. furosemide) are the mainstay therapy for fluid removal in patients with heart failure. Other common diuretics used are thiazide and aldosterone antagonists. Answer A:Inhibiting sodium reabsorption in the distal tubules is the mechanism of thiazide diuretics (i.e. hydrochlorothiazide). They are diuretics often used when a patient's CrCl > 30 ml/min. Answer B:Spironolactone, a potassium sparing diuretic, competes with aldosterone receptors in the distal renal tubule. This increases sodium and chloride excretion while conserving potassium. Answer C:Inhibiting the enzyme carbonic anhydrase is the mechanism of acetazolamide. By reversibly inhibiting carbonic anhydrase, there is a reduction in hydrogen ion secretion and an increase in renal secretion of sodium, potassium, bicarbonate, and water. Bottom Line:Loop diuretics (i.e. furosemide) remain the mainstay diuretic for the management of fluid overload in heart failure. They work by inhibiting the sodium-potassium-chloride cotransport system located within the thick ascending limb of the loop of Henle.

Which of the following side effects is not commonly seen with divalproex? A.) Thrombocytopenia B.) Hair loss C.) Hepatotoxicity D.) Insomnia E.) GI upset

D.) Insomnia Divalproex (Depakote®) is not commonly associated with insomnia. Divalproex is commonly associated with somnolence, dizziness, GI upset and headache. Thrombocytopenia is a rare but serious reaction to divalproex and requires monitoring CBC with differentials. Divalproex also has black box warnings for hepatotoxicity, hemorrhagic pancreatitis, and teratogenicity.

A patient with ESRD on peritoneal dialysis has been receiving an ESA for treatment of anemia and now requires IV iron to correct iron deficiency. Which of the following is an appropriate dosing regimen of IV iron to safely administer to this patient? A.) Ferumoxytol 1000 mg IV push over 5 minutes B.) Iron dextran 500 mg over 30 minutes C.) Iron dextran 1000 mg over 1.5 hours D.) Iron sucrose 400 mg IV over 2.5 hours E.) Sodium ferric gluconate 500 mg over 1.5 hours

D.) Iron sucrose 400 mg IV over 2.5 hours From this list the only approved dosing regimen is iron sucrose 400 mg IV over 2.5 hours. This is part of the dosing regimen to administer a total dose of 1000 mg in divided doses. The other regimens listed are too aggressive and put the patient at risk for adverse events. See product labeling for each agent.

Which of the following statements about alprostadil administration is true? A.) It is available as an oral formulation. B.) Repeat the dose if no erection occurs in one hour. C.) Fatty meals may impair absorption. D.) It is available as an intracavernosal injection. E.) Avoid administration with nitrates.

D.) It is available as an intracavernosal injection. Alprostadil is available only as an intracavernosal injection and an intraurethral suppository. Administration is limited to one dose in 24 hours for the injection and two doses per 24 hours (maximum of 3 doses per week) for the suppository. Alprostadil can be taken regardless of concomitant nitrates or food.

Compared with IV morphine, which of the following adverse effects is less likely to occur with IV fentanyl? A.)Constipation B.) Respiratory depression C.) Drowsiness D.) Itching E.) Confusion

D.) Itching EXPLANATION: The itching and peripheral vasodilation associated with opioids result from histamine release. Morphine and other naturally occurring agents are more potent histamine releasers than is fentanyl.

Which of the following best describes probenecid? A.) Like febuxostat, it decreases the body's production of uric acid. B.) It can be used in renal failure. C.) It blocks the excretion of uric acid into the urine. D.) Like Anturane, it is a uricosuric agent that blocks reuptake of uric acid at the proximal convoluted tubule.

D.) Like Anturane, it is a uricosuric agent that blocks reuptake of uric acid at the proximal convoluted tubule. Benemid is a uricosuric agent that blocks reuptake of uric acid at the proximal convoluted tubule. It does not affect the body's ability to produce uric acid and cannot be used in renal failure. It does not block the excretion of uric acid into the urine.

The treating physician confirms this patient's new diagnosis of heart failure. Which one of the following features will help to distinguish the New York Heart Association (NYHA) Functional Class of this patient? A.) B-type Natriuretic peptide B.) Ejection Fraction C.) Jugular vein distention D.) Limitations to physical activity E.) Pulse Pressure

D.) Limitations to physical activity Correct answer: D New York Heart Association (NYHA) Functional Classification is dependent on a patient's exercise capacity and severity of symptoms. A patient's classification can fluctuate frequently with treatment or the presence of exacerbating factors. NYHA Functional Class is an independent predictor of mortality and is used in clinical practice and research to guide treatments and interventions.

Which of the following is FDA approved for the treatment of moderate to severe binge-eating disorder? A.) Bupropion B.) Sertraline C.) Citalopram D.) Lisdexamfetamine E.)Fluoxetine

D.) Lisdexamfetamine Lisdexamfetamine (Vyvanse) is FDA approved for moderate to severe binge-eating disorder.

A patient presents to the pharmacy with a bottle of ibuprofen and is curious if there is a drug interaction with any of his blood pressure medications. The patient has difficult to control hypertension and takes aliskiren, lisinopril, clonidine, atenolol, and amlodipine. Which of his agents may have decreased effectiveness if used in combination with the ibuprofen? A.) Clonidine B.) Atenolol C.) Amlodipine D.) Lisinopril E.) Aliskiren

D.) Lisinopril ACE inhibitors work on efferent arterioles in the kidneys and cause vasodilation. Non-steroidal anti-inflammatory agents work on the afferent arterioles in the kidneys and blunt the effects of ACE inhibitors and ARBs.

Which of the following statins must be dosed with the evening meal? A.) Atorvastatin B.) Simvastatin C.) Pitavastatin D.) Lovastatin E.) Rosuvastatin

D.) Lovastatin Lovastatin is the only statin that must be dosed with the evening meal to increase its absorption.

Of the following medications being taken by an 85-year-old woman, which one would be of the most concern for causing an adverse event? A.) Lorazepam 0.5 mg at bedtime B.) Digoxin 0.125 mg daily C.) Docusate 250 mg daily at bedtime D.) Meperidine 50 mg daily for arthritic pain E.) Timoptic 0.25% drops one drop twice daily

D.) Meperidine 50 mg daily for arthritic pain EXPLANATION: Meperidine has an increased risk of causing respiratory and circulatory depression, and this risk is greater for the elderly patient. Considering the alternatives, meperidine should be avoided in the elderly.

Which of the following would not be considered for monotherapy of glaucoma? A.) Latanoprost B.) Dorzolamide C.) Carteolol D.) Methazolamide E.) Brimonidine

D.) Methazolamide All of the other choices could be considered as monotherapy for glaucoma. Methazolamide is an oral carbonic anhydrase inhibitor and is used in conjunction with ophthalmic drops

Of the following stimulants, which is available as a liquid and a chewable tablet? A.) Adderall B.) Concerta C.) Dexedrine D.) Methylin E.) Ritalin

D.) Methylin Methylin (methylphenidate) is the only stimulant product that is available as a liquid and a chewable tablet.

Which of the following contraceptives would you suggest for a patient who is 1 month postpartum and breast-feeding? A.) Yaz B.) Ortho Evra C.) Loestrin D.) Micronor

D.) Micronor EXPLANATION: A progestin-only oral contraceptive (Micronor) is recommended for new mothers (< 6 weeks postpartum) who are breast-feeding because estrogen may interfere with establishing milk supply. However, after breast-feeding is established, combination oral contraceptives may be used.

The most common organisms associated with CAP in adults treated as outpatients are A.) Pseudomonas aeruginosa, Mycoplasma pneumoniae, and Haemophilus influenzae. B.) Streptococcus pneumoniae, Haemophilus influenzae, and Klebsiella pneumoniae. C.) Mycoplasma pneumoniae, Streptococcus pneumoniae, Haemophilus influenzae, and Klebsiella pneumoniae. D.) Mycoplasma pneumoniae, Streptococcus pneumoniae, Haemophilus influenzae, and Chlamydophila pneumoniae. E.) Mycoplasma pneumoniae, Streptococcus pneumoniae, Haemophilus influenzae, and Pseudomonas aeruginosa.

D.) Mycoplasma pneumoniae, Streptococcus pneumoniae, Haemophilus influenzae, and Chlamydophila pneumoniae. Pseudomonas aeruginosa is more likely in patients with risk factors for multidrug-resistant bacteria such as late-onset HAP or VAP. Klebsiella pneumoniae is also not commonly associated with CAP.

According to the federal FDCA, the following definition applies to which term? "Any drug, the composition of which is such that such drug is not generally recognized, among experts qualified by scientific training and experience to evaluate the safety and effectiveness of drugs, as safe and effective for use under the conditions prescribed, recommended, or suggested in the labeling thereof." A.) Counterfeit drug B.) Food C.) Investigational drug D.) New drug E.) Pesticide

D.) New drug

Which of the following medications will interact with epoetin? A.) Insulin B.) Simvastatin C.) Atenolol D.) None of the above E.) All of the above

D.) None of the above There are no known drug interactions with epoetin.

Which of the following is the general vasopressor of choice in septic shock? A.) Dopamine B.) Epinephrine C.) Phenylephrine D.) Norepinephrine E.) Vasopressin

D.) Norepinephrine Norepinephrine is the preferred vasopressor recommended in the Surviving Sepsis Guidelines.

Which equation describes the rate of drug dissolution from a tablet? A.) Fick's law B.) Henderson-Hasselbalch equation C.) Michaelis-Menten equation D.) Noyes-Whitney equation

D.) Noyes-Whitney equation The Noyes-Whitney equation describes the rate of drug dissolution from a tablet. Fick's first law of diffusion is similar to the Noyes-Whitney equation in that both equations describe drug movement attributable to a concentration gradient. The Michaelis-Menten equation involves enzyme kinetics, whereas Henderson-Hasselbalch equations are used for determination of pH of the buffer and the extent of ionization of a drug molecule.

Which of the following is the correct reason that oral polio vaccine is no longer indicated in the United States? A.) It is actually indicated and routinely used. B.) There is no risk of polio; therefore, polio vaccination is no longer recommended in the United States. C.) Oral polio vaccine (OPV) is less effective than inactivated polio vaccine (IPV). D.) OPV has more serious adverse effects than IPV.

D.) OPV has more serious adverse effects than IPV. OPV's use was discontinued in the United States because of the vaccine-associated paralytic polio (VAPP) risk with the oral, live vaccine. This effect does not occur with IPV.

A patient has severe persistent asthma, and her physician is considering anti-IgE therapy. Which of the following medications is the correct generic (and trade name) for this therapy? A.) Omalizumab (Xopenex) B.) Montelukast (Singulair) C.) Zafirlukast (Accolate) D.) Omalizumab (Xolair) E.) Zileuton (Zyflo)

D.) Omalizumab (Xolair)

In a randomized controlled trial that follows a parallel design, which of the following is true? A.) Patients serve as their own control. B.) All patients end up receiving all of the interventions in random different orders, depending on group assignment. C.) Only patients who complete the entire protocol are included in the final analysis. D.) Patients are assigned to groups that receive a particular treatment over time; the only planned difference in the groups is the intervention.

D.) Patients are assigned to groups that receive a particular treatment over time; the only planned difference in the groups is the intervention. In a randomized controlled trial that follows a parallel design, patients are assigned to groups that receive a particular treatment over time; the only planned difference in the groups is the intervention. Patients do not serve as their own control. In general, patients receive only one study treatment depending on group assignment. And all patients are accounted for in the final analysis, including patients who do not complete the study protocol.

The treatment of choice of neurosyphillis is intravenous penicillin. Which of the following forms of penicillin can be administered intravenously? A.) Penicillin V Potassium B.) Penicillin V C.) Penicillin G Procaine D.) Penicillin G Aqueous E.) Penicillin Benzathine

D.) Penicillin G Aqueous The treatment of neurosyphilis necessitates intravenous penicillin due to the severity of the disease state. Penicillin can be administered in a multitude of different formulations. Penicillin G aqueous is given intravenously.

Which of the following should be taken once daily in the morning with food? A.) Azathioprine B.) Cyclosporine C.) Mycophenolate mofetil D.) Prednisone E.) Sirolimus

D.) Prednisone Prednisone is taken once daily with food.

Which of the following disease states decrease theophylline elimination and often result in reduced dosage requirements? A.) Hepatitis B.) Heart failure (decompensated) C.) Cirrhosis D.) A and C E.) A, B, and C

E.) A, B, and C Hepatitis, cirrhosis, and decompensated heart failure can dramatically reduce theophylline clearance.

Which of the following is required for diagnosis of prostate cancer? A.) Elevated prostate-specific antigen B.) Abnormal prostate finding on digital rectal exam C.) Elevated prostate-specific antigen and Abnormal prostate finding on digital rectal exam D.) Prostate biopsy E.) MRI imaging of the prostate

D.) Prostate biopsy The diagnosis of prostate cancer is based on tissue histology obtained during prostate biopsy. Prostate cancer cannot be diagnosed by prostate-specific antigen (PSA) result, digital rectal exam (DRE), adjunctive laboratory testing, imaging studies, or any other clinical findings. Clinical signs associated with prostate cancer include an elevated PSA and an abnormal prostate finding on digital rectal examination. PSA elevation often is present in men with prostate cancer, however, it is not specific for cancer and can occur in many benign conditions. A higher PSA is associated with an increased likelihood of prostate cancer, however, a normal PSA doesn't rule out the possibility of prostate cancer. A DRE may detect prostate nodules or irregularities that can occur with prostate cancer, however, due to the physical limitations of the exam, prostate cancer is often not detectable by DRE. DRE only can detect tumors in the posterior and lateral areas of the prostate (palpable via the rectum). DRE generally is not recommended as a routine screening test for evaluation of the prostate or rectum. Answer A: The diagnosis of prostate cancer is based on tissue histology obtained on prostate biopsy. PSA elevation often is present in men with prostate cancer, however, it is not specific for cancer and can occur in many benign conditions. A higher PSA is associated with an increased likelihood of prostate cancer, however, a normal PSA doesn't rule out the possibility of prostate cancer .Answer B: The diagnosis of prostate cancer is based on tissue histology obtained during prostate biopsy. A DRE may detect prostate nodules or irregularities that can occur with prostate cancer, however, due to the physical limitations of the exam, prostate cancer is often not detectable by DRE. DRE only can detect tumors in the posterior and lateral areas of the prostate (palpable via the rectum). DRE generally is not recommended as a routine screening test for evaluation of the prostate or rectum.Answers C & E: The diagnosis of prostate cancer is based on tissue histology obtained on prostate biopsy. Prostate cancer cannot be diagnosed by PSA result, DRE finding, adjunctive laboratory testing, imaging studies, or any other clinical findings.

The addition of an aminoglycoside antibiotic in hospital-acquired pneumonia increases the likelihood of protection against which bacterial pathogen? A.) S. pneumoniae B.) Haemophilus influenzae C.) M. pneumoniae D.) Pseudomonas aeruginosa E.) Staphylococcus aureus

D.) Pseudomonas aeruginosa EXPLANATION: Aminoglycosides are active against aerobic Gram-negative rods such as Pseudomonas, and their use in pneumonia increases the likelihood of having at least one antibiotic with activity against resistant Gram-negative pathogens. Vancomycin is added to protect against methicillin-resistant S. aureus. H. influenzae, M. pneumoniae, and S. pneumoniae are common pathogens in community-acquired pneumonia and do not require double coverage.

Which of the following psoriasis treatments inactivates Dovonex (calcipotriene) if used after it is applied? A.) Oral corticosteroids B.) Retinoids (Soriatane) C.) Methotrexate D.) Psoralens (combined with ultraviolet light therapy) E.) Etanercept (Enbrel)

D.) Psoralens (combined with ultraviolet light therapy) Dovonex is inactivated by psoralen with UVA (PUVA). Therefore, Dovonex should be applied after the light treatment.

Which asthma controller drug is preferred for mild persistent asthma in children under 12 years of age? A.) Accolate B.) Singulair C.) Xolair D.) Pulmicort E.) Medrol

D.) Pulmicort Pulmicort or other inhaled corticosteroid is preferred (see NIH EPR-3).

Based on current evidence, which of the following is the best agent for treating ICU delirium? A.) Propofol B.) Dexmedetomidine C.) Haloperidol D.) Quetiapine E.) Ziprasidone

D.) Quetiapine Quetiapine currrently has the most data supporting its use for treating ICU delirium.

Which of the following foods would be most useful in preventing an individual from acquiring folic acid deficiency? A.) Fried catfish B.) White bread C.) Honey bun D.) Raw mushrooms E.) French fries

D.) Raw mushrooms Fruits and vegetables that are steamed or raw have high folic acid content.

Because combination DMARD therapy may be more efficacious in the refractory RA population, which of the following represents the best choice for combination therapy? A.) Arava 20 mg once daily + Rheumatrex 25 mg once weekly B.) Remicade 3 mg/kg IM + Rheumatrex 25 mg once weekly C.) Enbrel 50 mg weekly + Orencia 125 mg weekly D.) Remicade 3 mg/kg IV + Rheumatrex 25 mg once weekly

D.) Remicade 3 mg/kg IV + Rheumatrex 25 mg once weekly Arava plus methotrexate (Rheumatrex) may be a very efficacious combination, but it increases the risk of liver toxicity significantly. Biologics like enbrel and orencia should never be combined. Remicade, approved only for use in combination with Rheumatrex, is given IV and not IM; this combination represents the best choice.

Weight loss is most often associated with which of the following? A.) Donepezil B.) Galantamine C.) Mirtazapine D.) Rivastigmine E.) Memantine

D.) Rivastigmine Weight loss, probably because of nausea and vomiting, is a warning for rivastigmine. In controlled trials, approximately 26% of women on doses of 9 mg/day or greater had weight loss of equal to or greater than 7% of their baseline weight. There is less reported weight loss with donepezil, galantamine, and memantine. The antidepressant mirtazapine is associated with weight gain in the elderly.

Which of the following drugs activates 5-HT1B/5-HT1D and to a lesser extent 5-HT1A or 5-HT1F receptors? A.) Methysergide B.) Propranolol C.) Ergotamine D.) Rizatriptan E.) Amitriptyline

D.) Rizatriptan Rizatriptan is a triptan (5-HT 1 agonist).

Atomoxetine labeling contains warnings for all of the following except: A.) Cardiovascular events B.) Psychotic episodes C.) Severe liver injury D.) Seizure activity E.) Suicidal ideations

D.) Seizure activity Atomoxetine carries a boxed warning for suicidal ideation in children and adolescents and other warnings including the risk of severe liver injury, serious cardiovascular events, effects on blood pressure and heart rate, psychosis/mania, and aggressive or violent behavior. There is not a warning for seizure activity with atomoxetine. Stimulants used for ADHD, however, may lower the seizure threshold.

Which of the following is a US Boxed Warning of lamotrigine? A.) QTc Prolongation B.) Suicidal Ideation C.) Heart Failure D.) Serious Skin Rashes E.) Hepatotoxicity

D.) Serious Skin Rashes Correct answer: D Lamotrigine has a US Boxed Warning for serious skin rashes, including Stevens-Johnson syndrome, that may require hospitalization and discontinuation of treatment. The incidence of these rashes is very small (0.3% to 0.8% in pediatric patients and 0.08% to 0.3% in adults receiving lamotrigine). As you can see by the reported incidences, the risk is much greater in children versus adults. Age is the only known risk factor for lamotrigine-induced serious skin rashes.The risk of serious rash is expected to be the same whether the patient is on the extended-release or immediate-release formulation. Nearly all cases of life-threatening rashes associated with lamotrigine have occurred within 2 to 8 weeks of treatment initiation. Although some rashes induced by lamotrigine may be benign, it is not possible to predict which will be serious. Therefore, lamotrigine should be discontinued at the first sign of rash unless the rash is clearly not drug-related. Answer A:QTc prolongation is not a known adverse effect of lamotrigine. This is a common adverse effect of anti-psychotics, macrolide antibiotics, and antiarrhythmics. Answer B: Lamotrigine does not have a boxed warning against suicidal ideation. There have been reports of anticonvulsants increasing suicidal ideation, however, it is not a US boxed warning for lamotrigine. Answer C:Heart failure is not a known adverse effect of lamotrigine. There are certain medications that may worsen signs and symptoms associated with heart failure. For example, a major adverse effect of amlodipine is peripheral edema, which may worsen certain signs and symptoms of heart failure. Answer E:Hepatotoxicity is not a boxed warning against lamotrigine. While lamotrigine does require hepatic metabolism for elimination, it does not have a boxed warning for hepatotoxicity. Medications that may be associated with an increase in hepatotoxicity include statins, acetaminophen, and amiodarone. Bottom Line:Lamotrigine has a US Boxed Warning for serious skin rashes, including Stevens-Johnson syndrome, that may require hospitalization and discontinuation of treatment.

A 21 year old female patient is interested in getting the Gardasil vaccine. Which of the following is the correct regimen? A.) She should get 2 doses total one at 0 months and one at 6-12 months. B.) She should get 3 doses total, at months 0, 4, and 6 C.) She should get 2 doses total at months 0 and 6 D.) She should get 3 doses total at months 0, 1-2, and 6 E.) The vaccine is not appropriate for patient age

D.) She should get 3 doses total at months 0, 1-2, and 6 Correct answer: D GARDASIL9 (Human Papillomavirus 9-valent Vaccine, Recombinant) helps protect individuals ages 9 to 45 against the following diseases caused by 9 types of HPV: cervical, vaginal, and vulvar cancers in females, anal cancer and genital warts in both males and females It may be given as 2 or 3 shots. For persons 15 through 45 years of age, GARDASIL 9 is given using a3-dose schedule; the second shot should be given 2 months after the first shot and the third shot should be given 6 months after the first shot.

Which drug is a once-daily anticholinergic bronchodilator? A.) Atrovent B.) Serevent C.) Foradil D.) Spiriva E.) Proventil

D.) Spiriva Spiriva (tiotropium) is inhaled once daily.

A 50 kg patient with a history of multiple gastrointestinal surgeries is started on a regular diet 5 days after a small bowel resection. He has good bowel sounds and had a bowel movement recently. He tolerated the diet with no nausea or emesis; however, a large amount of fluid begins to leak from his incision site. He is diagnosed with a fistula. Which of the following treatments represents the best nutrition intervention for this patient? A.) Start peripheral parenteral nutrition. B.) Change to a low-residue diet. C.) Place nasogastric tube and begin enteral nutrition. D.) Start central parenteral nutrition.

D.) Start central parenteral nutrition. EXPLANATION: Fistulas require bowel rest to heal; therefore, changing this patient's diet or starting enteral nutrition or doing both will not allow for resolution. Parenteral nutrition will be necessary to facilitate healing of the fistula. However, it will be difficult to provide the patient's caloric and protein needs sufficiently with peripheral parenteral nutrition. Starting central parenteral nutrition is the best answer.

What agency generally regulates traditional pharmacy compounding? A.) FDA B.) USP C.) PCAB D.) State boards of pharmacy

D.) State boards of pharmacy The regulation of pharmacy compounding is generally regulated by individual state boards of pharmacy. This is true for pharmacies operating as traditional pharmacies (i.e., compounding is performed for specific patients, pursuant to a valid prescription). Pharmacies operating outside of this scope may be subject to FDA jurisdiction.

Which of the following is true about supraventricular tachyarrhythmias with a narrow QRS interval? A.) The arrhythmia originates below the Purkinje fibers. B.) Amiodarone is the acute treatment of choice. C.) The heart rate is less than 50 bpm. D.) The acute treatment of choice is adenosine. E.) They are caused by early afterdepolarizations

D.) The acute treatment of choice is adenosine. Supraventricular tachycardia with a narrow QRS interval is caused by reentry. The acute treatment of choice is adenosine to break the reentrant cycle.

Which of the following counseling points regarding the use of Oxytrol for Women transdermal patch is the most appropriate? A.) When used with lifestyle modifications, the patch can reduce urinary accidents by 10% more than by using lifestyle modifications alone. B.) It will take up to 6 weeks to see the full benefit from the medication. C.) The best place to apply the patch is on the upper arm. D.) The patient must reapply the patch every 4 days. E.) This product should not be used by women < 65 years of age.

D.) The patient must reapply the patch every 4 days. The patient must reapply the Oxytrol for Women patch to the abdomen, hip, or buttock every 4 days. When the patch is used with lifestyle modifications, urinary accidents can be reduced by 25% more than by using lifestyle modifications alone. The full benefit from this medication will be seen in about 2 weeks.

Which is an important counseling point regarding nonprescription nicotine replacement therapy? A.) The patient can smoke for the first week of therapy but must stop smoking after 7 days. B.) The gum is the best choice for patients with transmandibular joint problems C.) Use of NRT does not prevent or treat withdrawal symptoms. D.) The patient should stop smoking prior to starting therapy. E.) Chewing the NRT lozenge increases the efficacy of the dosage form

D.) The patient should stop smoking prior to starting therapy. The patient should completely stop smoking when starting nonprescription NRT therapy unlike the prescription options for smoking cessation, which allow for the patient to continue smoking for the first week of therapy.

A physician writes a prescription for 100 tablets of hydromorphone 4 mg. She notes on the prescription that the patient is terminally ill. How long does the pharmacist have to dispense in partial quantities up to the 100 tablets ordered? A.) The pharmacist must dispense all of the tablets at once. B.) The pharmacist must complete the dispensing within 72 hours. C.) The pharmacist may partially fill the prescription for 30 days (the total quantity dispensed in all partial fillings must not exceed the total quantity prescribed). D.) The pharmacist may partially fill the prescription for 60 days (the total quantity dispensed in all partial fillings must not exceed the total quantity prescribed). E.) The pharmacist may partially fill the prescription for 90 days (the total quantity dispensed in all partial fillings must not exceed the total quantity prescribed).

D.) The pharmacist may partially fill the prescription for 60 days (the total quantity dispensed in all partial fillings must not exceed the total quantity prescribed). A prescription for a schedule II controlled substance written for a patient with a terminal illness may be filled in partial quantities to include individual dosage units. Such prescriptions are valid up to 60 days after the issue date. The applicable section of the DEA Pharmacist's Manual is Section X.

Which of the following statements is true? A.) Parametric tests are always preferred over nonparametric tests. B.) When a sample size is large, a nonparametric test has the advantage of being more easily and quickly applied than a parametric test. C.) The investigator needs to know the form of the population distribution to use a nonparametric test. D.) The sign test takes into consideration only the positive and negative signs of the data.

D.) The sign test takes into consideration only the positive and negative signs of the data. The sign test takes into consideration only the positive and negative signs of the data. The raw data used in the calculation of the test statistic for the sign test are plus and minus signs.

Ms. Clark just underwent oral surgery and asks if she can crush the Concerta tablet. What is the appropriate response? A.) Yes, it can be crushed B.) The capsule could be opened and sprinkled into applesauce C.) The tablet can be dissolved in water and then consumed immediately D.) The tablet cannot be crushed E.) The tablets are scored and can be cut in half but cannot be crushed

D.) The tablet cannot be crushed Correct answer: D Concerta (methylphenidate ER) is an osmotic controlled release formulation (OROS) of methylphenidate and FDA approved for attention deficit hyperactivity disorder (ADHD). The tablet has an immediate-release overcoat that provides an initial dose within 1 hour, and the remaining is released at a controlled rate over 5 to 9 hours. The trilayer core comprises 2 layers containing the drug and excipients, and 1 layer of osmotic components. As water from the gastrointestinal tract enters the core, the osmotic components expand and methylphenidate is released. The tablets thereforeshould not be crushed. Answer A: Concerta is an OROS formulated tablet and should not be crushed. Answer B: Concerta is a tablet, not a capsule. Answer C: Concerta cannot be dissolved in water. Answer E: Concerta is not scored and should not be cut in half. Bottom Line:Concerta (methylphenidate ER) is an osmotic controlled release formulation (OROS) of methylphenidate and FDA approved for attention deficit hyperactivity disorder. Concerta cannot be crushed, chewed, or divided.

Which of the following is true regarding digoxin therapy in patients with HFrEF? A.) Digoxin reduces mortality. B.) Concomitant amiodarone therapy decreases digoxin plasma concentrations. C.) Digoxin is contraindicated in patients with HFrEF and atrial fibrillation. D.) The target digoxin plasma concentration is 0.5-1 ng/mL. E.) Concomitant glyburide therapy increases digoxin plasma concentrations.

D.) The target digoxin plasma concentration is 0.5-1 ng/mL. The 2017 ACC-AHA Guidelines for the Management of Heart Failure suggest a target digoxin plasma concentration of 0.5-1 ng/mL. Digoxin does not improve survival in patients with heart failure—it only improves symptoms. Amiodarone increases digoxin plasma concentrations, and glyburide does not affect digoxin concentrations. Digoxin is useful in the management of patients with heart failure who also have atrial fibrillation.

Which of the following anticonvulsants is FDA approved for migraine headache prophylaxis? A.) Valproic acid B.) Amitriptyline C.) Gabapentin D.) Topiramate E.) Propranolol

D.) Topiramate

Patients should be instructed to drink plenty of fluids while taking which of the following anticonvulsants? A.) Gabapentin B.) Carbamazepine C.) Phenobarbital D.) Topiramate E.) Levetiracetam

D.) Topiramate EXPLANATION: Topiramate and zonisamide use have been associated with an increased risk for calcium phosphate kidney stones. The risk of kidney stones may be increased in those who have a personal or family history of kidney stones. Patients should be instructed to drink plenty of fluids (6-8 ounces of water per day) to decrease the likelihood of stone formation.

Which of the following medications is in pregnancy category D during the second and third trimesters? A.) Milrinone B.) Torsemide C.) Carvedilol D.) Valsartan E.) Dobutamine

D.) Valsartan Valsartan is an angiotensin receptor blocker (ARB) and all ARBs (and ACE inhibitors) are in pregnancy category D for the second and third trimesters.

Lisinopril may cause hemodynamically mediated AKI by preventing which compensatory mechanism by the kidney? A.) Vasodilation of the afferent arteriole B.) Vasoconstriction of the afferent arteriole C.) Vasodilation of the efferent arteriole D.) Vasoconstriction of the efferent arteriole

D.) Vasoconstriction of the efferent arteriole Angiotensin-converting enzyme inhibitors may contribute to development of AKI in patients with conditions resulting in prerenal kidney disease (e.g., conditions resulting in decreased perfusion of the kidney, hypovolemia, heart failure, liver disease). ACEIs (and ARBs) prevent the compensatory vasoconstriction of the efferent arteriole mediated by angiotensin II that occurs in an attempt to increase GFR.

Which of the following medications would be appropriate for the secondary prevention of stroke in a 75-year-old patient with hypertension and atrial fibrillation as a result of mitral stenosis (valvular damage)? A.) Aspirin B.) Pradaxa C.) Apixaban D.) Warfarin

D.) Warfarin The patient has a CHADS2 score of 4 placing him at high risk of recurrent stroke. He would benefit from the use of anticoagulation over antiplatelet therapy alone. NOACs are not indicated in those patients with valvular atrial fibrillation.

What would be the most likely outcome if a patient with Parkinson's disease taking levodopa were also prescribed haloperidol? A.) Excessive nausea and vomiting B.) Hypertensive crisis C.) Tachycardia and possible chest pain D.) Worsening symptoms of Parkinson's disease E.) Excessive somnolence

D.) Worsening symptoms of Parkinson's disease Haloperidol and other antipsychotics block dopamine activity and can worsen PD. They can also block the benefits of PD medications, which increase dopamine activity.

Which of the following is not consistent with the clinical presentation of systemic lupus erythematosus? A.) Malar rash B.) Photosensitivity C.) Proteinuria D.) Xerostomia

D.) Xerostomia

All of the following patients are inappropriate for self-treatment of vulvovaginal candidiasis (VVC), except A.) an 11-year-old female complaining of vaginal discharge and painful urination. B.) a 15-year-old pregnant female complaining of vaginal discharge and painful urination. C.) a 16-year-old female seeking treatment again after appropriate treatment of a vulvovaginal candidiasis (VVC) infection 20 days earlier. D.) a 16-year-old female whose VVC symptoms have persisted for 3 days on therapy. E.) a 20-year-old male whose female sex partner was just diagnosed with VVC.

D.) a 16-year-old female whose VVC symptoms have persisted for 3 days on therapy. The patient in choice D may not get full relief of symptoms until at least 1 week of therapy. Therefore, the patient can still receive OTC therapy for VVC. OTC therapy is excluded because the patient in choice A is under 12 years of age; because the patient in choice B is having the symptom of painful urination, which may be caused by a sexually transmitted infection requiring physician assessment; and because the patient in choice C has symptoms after 1 week of therapy.

Adverse effects of amiodarone include all of the following conditions except A.) pulmonary fibrosis. B.) blue-gray skin discoloration. C.) hypothyroidism. D.) acute renal failure. E.) hyperthyroidism.

D.) acute renal failure. EXPLANATION: Amiodarone is not nephrotoxic. However, it can affect thyroid function (causing both hyper- and hypothyroidism), cause blue-gray skin discoloration (especially in sun-exposed areas), and cause pulmonary fibrosis.

Tertiary syphilis in adults should be treated with A.) benzathine penicillin G 2.4 million units for 1 day. B.) azithromycin 1 g orally once. C.) aqueous crystalline penicillin G 4 million units q4h for 10-14 days. D.) benzathine penicillin G 2.4 million units once a week for 3 weeks. E.) doxycycline 100 mg orally bid for 7 days.

D.) benzathine penicillin G 2.4 million units once a week for 3 weeks. Benzathine penicillin G. 2.4 million units once a week for 3 weeks is standard therapy for either late latent syphilis or tertiary syphilis. Single-dose benzathine penicillin (2.4 million units) is appropriate for primary, secondary, or early latent syphilis. Response C is the treatment for neurosyphilis.

When compared to the cephalosporins, the peincillins have A.) better atypical bacterial coverage B.) better staphylococcal coverage C.) better streptococcal coverage D.) better enterococcal coverage E.) better aerobic Gram-negative coverage

D.) better enterococcal coverage Penicillin are highly active against most enterococcus, and cephalosporins are not active against enterococcus.

Valsartan benefits patients with heart failure by A.) inhibition of phosphodiesterase III. B.) blockade of β1-receptors. C.) inhibition of the conversion of angiotensin I to angiotensin II by angiotensin converting enzyme. D.) blockade of angiotensin II receptors.

D.) blockade of angiotensin II receptors. EXPLANATION: Valsartan's benefits in patients with heart failure are mediated by blockade of the angiotensin receptor.

Which of the following drugs may be used to diagnose adrenal insufficiency? A.) desmopressin B.) clemastine C.) captopril D.) cosyntropin E.) aminoglutethimide

D.) cosyntropin Cosyntropin (Cortrosyn) is a synthetic analog of ACTH that is used to diagnose adrenal insufficiency. It works by stimulating the adrenal cortex to secrete cortisol. If cosyntropin administration does not result in an appropriate increase in cortisol release, adrenal insufficiency is present.

Decreased urine production is an effect of A.) carmustine. B.) propylthiouracil. C.) ACTH. D.) desmopressin. E.) SSKI

D.) desmopressin. Desmopressin (DDAVP) is a synthetic analogue of vasopressin, or antidiuretic hormone. Thus, it decreases urine production by increasing water resorption

The treatment of choice for tinea unguium is A.) clotrimazole. B.) miconazole. C.) undecylenic acid. D.) griseofulvin. E.) tolnaftate.

D.) griseofulvin. The treatment of choice for tinea unguium (fungal infection of the toenails and fingernails) is a systemic antifungal agent such as griseofulvin. Topical therapy is generally not effective for fungal infections of the nails.

Which of the following is not a common sign or symptom of Addison's disease? A.) weakness B.) fatigue C.) weight loss D.) hypertension E.) hyperpigmentation

D.) hypertension Explanation: All of the above are typical signs and symptoms of Addison's disease except hypertension. Typically patients with Addison's will have trouble maintaining both blood pressure and blood sugar.

When using a peak flow meter, the patient should A.) hold the device 2 inches away from his or her open mouth. B.) use the average of three attempts. C.) record the best of two attempts. D.) place the mouthpiece into the mouth on top of the tongue. E.) breathe in normally and breathe out fast.

D.) place the mouthpiece into the mouth on top of the tongue. EXPLANATION: It is important to place the mouthpiece well into the mouth. The patient should inhale fully and then blast. The patient should record the best of three attempts.

A patient who is suffering from heat intolerance, weight loss, tachycardia, tremor, and anxiety may be treated with A.) acetaminophen. B.) mitotane. C.) cyproheptadine. D.) propylthiouracil. E.) diazepam

D.) propylthiouracil. Heat intolerance, weight loss, tachycardia, tremor, and anxiety are cardinal features of hyperthyroidism. Propylthiouracil is effective at reducing the excessive thyroxine level.

A 65-year-old man who weighs 78 kg with a creatinine clearance of 45 mL/min was admitted to the hospital for a submassive pulmonary embolism. The patient was initially anticoagulated with an infusion of unfractionated heparin. The physician team would like to discharge the patient home tomorrow and would like a recommendation for discharging the patient on warfarin. Which treatment regimen would be best for this patient? A.) Warfarin 10 mg X 2 days, followed by 5 mg daily B.) Enoxaparin 80mg daily for a minimum of 5 days and 2 INRs >2 and warfarin 5 mg daily C.) Enoxaparin 160 mg daily for a minimum of 5 days and 2 INRs >2 and warfarin 5 mg daily D.)Enoxaparin 80mg twice daily for a minimum of 5 days and 2 INRs >2 and warfarin 5 mg daily E.) Enoxaparin 80mg twice daily for a minimum of 3 days and 2 INRs >2 and warfarin 5 mg daily

D.)Enoxaparin 80mg twice daily for a minimum of 5 days and 2 INRs >2 and warfarin 5 mg daily Correct answer: D Given the patient's new diagnosis of a pulmonary embolism, the patient needs to be discharged on therapeutic anticoagulation. This would include subcutaneous enoxaparin, a low molecular weight heparin, and warfarin for at least 5 days or at least 2 consecutive therapeutic INRs(2-3).The recommended enoxaparin dose is 1 mg/Kg twice daily plus warfarin 5 mg daily with a goal INR of 2 to 3. Answer A:Patients with a pulmonary embolism, if being initiated on warfarin therapy, should be bridged with a minimum of 5 days of therapy with enoxaparin or heparin. Warfarin 10 mg x 2 days followed by 5 mg daily is the dosing schema recommended for total knee replacement patients. Answer B:The enoxaparin dose for this answer choice of ~1mg/Kg daily is lower than the recommended dose for this patient given the creatinine clearance of 45 mLs/min. The correct dose is 1 mg/kg twice daily. Answer C:The enoxaparin dose for this answer choice of ~2mg/Kg daily is higher than the recommended dose for this patient given the creatinine clearance of 45 mLs/min. The correct dose is 1 mg/kg twice daily. Answer E:The enoxaparin therapy should be continued for a minimum of 5 days (not 3 days) to cover the half-lives for natural anticoagulants C and S that are found in warfarin.Bottom Line:Patients who are initiated on warfarin therapy following a pulmonary embolism should receive a minimum of 5 days of bridge therapy with a low molecular weight heparin. Therapy should be continued until the patient has 2 consecutive INR readings of greater than or equal to 2.

SS is a patient with chronic kidney disease, anemia, hypertension, and diabetes. What is the target hemoglobin concentration for chronic kidney disease patients like SS receiving epoetin-alfa or darbepoetin-alfa? A) 7 g/dL B) 8 g/dL C) 9 g/dL D) 10 g/dL E) 11 g/dL

E) 11 g/dL Patient survival may be decreased if the hemoglobin level is titrated above 11 to 12 g/dL. Therefore, the target hemoglobin concentration is 11 to 12 g/dL. Answer (A) is incorrect. A hemoglobin concentration of 7 g/dL is the value to consider red blood transfusion, but not the target concentration for epoetin-alfa and darbepoetin-alfa. Symptomatic anemia patients with a hemoglobin concentration less than 7 g/dL are candidates for transfusion of red blood cells. Answer (B), (C) and (D) incorrect. These values are not high enough for the goal of 11 to 12 g/dL. 30% of users answered correctly.

Which of the following should caution for anaphylactoid reaction be used with digoxin immune Fab fragments administration? A) Papaya or papain allergy B) Patients treated previously with digoxin immune Fab fragments C) Sheep protein allergy D) Latex allergy E) All of the above

E) All of the above All of the above are instances when precautions for emergency intervention (oxygen, H1 and H2 antagonist, corticosteroids, and airway management) should be considered prior to administration of digoxin immune Fab fragment. Choice (A) is correct. Papain is used to segment whole antibody in the production of digoxin immune Fab fragments. Caution is warranted for patients with papain, papaya extract, or the pineapple enzyme bromelain. Choice (B) is correct. By design, the Fab fragment lacks the antigenic determinants of the Fc fragment and is associated with significantly reduced potential of anaphylactoid reaction compared to intact immunoglobulin. However, patient previously treated with digoxin immune Fab fragment are at increased risk of immunogenic reaction. Choice (C) is correct. Patients with known sheep protein allergy are at increased risk of immunogenic reaction as the manufacturer of digoxin immune Fab fragments utilizes sheep (ovine) immunized with a digoxin derivative to form the antidigoxin antibodies. Choice (D) is correct. Certain latex and dust mite allergens share antigenic structures with papain and patients with these allergies may be allergic to papain.

JS is a 16-year-old patient with purulent nasal discharge, headache, cough and congestion, bad breath, and anosmia (loss of smell) for the past 2 days. He has a temperature of 38.3°C (100.9°F). He has no known drug allergies. He has no known past medical history; however, he did receive 5 days of an unknown antibiotic to treat a skin and soft tissue infection about 20 days ago. He is 140 lb. After providing JS with an appropriate treatment option, 4 to 5 days after his initial presentation, he started to feel much better. On day 5, he even decided to go outside and ride his scooter with his friends. However, the next morning he woke up with chills, nausea, headache, and increased nasal discharge. He had a temperature of 38.3°C (100.9°F). Which of the following is the most appropriate recommendation? A) Acetaminophen 325 mg po q4h prn B) Amoxicillin 875 mg po tid C) Levofloxacin 500 mg po daily D) Amoxicillin/clavulanate 875 mg po bid E) Amoxicillin/clavulanate 2 g XR po bid

E) Amoxicillin/clavulanate 2 g XR po bid Answer e is correct. The patient is at risk for drug resistant S. pneumoniae and, therefore, would need a higher dose to overcome potential resistance. Answer a is incorrect. Although it would be appropriate to give JS acetaminophen analgesia and thermoregulation, this answer is not the most appropriate. At this time, JS needs antimicrobial therapy. When a viral respiratory infection occurs, patients can be at risk for developing a secondary bacterial infection. This phenomenon is sometimes referred to as "double-sickening" and can be easily identified following a viral URTI (lasting 5-6 days) that was initially improving followed by a new onset of worsening symptoms, that is fever, headache, or increase in nasal discharge. Answer b is incorrect. Amoxicillin is no longer recommended empirically due to the increasing prevalence of β-lactamase producing respiratory pathogens (H. influenzae and M. catarrhalis). Answer c is incorrect. The patient does not have a history of hives/anaphylaxis to penicillin. Answer d is incorrect. The patient has received an antibiotic within the previous 30 days and, therefore, is at risk for drug resistant S. pneumoniae which will not be overcome with a β-lactamase inhibitor. 24% of users answered correctly.

What fluoroquinolone inhibits the CYP-450 system? A) Doxycycline B) Levofloxacin C) Trimethoprim-sulfamethoxazole D) Nafcillin E) Ciprofloxacin

E) Ciprofloxacin Ciprofloxacin (E) is a fluoroquinolone antimicrobial and inhibits the CYP-450 system (1A2-strong and 3A4-weak). While doxycycline (A) is an inhibitor of the CYP-450 3A4 (moderate), it is a tetracycline antimicrobial. While levofloxacin (B) is a fluoroquinolone antimicrobial, it does not affect the CYP-450 system. While trimethoprim-sulfamethoxazole (C) is an inhibitor of the CYP-450 (Sulfamethoxazole-substrate CYP 2C9 [major], 3A4 [minor], inhibits 2C9 [moderate]; Trimethoprim-substrate 2C9 [major], 3A4 [major], inhibits 2C8 [moderate], 2C9 [moderate]), it is a sulfonamide antimicrobial. While nafcillin (D) does affect the CYP-450 system (P-450 3A4 inducer), it is a penicillin antimicrobial.

Which of the follow is the only FDA-approved combination tyrosine kinase oral chemotherapy regimen approved for the treatment of unresectable or metastatic V600E or V600K mutated melanoma? A) Ipilimumab + Dacarbazine B) IL-2 + Temozolomide C) Interferon-alpha 2b + Temozolomide D) Vemurafenib + Dabrafenib E) Dabrafenib + Trametinib

E) Dabrafenib + Trametinib Answer e is correct. Combination use of trametinib and dabrafenib has recently obtained FDA accelerated approved for use in the treatment of metastatic or unresectable melanoma as the combination therapy allows for greater inhibition of the MAPK pathway and improved outcome in patients with BRAF V600 mutation (+) disease. Answer a is incorrect.Ipilimumab is an intravenous immunotherapy agent and Dacarbazine is an intravenous chemotherapy agent. Answer b is incorrect. IL-2 is an intravenous immunotherapy agent and Temozolomide is an oral alkylating chemotherapy agent. Answer c is incorrect. Interferon-alpha 2b is an intravenous immunotherapy agent and Temozolomide is an oral alkylating chemotherapy agent. Answer d is incorrect. Both vemurafenib and dabrafenib are tyrosine kinase oral chemotherapy regimen approved for the treatment of unresectable or metastatic V600E or V600K mutated melanoma; however, its approval is for single agent usage and not for combination use. 36% of users answered correctly.

AA is diagnosed with cellulitis. Pharmacologic treatment of cellulitis is with an antibiotic that is active against penicillinase-producing strains of S. aureus (MSSA). Select the penicillin that is active against penicillinase-producing strains of S. aureus. A) Cephalexin B) Amoxicillin C) Clindamycin D) Doxycycline E) Nafcillin

E) Nafcillin Nafcillin is stable against penicillinase-producing strains of S. aureus and is a penicillin. Nafcillin is referred to as a penicillinase-stable penicillin or an antistaphylococcal penicillin. Other antibiotics in this class are dicloxacillin, methicillin and oxacillin. Note: Methicillin is no longer used clinically, but is referred to in S. aureus resistance (i.e., methicillin-resistant S. aureus). Answer (A) is incorrect. While cephalexin is stable against penicillinase-producing strains of S. aureus, it is a cephalosporin. Note: All cephalosporins are stable/active against beta-lactamase (penicillinase) production. A newer type of beta-lactamase production is called extended-spectrum beta-lactamase (ESBL); most cephalosporins are not stable/active against ESBL production. Examples of bacteria that have ESBLs are the Enterobacteriaceae (e.g., Escherichia coli).

Procainamide is metabolized by and inhibits the metabolism of which cytochrome P450 enzymes, respectively? A.) CYP2D6 and P-glycoprotein B.) CYP3A4 and CYP2D6 C.) CYP2C19 and CYP2C9 D.) P-glycoprotein and CYP3A4 E.) Acetylation by N-acetyltransferase and no effect on cytochrome P450 enzymes

E.) Acetylation by N-acetyltransferase and no effect on cytochrome P450 enzymes Procainamide is not metabolized by nor does it inhibit cytochrome P450 enzymes. It is acetylated by N-acetyltransferase to form the pharmacologically active metabolite N-acetylprocainamide.

JT is a 24-year-old woman brought to the hospital by her roommate. She states that she has been having diarrhea and vomiting for three days. She reports a 3 kg weight loss and cannot keep down anything, even water. In the emergency department, her BP is 96/46 mmHg, HR 120 beats/min, temp is 102.6°F, weight is 48 kg. On examination, her mucous membranes are dry, and she has no peripheral edema. She does not recall the last time that she urinated but thinks it may have been yesterday. Which of the following findings would you expect from her serum and urine laboratory analysis? A) Specific gravity 1.016, 2+ protein, light yellow urine, many WBC and RBCs B) Specific gravity 1.035, 0 protein, hazy red urine, granular casts, FeNa >2% C) Specific gravity 1.016, 2+ protein, light yellow urine, many WBC and RBCs D) Specific gravity 1.005, 0 protein, hazy red urine, granular casts, FeNa >1% E) Specific gravity 1.035, 0 protein, dark yellow urine, no casts, FeNa <1%

E) Specific gravity 1.035, 0 protein, dark yellow urine, no casts, FeNa <1% The high specific gravity, dark color urine, decreased FeNa (<1%), and the absence of cellular matter and protein, are all consistent with a pre-renal AKI secondary to circumstances such as dehydration. Answer (A) is incorrect. The patient is volume depleted (sudden weight loss, vomiting and diarrhea, temperature, dry mucous membranes, etc.). In a state of dehydration, the kidneys should retain sodium and water, and excrete only a small amount of dark, concentrated urine. The specific gravity of 1.016 is near normal, which does not suggest a concentrated urine. Answer (B) is incorrect. The FeNa >2% suggests that the tubules are not reabsorbing as much sodium as possible. In a volume-depleted state (pre-renal) the tubules should reabsorb as much water as possible and excrete as little as possible. In this situation, the FeNa should be very low (<1%). Answer (C) is incorrect. Dehydration causes a pre-renal AKI. There is no intrinsic damage to the nephrons, so there should not be any cellular matter or protein in the urine (a bland sediment). Answer (D) is incorrect. This is a very dilute urine (specific gravity of 1.005). The FeNa is inconsistent with pre-renal AKI, as is a hazy red urine color and the presence of casts.

A 65-year-old patient brings in her tamoxifen prescription for a refill. Upon reviewing her medication profile, you discover that she began taking tamoxifen 20 mg daily in June 2000 for the prevention of breast cancer and she has been having it refilled regularly since then. What if anything should you discuss with the patient's physician? A) The recommended duration of tamoxifen when prescribed to decrease the risk of breast cancer is 10 years. There is no need to clarify this patient's tamoxifen regimen. B) The merits of increasing the dose to 40 mg daily based on results of a recent study documenting superior efficacy of a 40 mg daily dose. C) Switching this patient to an AI based on recent studies that have documented improvement in efficacy and tolerability with AIs. D) The merits of decreasing the dose to 10 mg daily based on results of a recent study documenting equal efficacy but superior tolerability of a 20 mg daily dose. E) The recommended duration of tamoxifen when prescribed to decrease the risk of breast cancer is 5 years.

E) The recommended duration of tamoxifen when prescribed to decrease the risk of breast cancer is 5 years. Answer e is correct. Results from the National Surgical Adjuvant Breast Project (NSABP) Breast Cancer Prevention Trial established that the risks associated with continuing tamoxifen beyond 5 years (increased incidence of uterine cancer, deep venous thrombosis) exceeded the benefits. Answer a is incorrect. Results from the NSABP Breast Cancer Prevention Trial established that the risks associated with continuing tamoxifen beyond 5 years (increased incidence of uterine cancer, deep venous thrombosis) exceeded the benefits. Answer b is incorrect. The dose of tamoxifen established as effective in the prevention of breast cancer is 20 mg daily. Answer c is incorrect. Results from the NSABP Breast Cancer Prevention Trial established that the risks associated with continuing tamoxifen beyond 5 years (increased incidence of uterine cancer, deep venous thrombosis) exceeded the benefits. Answer d is incorrect. The dose of tamoxifen established as effective in the prevention of breast cancer is 20 mg daily.

The therapeutic range for phenobarbital is: A.) 1-2 mg/L B.) 4-12 mg/L C.) 10-20 mg/L D.) 50-100 mg/L E.) 15-45 mg/L

E.) 15-45 mg/L In assessment of chronic therapy the reference range is considered to be 15-45 mg/L. Because the anticonvulsant effect is concentration related and is linear, many practitioners will target higher concentrations in patients who are acutely seizing. Some studies report concentrations as high as 300 mg/L.

Your prescription needs 15 mL of a liquid that has a specific gravity of 1.3. If you decide to weigh the liquid rather than measure its volume, how much should you weigh out? A.) 17.5 g B.) 15.3 g C.) 11.5 g D.) 15.0 g E.) 19.5 g

E.) 19.5 g

Which of the following is the best daily dose of prednisone for home management of an acute exacerbation of asthma in a 30 kg child? A.) 15 mg B.) 60 mg C.) 10 mg D.) 20 mg E.) 30 mg

E.) 30 mg The dose is 1 mg/kg or 30 mg for a child who weighs 30 kg according to the NIH Expert Panel Report 3 guidelines.

What is the milliosmolarity (ideal) of normal saline (NaCl, formula weight = 58.5)? A.) 100 mOsm/L B.) 154 mOsm/L C.) 254 mOsm/L D.) 287 mOsm/L E.) 308 mOsm/L

E.) 308 mOsm/L

What is the highest dose of estrogen (ethinyl estradiol) offered in an oral contraceptive? A.) 25 mcg B.) 30 mcg C.) 35 mcg D.) 40 mcg E.) 50 mcg

E.) 50 mcg The highest dose of estrogen (ethinyl estradiol) offered in an oral contraceptive is 50 mcg.

How many grams per liter are needed to prepare 2N (normality) of H2SO4? A.) 25 g B.) 35 g C.) 78 g D.) 49 g E.) 98 g

E.) 98 g Normality is the number of equivalents in one liter of solvent. The expression "2N normal" means two equivalents dissolved in one liter. Because one equivalent of sulfuric acid equals 49 g, 2N is equivalent to dissolving 98 g of sulfuric acid in 1 L of solvent.

The indications for the colony stimulating factors filgrastim and sargramostim that are supported by clinical and pharmacoeconomic research are: A.) Primary prophylaxis of febrile neutropenia. B.) Secondary prophylaxis of febrile neutropenia. C.) Treatment of established febrile neutropenia. D.) Priming a patient for stem cell harvest. E.) A and B above. F.) A, B, and C above.

E.) A and B above. There is extensive clinical and pharmacoeconomic research to support the use of the colony stimulating factors filgrastim and sargramostim in the primary (use of colony stimulating factors with each cycle of chemotherapy beginning with the first cycle) and secondary (use of colony stimulating factors with each cycle of chemotherapy beginning with the cycle following the cycle in which febrile neutropenia first occurs) prophylaxis of febrile neutropenia. There is also extensive research to support the use of colony stimulating factors in priming a patient for a stem cell harvest. Using colony stimulating factors to treat occurrences of febrile neutropenia does not alter the mortality of febrile neutropenia, so there is no clinical or pharmacoeconomic benefit to support the considerable cost of colony stimulating factors in this setting.

Which of the following is a potential benefit of phytoestrogens? A.) Weight loss B.) Improvements in lipid profile C.) Decreased risk of fracture D.) Increased bone mineral density E.) A and B only

E.) A and B only Phytoestrogens (soy, flaxseed, isoflavones) may have beneficial effects on lipids, weight, and blood pressure. Data regarding vasomotor symptoms is conflicting, and although phytoestrogens may decrease bone loss, they do not increase bone density or decrease fracture risk.

What are common side effects of aldosterone antagonists such as spironolactone and eplerenone? A.) Hyperkalemia B.) Sedation C.) Headache D.) Rash E.) All of the above

E.) All of the above Hyperkalemia, sedation, headache, and rash are all side effects of aldosterone antagonists. Spironolactone can also cause gynecomastia, but eplerenone is less likely due to no reported activity on androgen receptors.

KF is a 72" tall, 173lb, 30 year-old male with normal renal function that has been on a ventilator in the trauma ICU after a motor vehicle accident. Five days into his hospital stay KF spikes a temperature of 39.4°C, labs reveal WBC count of 17, and there is an infiltrate noted on chest x-ray. The attending surgeon requests assistance with antibiotic coverage, and would like an agent to cover MRSA and two agents to double-cover for P. aeruginosa until cultures and sensitivities are known. Which of the following regimens will cover for MRSA and offer double coverage for P. aeruginosa? A.) Maxipime, Avelox, and Zyvox B.) Azactam, Timentin, and tobramycin C.) Zosyn, amikacin, and daptomycin D.) Unasyn, Rocephin, and amikacin E.) Amikacin, Fortaz, and vancomycin

E.) Amikacin, Fortaz, and vancomycin Amikacin, Fortaz (ceftazidime), and vancomycin is the only regimen that contains 2 anti-pseudomonal agents and MRSA coverage in pulmonary infections. Response C is incorrect because daptomycin cannot be used to treat pneumonia. Responses A and D only contain one anti-pseudomonal agent and option B does not cover MRSA.

A 68-year-old male patient with a past medical history of myocardial infarction, heart failure with a left ventricular ejection fraction of 30%, and atrial fibrillation presents to a clinic complaining of increased fatigue and lethargy. Laboratory studies are significant for increased levels of thyroid-stimulating hormone, suggesting hypothyroidism. His medications include aspirin, carvedilol, furosemide, lisinopril, spironolactone, amiodarone, and warfarin. Which of his medications could be the cause of his hypothyroidism? A.) Carvedilol B.) Furosemide C.) Lisinopril D.) Spironolactone E.) Amiodarone

E.) Amiodarone EXPLANATION: The amiodarone drug molecule contains iodine and, thus, can affect thyroid function. Both hyper- and hypothyroidism can occur with amiodarone.

Which of the following is affected by medications that induce or inhibit the cytochrome P450 enzyme system? A.) Tegaserod B.) Alosetron C.) Fibercon D.) Polyethylene glycol E.) Amitriptyline

E.) Amitriptyline Tricyclic antidepressant serum concentrations are affected by drugs that alter cytochrome P450 activity

Which of the following medications is most likely to be associated with lower-extremity edema? A.) Trandolapril B.) Clonidine C.) Bumetanide D.) Carteolol E.) Amlodipine

E.) Amlodipine EXPLANATION: Amlodipine is a dihydropyridine calcium channel blocker and can be associated with peripheral edema. Trandolapril (an ACE inhibitor), bumetanide (a loop diuretic), clonidine (a centrally acting α2-agonist), and carteolol (a β-blocker) are not typically associated with edema.

A 56-year-old female presents to the local emergency room complaining of crushing, substernal chest pain × 3 hours, which has been unrelieved by SL NTG. Past medical history is significant for hypertension, type 2 diabetes mellitus, TIA × 2, hypercholesterolemia, and metabolic syndrome. Heart rate and rhythm are regular, and no S3 or S4 sounds are present. Vital signs include BP 184/119 mmHg, HR 100 bpm, and RR 32 breaths/min. Laboratory tests show CrCl 63 mL/min. ECG shows ST-segment elevation greater than 1 mm in leads II, III, and aVF. She is immediately admitted to the chest pain center and started on oxygen. The catheterization lab personnel have been notified that she is being transported to the catheterization lab for primary PCI. Which of the following regimens is most appropriate for this patient at this time? A.) Aspirin 81 mg, clopidogrel 600 mg, and enoxaparin 1 mg/kg subcutaneous twice daily B.) Aspirin 81 mg, clopidogrel 600 mg, and heparin 60 unit/kg bolus followed by heparin 12 units/kg/h C.) Ticagrelor 180 mg and eptifibatide 180 mg/kg/IV bolus followed by eptifibatide 2 mcg/kg/h infusion D.) Aspirin 325 mg, prasugrel 60 mg, and bivalirudin 0.75 mg/kg IV bolus followed by bivalirudin 1.75 mg/kg/h E.) Aspirin 325 mg, ticagrelor 180 mg, and bivalirudin 0.75 mg/kg IV bolus followed by bivalirudin 1.75 mg/kg/h

E.) Aspirin 325 mg, ticagrelor 180 mg, and bivalirudin 0.75 mg/kg IV bolus followed by bivalirudin 1.75 mg/kg/h An appropriate PCI regimen includes aspirin, a P2Y12 receptor antagonist, and an anticoagulant agent (UFH, enoxaparin, bivalirudin). In some cases, a GPI may also be added. Answer C is incorrect because it does not include aspirin. The appropriate dose of aspirin for ACS is at least 162 mg; therefore, Answers A and B are incorrect because the dose of aspirin is not high enough. History of TIA or stroke is an absolute contraindication for prasugrel (Answer D).

Which of the following statements is true concerning HEPA filter certification, ISO room classifications, or both? A.) Certification of the PEC shall occur every 12 months during compounding of low- and medium-risk CSPs. B.) The environment of the buffer area shall be tested according to the requirements of ISO 14644 only. C.) The required number of air changes per hour for the buffer area must be determined only when the clean room is certified initially or if extensive remodeling is done to the area. D.) The buffer area, with no exceptions, must always maintain positive pressure in relation to the adjoining rooms or corridors of the pharmacy. E.) At least 30 air changes per hour shall occur, with the PECs allowed to provide up to 15 of the 30 required air changes per hour.

E.) At least 30 air changes per hour shall occur, with the PECs allowed to provide up to 15 of the 30 required air changes per hour. Certification of all ISO-classified environments shall occur every 6 months and shall be tested according to ISO 14644. Part of the certification includes determination of the number of air changes per hour for the ante-area and the buffer area and determination of the differential pressures between the areas. This is required by USP <797>. As stated before, 30 air changes per hour are required, but the PECs are allowed to provide up to 15 of the 30 air changes. The buffer area should maintain positive pressure in relation to the rest of the pharmacy unless compounding of cytotoxic drugs takes place in the area. When talking about the efficiency of a filter, you want to make sure you are measuring the efficiency of the most penetrating size of particle. The size of particle most penetrating with a HEPA filter is 0.3 micron, and the HEPA filter is 99.97% efficient at removing particles 0.3 micron in size. That means particles larger and smaller will be removed at an even higher efficiency. The required air changes per hour are 30, with the PECs allowed to provide up to 15 of the 30 required air changes per hour. If the buffer area is used to compound cytotoxic drugs, 0.01 inch of water-column negative pressure is required. Refrigerators, freezers, and printers are not prohibited from the buffer area, but the environment should be monitored, and their effect on the environment should be evaluated. The CAI is vertical flow and requires a different technique from that used in horizontal flow. The area inside a CAI is sanitized—not sterilized—so good aseptic technique is still required.

Which of the following agents may cause nephrolithiasis? A.) Radiocontrast dyes B.) Analgesic combinations C.) Cyclosporine D.) Amphotericin B E.) Indinavir

E.) Indinavir Indinavir may crystallize within the renal tubules and cause nephrolithiasis, especially in patients with hypovolemia. Other agents listed are causes of drug-induced kidney injury but not specifically nephrolithiasis.

Which of the following is a black box warning of Concerta ? A.) Concerta has been associated with serious cardiovascular events and should not be used in adults with pre-existing structural cardiac abnormalities or serious heart problems B.) Hypersensitivity reactions, such as angioedema and anaphylactic reactions have been reported C.) Stimulants are associated with peripheral vasculopathy D.) Stimulants can cause dangerous and prolonged erections (priapism) E.) Concerta has a high risk of abuse and dependence

E.) Concerta has a high risk of abuse and dependence Correct answer: E CNS stimulants have a black box warning for abuse and dependence. The risk of abuse should be assessed prior to prescribing. This patient has a history of substance use disorder (now in remission) - but should be counseled appropriately and monitored closely for signs of abuse and aberrant behavior. Answer A:Cardiovascular events are a warning for stimulants but not a black box warning. She should be monitored closely given her history of congestive heart failure and hypertension. Answer B:Hypersensitivity reactions are a warning for stimulants but not a black box warning. However, she should be monitored closely given her history of hypersensitivity with other medications. Answer C:Peripheral vasculopathy is a warning for stimulants but not a black box warning. She should be monitored closely given her history of Raynaud syndrome (a rare condition in which there is reduced blood flow to some areas of the body such as fingers and toes). Answer D:Priapism is a warning for stimulants but not a black box warning and this patient is female, thus this does not apply to her. Bottom Line:CNS stimulants, including Concerta, have a black box warning for abuse and dependence.

JH is a 74 year-old man with a past medical history significant for hypertension, diabetes, and emphysema. JH has been experiencing a productive cough for the past 2 weeks, but over the past 4 days it worsened and he has been unable to do daily activities due to shortness of breath. What antibiotic therapy should be offered to JH? A.) Antibiotic therapy should not be offered B.) Amoxicillin-clavulanate plus moxifloxacin C.) Ceftriaxone D.) Telithromycin E.) Doxycycline

E.) Doxycycline Doxycycline is the only choice listed that is appropriate for the treatment of an acute exacerbation of chronic bronchitis. Response A is appropriate treatment for acute bronchitis but not an exacerbation of chronic bronchitis.

What is the trade name for mometasone + formoterol MDI? A.) Foradil B.) Pulmicort C.) Combivent D.) Symbicort E.) Dulera

E.) Dulera

A.T. is a 19 year old female with new onset schizophrenia, paranoid type. The psychiatrist starts A.T. on haloperidol 2.5 mg po BID. Shortly after her second dose A.T. complains of sharp muscle pains in her neck and back. What side effect is A.T. experiencing and how should it be treated? A.) Pseudoparkinsonism- Give amantadine 100 mg po BID B.) Tardive dyskinesia- Discontinue haloperidol and give dantrolene 50 mg po TID C.) Tardive dyskinesia- Give benztropine 2 mg po every 30 minutes until symptoms resolve D.) Dystonia- Give dantrolene 50 mg po TID E.) Dystonia- Give benztropine 50mg IM every 30 minutes until symptoms resolve

E.) Dystonia- Give benztropine 50mg IM every 30 minutes until symptoms resolve Explanation: A.T. is experiencing a dystonic reaction. Dystonia is an extrapyramidal symptom that presents shortly after initiating or increasing the dose of an antipsychotic. Dystonia is characterized by painful, involuntary muscle spasms usually located in the face, neck or back. Dystonic reactions need to be treated urgently with an intramuscular anticholinergic such as benztropine or diphenhydramine. The anticholinergic dose may be repeated every 30 minutes until symptoms resolve.

JU is a 30 year-old Caucasian female on chronic hemodialysis. Within the first hour of her Wednesday hemodialysis session, she develops a fever (38.4°C), chills, and tachycardia (HR =131). Blood cultures from her dialysis catheter and a percutaneous site are drawn. A laboratory analysis is also performed. Laboratory results: WBC: 17.6 Neutrophils: 82% Bands: 14% What is the most likely causative agent for JU's infection? A.) Gram-negative bacilli B.) Anaerobic bacteria C.) Candida albicans D.) Atypical bacteria E.) Gram-positive cocci

E.) Gram-positive cocci Gram-positive cocci are the most common pathogens in catheter-related infections.

Which of the following conditions are the most common electrolyte disorders caused by cisplatin? A.) Hyponatremia and hypophosphatemia B.) Hyperkalemia and hypermagnesemia C.) Hyperkalemia and hypomagnesemia D.) Hypokalemia and hypermagnesemia E.) Hypokalemia and hypomagnesemia

E.) Hypokalemia and hypomagnesemia EXPLANATION: According to the manufacturer's information, cisplatin causes hypokalemia and hypomagnesemia.

Which of the following opioids is metabolized through the cytochrome P450 hepatic enzyme system? I. Hydrocodone II. Oxycodone III. Fentanyl A.) I only B.) II only C.) I and III only D.) II and III only E.) I, II, and III

E.) I, II, and III All are metabolized by the cytochrome P450 hepatic enzyme system.

Which of the following statements concerning drugs used to treat glaucoma are true? I. Carbonic anhydrase inhibitors, with the exception of acetazolamide and methazolamide, can be used as monotherapy. II. Latanoprost, bimatoprost, and travoprost can alter eye color. III. Nonselective beta adrenergic antagonists are often the first choice for treatment of open-angle glaucoma. A.) I only B.) III only C.) I and II only D.) II and III only E.) I, II, and III

E.) I, II, and III All of the statements are true. Acetazolamide and methazolamide are oral drugs and are not used for monotherapy.

Analysis of the cerebrospinal fluid may give valuable clues to the identity of the pathogen in meningitis. Given the following results, what would be indicative of a fungal infection? I. Increase in WBCs II. Decreased glucose III. Increased protein A.) I only B.) II only C.) I and III only D.) II and III only E.) I, II, and III

E.) I, II, and III Although fungal CNS infections show relatively slight changes in WBCs, protein, and glucose compared to bacterial infections, the trend is similar.

Which of the following are risk factors for a febrile seizure? I. A first-degree relative with a history of febrile seizures. II. Delay discharge from the neonatal nursery (more than 28 days). III. Day care attendance A.) I B.) I and II C.) I and III D.) II and III E.) I, II, and III

E.) I, II, and III In addition to the three risk factors that are listed, developmental delay is a risk factor.

Which of the following drugs is used to treat Cushing's disease? I. ketoconazole II. aminoglutethimide III. mitotane A.) I only B.) III only C.) I and II only D.) II and III only E.) I, II, and III

E.) I, II, and III Ketoconazole, aminoglutethimide, and mitotane are all used to treat Cushing's disease. Ketoconazole is most commonly known as an antifungal agent, but it inhibits cortisol synthesis at high doses (800-1200 mg daily).

M. L. is a 38-year-old African American female with ESRD on peritoneal dialysis. At her latest appointment, her PTH was 480 pg/mL, her corrected calcium was 10.8 mg/dL, and her phosphorus was 6.9 mg/dL. Which of the following is the most appropriate phosphorus binder for M. L.? A.) Calcium carbonate B.) Magnesium hydroxide C.) Aluminum hydroxide D.) Calcium acetate E.) Lanthanum carbonate

E.) Lanthanum carbonate M. L.'s calcium is already above the goal for a patient with ESRD, so the use of calcium-based phosphate binders is not advised. Magnesium and aluminum are not first-line agents because of the risk of accumulation in patients with kidney disease. Lanthanum carbonate is a non-calcium-containing binder that, when taken appropriately, should lower phosphorus without increasing serum calcium.

KF is a 72" tall, 173lb, 30 year-old male with normal renal function that has been on a ventilator in the trauma ICU after a motor vehicle accident. Five days into his hospital stay KF spikes a temperature of 39.4°C, labs reveal WBC count of 17, and there is an infiltrate noted on chest x-ray. The attending surgeon requests assistance with antibiotic coverage, and would like an agent to cover MRSA and two agents to double-cover for P. aeruginosa until cultures and sensitivities are known. What is the goal trough for extended-interval tobramycin dosing for KF? A.) 4 to 5mcg/mL B.) 15 to 20mcg/mL C.) Less than 10mcg/mL D.) Less than 3mcg/mL E.) Less than 1mcg/mL

E.) Less than 1mcg/mL The recommended goal trough for tobramycin and gentamicin is < 1mcg/mL regardless of treatment indication.

Which of the following medications is preferred to control the ventricular rate in a patient with atrial fibrillation and a recent myocardial infarction? A.) Digoxin B.) Verapamil C.) Amlodipine D.) Amiodarone E.) Metoprolol

E.) Metoprolol Metoprolol is a beta-blocker and is the preferred agent in this case as it will effectively control the rapid ventricular response associated with atrial fibrillation and it will reduce the risk of mortality and recurrent myocardial infarction in this patient with a previous MI.

Which drug is NOT indicated in COPD patients.? A.) Long-acting inhaled β2 agonists B.) Tiotropium C.)Inhaled corticosteroids D.) Methylxanthines E.) Montelukast

E.) Montelukast Although montelukast has had very limited study in COPD patients, it is officially indicated for asthma and allergic rhinitis, but not COPD

R. T. is a 40-year-old male admitted to the medical ICU following a severe asthma exacerbation. R. T.'s serum phosphorus is 0.9 mEq/L, and his body weight is 70 kg (100% of ideal). Which of the following acute phosphorus supplementation regimens is most appropriate? A.) 45 mmol of sodium phosphate IV over 6 hours B.) 45 mmol of sodium phosphate IV over 10 minutes C.) 15 mmol of po phosphorus (e.g., Neutra-Phos) over the next 24 hours D.) 15 mmol of IV sodium phosphate over 2 hours E.) No acute phosphorus therapy is required.

E.) No acute phosphorus therapy is required. R. T. is severely hypophosphatemic and requires high-dose IV therapy (0.64 mmol/kg × 70 kg = 44.8 mmol). The dose should be infused at 7.5 mmol/h (total time: 6 hours) to avoid precipitation with calcium.

Which one of the following medications interacts with belatacept ? A.) Azithromycin B.) Diltiazem C.) Erythromycin D.) Voriconazole E.) None of the above

E.) None of the above Azithromycin, diltiazem, erythromycin, and voriconazole interact with cytochrome P450 enzymes. To this point, no drug interactions have been reported with belatacept.

Sotalol is metabolized by and inhibits the metabolism of which cytochrome P450 enzymes, respectively? A.) CYP 3A4 and CYP 2D6 B.) CYP 2D6 and CYP 3A4 C.) P-gp and CYP 2D6 D.) P-gp and CYP 3A4 E.) None of the above

E.) None of the above Sotalol is eliminated almost entirely by the kidneys and is not affected by CYP450 enzymes.

Which of the following conditions is not a microvascular complication of diabetes? A.) Retinopathy B.) Nephropathy C.) Polyneuropathy D.) Autonomic neuropathy E.) Peripheral vascular disease

E.) Peripheral vascular disease EXPLANATION: Peripheral vascular disease is a chronic complication of diabetes that also includes coronary atherosclerosis and cerebrovascular atherosclerosis. The other answers are considered microvascular complications of diabetes.

Which of the following medications are subject to control under the Federal Controlled Substances Act of 1970? A.) Phenytoin and phenobarbital B.) Phenobarbital and levetiracetam C.) Levetiracetam and phenytoin D.) Gabapentin and pregabalin E.) Pregabalin and phenobarbital

E.) Pregabalin and phenobarbital EXPLANATION: Pregabalin and phenobarbital are both subject to control under the Federal Controlled Substances Act of 1970. Pregabalin and phenobarbital are both schedule V controlled substances

Which of the following is most commonly associated with bacterial conjunctivitis beyond the first month of life? A.) Chlamydia B.) Clostridium C.) E. coli D.) Neisseria E.) Staphylococcus

E.) Staphylococcus The most common pathogens in neonatal bacterial conjunctivitis are Neisseria gonorrhoeae, Chlamydia trachomatis, Staphylococcus aureus, Staphylococcus epidermidis, Streptococcus pneumoniae, and Escherichia coli. Bacterial conjunctivitis beyond the first month of life is most commonly caused by Staphylococcus aureus, Staphylococcus epidermidis, Streptococcus pneumoniae, and Haemophilus influenzae. Clostridium, an anaerobe, is not a common bacterial pathogen in conjunctivitis.

Which of the following agents would not be administered at the same time as heparin? A.) tPA B.) Reteplase C.) Eptifibatide D.) TNKase E.) Streptokinase

E.) Streptokinase A combination of UFH with streptokinase is not desirable because streptokinase is a nonspecific fibrinolytic, and UFH may increase the risk of bleeding because of streptokinase's long half-life. Therefore, answer E is the correct choice. Heparin should be administered for at least 48 hours with the other lytic choices (Answers A, B, and D) to reduce risk of re-occlusion. A GPI should be administered with heparin, and therefore C is not the correct answer.

Which parts of the needle are considered critical sites? A.) The hub only B.) The needle shaft only C.) The hub and the bevel and bevel tip only D.) The needle shaft and the bevel and bevel tip only E.) The hub, the needle shaft, and the bevel and bevel tip

E.) The hub, the needle shaft, and the bevel and bevel tip The hub, the needle shaft, and the bevel and bevel tip of the needle are all critical sites.

Which anticonvulsant should be used with caution in a pediatric or adult patient with angle closure glaucoma? A.) Rufinamide B.) Pregabalin C.) Levetiracetam D.) Lamotrigine E.) Topiramate

E.) Topiramate An ocular syndrome characterized by acute myopia and secondary angle closure glaucoma has been reported. Symptoms consist of blurred vision and ocular pain and generally present in the first month of use. Patients who develop blurred vision or eye pain while on Topamax should seek immediate medical attention. An eye exam may also find myopia, anterior chamber shallowing, redness, increased intraocular pressure and possibly extended dilation of the pupil.

In a patient with documented heparin-induced thrombocytopenia, what is the appropriate duration of therapy for a direct thrombin inhibitor (DTI)? A.) 28-35 days B.) 3 months C.) 6-12 months D.) Until discharged from the hospital E.) Until platelets are greater than 150,000/mcL

E.) Until platelets are greater than 150,000/mcL EXPLANATION: The American College of Chest Physicians' guidelines indicate that DTI therapy should continue at least until the platelets are greater than 150,000/mcL.

Urinary incontinence secondary to detrusor hyperactivity is most often associated with which type of incontinence? A.) Functional B.) Over-flow C.) Stress D.) Transient E.) Urge

E.) Urge Uninhibited detrusor muscle contraction of the bladder results in loss of control with bladder emptying and is termed Urge Incontinence. This condition is more often seen in women and is treated by inhibition of acetylcholine mediated detrusor contractions using an anticholinergic.

Which of the following best describes the site and mechanism of action of acarbose in diabetes? A.) Within the liver to impair glucose release B.) Within the pancreas to increase insulin production C.) Within the muscles and fat to decrease insulin resistance D.) Within the stomach to slow gastric emptying E.) Within the intestine to impair carbohydrate absorption

E.) Within the intestine to impair carbohydrate absorption Alpha-glucosidase inhibitors, such as acarbose, cause a delay in the digestion of carbohydrates into simple sugars and their subsequent absorption in the small intestine.

Which drug(s) are best for long-term management of severe persistent asthma? A.) Albuterol MDI as needed and before exercise + high dose Flovent B.) Montelukast at bedtime + high dose Pulmicort C.) Salmeterol twice daily + Montelukast at bedtime D.) Theophylline at bedtime + Formoterol twice daily + high dose Qvar E.) high dose Advair 500 Diskus twice daily + prn albuterol

E.) high dose Advair 500 Diskus twice daily + prn albuterol High dose Advair (500/50) twice daily plus prn albuterol. High dose Symbicort or Dulera would also be appropriate

The following statements concerning acetazolamide drug interactions are true except that A.) it interacts with cyclosporine to cause increased cyclosporine levels. B.) it may increase aspirin levels and cause toxicity. C.) it interacts with phenytoin to cause an increased risk of osteomalacia. D.) it interacts with lithium to cause either increased or decreased lithium levels. E.) it interacts with lisinopril to cause alteration of potassium.

E.) it interacts with lisinopril to cause alteration of potassium. EXPLANATION: Acetazolamide can cause initial hypokalemia, which is self-limiting, but it has no interaction with lisinopril. The other interactions are reported when the listed medication is given with acetazolamide.

Which of the following should be avoided in patients taking gemfibrozil? A.) metoprolol B.) losartan C.) thiamine D.) niacin E.) lovastatin

E.) lovastatin Previous FDA labeling guidelines stated a max daily dose of 20mg lovastatin with concurrent use of gemfibrozil. However, the new labeling states that concurrent use of lovastatin and gemfibrozil should be avoided.

Which of the following drugs works by decreasing cortisol synthesis? A.) Cortrosyn B.) ACTH C.) oxandrolone D.) prednisone E.) metyrapone

E.) metyrapone Metyrapone (Metopirone) inhibits 11-hydroxylase activity, and thus decreases cortisol synthesis.

The possible benefits of LMWH over UFH include all of the following except A.) predictable response. B.) ease of administration. C.) no recommended routine monitoring. D.) lower incidence of heparin-induced thrombocytopenia. E.) no renal adjustment necessary.

E.) no renal adjustment necessary. Renal adjustment is necessary with LMWH in patients with a CrCl < 30 mL/min whereas UFH does not require dosage adjustment. LMWH has advantages over UFH in ease of administration (Answer B), reduced incidence of heparin-induced thrombocytopenia (Answer D), more predictable therapeutic response (Answer A), and lack of required monitoring (Answer C).

Which one of the following should be used with caution in patients with a baseline viral load over 100,000 copies/mL? A.) tenofovir/emtricitabine + nevirapine B.) tenofovir/emtricitabine/efavirenz C.) tenofovir/emtricitabine + atazanavir + ritonavir D.) tenofovir/emtricitabine + raltegravir E.) tenofovir/emtricitabine/rilpivirine

E.) tenofovir/emtricitabine/rilpivirine When used in patients with viral loads greater than 100,000 copies/mL, the patients receiving the components of tenofovir/emtricitabine/rilpivirine (Complera) experienced a higher incidence of virologic failure and resistance as compared to patients receiving the components of tenofovir/emtricitabine/efavirenz (Atripla).

R. C. is a 33-year-old female diagnosed with stage 2 breast cancer. R. C. is 5'4", weighs 200 lb, and is considered obese because of her body mass index of 34.3. Pathology reports show that her breast cancer is estrogen receptor-negative, progesterone receptor-negative, and HER2 negative, known as "triple negative" breast cancer, an especially aggressive tumor type. After a modified radical mastectomy with sentinel lymph node biopsy, R. C. will receive chemotherapy with the dose-dense AC followed by T regimen (doxorubicin 60 mg/m2 intravenous push over 3-10 minutes and cyclophosphamide 600 mg/m2 intravenous infusion over 30 minutes, both given once every 2 weeks for 4 cycles, followed by paclitaxel 175 mg/m2 intravenous infusion over 3 hours once every 2 weeks for 4 cycles, with pegylated filgrastim 6 mg subcutaneous given the day after treatment in each of the 8 cycles to prevent febrile neutropenia). The combination of doxorubicin and cyclophosphamide is highly emetogenic, whereas paclitaxel has low emetogenicity. Before this diagnosis, R. C. had never had cancer or cancer chemotherapy medications. She has smoked 1 pack of cigarettes per day for 20 years and has never consumed alcohol. She is gravida 3, para 3; all children are alive and healthy. Her family history is negative for breast cancer. R.C. is married and lives with her husband and children about 15 minutes from the cancer center. Because R. C.'s tumor is estrogen receptor-negative, she is not a candidate for therapy with which of the following? A.) Tamoxifen B.) Anastrozole C.) Trastuzumab D.) Bevacizumab E.) Lapatinib F.) A and B only

F.) A and B only Randomized studies have established that tamoxifen, an estrogen receptor agonist/antagonist, and aromatase inhibitors such as anastrozole, which block the production of estrogens from nonovarian sources, are effective only against breast cancer that is estrogen receptor-positive.


Set pelajaran terkait

SCM 200 Final Exam True or False

View Set

Study set 15 for RN NCLEX (Kaplan)

View Set

algebra 2a - unit 2: key features and graphing quadratics lesson 5-8

View Set

Citizenship handbook notes- Law making process

View Set